You are on page 1of 188

2015 DISCLAIMER

QU This is solely a study material which is compiled and edited. There is no such a copyright to
restrict the knowledge and efforts of anyone in the field of education. There is no such an
intention to hurt anybody to whosoever have prepared. I have tried to collect all the material
whichever available for only and only ment for STUDY purpose.

Sources:-

 Editor‟s question bank


 Editor‟s beloved seniors and teacher efforts
 Amigos (TNPSC notes)
 Vetcos
 Internet souces
 Karnataka state Question bank

QUESTION BANK

Editor dedicated this compilation to the vets and student who preparing for ICAR-JRF and
make so much easy for them to read. It‟s very difficult to get proper reading material from
here and there. So, editor efforts to compile this material and to serve directly towards the
student for the studing.

Edited by: - Dhruv N Desai


2nd Edition
(Revised and large compilation)

Dhruv N Desai
B.V.Sc & A.H (Navsari, Gujarat)
Pursuing M.V.Sc
ICAR-IVRI
Izatnagar, Bareilly Declairation
Editor doesn‟t have any copyright for this material even didn‟t put any rights for their own
E-mail: dhruvdesai24@gmail.com material which is compiled.

d[Type text] Page 1 2


Dhruv N Desai
 Volume of semen in boar - 250 ml  Infective stage of schistosoma – Cercaria
 Drug of choice for theileriosis – Buparvaquone  Cuboni test for pregnancy diagnosis done in which animal – Mare
 Most effective drug for cestodiasis – Praziquntal  Duration of spermatogenesis in buffalo – 64 days
 Lambert suture not applied in which of the following organ a) rumen b) uterus c) stomach d)  Drainage of middle ear provided through – Zepps operation
esophagus(answer)  Feline distemper is known as – Feline panleukopenia
 Anti nutritional factor present in cotton seed – Gossypol  Ketone bodies in the urine is due to – Hypoglycemia
 Drug which prevent platelet aggregation – Aspirin  Smallest virus – FMD virus
 Range of rumen ph – 5-7  Which one of the following is a live vaccine –Brucella s-19 vaccine
 Number of lumbar vertebrae in dog – 7  Breech presentation is also known as – Posterior longitudinal
 Longest muscle in the body – Longissmus dorsi  Diagnosis of camphylobactor is done by – Vaginal mucous agglutination test
 Organism which causes aflatoxicosis -Penicillium leberi ( aspergillus flavus not found in the  Dolphins and whales breath through a) gills b) Lungs(answer) c)spiracles
answer choices)  Agar contains - a) protein b) lipid c) Carbohydrates d) all the above
 Organism causing brooder pneumonia - Aspergillus fumigatus  Raw egg feeding in dogs causes deficiency of – Biotin
 Gnrh secreted from – Hypothalamus  Paralysis of hind quarters is known as – Paraplegia
 Hormone that is released in high amount in 2 nd stage of labour – Oxytocin  Garlic odour of food content is seen in -Phosphorus poisonining
 Dilated pupil and fish eye appearance seen in which stage of anesthesia – Stage iv  Antidote for lead poisoning – Ca-Na EDTA
 Heart of bovine is attached to – Pericardio sternal ligament  Bronze coloured liver is seen in a)Fowl typhoid b) pullorum disease c)fowl cholera
 Largest antibody type - Ig M  Ulcers in abomasum is seen in –Theileriosis
 Antibody which is lining in the mucosal surface – Ig A  Caecal coccidiosis is caused by – Eimeria tenella
 Viral genome contain a) RNA b) DNA c) Either RNA or DNA ( answer)  Sugar needed for multiplication of brucella – Erythrital
 Virus which contain reverse trascriptase enzyme – Retroviridae  Brucella ovis in sheep causes – a) orchitis b) prosthitis c)Epididymitis
 For closing of uterine incision sutures should be started from a) cervical end b) ovarian end  Ovulation occurs after end of estrous in which species of animal – bovine
(answer)  Largest deer species in india a) sambar b) spotted deer c) Barasinga
 Catgut obtained from sub-mucosa of – Sheep  Route of infection in anchylostoma species – Skin penetration
 Horse is which type of animal – Seasonally polyestrous  Vagus nerve is a a)sensory b)motor c) mixed
 Purkinje cells present a) myocardium b)cerebrum c) cerebellum (answer)  Pearls is seen in a) Squamous cell carcinoma b) basal cell carcinoma
 Drug inhibiting cell wall synthesis – Penicillin  Ingestion of lantana plant associated with – Photosensitivity
 Organism devoid of cell wall – Mycoplasma  Motility of bacteria is due to a) plasmid b) pili c) Fimbriae
 Microscopic lesion in bse - Vacuolation of neurons  Which one of the following is an extracellular organism a) babesia b)anaplasma c) Trypanasoma
 Cyclozoonosis related to – Ecchinococcosis  Diabetes insipidus is due to deficiency of – Antidiuretic hormone
 Long acting local anesthetic – Bupivaquone  Recent out break in avian influenza is due to – H5N1
 Thawing temperature of semen- 37C for 30 sec
 Which dose is related to highly toxic drugs – 1-50 mg / kg body weight  Hjarres disease is caused by – E coli
 Accepted fluoride content in water – 1mg / lit water  Chemical used for control of snail population – Copper sulphate
 Vector for leishmaniasis – Phlebotomus  Agent for blister - Bin iodine of mercury
 Stomach fluke of bovine – Paramphistomiasis  Dry matter feeding of cattle a) 3% of body weight b) 3% of metabolic body weight
 Pipe stem liver is caused by - Fasciola hepatica  In right side torsion animal should be casted on a) right side b) left side
 Organism not having filaments – Mycobacterium  Nerve block done for examination of penis and prepuce – Pudental block
 Irritable non ionic drug should be administered in which route – Intravenous  Destruction of free radicals associated with which vitamin – Vitamin-E
 Example of brachiocephalic breed – Pug  Example for angiotensin converting enzyme inhibitor – Captopril
 Acute gangrenos myositis associated with – Black quarter  For horn surgery in goat nerve to be blocked a) cornual nerve b) infratrochlear nerve c) cornual
 Organism causing interstitial nephritis – Leptospira nerve &infratrochlear nerve
 Most abundant leukocytes type in bovines – Lymphocyte  For preventing radiation hazard wear a apron made of – Lead
 Blood cells containing granules toxic to parasite – Eosinophils  Spermatid is basically – Haploid
 Nucleated thrombocytes seen in – Fowl  Epsilon toxin is associated with – Enterotoxaemia
 Camel is which type of animal – Induced ovulator  Which ligament should be cut in CLP – Median patellar ligament
 Diffused pus in the connective tissue is known – Phlegmon  Navicular bone is the name of – Distal sesamoid bone
 Bilateral tubular inpatency causes a) anestrous b) Repeat breeding(answer) c) early embryonic  Vitamin administered in dicoumarol poisoning – Vitamin-K
mortality  ‗Bioterrorism‘ associated with which disease – Anthrax
 Reference test for rabies – FAT  Visceral larva migrants is caused by – Toxocara canis
 Rabies is which type of virus – Neurotrophic  Organism present in pulmonary artery – Dirofilaria immitis
 Gasping observed in which disease – Infectous laryngeo tracheitis  Programmed cell death is known as – Apoptosis
 Immuno deficiency in IBD is due to destruction of - B-Lymphocytes  Gestation period of horse – 11months 11days
 T-cell maturation occurs in – Thymus
3 4
Dhruv N Desai Dhruv N Desai
 Organ to be examined in post mortem examination of trichinella spiralis a) lung b) liver c) intestine 7. Rumen fluid pH in acid indigestion - 4-5.
d) Diaphragm 8. Rumen fluid pH in protein putrefaction/alkaline indigestion-8-10.
 Glial cells present in - Brain 9. Pulse is characterized by rate, rhythm and quality or volume.
 Calcitonin secreted from - a) Thyroid gland b) parathyroid gland 10. Site of blood collection
 Which one is not related to mycoplasma – a)CCPP b) CBPP c) CRD d) BSE a. Cattle, Buffalo, sheep & goat - Jugular vein, ear vein.
 Liver develops from – a) ectoderm b) Endoderm c) mesoderm b. Horse - Jugular vein
 Which one of the following is called as endogenous antigen a) MHC I b) MHC II c) MHC III c. Dog - Recurrent tarsal vein, cephalic vein.
 Inflammation of hoof is known as – a) bursitis b) synovitis c) laminitis d. Pig - Ear vein, anterior vena cava.
 Lung is distinctly lobulated in a) cattle b)fowl 11. Site of CSF collection:
a. Cattle & Buffalo- lumbo sacral or 1st, 2nd coccygeal region
b. Horse - Sub occipital or lumbo sacral region.
CLINICAL MEDICINE c. Dog - cisternal puncture.
d. Sheep & goat - suboccipital or lumbo-sacral region.
1. Methods of physical examination in cattle 12. Normal pulse and respiration ratio - 4:1
a. Inspection- observing through eyes. 13. Bloat - Drum like sound(on percussion)
b. Palpation - feeling through hands. 14. Left abomasal displacement - simultaneous auscultation and percussion over an area between the
c. Percussion - tapping through fingers. upper third of the 9th and 12th ribs of abomasal wall - sounds heard are; Ping sound/metallic
d. Auscultation - listening through stethoscope. sound/ penny dropping sound/ tinkling sound/ splashing sound/ pebble in well sound.
2. The phonendoscope is used for auscultation purpose in case of ruminants. 15. William‟s auscultation is the simultaneous auscultation of reticulum and palpation of rumen in
3. Sites of recording pulse rate. Traumatic reticulo peritonitis.
a. Cow &Buffalo - Coccygeal artery, Maxillary artery, facial artery. 16. Recording and Measurement of heart sounds is done by Phonocardiography.
b. Calf- femoral artery. 17. CSF pressure is measured by Manometer.
c. Dog & cat - femoral artery. 18. The 2nd to 5th intercostals space is the site for the auscultation of heart.
d. Sheep & goat - femoral artery. 19. Somnolence is the state when animal is depressed to the point that it is unable to hold its head.
e. Horse, donkey& mule - External maxillary artery, median artery, facial artery. 20. Dorsal bending of spinal column is known as Kyphosis.
4. Types of respiration 21. Ventral bending of spinal column is known as Lardosis.
a. Costal (Thoracic) - dog and cat. 22. Lateral bending of spinal column is known as Scoliosis.
b. Abdominal - cattle, buffalo, sheep and goat. 23. A cow sits on the sternum and rests on the flank in Milk fever.
c. Costo-abdominal - Horse, mule, donkey and Ass. 24. In downer cow syndrome, a cow has no defect in eating, defecation or urination but will be unable
5. Normal parameters to stand and remains in sternal recumbancy.
25. Amaurosis is total blindness.
Species Respiratory rate Normal temperature Pulse rate 26. Amylopia is partial blindness.
27. Abducted elbow with extension of head and neck in a cow with brisket edema and jugular pulsation
Cattle 10-30/minute 38.5ºC(101.5ºF) 60-80/minute is seen in traumatic pericarditis.
28. A male dog urinating like a bitch in cystitis and urolithiosis.
Horse 8-10/minute 38.0ºC(100.5ºF) 29. A horse adopting a dog sitting posture with kicking at the belly is seen in acute gastric dilatation.
30. Goose stepping gait in pigs is due to the deficiency of pantothenic acid.
Sheep 10-20/minute 39.0ºC(102.0ºF) 70-90/minute 31. High stepping gait with rigidity of limbs is seen in tetany.
32. Knuckling of fetlock is seen in neuritis and nerve paralysis.
Goat 25-35/minute 39.5ºC(103.0ºF) 70-90/minute
33. A pulse which is brief, small and hard in nature is called the wiry pulse.
34. When the pulse wave is small,weak and prolonged it is termed as thready pulse.
Pig 10-20/minute 39.0ºC(102.0ºF)
35. The respiratory centre is located in the medulla oblongata.
Dog 14-30/minute Large-37.5-38ºC Large-70- 36. Euphoea is the normal quiet breathing with usual respiratory rate in an animal.
37. Hyperpnoea is the increased rate of respiration with an increased or decreased depth of respiration.
(99.5-101.5ºF) 90/minute
38. Polypnoea is the increased rate of respiration with reduction in depth.
39. Oligopnoea is the decreased rate of respiration.
Small-38.5-39.5ºC Small-90-
40. Apnnoea is the complete cessation of breathing.
(101.5-102.5ºF) 120/minute 41. Cheyne-strokes respiration is the gradual decrease in the depth of respiration in a renal and
cardiac disease.
42. Biot‟s respiration is characterised by altering periods of Hyperpnoea and apnea ( in meningitis).
43. Kussmaul‟s respiration or air hunger is the respiration which is forceful and regular but expiration
is unaffected ( in uremia, diabetic ketoacidosis).
6. Normal rumen fluid pH - 6.2-7.2. 44. Chronic pulmonary obstructive disease/heaves in horse is characterized by double expiration.
45. Septic shock occurs predominantly due to Gram positive organism.
5 6
Dhruv N Desai Dhruv N Desai
46. Depraved or perverted appetite is known as pica/ allotriophagia. 94. The prepartum diet contains high amount of sodium and potassium (cations) which increases the
47. Coprophagia is the eating of its own or other animal‘s faeces. chance of milk fever.
48. Pilophagia is the licking of hair and body coat. 95. Eclampsia in bitches is characterised by clinical signs like opisthotonus arch, tonoclonic
49. Normal rumen motility is 7-12/5 minutes. conulsion, high rise in temperature (108ºC).
50. The interval between two rumen motility should not be more than 2 minutes. 96. Phosphorus deficiency may reduce glucose utilization leading to reduced production of ATP
51. Rumen hypermotility is seen in conditions like bloat, acidosis, simple indigestion, TRP. which is essential to maintain the integrity of RBC.
52. Grunting can be heard on auscultation of traumatic reticulo- peritonitis. 97. Phosphorus deficiency may cause reduction of red cell glycolysis and decrease in ATP synthesis.
53. Pole or Bamboo test is the raising of Animal forcefully by placing the pole on the xiphoid 98. Normal level of phosphorus is 4-7mg/dl. In case of deficiency it decreases to 0.5 to 3 mg/dl.
cartilage of sternum in lower third of the chest. It will make the animal to grunt. Used in TRP. 99. In horse azoturia the serum creatinine phospho kinase(CPK) level is highest in skeletal and
54. In dogs the liver enzyme SGPT is measured. cardiac muscle degeneration. The AST level is also increased.
55. In large animals the liver enyzme SGOT is measured. 100. Normal serum magnesium level is 1.7 to 3 mg/dl. In hypomagnesaemia it reduces to 0.5mg/dl.
56. Diaphragmatic hernia is common in Buffaloes. 101. Fatty liver syndrome is treated by administering choline chloride ( which facilitates the
57. Liptak test is used for the diagnosis of LDA. transportation of fatty acids from liver to fat deposits) and vitamin B12.
58. Abomasal pH is 2-4. 102. Baby pig disease due to hypoglycaemia.
59. ECG means atrial depolarization and ventricular repolarization. 103. Thiamine hydrochloride helps in lactate metabolism and is given for acid indigestion.
60. In ECG, prolongation of P wave indicates left atrial enlargement. 104. TRP is also called as hardware disease.
61. In ECG, increased P wave amplitude indicates right atrial enlargement. 105. Vagus indigestion is also called as Hoflund syndrome.
62. In ECG, tall R wave and prolongation of QRS complex indicates left ventricular enlargement. 106. In vagus indigestion constipation is an important clinical sign.
63. In ECG, deepening of S wave indicates Right ventricular enlargement. 107. In vagus indigestion failure of two sites is seen;
64. In ECG, peaked T wave indicates hyperkalemia. a. Omasal transport failure
65. The predominant bacteria in rumen fluid are gram negative. In case of acidosis it is gram b. Pyloric outflow failure-causes hypochloric metabolic alkalosis
positive. 108. Bradycardia can be considered as a diagnostic factor for vagal indigestion.(ATROPINE test: given
66. Acid indigestion due to ingestion of large amounts of highly fermentable carbohydrates. 30 mg of atropine sulphate s/c and heart beat is monitored for every 2 to 5 minutes)
67. Diagnostic tests used for simple indigestion are sediment activity test and cellulose digestion test. 109. LDA is common in high yielding dairy cows rather than RDA.
68. Acute pancreatitis in dogs leads to a praying stance. 110. Ping sound (high-pitched) is heard in case ofLDA,RDA, cecal dilatation and tortion, intestinal
69. TRP is characterised by leucocytosis with neutrophilia -left shift. tympany associated with acute obstruction or paralytic ileus, pneumoperitonium, tortion of the coile
70. Vagus indigestion has papple shaped abdomen that is pear shaped in the right side and apple colon.
shaped in right side. 111. Pung(low pitched ping) is heard in gas filled rumen.
71. Normal blood calcium level is 9-11mg/dl in a cows 112. The etiology for secondary bloat is oesophageal obstruction,vagus indigestion, diaphragmatic
72. Time of occurrence of milk fever is within 48 hours. hernia, tetanus, TRP, tumors, hypocalcemia, hydatid cyst, worm infestation(amphistomosis)
73. The rectal temperature in case of milk fever is 36-38ºC. 113. Treatment of choice for acid indigestion is 5% sodium Bicarbonate.
74. CPK and ALT liver enzyme levels are increased in downer cow syndrome. 114. Treatment of choice for milk fever is 25% calcium borogluconate.
75. Ketosis in cows mainly due to negative energy balance. 115. Treatment of choice for eclampsia in bitches is 10% calcium borogluconate.
76. Pregnancy toxaemia in sheep is ketosis and in cattle is called fatty liver syndrome. 116. Treatment for ovine ketosis is 5% dextrose
77. Lactation tetany in horse is due to hypocalcemia. 117. Treatment of choice in bovine ketosis is 50% dextrose (replacement therapy). As hormonal
78. Lactation tetany in cattle is due to hypomagnesemia. therapy it is dexamethasone.
79. Eclampsia in bitches is due to hypocalcemia and hypoglycaemia. 118. Treatment of hypomagnesaemia is with 25% calcium borogluconate and 5% magnesium
80. Nervous signs of ketosis due to production of isopropyl alcohol which is a breakdown product of hypophosphate.
acetoacetic acid in the rumen. 119. Milk fever is common in Jersey cows.
81. Blood glucose level in case of ketosis is 20-40mg/dl. 120. Ketosis is common in Guernsy cows.
82. The normal blood ketone body level is 50mg/dl. 121. Downer cow syndrome is common in Holstein Friesian.
83. Diagnostic choice for fatty liver syndrome is liver biopsy.
84. Time of occurrence of bovine ketosis is 60 days after parturition.
85. Time of occurrence of post parturient haemoglobinuria is 2-4 weeks after parturition.
86. Low milk fat syndrome due to a decreased formation of acetate in rumen is due to low fibre diets.
87. Milk is a poor source of copper and iron. It is a rich source of calcium and phosphorus.
88. Sulkowich test is used for the estimation of calcium in the urine during hypocalcemia.
89. Xylidill test used for estimation of magnesium in urine in hypomagnesemia.
90. Occurrence of post parturient haemoglobinuria is due to feeding of cruciferous plants.
91. Myoglobinuria and muscle degeneration are the pathological changes seen in the case of azoturia
in horses.
92. The normal ratio of Calcium and Magnesium in blood is 6:1.
93. Prepartum diet containing high amount of sulphur and chloride(anions) reduces the chances of
milk fever.
7 8
Dhruv N Desai Dhruv N Desai
Lectins or Castor bean Heat treatment
Ricin(haemagglutinins)

Saponin Lucerne Bloat in


ruminants
2.NUTRITION Soyabean
Poultry-
PROXIMATE COMPOSITION OF FEEDS (Weende analysis) depression of
growth
Moisture Drying at 100-105oC
Poly phenolic Sal seed meal Physical–soaking & working
Estimated indirectly by estimation of N2 and Multiplying
compounds
by 6.25 (i.e.100 units of protein contains 16 units of Sorghum Chemical – Poly ethylene
Total Protein Nitrogen) Tannins glycol (PEG),Polyvinyl
pyroldone (PVP)
Protein= N2 * 6.25
Alkali (NaOH)Formaldehyde,
Ether extract (fat) Extraction with petroleum ether in soxhlet apparatus
Methanol
Done with ether extracted sample.
Substances interfering with mineral utilization
Crude fiber Boiling with weak acid (HCl) & weak alkali (NaOH).
Phytic acid Soya bean, Forms Zn-phytate
Residue left represents CF.
complex.
Cotton seed
Ash Ignite at 500oC to 600oC in muffle furnace.
meal Ruminants can
NFE ( Nitrogen Free Extract ) 100 - Sum of other fractions. hydrolyse using
phytase enzyme
(Not estimated by analysis) calculated by difference
Oxalic acid Beet Forming insoluble
 None of the proximate principles is a chemical compound calcium oxalate
Spinach complex.Causes
Order of priority for nutrients hypocalcaemia
Water, Energy, protein, Minerals, Vitamins (Water is also a nutrient.)
Glucosinolates Brassica sp – Depress Synthesis
ANTI NUTRITIONAL FACTORS cabbage , of thyroid hormone.
turnips Cooking
Anti nutritional factor Source Remarks Methods of removal mustard seed Ruminants appear
to be less
Substances decreasing metabolic utilization of protein susceptible.
Protease inhibitors Seeds of Heat treatment Toxic to pigs &
legumes poultry
Trypsin inhibitors
(Kunitz inhibitors Soya bean
&Bowman Brick
inhibitors) Gossypol Cotton seed Forms complex Toxic effect can be
with Iron. overcome by
supplementing ferrous
Pigs & rabbits more sulphate
susceptible.

9 10
Dhruv N Desai Dhruv N Desai
Horses& Ruminants Moisture content of Haylage is 40-45%
more resistant. Moisture percentage of hay should not exceed 15%
Hay prepared from mixed crops of legumes and non-legumes is known as mixed hay.
Anti vitamins The best time for cutting a crop for hay making is when it is one third to a half in blossom.
The loss of nutrients in hay making occurs through bleaching, leaching and shattering.
Raw soya Lowering of vit A Heating in steam Which feed supplies both by pass protein and by pass fat? cotton seed meal.
Anti vitamin A beans carotene in Blood Feeding schedule of animal should be based on body weight
(Lipoxygenase) plasma Feed additive is a non – nutritive product that affect the feed utilization or performance of animal
Oyster shell and lime stones are used as grit in poultry feeds.
Anti vitamin D Isolated soya Autoclaving Antibiotics as feed additives is recommended only in pigs & poultry
protein Kernel is a dehulled seed.
A uniform mixture of one or more micro ingredients and a carrier used in the introduction of micro
Anti vitamin E Raw kidney Muscular dystrophy Autoclaving
ingredients into a larger mixture is known as pre-mix
bean  Alkaloid in legume which predispose bloat is Saponin
 In a digestibility trial, the causal faecal collection period for ruminants is about 10 days
Anti vitamin K Sweet clover Sweet clover
 The dry matter of plant origin according to Van-Soest method consists of cell wall contents and cell
(Dicoumarol) disease
contents
Anti Pyridoxine Linseed meal Autoclaving Rumen
(Linatine)  The urease activity of rumen bacteria converts urea to ammonia in rumen.
 The unsaturated fatty acids under go Biohydrogentaion reaction inside the rumen.
Anti Biotin Raw egg white Avidin binds with Heating
 The temperature of rumen varies from 38-42 oC with an average of 39oC
biotin.
 The main gases inside reticulo-rumen CO2, CH4 and N2 occurs in 65%, 25% and 7% respectively.
 Ruminal gas production 30 lit /hr after feeding of animals
 Ruminal gases CO2, CH4, H2.N2.
Cyanogens - Cyanogenics glycosides.  4.5 gms of CH4 Produced from 100 gms of carbohydrates
 Most of the rumen protozoa are ciliated
 Converted to prussic acid or hydrocyanic acid.  Ruminal fermentation by anaerobic bacteria and protozoa‟s
 Produce anoxia of the central nervous system.  Starch provides carbon skeleton for better utilization urea
 Ruminants are more susceptible especially cattle.  Methane production require 8% of gross energy intake
 Immature sorghum green fodder/ tapioca leaves feeding should be avoided.  Average number of micro-organisms in rumen liquor/ ml
 Rx- Cattle 3g sodium nitrate & 15g sodium thiosulaphate in 200ml H2O o Total bacterial count - 1 x 1010
Sheep 1g sodium nitrate & 2.5g sodium thiosulaphate in 50ml H2O o Protozoa - 1 x 106
o Oscilospira - 1 x 104
(Injected intravenously) o Yeast - 1 x 103
 Buytric acid converted to ketone bodies by the the ruminal epithelium.
 Urea toxicity results when the rumen ammonia level exceeds 80 mg/100ml.
 Under normal conditions the calf rumen becomes functional in about six to eight weeks
Fodder
 E.coli produces panthothenic acid
 The ensiling process requires 2-3 weeks for converting forage into silage. Water
 Chief acid of silage is Lactic acid
 The bulk of the water in extracellular and intracellular fluids which acts as solvent for inorganic and
 Flieg index is a commonly used method for evaluation of Silage quality
organic compounds is known as free water.
 Zero grazing or soiling is a method in which herbage is cut each day and brought to animals in
 The water available to the animal body by biochemical reaction is known as metabolic water.
containment.
 Water bound with proteins in colloidal system or water present inside cells as hydrated ions is bound
 The central fodder seed production farm is located at Hesserghata
water.
 Oat and Berseem are rabi crops
 Approximately loss of more than 10% of water may result in death
 Sorghum maize and soya bean are kharif crops
 Water content in the boby of new born calf is about 80%
 Molasses Brix is a term used to refer the amount of sugar content in molasses
 Molasses can be used upto10-15% in ruminant ration and 25% in poulty feed.
 Dry matter consumption in cattle‘s about 2-2.5 kg for every 100 by of live weight. Energy
 Domesticated ruminant with high dry matter consumption is goat (5-7%).
11 12
Dhruv N Desai Dhruv N Desai
 Which nutrient is considered first while formulating ration? Energy  Biological value of microbial protein is about 80
 1 kg of TDN is equivalent to 4400 Kcal of DE and 3520 Kcal of ME  The great demand of undegradable protein is in the diet of high yielders and young ruminants
 Bomb calorimeter is used to estimate Gross energy of feed. (rumen not developed.)
 Reference standard in a Bomb calorimeter is Benzoic acid  Methionine hydroxyl Analogue (MHA) is an amino acid analogue used in by pass protein
 Digestible energy is represented by portion of feed energy consumed which is not excreted in principle.
faeces.  Absorption of amino acid occurs mostly in proximal jejunum of SI.
 Net energy is the ideal method of expressing the nutritive energy of feed.  The amino acids which give rise to Acetyl Co-A and consequently the potential fatty acid producers
 Any surplus in the concentration of ATP favours formation of phosphocreatine, a major storage are called as ketogenic aa .
form in all domestic animals  Leucine is the only true ketogenic amino acid.
 The calorific value of glucose is 673 kcal  More than 80% of the urinay nitrogen is excreted in the form of urea
 In glycogenesis 2 ATP are used in incorporation of glucose into glycogen  The approx metabolic faecal nitrogen in ruminants is 5 gram
 Muscle glylogen serve as ATP or fuel reserve where as liver serve as glucose reserve.  Uric acid is a catabolite of purine.
 Calorific value of fat is 9.3 kcal/ gram  For the conversion of uric acid to allantoin uricase enzyme is required.
 The net gain of ATP from oxidation of mole of a palmitic acid is 130  Chief end product of purine metabolism in ruminants is Allantoin
 In glycogen molecule straight chain bonding is of alpha 1, 6 glucosidic bond and branching takes  The minimum nitrogen excretion from a animal on a protein free diet through faeces and urine is
place at alpha 1,6 glucosidic bond. known as MFN and EUN respectively.
 Gluconeogenesis is almost reverse of glycolsis  The excess body amino acids are disposed by transamination & oxidative deamination process.
 Gluconeogenesis differs form glycolysis by four enzymes.  Citrulline is an amino acid produced in the urea cycle
 Glucose is also known as Grape Sugar or Dextrose.  Protein does not store as a reserve like fat, CH2O
 D-Glucose is the sugar of the body.  Sequence of protein depletion Liver>kidney>heart>skeletal muscles
 Only naturally occurring ketohexose is fructose Fat
 Sweetest of all sugar is fructose.
 Cow milk contains about 4.5% lactose (milk sugar)  The referral temp at which lipid (fat) is a solid & lipid (oil) is a liquid is 25oC
 In terms of structure glycogen is similar to amylopectin  The diff in melting point of lipid reflects the degree of unsaturation of fatty acid constituents.
 Break down of glycogen is catalyzed by phosphorylase enzyme  In the body linoleic acid is converted to Arachidonic acid.
 Complete hydrolysis of cellulose yields only D-glucose while partial hydrolysis yields cellobiose.  Phosphatidyl choline is commonly known as lecithin
 In horse VFA is are absorbed through Caecum & colon  Phosphatidyl ethanolamine is commonly known as cephalin.
 Carbohydrate provides more than 50% of the energy value of the diet.  The simplest glycolipid is cerebroside and the complex one is ganglioside
 Ketosis and acidosis are the result of imbalances between input and output of energy in ruminant  Low density lipoproteins are the chief carries of cholesterol.
animals.  HMP shunt or pentose phosphate pathway is an important way to produce NADPH for fat synthesis.
 Ketosis is said to be developed if the ketone concentration of blood 50mg/100ml or more  Dietary fatty acids apperars in the lymph as chylomicrons .
 The branching enzyme of glycogen synthesis is Glulosyl 4,6 transferase and the de branching  In ruminants glucose cannot be converted in to fat as it lacks ATP citrate lyase and NADP malate
enzyme is Glucosidase dehydrogenase
 VFA having maximum absorption rate is Butyrate  Fatty acid synthase complex contains seven number of enzymes.
 Starch digestibility in rumen ranges from 63-70%.  Mitrochondrial elongation of fatty acid starts usually with palmitic acid
 Chief cellulose degrading bacteria of rumen is Fibrobacter succinogens  The catabolism of fatty acids to Co2 & H20 occurs by sequential combination of beta- oxidation
 Only VFA present in appreciable quantity in peripheral blood as an important energy source is cycle and TCA cycle
Acetate  Fatty liver syndrome due to feeding of cereal grains – deficiency of BIOTIN
 The products of CHO fermentation in ruminants supply energy and carbon skeleton for the synthesis  Nervous form ketosis is caused by Isoproponol
of amino acid for microbial protein synthesis  Biosynthesis of fatty acids pigs – adipose tissue Birds- liver, cattle, sheep – liver and adipose
Protein tissue
Minerals
 Rice bran must have 14% CP and less than 14% crude fiber
 Microbial yield of protein ranges from 90g- 230g for kg of organic matter digested.  Enzootic neonatal ataxia is caused due to the deficiency of copper in young animals.
 Iodinated casein has the same physiological effect as thyroxine.  The animal feed mostly contains iron as ferric (Fe+++) which is converted to ferrous by the acid
 DTP + CP = protein equivalent medium of stomach.
2  Deficiency of iron causes Microcytic hypochromic anaemia in pigs and chicks, but in calves it
causes microcytic normochromic anaemia.
 The conversion factor for converting milk‘s nitrogen to CP is 6.38 instead of 6.25  Enzootic marasmus is a deficiency disease of cobalt.
 Stutzer‟s reagent is used for the determination of true protein.  Phosphorus content of bran is 1.2-1.5%
 The net protein utilisation of bacterial protein is about 0.59  Transport form of copper Ceruloplasmin with α2 globulin in blood and plasma

13 14
Dhruv N Desai Dhruv N Desai
Vitamins c. Sheep
d. Cattle
 Vitamin D3 is cholecalciferol  Most promising initial symptomof Vitamin A deficiency in cows and horses?
Rhodopsin is also known as visual purple. a. Copius lacrymation
 If no green is fed to ruminants the concentrate mixture should have Vitamin A at the rate of 5000 b. Copius salivation
IU/Kg c. Xeropthalmiia
 Fatal syncope in calves and pigs is due to deficiency of Vitamin E d. Night blindness
Others  ……….% NDF in total ration is critical for maintenance of normal milk fat
a. 66%
 Growth rate of male and female calves is similar up to age of 3 months b. 18%
 In Camels, Llamas, Alpacas and Vicunas which are also ruminants but omasum is missing, so may c. 73%
be called as pseudo ruminants d. 36%
 Maximum permitted level of aflatoxin in animal feeds (as per prevention of food adulteration act is  Urea supplement is not recommended if CP content of ruminant diet is above ?
about 30 ppb (0.03 ppm) a. 18%
b. 25%
 N: S ratio of wool is 5 :1
c. 7%
 Richest source of Prostaglandins is seminal fluid of sheep
d. 13%
 Synthetic analogues of naturally occurring prostaglandins are called prostanoids. DAIRY SCIENCE
TRUE/FALSE

 Animals yielding as high as 10 liters of milk can be maintained solely on green fodders T/F COMPOSITION OF MILK
 Most of the rumen bacteria are non-spore forming gram positive anaerobes T/F
 Considering energy yield to cell and anaerobic glycolysis is the more efficient mechanism. T/ F Water Fat SNF Protein Lactose Ash
 Glycolysis may proceed in the presence or absence of oxygen -T /F.
 Glucose and sucrose have same calorific value but glucose less sweetest than sucrose. T/F Cow 86.6% 4.6 9.25 3.4 4.9 0.7
 Insulin secretion in the ruminant is stimulated by a rise in VFA concentration -T/F
 If blood glucose falls milk yield tends to fall in parallel- T/F Buffalo 84.2% 6.6 9.86 3.9 5.2 0.8
 The digestibility of protozoa protein is lower than the bacterial protein T/F
 In denaturation hydrolysis of peptide bonds of proteins occur T/F Goat 86.5% 4.5 7.75 3.5 4.7 0.8
 While formulating a ration single protein source is always recommended- T/ F(Only combination of
protein source is recommended.) Ewe 79.4% 8.6 11.39 6.6 4.3 1.0
 Each amino acid has its own characteristic isoelectric PH T/F
 Bile juice contains no enzymes T/F Sow 89.6% 4.8 5.86 1.3 3.4 0.9
 Chylomicrons do not enter the portal blood directly but enter body primarily through lymph system-
T/F Human 87.7% 3.6 8.82 1.8 6.8 0.1
 Chemical substances that increase bile secretion are called choleretics.-T/F
Ass 90.0% 1.3 8.44 1.7 6.5 0.5
 Body doesn‘t excrete iron T/F
 Alkali disease is due to the deficiency of selenium T/F

Questions
 Milk – clean lacteal secretion with SNF not less that 8.5% and fat not less than 3.5% after 72 hours
 Rumen degradable protein content is highest for ? of calving or free from colostrum.
a. Soybean meal  Water act as carrier for other constituents
b. Coconut cake FAT:
c. Groundnut cake
d. Fish meal  Fat is the most variable factor
 Which of the following is most important in inhibiting the digestibility of paddy straw ?  Size of fat globule become smaller and more in number as lactation in advance.
a. Lignin
 Larger size of fat leads to quicker rise to cream and easy to churn
b. Silica
 Small fat globules are best suited for cheese making since less fat is lost in whey.
c. Hemicellulose
d. Oxalate  Melting point of fat is 33 – 33.50C
 Concentration of Ammonia and Total VFA in rumen is highest for?  Milk fat is rich in vitamin A and D
a. Goat MILK PROTEIN:
b. Buffalo
15 16
Dhruv N Desai Dhruv N Desai
 Casein, α-lactalbumin, β-lactoglobulin  Boilingt point of milk - 100.170C to 1010C
 Rich in lysine and valine BACTERIA IN MILK:
 Casein constitutes 80% of total protein
 Casein exist as Ca-caseinate phosphate  Pscyrophillic: 3 - 20 0C eg: Pseudomonas fragi, P.flourescens
 α-lactalbumin is not coagulated by rennet and acids but by heat (Optimum is 7 0C)
 lactoferritin and lysozyme have bioprotective effect
 Mesophillic: 20 - 50 0C eg: Streptococcus cremoris, S.lactis
LACTOSE:
(Optimum is 37 0C)
 Whey is the rich source of lactose
 Thermophillic: Above 50 0C eg: Lactobacillus thermophillus,
 Lactose is least variable factor
Bacillus calidolactis
 Important factor for cheese, dahi, and buttermilk
MINERALS:  Lactose fermenters – Homofermentative - Strep.cremoris, S.lactis
Heterofermentative - Lactobacillus sp, Leuconostoc sp,
 Buffalo milk have more of Ca and less of inorganic P than cow milk
 In mastitis milk chloride : lactose ration is high  Proteolytic bacteria: B.subtilis, B.cremori, Pseudomonas putrifaciens
VITAMINS:  Lipolytic: P.fragi, P.flouresence
CHANGE IN COLOUR OF MILK:
 Good source of thiamine & riboflavin
 Poor source of Vit C & D  Blue – Pseudomonas syncyanea (Bluish Gray), S.lactis (Dark Blue)
PIGMENTS:  Yellow – P synxanthia, Flavobacterium
 Red - Serratia macescans, Brevibacterium erythrogenes, Micrococcus rosenes
 Fat soluble - carotene & xanthophylls
 Brown - P.putreafaciens, P.flourescens
 Water soluble - riboflavin
ADULTERATION:
FLAVOUR:
Water up to 3% allowed
 Growth of bacteria – fruity, malty ,acid
Presence of neutralizers is identified by Rosalic acid test
 Lipase – rancid
In Rosalic acid test appearance of pink color - NaOH, KOH,CaOH
 Processed milk – cooked flavour due to sulphydrl group
In Rosalic acid test appearance of rose red color – sodium carbonate or sodium bi carbonate
 Oxidation – cardboard flavour
In Rosalic acid test appearance of brown color – Absence of any neutralizers
 Dried milk – tallowy
Presence of starch is identified by Iodine test. Blue color indicates positive.
 Other products – metallic or paint
Gelatin is identified by Picric acid test. Yellow precipitate is positive.
PHYSICAL PROPERTIES OF MILK:
Cane sugar is identified by Resorcinol test. Red color is positive
 PH -6.5-6.7 Glucose is identified by Barfoed test. Red precipitate is positive.
 Alkaline PH - Mastitis milk Buffalo milk in cow‟s mild is identified by Hansa test.
 Acidic PH - Added colostrums or bacterial deterioration Skin milk powder is identified by Nitric acid test. Orange color is positive. Yellow color is normal
 Natural acidity is due to casein, acid phosphatase and citrate milk.
 Developed acidity is due to lactose  Vanaspathi in milk is identified by Baudoin test. This is because vanaspathi contains sesame oil.
SPECIFIC GRAVITY:  Delvotest kit detects Anti biotics and sulpha residues.
PASTEURIZATION :
 Specific gravity of Cow milk is 1.028-1.032
 Specific gravity of Buffalo milk is 1.030-1.034  Holding and continuous
 Determined by lactometer at 15.60C  LTLT - 63 0C for 30 min
 More fat content leads to low SG and vice versa  HTST – 72 0C for 15 sec
 Addition of water decreases SG  UHT – 135 to 150 0C for fraction of seconds / with no hold.
 Addition of solids increases SG  Glycol is used as coolant in HTST.
FREEZING POINT: THERMISATION:

 Cow milk = -0.512 to -0.572  Heating below pasteurization temperature to temporally inhibit bacterial growth.
 Buffalo milk = -0.521 to -0.575  63 to 65 0C for 15 sec
 Determined by Cryoscopy ULTRA PASTEURIZATION :
BOILING POINT:
 115- 130 0C for 2 to 4 seconds and cooling below 4 0C.
17 18
Dhruv N Desai Dhruv N Desai
 Extend milk to 15 to 30 days  Potential source of energy - 9.3 cal/g
HOMO GENISATION :  Melting point 28 to 44 0C specific gravity is 0.93 to 0.94
 Antioxidants like ethyl gallate and hydroquinine is used
 Reducing fat globule size to 1 micron or less. CHEESE:
 Disintegration of fat globule is achieved by turbulence and cavitations
 Raw milk upon homogenisation before pasteurization results in rancidity due to activation of lipase Water Fat Protein Ash
activity.
Cheddar 36.8 33.8 23.7 5.6

Cottage 69.8 1.0 23.3 1.9

 By coagulating casein
 Hard cheese not more than 43% moisture not less than 42 % fat
CREAM:  Freezing done at – 4.5 C for fresh cheese and -14.5 C for one year old cheese
 Cottage cheese is prepared from pasteurized skim milk
Water Fat Protein Lactose Ash Total solid SNF
 Mozzarella cheese is best suited for pizza making
68.2% 25% 2.54% 3.71% 0.56% 31.8% 6.8% ICE CREAM:

Fat SNF Sugar Total solid


 Not less than 25% fat.
 Pasteurization temperature for cream by LTLT is 71 0C for 20 mts Economy 10-12 10-11 13-15 35-37
 Pasteurization temperature for cream by HTST is 95 -100 0C for 15–16 sec
 Cream separator works by strokes law (centrifugal force). Good ice cream 12-14 8-9 13-16 37-39
 If fat screw IN – More fat in cream
 If skim milk screw OUT – more fat in cream Deluxe 16-20 5-9 13-17 40-41
Types of cream:
 Not less than 10% fat
 Table Cream, Light Cream, Coffee Cream – 20 – 25 % milk fat  Rapid freezing of pasteurized milk with agitation to in corporate air
 Whipping Cream, Heavy Cream – 30 – 40 % milk fat  Emulsifiers not more than 0.5 %
 Plastic Cream - 65 -80 % milk fat  Over run in ice cream not exceeding 80 %
BUTTER :  Phosphatase test negative
Moisture 16.2%

Fat 80.2%
MILK POWDER:
Curd 1.1%
 Drying under low temperature
Salt 2.5%  Moisture less than 5 %
 Fat not less than 26%
 Not less than 80% fat not more than 16% moisture and not more than 3 % salt
Whole milk powder Skim milk powder
 Flavouring agent for butter is Diacetyl. It must not exceed 4 ppm.
 Cream for butter making should contain 30 – 40 % fat and pasteurized at 630C for 1 hour or 88 C for Moisture 3.5% 3.5%
10 mins
 Churning is done at 10 0C not exceeding 30 – 40 min, lose in butter milk should not exceed fat of Fat 27.5% 0.8%(not more than 1.5%)
0.2 %
 Sweat cream butter have acidity with in limit of 0.2 % if it exceeds 0.2 % then it is sour cream butter. Protein 26.4% 35.4%
 Neutralizers for butter is soda (sodium carbonate and sodium bicarbonate) and lime (CaOH, CaO)
Lactose 37.7% 52.3%

GHEE: Ash 5.9% 8.0%

 Clarified milk fat


 99% or more of fat, moisture not more than 0.5%
19 20
Dhruv N Desai Dhruv N Desai
Standardized Toned milk Double Skin milk Recombined Milk -0.525 to -0.5650C
milk toned milk milk
Evaporated milk -1.30C
Fat 4.5 3.0 1.5 0.5 3.0
Condensed milk -14. 90C
SNF 8.5 8.5 9.0 8.7 8.5
Cheddar cheese -4.50C

Meat -1 to -1.50C
PASTEURIZATION TEMPERATURE

Batch method Continuous method


PFA STANDERDS
Milk 63 0C for 30 min 72 0C for 15 sec
Ice cream 68 0C for 30 min 80 0C for 25 sec PRODUCT MORE THAN(>) LESS THAN(<) STORAGE TEMP
Cream 71 0C for 20 min 95 – 100 0C for 20 sec
Cream 25% Fat 5-10 0C

Butter 80% Fat 1.5% Curd -23 to -280C


3% Salt

Ice cream 10% Fat 3.5% protein -23 to -280C


36% Total solids
0.5% Stabilizers&
Emulsifiers

IMPORTANT TEMPERATURES Hard cheese 42% Fat 43% Moisture Cold curing(0-40C)

PROCESS TEMPERATURE Warm curing(10-160C)

Crystallization of condensed milk 35-400C Whole milk powder 26% Fat 5% Moisture 240C
1.2% Acidity
Bactofugation 55-600C
Skim milk powder 5%Moisture 240C
Homogenization 60-650C
1.5%Fat
LTLT 630C(15 mts)
1.5%Acidity
Thermization 680C(15 sec)
Unsweetened 8% Fat 5-160C
HTST 0
72 C(15 sec) condensed
milk(Evoporated 26% Milk solids
0 milk)
Stassanization 74 C (7 sec)

Clarification of ghee 1100C Sweetened 9% Fat 100C


condensed milk
31% Milk solids
Pilot sterilization 1170C(15 mts)
40% Cane sugar
Ultra pasteurization 1300C(2-4 sec)
Ghee 99% Fat 0.5% Moisture 210C
UHT milk 135-1500C

Freezing points
ANESTHESIOLOGY
21 22
Dhruv N Desai Dhruv N Desai
A. Succinyl Choline B. Atracurium C. Pancuronium D. Gallamine
1. The action of opiates can be reversed with
16. IPPV is not required during Thoracotomy in which species
A. Butarphanol B. Naloxone C. Yohimbine D. Glycopyrrolate
A. Bovine B. Equine C. Feline D. Canine
2. Which of the following is Neuroleptanalgesic?
17. Triple mixture for anesthesia in Ruminants contains
A. Ketamine & Xylazine B. Fentanyl & Morphine
A. Xylazine, Ketamine & Guaifenesin B. Xylazine, Ketamine & Atropine
C. Fentanyl & Droperidol D. Oxymorphone & Atropine
C. Ketamine, Diazepam & Guaifenesin D. Xylazine, Diazepam & Atropine
3. The following is ultra short acting barbiturates
18. The pleural pressure of an animal required for initiation of inspiration is
A. Phenobarbitone B. Pentobarbital C. Methohexital D. Oxybarbiturate
A. – 5 cm of H 0 B. – 4 cm of H 0 C. – 3 cm of H 0 D. – 2 cm of H 0
4. The surgical plane of anesthesia is generally considered to be 2 2 2 2

A. Stage II B. Stage III Plane I 19. The local analgesic may affect
C. Stage III Plane II D. Stage III Plane III A. Sensory Neuron B. Motor Neuron C. Both A & B D. None of the above
5. The term Atelectasis refers to 20. Which of the following is/are dissociative anesthetic?
A. Increased fluid in alveoli B. Hyperinflation of alveoli i. Tiletamine ii. Profopol iii. Thipental iv. Ketamine.
C. Collapsing of alveoli D. A decrease in blood perfusion around alveoli A. iv only B. ii, iii & iv C. i & iv D. i, ii & iv
6. The barbiturate used as anticonvulsant is 21. In controlled ventilation the inspiration expiration time ratio should be maintained at
A. Phenobarbitone B. Pentobarbital C. Methohexital D. Thiamylal A.1:2- 1-3 B. 1:1 C. 2:3 D. 1:4
7. ‗Second Gas effect‘ is seen in anesthesia using 22. Local anesthetic will have direct effect on the
A. Ether B. Nitrous Oxide C. Halothane D. Isoflurane A. Peripheral nervous system B. Central nervous system
8. Which of the following can be delivered using the same precision Vaporizer? C. Both A & B D. None of the above
A. Ether & Methylflurane B. Nitrous Oxide & Halothane 23. The colour of oxygen cylinder is
C. Halothane & Isoflurane D. Isoflurane & Seviflurane A. Blue B. White C. Yellow D. Black
9. The dose rate of atropine used as preanaesthetic is 24. The centrally acting muscle relaxant is
A. 0.1-0.2 mg/kg B. 0.01-0.02 mg/kg C 0.2-0.4 mg/kg D. 0.02-0.04 mg/kg A. Succinyl Choline B. Guaifenesin C. Tubocurium D. Diazepam
10. The site for epidural anesthesia in horses is 25. The paravertebral block in cattle aims which of the following spinal nerves
A. Lumbo-Sacral B. Sacro-Coccygeal C. InterCoccygeal1-2 D. Thoraco-Lumbar A.T12,T13, L1 B. T13, L1, L2 C. L1,L2,L3 D. None of the above.
11. Local anesthetic agents works well when applied
i. Topically on epidermis ii. Topically on Cornea iii. Topically on Mucous Membrane 1 B 6 A 11 B 16 A 21 A
iv. By injection. 2 C 7 B 12 D 17 A 22 A
3 C 8 C 13 C 18 A 23 B
A. Above all B. ii, iii & iv only C. i & iii only D. iv only 4 C 9 D 14 B 19 C 24 B
12. IVRRA is used in which of the following surgical procedure 5 C 10 C 15 A 20 C 25 B
A. Amputation of Horn B. Evisceration C. Laprotomy D. Distal limb surgery
13. The effect of Local anesthetic can be prolonged by addition of
A. Ether B. Hyaluronidase C. Epinephrine D. Dexamethasone
14. Amputation of horn requires blocking of cornual branch of both lacrimal and infra trochlear nerve b in
which species
A. Sheep B. Goat C. Ox D. Buffalo
NATIONAL INSTITUTES
15. The depolarizing muscle relaxant is
23 24
Dhruv N Desai Dhruv N Desai
Central institute for research on Buffaloes – Hissar, Hariyana PPR Goat plaque
Pseudo rinder pest
National Equine research centre – Hissar, Hariyana Goat catarrhal fever
National bureau of animal Genetic resources – karnal, Hariyana (NDRI) Canine distemper Hard pad disease
Canine influenza
Central Sheep & Wool research institute – Avikanagar, Rajasthan Carre‘s disease
Orthomyxovirida Type A influenza Avian influenza Fowl plaque
National Camel research institute – Bikaner, Rajasthan e virus
Swine influenza
Central institute for reseach on Goats – Mukdoom, UP
Equine influenza Equine distemper
Indian Grassland & forage research institute – Jansi, UP Pink eye
Stable pneumonia
Central Avian research institute – Izatnagar, Utranchal Rhabdoviridae Lyssa virus Rabies Mad dog disease
Hydrophobia (in man )
Vector control research centre – Pondycherry Lyssa.
Institute of Cytology & Preventive Oncology – New Delhi Ephemero virus Ephimeral fever 3 Day sickness
Dragon boat disease
National research center on Yak – Dirang,Arunachal pradesh Bovine epizootic fever
Rhabdo virus Vesicular stomatitis Pseudo FMD
Sore mouth
Sore nose
Positive ( + ) sense RNA viruses

Order – Nidovirales (nested viruses)


Coronoviridae Coronovirus Infectious bronchitis(IB)
Transmissible gastero
enteritis(TGE)
Flaviviridae Pestivirus Classical swine fever Hog cholera
Bovine viral diarrhea Mucosal disease
Border disease Hairy shaker‘s disease
Equine viral arteritis
Picornaviridae Aphthovirus FMD Aphthus fever
Enterovirus Duck viral hepatitis I
Avian encephalomyelitis Epidemic tremor
New England Disease
Asteroviridae Asterovirus Duck viral hepatitis II
Togoviridae Alphavirus Equine Blind staggers
encephalomyelitis
Ovine encephalomyelitis Louping ill
Double Stranded RNA viruses
Reoviridae Orbivirus Blue tongue Muzzle disease
VIROLOGY
Pseudo FMD
FAMILY GENUS DISEASE SYNONYMS Sore mouth
Negative ( - ) sense RNA viruses Sore muzzle
African horse sickness Horse plaque
Order – Mono Nega Virales Rota viral diarrhea
Paramyxoviridae Avula virus New castle disease Avian distemper Birnaviridae Avibirna virus IBD Gumboro disease
Avian pest Infectious nephrosis
Black eye Reverse transcribing RNA viruses
Doyles disease
Morbili virus Rinder pest Cattle plaque Retroviridae Lentivirus Equine infectious anemia Swamp fever
Bovine typhus ( EIA )

25 26
Dhruv N Desai Dhruv N Desai
Maedi / Visna Staphylococcus S.aureus Tick pyemia – lambs
Lymphoid leucosis Big liver disease Botryiomycosis/scirrous cord - horse
Avian sarcoma Bumble foot – poultry
Ambisense RNA virus Ritters disease/expoliative skin
Bunyaviridae Rift valley fever disease – children
Nairobi sheep disease Toxic shock syndrome
Double Stranded DNA viruses S.intermedius Canine pyoderma
S.Hyicus Greasy pig disease/expoliative
Herpesviridae Varicellovirus Pseudo rabies Mad itch epidermitis
Aujeskey‘s disease Streptococcus S.pyogenes Scarlet fever/septic sore throat
Infectious bulbar paralysis S.Equi Strangles/infectious adenitis
Mardivirus Marek‘s disease Fowl paralysis S.dysagalactiae Acute mastitis
Range paralysis S.agalactiae Chronic mastitis
Grey eye S.uberis Dry cow mastitis
Pearl eye Corynebacterium C.diphtheria diphtheria
Gallid herpes I ILT Avian diphtheria,
Hemorrhagic tracheitis C.pseudotuberculosis Ulcerative lymphangitis
Bovine herpes virus IBRT Red nose Caseous lymphadenitis
I Infectious pustular C.renale Ulcerative balanoposthitis/pizzle rot
vulvovaginitis Bovine pyelonephritis
Necrotic rhinitis C.equi Suppurative bronchopneumonia
Bovine herpes virus Malignant catarrhal fever Gangrenous coryza (Rhodococcus equi)
III C.pyogenes(aracanobacteri Summer mastitis
Adenoviridae Adenovirus Egg drop syndrome um pyogenes)
Mast adenovirus Infectious canine Rubarth‘s disease Actinomycosis A.bovis Lumpy jaw
hepatitis ( ICH ) Blue eye A.viscosis Canine actinomycosis
Poxviridae Parapoxvirus Orf Contagious ecthema Actinobaculum A.suis Porcine pyelonephritis
Scabby mouth Nocardia N.farcinica Bovine farcy
Vaccinia virus Cow pox A.asteroides Canine nocardiosis
Capripox virus Lumpy skin disease Pseudo urticaria Dermatophilus D.congolensis Strawberry foot rot/lumpy
Double Stranded linear DNA virus wool/cutaneous streptothricosis
Irido virus African swine fever Wart hog disease Listera L.monocytogenes Circling disease/silage
Single Stranded DNA virus disease/meningo encephalitis
Para poxviridae Parvovirus Canine parvo virus Erysipelothrix E.rhusiopathiae Diamond skin disease/vegetative
Chicken anemia virus endocarditis
Feline panleukopenia Fading kitten syndrome Bacillus B.anthracis Wool sorters
Feline distemper disease/charbon/spleenic
fever/malignant pustule
Clostridium C.tetani Lock jaw/wooden horse/saw horse
stance
C.botulinum Lamsiekte/bulbar paralysis/loin
disease
Duck sickness
Limber neck - poultry
Shaker foal syndrome
C.chauvoei Black quarter/symptomatic anthrax
C.septicum Malignant edema – cattles
Braxy/Bradsot – sheep
Para anthrax - pigs
BACTRIOLOGY
C.novyi - Type A Big head
ORGANISM GENUS DISEASE Type B Black disease/infectious necrotic
hepatitis
27 28
Dhruv N Desai Dhruv N Desai
C.hemolyticum Bacillary hemoglobinuria B.burgdorferi Lyme disease
C.perfringens Type- A Gas gangrene Bortodella B.avium Turkey coryza
Type – B Lamb dysentery B.parapertusus Wooping cough – children
Type – C Struck B.bronchiceptica Kennal cough/atropic rhinitis
Type – D Pulpy kidney disease/over eating Morexella M.bovis Infectious kerato conjunctivitis/Pink
disease/enero toxemia eye/new forest disease
Type - E Hemorrhagic enteritis Fusobacterium F.necrophorum Bull nose – pigs
C.piliformae Tyzzer disease - mice Thrush – horse
C.colinum Quail disease Bush foot – pigs
Mycobacterium M. Tuberculosis Pearls disease/king of disease/white Black pox/black spot
disease Bacterioides B.nodosus Foot rot - sheep
M.paratuberculosis Crohn‘s disease/johne‘s Mycoplasma M.mycoides CBPP
disease/chronic bacillary dysentery CCPP
Escherichia E .coli Calf scour/white scour/neonatal M.agalactiae Contagious agalactiae
diarrhoea - calves M.gallicepticum Chronic respiratory disease
Honeymoon cystitis –calves Infectious sinusitis - turkey
Watery mouth – lambs M.hypopneumonia Enzootic pneumonia/endemic
Edema disease – pigs pneumonia - pigs
Coli granuloma/hjarres disease M.meleagridis Air sacculitis - turkey
Mushy chick disease/yolk sac
infection
Salmonella S.pullorum Bacillary white diarrhoea
S.gallinarum Fowl typhoid GOLD STANDERD TEST
S.typhimurium Paratyphoid
S.arizonae Para colon infection Glanders Compliment fixation test(CFT)
S.anayum Keel disease Leptospira Microscopic agglutination test
Klebsiella K.pneumoniae Atrophic rhinitis - pigs Rinder pest Virus neutralization test
Shigella S.marcescens Human bacillary dysentery Rinder pest Compliment fixation test(CFT)
Yersinia Y.pestis Black death/bubonic plaque ( confirmatory test )
Y.pseudo tuberculosis Pseudotuberculosis PPR Virus neutralization test
Pseudomonas P.aeruginosa Fleece rot- sheep Influenza Haemagglutination inhibition test (HI)
Burkholderia B.mallei Glanders/farcy Rabies Fluorescent antibody test(FAT),
B.pseudomallei Melioidosis Fluorescent antibody virus neutralization test (FAVN)
Actinobacillus A.ligneresii Wooden tongue/timber tongue Swine fever FAVN
A.suis Pleuropneumonia – pigs FMD Competitive ELISA
A.equuli Sleepy foal disease Blue tongue Competitive ELISA, AGID, PCR
Pasturella P.multocida Type - A Shipping fever– cattle African horse sickness Indirect ELISA, CFT
Fowl cholera – poultry IBD Quantitative AGID
Snuffles - rabbits Equine infectious anemia Coggins test
Type - B Hemorrhagic septisemia/stockyardis
disease/barbone
Type - D Atropic rhinitis
P.hemolytica Gangrenous mastitis - cattle DON‟T CONFUSE
Haemophilus H.parasuis Glassers disease
H.paragallinarum Infectious coryza Fowl coryza Hemophilsalis gallinarum
H.somnus Sleeping syndrome Turkey coryza Bartodella avium
Brucella B.abortus Contagious abortion/enzootic Gangrenous coryza Malignant catarrhal fever
abortion Nutritional coryza Vitamin A deficiency
Campylobacter C.jejuni Winter dysentery
C.coli Swine dysentery Mad dog disease Rabies
Leptospira L.icterohemorrhagia Weils diseae – human Mad cow disease Bovine spongiform encephalopathy
Borrelia B.anserina Avian spirochetosis
29 30
Dhruv N Desai Dhruv N Desai
Kennel sickness Salmonella ATP sparer Creatine phosphate
Kennel cough Bartodella bronchiceptica Fat sparer Insulin
Protein sparer Growth hormone
Sweet itch/Queensland itch Culicoides
Dairy man itch Sarcoptic mange Deadly night shade Atropa belladona
Golden/black night shade Solanum nigrum
Bush foot Fusiformis necrophorus Woody night shade Solanum dulcamera
Bush disease Chlamydia psittaci
Bush sickness Cobalt deficiency

Liver rot Fasciola


Fleece rot Pseudomonas
Foot rot(sheep) Bacterioides nodosus
Foot rot (cattle ) Fusiformis necrophorus
Wool rot Dermatophilus congolensis

Shipping fever (Cattle) Pasturella


Shipping fever (Horse) Equine influenza

Thrush (horse) Fusobacterium


Thrush (poultry) Candidiasis TOTAL CONTROL

BRAIN PART FUNCTIONS


Pink eye (cattle) Moraxella bovis
Cerebrum Memory, initiative, volition, intelligence
Pink eye (horse ) Equine influenza
Sensory impulses like sight, smell, taste, etc.,
Blue eye Infectious canine hepatitis
Fear, anger, emotion
Grey eye/pearl eye Marek‘s disease
Voluntary control of skeletal muscle
Black eye New castle disease
Cerebellum Unconscious control
Balance
Co-ordination
Atrophic rhinitis Pasturella ,Bartonella Hypothalamus Hunger
Necrotic rhinitis IBRT Thirst
Body temperature
Enzootic abortion (cattle) Brucella Sleep
Enzootic abortion(ovines) Chlamydia Thalamus Relay center between sensory nerves& cerebral cortex
Epizootic abortion(cattle ) Campylobacter Basal ganglia Control muscular activity
Medulla oblongata Involuntary reflex actions like, respiration, coughing,
Infectious jaundice Leptospira vomiting,
Malignant jaundice Clostridium novyi type - B Salivary secretion
Heart beat rate
Red water disease Clostridium hemoglobinuria Reflex part of swallowing
Red water fever Babesia Amygdala Social ranking
Limbic system Aggressive behavior
Spleenic fever Anthrax
Malta fever/undulent fever Brucella
Q fever Coxiella burnetti

Black tongue Niacin deficiency


Red tongue Biotin deficiency

Vitamin A sparer Vitamin E


Vitamin E sparer Selenium
31 32
Dhruv N Desai Dhruv N Desai
Cat plaque Feline panleukopenia
Rabbit plaque Tularensis
Duck plaque Duck viral enteritis
Bubonic plaque Yersinia pestis
Lung plaque CBPP
FATHERS White plaque Mycobacterium tuberculosis
BLACKS
Veterinary science Salihotra Black quarter Clostridium chauvoei
Anatomy Cladius galon Black leg Clostridium chauvoei
Bacteriology Robert koch Black head Histomonas meleagridis
Cellular & modern pathology Rudolph virchow Black disease Clostridium novyi - Type B
Chemotherapy Paul ehrlich Black flies Simulium indicum
Experimental pathology John hunter Black death Yersinia pestis
Immunology Edwerd jenner Black tongue Niacin deficiency
Microbiology Louis pasteur PSEUDOS
Medicine Hippocrates Pseudo FMD Blue tongue
Vet medicine Ranatus vegetius Pseudo cow pox Milkers nodule
Vet pharmacology Rudolph bucheu Pseudo rinder pest PPR
Parasitology Fraucisco redi Pseudo tuberculosis Yersinia pseudotuberculosis
Surgery Sasruta Pseudo rabies Aujeskeys disease/infectious bulbar paralysis
Modern toxicology M.J.B.Orfila Pseudo fowl plaque New castle disease
Vet protozoology leukart Pseudo glanders Equine epizootic lymphangitis
Vet radiology Richerd BODIES
Sociology August comte Bollinger bodies Fowl pox
Economics Adam smith Joset bodies Borna disese
Nutrition Lavoisier Dohle‘s bodies Aggregates of ribosomes in neutrophils
Modern embryology Karl ernst van baer Koch blue bodies Theileriosis
Meat inspection Ostertag Councilman bodies Yellow fever in man
Animal breeding Robert bakewell Negri bodies Rabies
Marketing Philip kotler Guarnieri bodies Cow pox

ORIGINS

ECTODERM ENDODERM MESODERM


Nervous system Skeletal system Digestive system
Pituitary gland Muscular system Respiratory system
Salivary gland Urinary system Liver
Sweat gland Genital system Pancreas
Mammary gland Blood vessels
Stomodium (mouth) Spleen
Enamel kidney
Skin - Epidermis Skin – Dermis & Corium
Sense organs(Ear&Eye) Middle ear
External ear
Internal ear
Nasal cavity
PLAQUES Anus
Cattle plaque Rinder pest Tongue & palate
Equine plaque African horse sickness Nail, hoof, horn &hair
Goat plaque PPR
Fowl plaque Avian influenza
33 34
Dhruv N Desai Dhruv N Desai
OIE RECOMMENDED QUARANTINE DAYS (ICAR BOOK)

DISEASE DAYS
Cattle
Rinder pest 21 days
HS & Theileriosis 28 days
Infectious bovine rhinotracheitis 30 days
Tuberculosis 90 days
Anaplasmosis 100 days
CBPP 180 days
Sheep & Goat
pox 21 days
Brucellosis 30 days
ANTIBIOTICS MICRO-ORGANISM Blue tongue 40 days
Bacillus colistinus Colistin(polymyxin E) CCPP 180 days
Bacillus polymyxa Polymixin B Swine
Bacillus subtilis(B.lichniformis) Bacitracin TGE 28 days
Chromobacterium violaceum Monobactam Aujesky‘s disease 30 days
Micromonospora purpuria Gentamicin Swine fever 40 days
Penicillium notatum Penicillin G Equine
Penicillin griseofulvin Griseofulvin Glanders 28 days
Streptomyces cattleya Carbapenam Dourine 28 days
Streptomyces erythreus Erythromycin Equine influenza 28 days
Streptomyces fradiae Neomycin/tylosin Contagious equine metritis 30 days
Streptomyces griseus Streptomycin Birds
Streptomyces kanamyceticus Kanamycin Fowl cholera 14 days
Streptomyces lincolnensis Lincomycin ND & fowl plaque 21 days
Streptomyces medeterranei Rifamycin Fowl typhoid 28 days
Streptomyces nodosus Amphotericin B Infectious bronchitis 28 days
Streptomyces orientalis Vancomycin Aspergillosis 45 days
Streptomyces venezuelae Chloramphenical Canines
Streptomyces spectabilis Spectinomycin Rabies 4 months
Streptomyces tenebrans Apramycin
Streptomyces rimosus Oxytetracycline
Streptomyces aureofaciens Chlortetracycline

PRESERVATIVE FOR VETEROLEGAL SPECIMANS

SPECIMAN PURPOSE PRESERVATIVE


Faeces Helminthic eggs 4-10% formalin
Coccidial oocyst 2.5% potassium dichromate
Pasitological Ticks & mites 70% alcohol
35 36
Dhruv N Desai Dhruv N Desai
specimans Bile pigment in urine Pouchet test
Insects, fleas, lice 70% alcohol or 50% formalin Gmelins test
Urine Chemical analysis Toluene or 40% formalin Bile salt in urine Hay‘s test
Ammonia, Creatine Thymol(0.1 gm /100ml of urine)
Calcium& phosphorous Conc Hcl Casoni‘s test Hydatidosis
ketosteroids Chloroform Tricin test Trichomoniasis
Cytological studies 40% ethanol Mucous agglutination test Trichomoniasis
Hormones No preservatives Capillary agglutination test Anaplasma & Babesia
Bacteriological No preservatives Sabin & Feldman dye test Toxoplasma
examination Farmal gel test Trypanasomiasis
Blood Stilbamidine test
Blood smears Bacteria Heat fixation Mercuric chloride test
Protozoa & DLC Methyl alcohol & absolute alcohol Coggins test Equine infectious anemia
Blood samples Bacterial culture No preservative Cuboni‘s test Pregnancy diagnosis mare(estrogen in urine)
Virus isolation Buffered glycerine Ascheim zondek test(AZ Test) Pregnancy diagnosis mare(PMSG in serum)
Biochemical Blood urea Potassium oxalate Malachite green test Test for imperfect bleeding of meat
analysis Hotis test Streptococcus agalactiae
Blood sugar Sodium fluoride or potassium oxalate Dick test Streptococcus pyogens
Calcium Heparinized blood Bacitracin test Streptococcus pyogens
Ketones Oxalate or sodium fluoride Elek‘s test Corynebacterium
Pyruvate Citrate /10% Trichloro acetic acid/3% Antons test Listeria
perchloric acid Ascoli‘s test Anthrax
Serum Serological test Phenol/methiolate/unpreserved String of pearls test Anthrax
CSF Electrolyte EDTA Stormont test Tuberculosis
Glucose Sodium fluoride Spoligo test Paratuberculosis
Milk Bacteriological Unpreserved in ice Eijkman test E.coli
TB organism 0.1% Boric acid Ames test Salmonella
Biopsy samples 10% formalin Card test Brucella
Tissues Bacteriological studies Unpreserved in ice Rivanol/Mercaptoethanol test
Viral studies 5% glycerine Abortus bang ring test
Histopathology 10% formalin Rose Bengal test
Synovial fluid EDTA/sodium citrate Kanagawa test Vibriosis
Coomb‘s test Brucellosis &Auto hemolytic anemia
TEST FOR YOU Halothane test Porcine stress syndrome
COFAL test Lymphoid leucosis
Mcfadean reaction Bacillus anthracis
Milk fever Sulkowitch test(Ca in urine) Nagler reaction Clostridium perfringens
Hypomagnesemia Xylidil test(Mg in urine) Stormy clot reaction Clostridium perfringens
Simple indigestion Sedimentation activity test Weilfelix reaction Ricketsia
Ketosis Rothra‘s test(ketone bodies in urine) Quelling/swelling reaction Klebsiella
Ross test/Cow side test(ketone bodies in milk) Caslicks operation Pneumovagina in horse
LDA Liptak test Whipples operation Vaginal prolapse in dog
Mules operation For prevention of blow fly strike
TRP Pole or Bamboo test
Hobday operation Roaring in horse
Cyanide poisoning Picrate test ADULTERATION OF MILK
Nitrate poisoning Diphenylamine blue test Storch test/Guaicol test Heated milk with fresh milk
Starch iodine test Freezing point depression Water in milk
Hematuria Benzedine test(RBC in urine) Rozalic acid test Alkali neutralizers in milk
Glycosuria Benedicts test(Glucose in urine) Iodine test Starch
Proteinuria Hellers test Picric acid test Gelatin
Biuret test Barfoed‘s test Glucose
37 38
Dhruv N Desai Dhruv N Desai
Hansa test Buffalo milk in cow milk Fatty liver kidney syndrome Avidin- Antri nutritional factor
Nitric acid test Skim milk powder in milk Choline Perosis /slipped tendon
Baudoin test Sesame oil in ghee
Pytosterol acetate test Vegetable oil in ghee
Phosphorous test Pasteurization efficiency
MINERALS

MINERAL DEFICIENCY REMARKS


Calcium Rickets - young Bood level = 9-12 mg/dl
VITAMINS Osteomalacia - adults
Parturient paresis - cattle
VITAMIN DEFICIENCY REMARKS Lactation tetany - horse
Vitamin A (Retinol) Night blindness Anti infectious vitamin Eclampsia - dog
Xerophthalmia Rods –dim light-rhodopsin Soft shelled eggs - poultry
Nutritional roup - chicken Cons – bright light - iodopsin Phosphorus Pica or Allotriophagy Blood level = 4-12 mg/dl
Lamsiekte (lame sickness) Pica predisposes botulism.
Vitamin D(calciferol) Rickets – young Anti rachitic factor
Potassium Muscle weakness Intra cellular ion
Osteomalacia –adults Calcifediol –circulatory form
Sodium Corneal keratinization Extra cellular ion
Rickety rosary -chickens Calcitriol –active form
Cannibalism &feather pecking - Birds
VitaminE(tocopherols) Crazy chick Anti sterility factor
Sodium chloride Heat exhaustion In the absence of aldosterone,
disease/encephalomalacia- Vitamin A sparer
(salt) Dehydration Nacl cannot be reabsorbed by
chicks Potent antioxident
kidney – Addison‘s disease.
Exudative diathesis-chicks
Nutritional muscular dystrophy
– chicks Sulphur For efficient utilization of urea a Sulphur containing
Stiff lamb disease – lamb nitrogen : sulphur ratio of 10:1 is Amino acid– Cystine,
White muscle disease-calf suggested. Cysteine,Methionine
Mulberry heart disease-pig Hormone – Insulin
Vitamin – Biotin&Thiamine
Vitamin K Haemorrhagic blemishes Dicumerol – Anti vitamin K
Sweet clover disease Treatment of sweet clover Magnesium Hypo magnesemic tetany/grass Blood level = 1.7-4 mg/dl
poisoning tetany/grass staggers/lactation tetany NH3 prevents absorption of Mg.
Wheat poisoning K depresses serum Mg.
Vitamin C(Ascorbic Haemorrhagic diathesis Stored in adrenal & pituitary
Stepping syndrome - pigs
acid/hexuronic acid) Scurvy gland
Enzyme L-gluconolactone Iron Piglet anemia Ferritin – storage form
oxidase necessary for its Thumps Transferrin – circulatory form
synthesis Absorption form – Fe2+ (ferrous)
Circulatory form –Fe3+(ferric)
Vitamin B1(Thiamin) Beri – beri/peripheral neuritis Energy releasing vitamin
Star gazing attitude Raw fish contain thiaminase Zinc Parakeratosis Stored in bones
Chastek paralysis enzyme Swallon hock syndrome
Wernick‘s encephalopathy Crooked leg
Vitamin B2(riboflavin) Curled toe paralysis –chicks Manganese Perosis Enlargement of Tibiometatarsal
Clubbed down condition Slipped tendon joint
Cheilosis - man Slipping of gastrochnemious or
Achilles tendon
Vitamin Black tongue – dogs Tryptophan is pro vitamin to
B3(Niacin/Nicotinic acid) Pig pellagra niacin Copper Salt sick - cattle High molybdenum causes
Enzootic ataxia - lambs conditional Copper deficiency
Vitamin B6(pyridoxine) Goose stepping gait Eluate factor
Steely wool - sheep
Vitamin Pernicious anemia Animal protein factor
Sway back/swing back/gingin Rickets ↑S ∫ Cu
B12(Cyanocobalamin) Microbes of family
– lambs&calves ↑Mo ∫ defieciency
Actinomycetacea can synthesis it
Falling disease - cattle
Pantothenic acid Scaly dermatitis Filtrate factor
Dissecting aneurysm - chicks
Dog sitting posture in pigs
Scouring/peat scours/teart - cattle
Folic acid Macrocytic anemia
Iodine Goiter
Biotin Scaly dermatitis Preventive against ―Egg white
Critinism – young
Red tongue injury‖
39 40
Dhruv N Desai Dhruv N Desai
Myxedema - old Fresh bone 45%
Cobalt Enzootic marasmus Silage 60 – 65%
Hill sickness RBC 60 – 75%
Bush sickness Animal body 65%
Coast disease Whole egg 65%
Wasting disease Meat 72%
Nakuritis Animal cell 70-85%
Pinning Milk 87%
Embryo 90%
Molybdenum Toxicity Plasma 91%
Teart Bile 97%
Peat scours Spinal fluid 99%
Fluorine Toxicity
Shifting lameness
Mottling teeth TRIVIALS
Selenium Muscular dystrophy (white muscle Influence Vit E absorption
disease) Antagonistic to sulphur  Amount of energy lost through methane production – 7%
Predisposes retained placenta and Improves hatchability of eggs  Rate of gas production in rumen – 30 litres/hour
abortion in sheep Toxicity
 Amount VFA in rumen fluid - 60 – 120 meq/lit
Blind staggers(Acute)
Alkali disease(Chronic)  Life span of RBC in birds – 28-35 days
 Feulgen stain is used for demonstration of nucleus in DNA
 Degnala disease is caused by feeding of Mycotoxin contaminated straw
RARE DISEASES
 Larva of Anguina agrostis causes nematode poisoning
Corridor disease Theileria lawrensi  Standard unit for measuring radiation - Becquerel
Border disease (sheep) Togoviridae  Suppression of immune response (anergy) can be treated with levamisole
Edema disease (pigs) E.coli  Temperature of blood in the body – 38 – 40 0C
Chewing disease(horse) Centaurea solstitialis  Bracken poisoning causes Bright blindness in sheep
Derzsy‘s disease(goose) Goose parvo virus I
 Cat has, proportionately to the size of its body, the largest brain
Sleepy foal disease Actionbacillus equli
Gilchrist disease(man) Blastomycosis  Edema disease in pigs is caused by E.coli
Graves disease Exophthalmic goiter  Reagent used in California mastitis test - Teepol Reagent
Grouse disease Trichostrongylus tenuis  Earliest recognized carcinogen – Soot
Morels disease Gram positive micrococci  False pregnancy in goat is also termed as – cloud burst
Nairobi sheep disease Bunyavirus  Removal of infected tissue from a wound surface is called debridement
New forest disease Infectious bovine keratitis
 A gap between front & cheek teeth in ruminants is diastema
Pullet disease Reo virus
Vent disease Treponema cuniculi  Drug used in flea collars – Dichlorvas
Schmorl‘s disease(rabbit) Bacteroides necrophorus  Dog sitting posture in pigs is seen in pantothenic acid deficiency
Creutzfeldt –jacob disease(human) Prions  ELISA was developed by Engvall & Perlmann
Tzaneen disease Theileria mutans  New market cough is a synonym for Equine influenza
Wesslsborn disease Flavi virus  Self replicating infectious protein is called as prions
Cat scratch disease Bartonella henselae
 Ursodeoxycholic acid is used for dissolving gallstones
Kysanur forest disease Flavi virus
Jacob disease Bovine spongiform encephalopathy  First recombinant viral vaccine developed was FMD vaccine
 Inflammation of tongue – Glossitis
 Guinea pigs do not synthesize Vitamin C hence more liable to scurvy
PERCRNTAGE OF WATER  Blood in sweat – Haematidrosis
Enamel 4%  Rat bite fever or haver hill fever is caused by Streptobacillus moniliformis
Cereals & straw 10 – 15%  Ivermctin is toxic to Collie breeds of dog
Hay 15%  Kitchen death is caused by carbon monoxide poisoning
41 42
Dhruv N Desai Dhruv N Desai
 Thyroid cartilage of larynx has Adams apple o Biconcave - Dog, Cow, Sheep
 Inflammation of vagina – colpitis/vaginitis o Shallow/flat - Goat
 Inflammation of nipple – mamillitis o Shallow concave - Horse, Cat
 Inflammation nail & claws – onychia o Elliptical, sickle shape - Camel , Deer
 Toxic principle in onion – n propyl disulphide o Elliptical & nucleated - Birds, Amphibians
 Removal of diseased ovary – Ovariotomy  Poikilocytosis – variation in cell shape
 Removal of normal ovary – Oopherectomy  Aniosocytosis – variation in cell s size
 Persistent erection of penis is called as priapism  Larger size RBC – in dog (7.3 micron)
 Trochlear nerve is also called as pathetic nerve  Smaller size RBC – in goat (4.1 micron)
 Papain is used to tenderize meat  Mature RBC derive energy from Anaerobic EMP pathway and HMP shunt since they have no
 Toxic principle in potato – solanine mitochondria
 Best time for abdominal palpation of pregnancy in dogs – 24 – 32 days  Aplastic anemia lacks functional bone marrow
 Urine drinking is a symptom seen in sodium deficiency  True PCV = venous PCV*0.96(Correction factor for trapped plasma)
 Adjuvant used in inactivated vaccine – Aluminium hydroxide  Rouleaux formation is seen in equines and dogs
 Vero cells are taken from kidney of African green monkey  ESR is negatively influenced by Reticulocyte and Albumin
 Site of synthesis of Monocytes - Lymphoid tissue of bone marrow and spleen
 In ruminants Haemal lymph nodes functions as spleen
 Vit B12 and folic acid are essential for maturation of RBC
I. PHYSIOLOGY  Methemoglobin is formed by oxidation of ferrous iron to ferric iron
 LOCOMOTION  Hb has 200 times more affinity for CO than O2
 Each gram of Hb binds with a maximum of 1.34 ml of O2
 PM Contraction - Rigor Mortis
 Average life span of RBC is 120 days (20-30 days in poultry)
 PM Cooling - Algor Mortis
 Destruction of RBC in dog is in bone marrow
 PM Staining - Livor Mortis
 Creatine phosphate in muscle is referred to as ATP sparer or energy buffer  RETICULO ENDOTHELIAL SYSTEM
 Each molecule of glucose produce – 38 ATPs
 About 5-6 hrs after death, all muscles of the body assume a state of contracture – Rigor Mortis o In liver and spleen –Stellate Or Kupfer Cells
 The efficiency of muscle contraction is – 45% o In tissues - Histiocytes Or Macrophages
 Muscle contraction without shortening in length – Isometric Contraction o In blood - Monocytes
 Whole cardiac muscle obeys all or none law because of Syncytium  The ratio of WBC to RBC is more in goats(1:1300) and less in cattle(1:800)
 Refractory period is the brief period during which muscles undergoing contraction for a first stimuli  Shift to left is an increase in number of immature neutrophils characteristic of bacterial infections
is unable to respond to a second stimuli  T-lymphotes provide cellular immunity
 The energy of contraction of muscle is directly proportional to the length of the fibre- Sterling law  B-lymphocytes provide humoral immunity
 Tetanisation is the fusion of successive twitches when the frequency of stimuli is given at a rapid  Suppressor or regulatory T cells regulate the activities of Cytotoxic T cells and helper T cells
rate  Life span
 Myasthenia gravis is a neuromuscular disorder in which auto antibodies are produced against Ach o Granulocyte - 4-8 hrs
receptors o T lymphocytes - 2-3 yrs
o B lymphocytes - 3-4 days
 BLOOD o Monocytes - 24 hrs
 Plasma constitutes about 55-70% of blood o Platelets - 8-11 days
 Viscosity in blood is provided by gamma globulins  Platelets are nucleated in birds and reptiles
 Arterial blood is more Alkaline than venous blood  Albumin acts as a primary carrier to fatty acids
 Yellow colour of the plasma is due to Bilirubin  Plasma proteins acts as blood buffer and thus maintains pH(7.4)
 Serum differs from plasma lacking fibrinogen, prothrombin and other coagulation factors  Prostaglandin
 RBC of species o PGG2,PGH2 - Platelet aggregation

43 44
Dhruv N Desai Dhruv N Desai
o PGI2 - Vasodilator
o PGI2 - Platelet aggregation inhibitor  Exchange between atmospheric air and pulmonary capillary is External respiration
 Vitamin K is necessary for the formation of prothrombin and clotting factors V, VII, IX and X  Gas exchange occurs in Alveoli
 Heller And Paul Mixture =Ammonium oxalate : potassium oxalate = 3:2  Dead space is the respiratory passage from the External nares to alveoli
 Sodium fluoride is ideal anticoagulant for estimation blood glucose level  Hyperventilation is increased in alveolar ventilation cause respiratory alkalosis
 Hemophilia A due to deficiency in factor VIII  Hypoventilation is decreased in alveolar ventilation cause respiratory acidosis
 Heparin is produced by mast cells and Basophils  Inspiration is an active process, Expiration is passive process
 Blood groups  In horse even under rest, expiration is active
o Dogs - 8 groups  Hypernea is state of breathing in which rate, depth or both are increased
o Horse - 8 groups  Polypnea - rapid, shallow panting type of respiration
o Cattle - 11 groups  Tidal volume – air breathed in during a quite normal respiration
o Sheep - 7 groups  The entry of air in to pleural cavity is called Pneumothorax
o Pigs - 13 groups  Residual volume represents the amount of gas remaining in the lung even after forced expiration
 Respiratory quotient(RQ)= Volume Of CO2/ Volume Of O2
 HEART AND CIRCULATION o RQ of CHO - 1
o RQ of Lipids - 0.7
 Systemic Circulation - 84% of Blood o RQ of protein - 0.8
 Pulmonary circulation - 8% of Blood  Partial pressure of O2 in alveoli pO2=100 mmHg
 Coronary circulation - 7% of Blood  Partial pressure of CO2 in alveoli pCO2=40 mmHg
 SA node controls the rate of heart  One gram of Hb can transport 1.34 ml of O2
 Excitatory stimuli originate outside SA node - Ectopic foci  Arterial blood venous blood
 Conduction velocity is fastest in Purkinje fibres
 Cardiac sounds can be recorded by using an instrument called phonocardiogram pO2= 100 mmHg pO2 =40 mmHg
 Lub sound (S1) – closure of AV valve
pCO2=40 mmHg pCO2=45 mmHg
 Dub sound (S2) – closure of pulmonary valve
 Electriocardiograph is an instrument to measure electrical changes of heart  Greater portion of CO2is transported in blood in chemical combination as HCO3
 QRS complex shows spreading of electrical potential through A.V node, bundle of his ,purkinje  Chloride shift or hamburger shift – in venous blood HCO3 ion comes out of RBC and to replace
fibres, ventricular muscles Cl ion goes in to RBC
 Output of each ventricle is referred as stroke volume  Binding of O2 to Hb displaces CO2– a phenomenon referred to as ―Haldane effect‖
 Cardiac output is the volume of blood ejected by either the left or right ventricle through the aorta  Asphyxia is hypoxia combined with hypercapnea
or pulmonary artery per minute  Hering Breuer reflex – inhibits inspiration so that prevents further inflation during over stretch
 Starlings law = Energy liberated by cardiac muscle is directly proportional to fibre length  Central chemoreceptor area is in medulla, peripheral chemoreceptor area is in carotid and aortic
 Vagus nerve is negative chronotropic and negative inotropic bodies
 Two baroceptors one in carotid sinus (sinus or buffer nerve) and one in aortic body (cardio
depressor nerve)  RESPIRATION IN BIRDS
 Endothelin is the most potent of all the mammalian vasoconstrictor substances
 True capillaries are the place of nutrient exchange  Both inspiration and expiration are active
 Pulse pressure is the difference between systolic and diastolic pressure  Syrinx - is the vocal organs in birds
 Blood pressure using Sphygmomanometer is measured using femoral artery in dogs and Coccygeal  Exchange of gases between lungs and blood occur in Parabronchi
artery in cattle  Since Air sacs are Avascular, no gaseous exchange occurs
 Increased blood flow to tissues in response to increased metabolic rate is called as Active  Diverticula from airsacs are connected to many bones , hence they are pneumatic
Hyperemia
 Cerebrospinal fluid produced from lateral, third and fourth ventricle of brain  NERVOUS SYSTEM

 RESPIRATION  Astrocytes closely attached to blood vessels of CNS

45 46
Dhruv N Desai Dhruv N Desai
 Microglia or microcytes formed from leucocytes o Sneeze trigeminal
 Schwann‟s cell (neurilemma) produce myelin sheath, insulating and coating nerve fibre  Sleep is a state of reversible unconsciousness
 Nodes of ranvier aid in flow of ions between ECF and ICF  Sleep inducing centre is located in thalamic reticular area
 Velocity of myelinated nerve fibre ranges from 30 -100m/sec, whereas in unmyelinated  Serotonin is a sleep inducing substance secreted by raphe nuclei
30 m/sec  The central motor control system include the motor cortex, basal ganglia and cerebellum
 The duration of activity of neurotransmitter in synaptic cleft last only for 1-2 m sec  Coordination of slow or ramp movements is associated with basal ganglia
 Specific type of neuron synthesis and release only one type of neurotransmitter substance at nerve  Membranous labyrinth is the functional part of vestibular apparatus
terminal – Dale‟s principle  The terminal portion of the spinal cord, the meninges and nerves are collectively referred to as cauda
 Neurotransmitters susceptible to anoxia and anesthetic agents equinae
 Excitatory neurotransmitters : Glutamate,Substance P, L-Aspartate  Brachial plexus – C6,C7,C8,T1&T2
 Inhibitory neurotransmitters: Glycine, GABA, Dopamine, Serotonine, Taurine, Morphine,  Lumbosacral plexus – L3,L4,L5,S1&S2
Endorphine  Mixed(both motor sensory) cranial nerves: Trigeminal, Facial, Glossopharyngeal, Vagus
 Both excitatory and inhibitory : Ach, Nor Epinephrine, Epinephrine, Histamine, PG  Sensory cranial nerves: Optic, Olfactory/Vestibulotrochlear
 Neurotransmitter Aminoacid Precursor
o Norepinephrine phenyl alanine
o Glycine serine  DIGESTIVE SYSTEM
o GABA L-glutamic acid  Salivary secretion in cattle – 100 – 200 lit/day
 Exteroreceptors and proprioreceptors are collectively called Somatoreceptors  Organ of prehension in dog & cat – fore limb
 Proprioreceptors – for posture and balance  Fundic glands
 Fore brain – Proscencephalon  Body chief cells/peptic cells – pepsin & rennin
 Mid brain - Mesencephalon  Neck chief cells – intrinsic factor & mucin
 Hind brain – Rhombencephalon  Parietal/oxyntic cells – HCl
 Two cerebral cortices are connected by transverse myelinated fibres known as the corpus collasum  Acetate : propionate :Butyrate ratio
 The ability of one hemisphere to control movement, reducing that burden for the other half is called  Forage diet – 70:20:10
Cerebral Dominance  Grain diet - 60:30:10
 Thalamus functions as sensory relay nuclei  Normal VFA content of rumen – 60 -120 meq/lit
 Limbic cortex primarily functions as visceral brain  Ketone bodies serves as energy source in CNS & heart
 Hypothalamus acts as a principal motor output pathway of the limbic system and controls the  Pancreas
vegetative functions of the body  Secretin – stimulates secretion of bicarbonates from pancreas
 Formation and recall of memory require the function of amygdala and hippocampus  CCK – stimulates secretion of enzymes from pancreas
 Melatonin released from pineal gland in response to darkness  Sphincter of oddi gaurds terminal part of common bile duct
 Melatonin inhibits gonadal activity  Jaundice
 Tri geminal, Abducent, facial, vestibular originates from pons  Obstructive jaundice – conjugated bilirubin
 Cerebellum is important in the planning and execution of ballistic movements  Hepatic jaundice – free bilirubin
REFLEX CRANIAL NERVE  Hemolytic jaundice – both free & conjugated
o Pupillary light Oculomotor  Duodenum acts as pace maker of intestine
o Corneal  CHO in the Small Intestine stimuli for GIP secretion
 Blink Ophthalmic division of trigeminal  Fat & protein in the Small Intestine stimuli for CCK secretion
 Lachrmation -do-  Acid in the Small Intestine stimuli for secretin secretion
o Salivary reflex Trigeminal, facial, glossopharyngeal  Reverse peristalsis is a normal feature in colon
o Emetic glossopharyngeal, vagus,vestibular  Absorption
o Sucking trigeminal and facial  Glucose & amino acid – active transport (Na+ Co transport )
o Swallowing vagus, hypoglossal, glossopharyngeal  Short chain fatty acid & glycerol – passive diffusion
o Mastication trigeminal, facial,GP  Intact protein & triglycerides – pinocytosis
o Cough reflex vagus  EXCRETORY SYSTEM
47 48
Dhruv N Desai Dhruv N Desai
 Serum magnesium level is constantly increased during hibernation
 Renal function is the total cardiac out put that passes through the kidneys (21% in man; 20%  Brown fat present in hibernating animals helps them to awake from hibernation
in dogs)  Estivation /summer sleep is observed in – frog, crocodiles & alligators
 Glomerular filtration rate – 180 lit/day  Amount of heat loss by evaporation of 1g of water is 580 calories
 Glomerular membrane is completely impermeable to plasma proteins  Sweat glands – Eccrine in man ; Apocrine in animals
 Filtration fraction – percentage of the renal plasma flow that becomes glomerular  Among farm animals sheep & cattle have lowest critical temperature
filtrate(normal plasma flow – 650 ml/min ; normal GFR – 125 ml/min)  Raise in 10c in body temperature can cause increase of 10 -20% in the basal metabolism
 Glucose &amino acid are reabsorbed entirely from glomerular filtrate  Oily secretion of preen glands makes the plumage resistant to wetting
 Ethology is the study of animal behavior
Part Amount Of Remarks
GFR  ENDOCRINOLOGY
Reabsorbed  Carbolic acid is the first disinfectant identified by joseph lister
Proximal tubule(action of 65% Decrease Ca excretion  Idoxuridine is the first antiviral drug identified by Kaufman
PTH)  Secretin is the first hormone identified by bayliss & starling
Descending loop of henle 15% More permeable to water  Oxytocin & vasopressin are the peptide hormones
Less permeable to urea & sodium
 Precursor of steroid hormones – cholesterol
Ascending loop of henle Less permeable to water  RECEPTORS
More permeable to urea o Protein ,peptide hormones
& catecholamines - cell surface
Distal tubule(action of 10% Active Na+ transport o Steroid hormones - cytoplasm
aldosterone) Secretion of K+ o Thyroid hormones - nucleus
 First messenger – hormone
Collecting tubule(action of 9.3% Permeable to water
 Second messenger – c AMP, Calmodulin, Cytosolic Calcium, Diacyl Glycerol, Inositol
ADH)
Triphophate
 Third messenger – phosphokinase
Plasma load – total amount of substance in the plasma that passes through the kidneys each

 Physiologically, the pituitary gland is a master gland
minute ( plasma load of glucose -600mg/min)
 Arachidonic acid is a precursor for prostaglandins
 Tubular load – fraction of plasma load that is filtered as glomerular filtrate(tubular load of
glucose – 125 mg/min)  Long day light promote reproduction in horse – long day breeder
 Urine is thick in horse  Sheep & goat – short day breeders
 Tubular fluid contain 2 buffer system namely – phosphate buffer & Ammonia buffer  α - receptors control catecholamine release from sympathetic nerve endings
 Presence of fat in urine is not pathological Hormone No of Type of action Remarks
 Glucosuria is a characteristic finding in enterotoxemia Amino
 When the pressure in bladder reaches 150 mm H2O, contraction of bladder begins acid
 Avian kidney has 2 types of nephrons – Mammalian type(25% glomerular filtrate) & reptilian Growth hormone 190 Anabolic hormone Deficiency-Dwarfism
type(75% glomerular filtrate)
 Metabolic end product of protein in mammals – urea where as in birds & reptiles it is uric Protein sparer Excess – gigantism (young)
acid Agromegaly (adult)
GROWTH & BEHAVIOR
 Only 3% of the cells with in adult body is capable of dividing Prolactin Lactogenic hormone Crop milk secretion – pigeons
 Shape of Growth curve is sigmoid or „S‟ shape Broodiness – birds
 GH does not have effect on growth during the fetal life
Maintenance of CL – sheep
 Krypton gas is used to measure body weight indirectly by dilution method
&goat
 Thermo neutral zone for most farm animals 60 -900F
 Torpor is a stage in which animals or birds make their metabolic activities decline Metarnal behavior - animals

49 50
Dhruv N Desai Dhruv N Desai
Vasopressin 8 Deficiency Catecholamines Non shivering thermogenesis
( adrenal medulla)
Diabetes insipides

Oxytocin 8 Let down of milk


Sperm transport
Contraction of uterus

Thyroxine Potent Galactopoitic Metamorphosis – amphibians


hormone
Moulting – birds II. VETERINARY ETHICS AND JURISPRUDENCE
Catabolic hormone
Deficiency ACTS YEAR
T4(Thyroxin) – more
predominant than T3 Cretinism(young) Livestock importation act 1898 modified on 1952
T3 is more active than T4 Myxedema(adult) The Glanders &Farcy act 1899
Para thyroid 84 Increases Ca absorption The dourine act 1910 modified on 1957
hormone
Minute to minute regulation of The poisoning act 1919 modified on 1952
(Chief cells of blood calcium
parathyroid gland) Dangerous drugs act 1930

Calcitonin 32 Rapid but short time regulation Drugs and cosmetics act 1940
of blood calcium Drugs and cosmetics rules 1945
(C cells of thyroid
gland – animals Prevent post prandial hyper Prevention cruelty to animal‘s act 1960
calcemia
Ultimobronchial Prevention cruelty to animals to drought& pack 1965
gland –reptiles, animals rules
amphibians &
birds) Prevention cruelty to animals (licensing of Farriers 1965
rule)
Insulin 51 Hypoglycemic factor Fat sparer
Prevention cruelty to captured and wild animals 1972
(β cells of Deficiency - diabetes mellitus
pancreas) Wild life ( protection ) act 1972
Glucagon 29 Hyperglycemic factor Project Tiger 1973
(α cells of Ketogenic hormone Prevention cruelty to animal‘s registration of cattle 1978
pancreas) premises

Mineralocorticoids Electrolyte balance Transportation of animals rules 1978


( zona glomerulosa
of adrenal cortex) Blood pressure homeostasis Experimental animals act 1982

Eg - aldosterone Animal welfare board of India 1982

Glucocarticoids Anti inflammatory Project Elephant 1992


(zona reticularis of
adrenal cortex) Deficiency –Addisons disease Note :-

Excess – cushing syndrome  Livestock importation act 1898 not permitting transport of following diseased
Eg - Cartisol
animals -Tickpest,Anthrax,Glanders,Farcy,Scabies
 Applicable in all states of India except in J&Kstate

51 52
Dhruv N Desai Dhruv N Desai
 Cloning in sheep - 1997, DOLLY.  There are two populations of T cells – T helper cells (TH cells) and cytotoxic T cells (TC cells).
 Phook or doomdev injection air or any materials in to the female genital organ  Differences Between Humoral & Cell Mediated Immunity
THE INDIAN PENAL CODE
Cell mediated immunity
THE INDIAN DEALS WITH Humoral immunity
PENAL Antigen Extracellular antigens Intracellular antigens
CODE
Responding B lymphocytes T lymphocytes
Section 44 Illegal harm to the animals lymphocytes
Section 47 Definition of animals - any living being other than human beings
Effector mechanism Antibody mediated Lysis of infected cell
Section 192 False entry in records elimination

Section 197 False certification of animals Transferred by Serum T lymphocytes


Section 204 Destruction of any documents
Section 269 Done by negligence to spread infectious disease
 The portions of antigen that are recognized by the immune system (by individual lymphocytes) are
Section 270 Done by intention to spread infectious disease called epitopes or antigenic determinants
 Apoptosis is programmed cell death.
Section 271 Not following quarantine rule
 Memory cells escape apoptosis through expression of a specific gene sequence called bcl2
Section 272 Adulteration of any food or drink  Immunologic unresponsiveness against individual‘s own antigen is referred as tolerance
Section 273 Sale of unfit food or drink-6 months prison with Rs -1000/ - fine.  The specific immune response that takes place after an antigen stimulus can be divided into three
phases Recognition Phase, Activation Phase & Effector Phase.
Section 274 Adulterating medicinal items for sale  There are three classes of lymphocytes – B-lymphocytes, T-lymphocytes and natural killer cells
Section 275 Sale of adulterated medicine (NK cells)
 Mature B cells do not produce antibody but it differentiates into two daughter cells upon antigenic
Section 289 Disobey any order, with any animal in his possession stimulation – plasma cells and memory cells
Section 304 Negligently causing death of any person  Plasma cells are the only cells in the body to produce antibodies
 The two important CD receptors of T cells are CD4 and CD8.
Section 377 Voluntary carnal inter course/ Beastiality with any animals – 10 yr prison with Rs
 The helper T cells have CD4 receptors and Cytotoxic T cells have CD8 receptors
10000/ fine
 The NK cells mediate a phenomenon called Antibody Dependant Cell Mediated Cytotoxicity
Section 420 Fraudulent cheating of persons altered animals - prison of 7yr (ADCC) that removed the antigen coated with immunoglobulins
Section 427 & Mischief, maiming, killing by poisoning – prison of 2 yr Sl.No Property B cells T cells
428
1. Site of Bone marrow, bursa, Thymus
Section 430 Causing decrease of water supply for animals development Peyer‘s patches

2. Distribution Lymph node cortex, Spleen periarticular


splenic follicles sheath

3. Circulate No Yes

III. IMMUNOLOGY 4. Antigen receptors BCR TCR

 Louis Pasteur who was credited with the formulation of germ theory did extensive work on fowl 5. Important surface Immunoglobulins CD2, CD3, CD4, CD8
cholera, anthrax and rabies and developed vaccines. antigens
 The innate immunity is also called as natural defense 6. Antigens Free foreign proteins Processed foreign
 Humoral immunity is mediated by antigen specific blood glycoproteins called antibodies.
53 54
Dhruv N Desai Dhruv N Desai
recognised proteins on MHC 4. Spleen – Filter in circulatory system
5. MALT – localised antibodies at major sites of pathogen entry
7. Tolerance Difficult Easy
induction  A microorganism is said to be pathogenic when it can cause infection and the ability to cause
8. Progeny cells Plasma cells, memory Helper T cells, infection is termed as virulence.
cells cytotoxic cells  Molecules with a molecular weight of 5000 or greater are good immunogens
 Haptens are small antigens that cannot elicit antibody response individually. But they become
9. Secreted protein Immunoglobulins Cytokines immunogenic when coupled with larger molecules.
 Exotoxins are highly immunogenic and stimulate the production of antibodies. The antibodies
10. Phenotypic Fc receptor, Class II Helper T cell
against exotoxins are called antitoxins.
markers MHC, CD19, CD21
CD3+, CD4+, CD-  When these exotoxins are precipitated by mild protein denaturing agents such as formaldehyde, the
Cytotoxic cell exotoxin loses its pathogenicity but retains its immunogenicity called toxoids
 Some times rarely immune response is elicited against normal body components. Such substances
CD3+, CD4-, CD+
are called as autoantigens
 Heterophile antigens are immunologically related groups of antigens that occur in the cells of some
bacterial species and also in some species of animals.
 The macrophages are considered as powerful phagocytic cells and are referred as big eaters or  Chemically the antibody molecules are glycoproteins.
garbage collectors
 The flexibility of an antibody molecule is due to a region called hinge region that is rich in proline
and cysteine residues.
 The antigen-combining site of an antibody is called as paratope.
 IgM is the biggest antibody molecule with a molecular weight of 970kD.
Different names are given for macrophages found in various organs.  IgM is the first antibody to class to appear in primary immune response
 IgG is the only antibody class that can pass through placenta
a. Blood stream - Monocytes  IgE presence in large amount is an indication of allergic condition.
b. Connective tissue - histiocytes
c. Liver - Kupfer cells  Macrophages and dendritic cells are called as antigen presenting cells (APCs)
d. Brain - Glial cells
e. Lung alveoli - Alveolar macrophages 1. Primary binding tests – In these tests the binding of antigen to an antibody is measured directly.
f. Lung capillaries - Pulmomary intravascular macrophages E.g. RIA, ELISA, IFA etc.
2. Secondary binding tests – In these tests, the results of antigen-antibody interaction
 The most important CD marker of macrophages is CD68, which is otherwise called as macrosialin
(agglutination, precipitation, fixation of complement) in-vitro are measured. E.g. HI, AGID, CFT
 The primary function of macrophage is phagocytosis
etc.
 The actual mechanism of macrophage-mediated phagocytosis is by receptor-mediated endocytosis 3. Tertiary binding tests – These tests are in-vivo tests and require a living system. In these tests,
followed by lysosomal enzyme degradation. the effects of actual protective effects of antibodies are measured in living system. E.g.
 The percentage of neutrophils in blood circulation among animals varies widely. It is 60-70% in Neutralisation assay
carnivores, 20-30% in ruminants and 50% in horses  The ability of an assay to detect only the target and not any other is referred as specificity of the test.
 Opsonin make the antigen palatable for phagocytic cells.  The sensitivity of an assay refers to ability of the test system to detect very minute amount of the
 Ig E is one of the isotypes of antibodies that is responsible for allergic reactions. target
 Interdigitating dentritic cells are scattered throughout the skin epidermis and called as Langerhans  In Fluorsescent immunoassays (IFA) fluorescent dyes like fluorescent isothiocyanide (FITC) or
rhodamine is used
cells.
 If the concentration of antibody is in excess it will not produce agglutination. This phenomenon is
The important roles of generative and peripheral lymphoid organs called as prozone reaction.

1. Bone marrow (mammals) and Bursa of Fabricius (birds) – B cell factories  Inactivated vaccines - Formalin and Beta propiolactone are the common inactivating agents.
2. Thymus and intestinal epithelium (payer‘s patches)– T cell factories
3. Lymph nodes – Junctional filters in lymphatic system
55 56
Dhruv N Desai Dhruv N Desai
 Plasmids: are also referred as extra chromosomal DNA
 Transposition: Certain genes in bacteria are capable of shifting from one location to another
IV. GENERAL MICROBIOLOGY location in the chromosome
 Disinfectants - chemicals that are used to kill microorganisms on inanimate objects.
 Antoni van Leeuwenhoek is called as father of bacteriology.  Antiseptics - chemicals that are relatively in toxic and are used to kill or inhibit microorganisms in
 Robert Hooke - identified cells using his compound microscope living tissues.
 Louis Pasteur is called father of Microbiology.  Generally the chemicals with antimicrobial actions are referred as germicides.
 Flagellum is the organ of locomotion for bacteria.  The substances obtained from microorganisms alone are referred as antibiotics.
 Monotrichous - Bacteria having single polar flagellum.  First report of viruses: by Dimitrii Ivanowsky attributed the causative agent of tobacco mosaic
 Lophotrichus - Having tufts of flagella at one end disease.
 Amphitrichous - Having flagella at both ends FIRSTS
 Peritrichous - Having flagella all around surface  Animal virus – Foot and mouth disease by Loeffler and Paul Frosch
 The flagellum is composed of three parts filament, hook and basal body  Human virus – Yellow fever by Reeds Commission
 The major component of cell wall of Gram positive bacteria is Peptidoglycan (80-90%).  Plant virus – Tobacco mosaic disease by Dimitrii Ivanowsky
 The LPS is also referred as Endotoxin  The DNA found in the chromosome is attached with a protein called histones and this protein is also
 Mycoplasma do not have cell wall. responsible for staining property (basic staining).
 The ribosomes of bacteria are 70S composed of 30S and 50S ribosomes subunits.  The two strands of DNA are joined together by hydrogen Bonds.
 Four nucleic acid bases form the deoxyribose-nucleotides. They are Adenine (A), Guanine (G),  The most commonly used stain for fungal identification is Lactophenol Cotton Blue.
Cytosine (C) and Thymine (T) of DNA.  The media that are commonly used for fungal isolation are the Sabouraud‟s dextrose agar
 The nucleotides of DNA are linked by 3‟-5‟ phosphodiester bonds.  The substances that pathogens produce that cause damage to phagocytes are referred to as
 RNA has got purine bases adenine (A) and guanine (G) and pyrimidine bases Cytosine (C) and "Aggressins".
Uracil(U).  Endotoxins are part of the outer cell wall of bacteria.
 DNA is used to code for the synthesis of RNA is called transcription.  Endotoxins are associated with cell wall of Gram-negative bacteria - Lipopolysaccharide
Extra cellular bacterial proteins that function as invasin
 GROWTH OF BACTERIA Invasin Bacteria Involved
 Generation is the interval for the formation of two cells from one cell  Coagulase Staphylococcus aureus
 Lag phase: The brief period of no activity is called as lag phase.  Leucocidin Staphylococcus aureus
 Exponential phase: During this phase there is rapid increase in the number of bacteria.  Lecithinase Clostridium perfringens
 Stationary phase: During this phase there is no increase in number of cells.  Anthrax Lf Bacillus anthracis
 Death phase: The death phase is also exponential but it is slow. It is due to lack of nutrients. V. MEAT SCIENCE
Bacteria are classified in to five categories.
 Psychrophile - (13oc)  Lateral retro pharyngeal lymph node is used to rule out TB
 Mesophile - (39oc)  Hemal lymph node absent in horses and pigs
 Thermophile - (60oc)  Water : protein ratio of young animal > 4:1
 Hyperthermophile - (88oc)  Muscle : Bone ratio for healthy animals - 4:1
 Hyperthermophile - (105oC).  Rigor mortis time of cattle = 9 hrs, birds = 2 hrs.
 Halophile - Salt loving, prefer Nacl concentration between 1-30%),  Chilled meat temperature is 7˚C
 Osmophile - grow in high sugar concentration  Chilled offal temperature is 3˚C
 Xerophile - grow in very dry conditions  Frozen temperature of meat is -18˚C
 Yield of ATP molecules in respiration : 38 ATP
 Cooked meat temperature is 72˚C
 Mutation: inheritable change in base sequence of nucleic acid
 Black cartridge used for slaughtering medium size of animal
 Point mutation: Mutations involving one or very few base pairs are referred as point mutation.
 In electrical stunning low voltage temperature is 70 volts/250mA, 7-10 sec
 Transduction: process in which DNA is transferred from cell to cell through viruses that infect
 In stunning if current is not sufficient it lead to Curarisation/Missed Shock
bacteria called as Bacteriophages.
 Act of slaughter in jewish method is shechita
 Conjugation: process of transfer of DNA directly from one bacterial cell to another cell by a
mechanism that requires cell-to-cell contact  First slaughter house → leonar, Mumbai
 Phosphorous level of meat and blood 55-60% and 80%
57 58
Dhruv N Desai Dhruv N Desai
 Mould formation is common in chilled meat  Giblet consisting of heart, liver, gizzard
 Process of freeze drying called lyophilisation  Poultry meat contain high level of oleic and linoleic acid and low level of cholesterol
 Sterilization by radiation called Radapperization  The onset of rigor mortis is enhanced at ambient temperature above 20˚C
 Marbling absent in horse flesh and venison  In plate type freezer achieved at the temperature of -10˚C & blast type freezer achieved at
 Musky odour seen in buffalo meat -10˚to -30˚C
 Glycogen content in horse 0.5-1%  Canned meat products have a self life of 2 yrs at ambient temperature
 Refractive index is high in horse fat  Hippophagia –consumption of horse meat
 Feed efficiency – poultry(1:1.8)>rabbit(1:2)>pig(1:3)>cattle(1:5)  Kynophagia – consumption of dog meat
 Dressing percentage of pig →70-75%  Weight taken 24hrs prior to slaughter is considered as Live weight of the animal
 Ritual method practiced in india is halal & jhatka method
 Gut sweat bread → Pancreas
 Reducing agent used in curing is Sodium Ascorbate(0.2-1%)  PLUCK in cattle – larynx, trachea, lungs, heart and liver
 Ultimate pH level of meat is 5.5-5.7 Sheep – spleen also
 Process of conversion of muscle to meat called rigor mortis Pigs – esophagus also
 Rigor mortis occurs 8-12 hrs after slaughtering  Meat inspector in his one day work(8hrs)can examine – 75 cattle/200 pigs/250 calves/400 sheep
 Autolytic lysosomal enzymes in meat is called cathepsin  Area size
 PSE occurs mostly in pig, DFD is common in beef Small abattoir Upto 30,000 units/year 1 – 2 acres
 In cold storage condition, Z line is distrupted Medium abattoir 50,000 + units/year 2 – 4 acres
 Thawing temperature of meat is 4-6˚C
 Freezing point of meat is (-1.5˚C) Large abattoir 1 lakh + units/year 4 – 6 acres
 Presence of watery or blood stained fluid from frozen meat is called weep/drip
 light intensity
 Scalding temperature of pig is 62-64˚C for 6 min
(Taken at the level of 0.9 m from floor)
 One animal unit =one bovine=2 pigs=3 calves = 5 sheep
 Overhead rails should be placed at the height of 3.3 m for cattle dressing All inspection points 540 lux units 50 foot candles
 Meat analogues are Soyabean protein and gluten of wheat
Slaughter hall & work room 220 lux units 20 foot candles
 Meat of deer is called venison
 Dressing % of veal =63% Other areas 110 lux units 10 foot candles
 Art of removing skin/hide is called flaying
 Fresh, emulsion type of pork sausage called ‗salami‘  Room temperature
 Vitamin B1 (thiamine) is higher in pork
Chilling room -1 to 50C
 Water level of meat is 65-80%
 Milk has an excellent source of Ca & P and low in Fe, cu, I and vitamin C. Detention room 200C
 Self life of vacuum packaging is 8-10 weeks at 0˚C
Edible offal room 30C
 Musty/earthy odour due to Achromobacter and fishy odour due to E.coli
 Keet is the name of young guinea fowl Meat cutting room 120C
 Cow slaughter is banned in india except in states of kerala and west Bengal
 Maillard reaction is responsible for development of brown color on the surface of cured meat  PPM level
 Black rot in eggs is cause by Proteus and Pseudomonas
Chlorine for carcass washing 100 ppm
 Red rot caused by Serratia
 In sausage making, salts added in the level of 4-4.5% Chlorine for equipment washing 250 ppm
 Functional unit of myofibrils called Sacromere
BOD of domestic sewage 250-300 ppm
 Commonly used humectants are glycerol/propylene glycol
 Buffalo meat is white due to absence of carotene BOD of slaughter house 1500-2000 ppm
 Vitamin A present in beef and mutton absent in Buffalo, Chevon And Pork

59 60
Dhruv N Desai Dhruv N Desai
Sodium nitrite level in cured meat 200ppm  First hormonal peptide to be synthesized found in the animals
 Oxytocin = rapid birth
Sodium nitrate level in cured meat 500ppm
 Two sites of origin – ovary, hypothalamus
 Dressing percentage  Contraction of oviduct, milk letdown
 Ovarian oxytocin - Luteal function – acting on endometrium – induce PGF₂α in turn leads to Lysis of
cattle 50 -54 % CL
sheep 45 -48 %  Estrogen enhances responsiveness of smooth muscle to oxytocin
GnRH
Goat 43 -50 %  Release of FSH, LH
FSH
Pig 70 -75 %
 Growth & maturation of graffian follicle
Poultry 65 – 70 %  Spermatogenesis – up to secondary spermatocytes
 Acts on receptors of sertoli cells leads to production of ABP
Rabbit 52 – 58%  Spermiation
 Secretion of inhibin from granulose cells of ovary and sertoli cells of testes
 Bleeding time
LH or ICSH
Species Bleeding Amount of blood Blood yield  Pre ovulatory LH surge
time(mts) (% in body wgt)  Maintains activity of CL
 Stimulates leydig cells
Cattle 6 3 - 4% 10 – 12 kg
Prolactin
Calves 6 5 – 6% 1.5 kg  Luteotropic properties in dogs, mice, rats
 Maternal behavior
Sheep 5 4 - 4.5% 1 -1.5 kg  Functions as metabolic hormone in lower forms of animals
Pig 6 3 -4% 2 -3 kg Placental hormones
 PMSG, hCG, PL, PSPB
poultry 1.5 - 2 PMSG
 Can be isolated from blood, not found in urine
 Endometrial cups of pregnant mare – these are formed by 40th day of gestation and persist till 85th
Slaughtering of pigs
day of pregnancy
 Desirable thickness of fat on the back is 1.5 inches for lean pork production
 Important for maintenance of pregnancy in mare
 Gaseous Stunning – 65-70%CO2,
 Clinical use – super ovulation, anestrum
 Electrical stunning - 60-80 volts, 5-10 sec
 More of FSH like activity
 Sticking – carotid arteries and jugular veins , 5-6 min
 Calcium deficient in meat of pig
 Pig – highest fat storing ability HCG
 Pork is rich in phosphorus, iron,energy  Syncytio trophoblastic cells of placenta of primates
 Vitamin A and D not present in pork  More of LH like activity
 Blue pig - crossing of white and black breed  Clinical use – induce ovulation, cystic ovaries
 Lard – pig fat Placental lactogen
 Store pig – 8 -15 weeks of age for market  GH like activity
 Scalding temperature – 60-63˚C for 5 min  Imp. Regulator of maternal nutrients to the growing foetus
Estrogen
VI. GYNAECOLOGY
 Sexual receptivity in female
Oxytocin  Secondary sexual characters
 Ductal development of mammary gland
61 62
Dhruv N Desai Dhruv N Desai
 Attachment of embryo to uterine wall  Dog – two parts bulbus glandis(proximal 1/3) , pars longa glandis(distal2/3)
 Anabolic effect  Cat – short, terminal part having several spines
 Negative feedback mechanism to the GnRH  Stallion – prominent urethral process, Groove - Fossa glandis
 Development of female reproductive tract Testicular descent
 Clinical use : Induction of heat, treatment of misalliance, as Ecbolic (Mummification, Pyometra),  Bull – 106 days of gestation
induces milk production in cow  Horse – Near birth
Progesterone  Ram, Boar – 70 days of gestation
 Source ; ovary, placenta, adrenal, testes  Dog – 3 – 4 days post natally
 Inhibits uterine contractions  High flankers – Testes reaches the inguinal canal but not descent in to the scrotum
 Increases endometrial secretions  Impotentia coeundi – Reduced to complete lack of sexual desire and ability to copulate
 Maternal behavior, nest building  Impotentia generandi – Inability to reduced ability to fertilize
 Induction of lactation  Balanitis – Inflammation of glans penis
Clinical use  Posthitis – Inflammation of prepuce
 Treatment of ovarian cysts not responds to GnRH  Balanoposthitis - Inflammation of penis and prepuce
 Cervico vaginal prolapsed  Phimosis – Unable to normally protrude the penis
 Early embryonic mortality  Paraphimosis – Unable to retract the penis in to the prepuce
 Habitual abortion  Diphallus – Double penis
 Cow, Goat, Sow – CL dependant  Phallocampsis – deviation of penis either ventral or lateral or spiral
 Rainbow penis – ventral deviation
TESTES  Corkscrew penis – lateral deviation
 Mediastinum testes absent in stallion
 Connective tissue capsule – Tunica albuginea Inherited sperm defects
 Medial septum of testes – Dartos
 Blood testes barrier – primary – Peritubular cells -prevent auto immune reactions Diadem effect Sign of disturbance in spermiogenesis, Feulgen stain & phase
Secondary – junctional complexes between sertoli cells Eversion of galea captis &crater shaped contrast microscopy helpful
 Testes – 4-6°c lower than body temperature depressions in the nucleus, in revealing this defect.
 Oxytocin, PGF2α, Ach, tends to alter the Epididymal Transit Time Nuclear pouch formation defect.
 Extra gonadal reserve (EGR) – Epididymis, vas deferens, Ampulla
 Time require to complete a cycle of seminiferous epithelium(Spermatogenesis) Knobbed Acrosomal defect, Eccentrically placed Eosin-B, Fast green stain&
Bull – 14 days spermatozoa thickening of the Acrosome. phase contrast microscopy
Boar – 9 days helpful in revealing this
Ram – 10 days defect.
Horse – 12 days
 RUT – Certain definite period of sexual excitement in some wild animals (Deer,
 Testicular hypoplasia – Giant cells, medusa cells, high incidence of cytoplasmic droplets
Camel, Elephant) – spermatogenesis occurs in this period
 Testicular degeneration – Large no. of primary abnormality
 Ampullae & Vesicular gland absent in dog and cat
Spermatogenesis
 Yellowish colour of bull semen is due to riboflavin
 Bull, Ram, Dog – 60-70 days
 Bulbouretheral gland absent in dog
 Stallion – 40-45 days
 Sigmoid flexure – Pre scrotal – Boar, Post scrotal – Bull, Ram
 Boar – 50-60 days
 Retractor penis muscle controls sigmoid flexure
 Azoospermia – no sperms
 Oligospermia – decrease in sperm concentration
Glans penis
Artificial insemination
 Bull – pointed
 1780 – Lazzaro spallanzani (ITALIAN) - AI in bitch
 Ram – urethral process
 1900 - Ivanoff (Russian) – used AI as a technique for breeding
 Boar – glans penis absent
63 64
Dhruv N Desai Dhruv N Desai
 1939 – in India – Kumaran -Palace dairy farm – Mysore Mare 36 – 38 days
Semen collection
Ewe 16 -18 days
Species Artificial vagina Tempeture
Bull 39-41°c Sow 13 – 20 days
Stallion 45-50°c
Boar 45-50°c
Ram & bucks 45-50°c  FSH & LH required for antrum formation
Dog 40-42°c  Cow – Metestrual or post Estrual bleeding – capillary bleeding due to the with drawl of
 Sperm cell concentration – Bull - 10% of the semen volume, Boar – 2-5% estrogen
 Species pH  Young animals – slight shorter length of estrous cycle
Bull, Ram 6.8 Sexual differentiation
Stallion, Boar 7.4 o Feline, porcine embryo – 30 days of gestation
Dog 6.7 o Ovine – 35 days
 Fructose – Normal sugar providing energy to spermatozoa in ruminants o Bovine – 45 days
 Sorbitol – Sugar alcohol can be oxidized to fructose and provides source of energy  Ovum ovulated - All species Metaphase II – 2nd meiotic division
 Inositol – Boar semen Mare, Dog, Fox – 1st meiotic division
 Ergotheionine – Boar, Stallion
 Glyceryl phosphoryl choline – Epididymal Secretion Species Estrus period Ovulation time
 Age at Puberty
Cow 14-18 hrs 12-18 hrs after the end of estrus
Species Female Male
Mare 4-7 days Last 2 days of estrus
Cattle 6-18 months 9-12 months
Sow 2-3 days Last day of estrus
Horses 10-24 months 18 months
Ewe 1-2 days Last day of estrus
Dog 6-12 months 7-10 months
Bitch 7-9 days First 3 days of estrus
Swine 5-8 months 5-7 months
Cat Induced 4 days if One day after mating
Cat 5-18 months 5-18 months
copulation occurs or else 9-
10 days
 Monoestrous – Wild Animals
 Polyestrous – Cow, Sow  Capacitation initiated in the uterus and completed in isthumus of oviduct
 Regular estrous cycle – Cow, Sheep, Mare, Bitch, Sow  Hyaluronidase – Bull acrosome
 Spontaneous ovulators – ovulation takes place but CL formed will not be functional until  Arylsulfalase – Boar acrosome
mating  Syngamy - Fusion of male and female pronuclei
 Induced ovulators – ovulation & CL formation depends upon the mating has occurred or  Pheromone – volatile substance secreted or released outside the body and perceived by
not ( cat, Rabbit, Mink ) the olfactory system of other individuals of the same species
 Uniparous / Monotocous – one ovum, one fetus( cow, mare, sow)  Boar – Saliva (sub maxillay gland), Prepucial pouch – 2 Attractants 3α androstenol, 5α
 Multiparous / Polytocous – 3 -15 ova, 3-15 fetus(dog, cat, sow) androstenone
 Nullipara – female that have never conceived / carried young one  Flehmen response – Bull, Ram, Stallion
 Primipara – conceived for the first time (1st gestation period)  Delayed ovulation, silent estrus, anovulation may be due to β-carotene deficiency
 Pluripara – conceived previously 2 or more times earlier  Early embryonic mortality occurs between 8-19 days after breeding
Species Implantation(Days after Conception)  Cow – best time of AI – middle to the end of standing heat ( mid estrus not metestrus )
 Card test – rapid, sensitive accurate test for field screening of brucellosis
Cow 22 – 35 days  Leptospirosis – gargety milk

65 66
Dhruv N Desai Dhruv N Desai
Disease organism Time of abortion Diffuse Mare, Sow

Vibriosis Vibrio fetus veneralis Early Embryonic death – common Zonary Bitch, Queen
4 th month to term – occasional ( II
trimester ) Discoidal Guinea pig

Trichomonosis Trichomonas foetus First trimester ( 2-4 months)


 Cattle –Caruncle - Arranged in 4 rows (70-120 in number)
Fungal abortion Aspergillus fumigatus 5th -7th months  True water(2nd ) bag - amnion
 Mare – cruciform or ‗T‘ shaped
Epizootic bovine Psittacosis,Chlamydia 6th -8th month
 Bitch and queen entire uterus lies in the abdominal cavity
Abortion Group of org
 Portion of cervix projects in to the vagina – Portio vaginalis
Listeriosis Listeria monocytogenes Last trimester(7th -9th month)
 Fornix – absent in sow, prominent in mare
Brucellosis Brucella abortus Last trimester of pregnancy  Remnants of wolffian duct – gartners duct
 Pregnancy diagnosis also known as cyesiognosis
Leptospirosis L.pomona,L.hardjo, Last half of gestation
 Positive signs of pregnancy – Amniotic vesicle, Fetal membrane slip, Fetus, cotyledons
L.grioppotypphosa
Days Palpable part @ pregnancy
IBR –IPV Herpes virus All 3 trimesters of the pregnancy
30 days Amniotic vesicle
Gonadal sex determination
 XX – Medulla inhibited and cortex develops - Female 35-90 days Fetal membrane slip
 XY – cortical development inhibited –testes develops - male
 Primary sex cords - Ancestors of spermatozoa About 90 days Fetal bump
 Secondary sex cords – Ancestors of oocytes 90-100 days Placentomes

Species Ovary shape More functional 120 days Fremitus

Bull , Ewe Almond Right  White heifer disease due to sex linked recessive gene is commonly seen in white short
horn cattle
Mare Bean Left  Uterine tubal patency test – phenolsulphonpthalene(PSP) dye test
Sow Mulberry Left  Follicular cyst – nymphonia (bullers), multiple in both ovaries,relaxation of sacrosciatic
ligament- upward displacement of coccyx – “sterility hump”
Bitch Oval  Luteal cyst – often single, anestrous, adrenal virilism

Mummification of fetus
 Ovarian hormones – Estrogen, Progesterone, Oxytocin ,Relaxin, Inhibin and Activin
 In cattle – hematic type – 3-8 months , papyraceous type occur in other species
 Oviduct – opening of infundibulum ―ostium tubae abdominal‖
 R/E - firmer,dryer leather like tissue with uterine wall without cotyledon
 Opening of utero tubal junction ―ostium tubae uterinum‖
RX
Uterus species
 PGF2 α -( Lutalyse,vetmate,iliren,dinofertin)
Bicornuate Cow, Ewe, Goat , Sow  Cattle – 25mg (total dose)

Simplex primates, humans Maceration of fetus


will occur at any stage – commonly 3rd month
Deciduate Bitch, Queen
 Trichomoniasis and vibriosis organisms invade the uterus cause infection and pus
Non-deciduate Cow, Doe, Ewe, Mare, Sow formation
 Dropsy of fetal membranes over all incidence 0.3%
Cotyledonary Ruminants  Hydroallontois ( 88%), most frequently encountered than hydroamnios (5-10%)

67 68
Dhruv N Desai Dhruv N Desai
 Hydroallontois - bloated bull frog like calf
 Uterine torsion – twisting or revolution of the gravid uterus on its long axis
 Signs of approaching parturition in mare – waxing of teat, patchy sweating  Post partum heat in pigs- 3 – 5 days
 Fetus decides the day of birth and dam decides the time of birth  At the time of deep freezing – 30 million sperms /ml
 Normal placental Expulsion time :  At the time of AI (post thaw) - minimum 10 million sperms / ml
 cattle - 8 - 12hrs,  Buck spermatozoa quite susceptible to cold shock
 Mare - 0.5 – 3hrs,  Buck semen – Presence of egg coagulating enzyme (Phospholipase A) prevents the
 sheep &goat - 3 – 6 hrs storage at 5 °c in yolk containing diluents
 Bitch – Placentophagy  Equilibrium of semen @ 5°c for 6 hrs to enable glycerol action
 Sow – Foetophagy  Sealing powder – poly vinyl alcohol
 During fetal expulsion – cow, Ewe, Doe – Sternal recumbency, Mare – lateral recumbency  Laboratory seal has to be cut during AI
 Post partum period – puerperium  Minimum of 10-15 million of viable sperms present after freezing and thawing in each
 Uterine involution completed by doses
 cattle - 26 – 52 days following parturition  The capacity of French mini straw – 0.25 ml
 Mare - 32 days
 Bitch - 4 – 5 week
 Post partum uterine discharge – Lochia
 Onset of estrus after parturition, cattle :- 33 – 90 days, buffaloes :- 4 – 6 months
 Foal heat – 5 -12 days post partum
 Bitch - the post partum Lochia is green colour is due to Uteroverdin – break down
product of Hemoglobin .
 Uterine incision is closed by – double row of lembert or cushing sutures
 Feeding sweet clover to sheep – Hyperestrogenisim can leads to uterine prolapsed.
 Downer cow – clinically parturient paresis but unable to rise after24 hours and two
calcium infusions
 Creeper cow – cow becomes alert and gains control following calcium injection but
remains recumbent due to inability to use hind quarters
 Synthetic analogues of GnRH – Buserelin(RECEPTAL) , Fertirelin(OVALYSE) ,
Gonadorelin (FERTAGYL)

Presentation P₁ Relation of the spinal axis of the fetus to that of dam. (eg;
longitudinal / transverse and anterioer / posterior )

Position P₂ Relation of the dorsum of the fetus in longitudinal


presentation or the head in transverse presentation to that of
quadrants of maternal pelvis.
(eg; dorso – sacral ,dorso-pubic etc., )

Posture P₃ Relation of the extremities or the head, neck and limbs of the
fetus to the body of its own. VII. LIVESTOCK PRODUCTION AND MANAGEMENT
(eg; shoulder flexion, hip flexion nape etc.,)
Four pillars of livestock management (or) LPM

1) Breeding 2) Weeding 3) Feeding 4) Heeding

 Turkey - Meleagris gallopavo


69 70
Dhruv N Desai Dhruv N Desai
 J. Quail - Coturnix coturnix japonica Goat 23
 Guirea foul - Numida meleagris
 Duck - Anas plathyrhynchos Equines 6
 Goose - Anser anser
 Watering of livestock
Common Terms and Definitions
 Horse
Species water intake /day
 Geld (or) gelding - castrated male horse.
 Broken horse - A well trained horse Cattle & buffalo 27 – 28 lit
 Unbroken horse - Untrained horse
Adult camel 70 – 90 lit
 Colt foal - Male young one
 Filly foal - Female young one Sheep & goat 18 lit
 Double rig - Cryptorchid (both testicles retained in the abdomen)
 Foaling - Act of giving birth to young one. Pigs 25 – 30 lit
 Mule - Mare x jack ass Poultry 250 ml
 Jennet/Jenny/hinny/Genet -stallion x she donkey
 Cattle Dog &cats 14 lit
 Heifer - Young female over one year, which has just attained maturity.
Horse 36 lit
 Slink calf - An aborted calf
 Bobby calf - Male calf about 1 week old.
 Free martin - Twin calves of different sexes are born
 The bull calf - Sexually normal . Species Water req.for all purposes / day
 female calf - Sterile (always)  Cow - 100 – 110 lit
 Sheep  Horse - 72 lit
 Wedder (or)wether - An adult castrated male sheep.  Pigs - 40 -50 lit
 Gimmer - Female sheep which is between 1 and 2 shearing.
 Seggy - an adult male castrated after service Potable water
 Frog – the central elevated portion behind the foot Standard physical qualities
 Chestnut – the horny growth situated below the hock on both the hind limb Organic matter 3ppm
 Hogging – clipping the mane PH range 7-8.5
 Pouring – pouring small quantity of dip into parts of the fleece along the back,sides and
belly Turbidity 5 turbidity scale
 Crutching – removing soiled dung-stained wool of Perineal and inguinal regions
Chemical qualities
 Scouring – removal of impurities in raw wool
Chloride, Sulphate 250ppm
 Mulling - castration by crude method
 Ringing – removal of wool from the region around the penis Fluoride 1ppm
 Eyeing – clipping of wool around the eye to prevent wool blindness
Species Number of defined breeds Ammonia
India Lead 0.1ppm
Cattle 28 Arsenic 0.05ppm
Buffalo 7 Iron 0.3ppm
Sheep 44
Hardness of water
71 72
Dhruv N Desai Dhruv N Desai
 temporary hardness – bicarbonates of calcium and magnesium  Wolf teeth – 1st pre molar of upper jaw in horse
 Permanent hardnes - Chlorides and sulphates of calcium and magnesium.  Dental star – a mark seen on the table surface of incisors in horse
 Chlorine demand for normal water – 0.9-1.8 ppm  Infundibulum – dark depression on the table surface of incisors in horse
 Brackish taste of water is due to presence of sodium chloride  Carnassials / sectorial teeth – in dogs.
 Sickle shaped horn – surti  4th cheek tooth of upper jaw ( 4th pre molar)
 Tallest Indian sheep breed – Nellore  5th cheek tooth of lower jaw (1st molar)
 Shortest Indian sheep breed – Mandya
 Galvayne‟s groove is a depression on the labial surface of the corner incisors
 Pelt breed – karakul
 Bishoping is an attempt to make the old animals to be mistaken for a young one
 Largest goat breed – jamnapari
 Dwarf breed of goat – Barbari
Dental formula
 Milk fat percentage highest in Jakffarabadi & lowest in Nili-ravi
species Temporary ( deciduous ) permanent
 Chegu and chanthangi are pashmina goat
 Gestation heat is also present in goat 2 (Incisors / canine / premolar) 2 (Incisors/canine/premolar/molar)
 Safe sanitary distance is 150-200 feet away from the sources of contaminations
Cattle/sheep/goat 0/4 , 0/0 , 3/3 20 0/4 , 0/0 , 3/3 , 3/3 32
 Glutaraldehyde (2%) aqueous solution used for sterilization of instruments
 Trap is a contrivance for preventing sewar gas escaping in to house drainage system Horse 3/3 , 0/0 , 3/3 24 3/3 , 1/1 , 3-4/3 , 3/3 40 - 42
 Presence of iron in water encourages the growth of iron bacteria such as crenothrix and
gallionella Pig 3/3 , 1/1 , 3/3 28 3/3 , 1/1 , 4/4 , 3/3 44
 Higher concentration of fluoride causes interference with calcification giving rise to 3/3 , 1/1 , 3/3 28 3/3 , 1/1 , 4/4 , 2/3 42
Dog
dental dystrophy known as mottled teeth
 Cooling power can be measured by kata thermometer Cat 3/3 , 1/1 , 3/2 26 3/3 , 1/1 , 3/2 , 1/1 30
 Air velocity 100ft/min at 70˚C is found to be comfortable for broilers
Camel 1/3 , 1/1, 3/2 22 1/3 , 1/1, 3/2 , 3/3 34
 Percentage of CO2 present in the atmosphere can be measured by Haldane‟s apparatus
 Short day breeders – sheep and goat
 Long day breeders - horse  Double dished face is characteristic of jersey and Guernsey
 Housing -East- west orientation – temperate regions  Golden yellow color milk is seen in Guernsey
 North – south orientation – tropical regions  Best milk production of world is Holstein Friesian
Identification of horse
 Key stone of arch in animal breeding – selection
 Grey – skin is black with admixture of black and white hairs
 Mass selection can be powerful for highly heritable traits
 Bay – varies from dull red to yellowish color, black mane, tail and the limb
 Piebald – irregular patches of white and black Species Sperm count/ml volume
 Star – a white mark on the forehead either large or small
 Stripe – a narrow white marking running down the face,may be thin or broad Bull 600-1200 million 2-10 ml
 Conjoined star and stripe – stripe in continuation of a star
Buffalo 600-1000 million 2-5 ml
 Blaze – a white marking covering almost the whole of the forehead between the eyes and
extending down the front of the face beyond the width of nasal bone and usually Ram 800-4000 million 0.6-2 ml
involving the muzzle
 White face – white covers the whole of forehead Stallion 50-200 million 30-280 ml
 Snip – any isolated white mark in between the nostrils Boar 25-1000 million 150-450 ml
 White muzzle – both lips will be white
 Whorls – any irregular setting of hairs
 Freeze branding - Dry ice – (- 70˚C) , Liquid nitrogen – (-196˚C)  Calf starter should be fed at 3 months of age (TDN -70%, CP -22%)
Teeth  Additional feeding during the pregnancy period – ‗Steaming up’
 Canine teeth absent in mare,cattle  Cows should be bred after calving within 60-90 days
 Tushes – canine teeth of pig  Ear notching is commonly practiced in pigs
73 74
Dhruv N Desai Dhruv N Desai
 Removal of testicles in fowl – Caponisation  The fiber from the Angora goat is known as Mohair
 Draught power of bullock – 0.75 HP  Fleece contain Suint and Grease
 Gestation period of goat is 145-155 days  Suint – water soluble salts present in the wool, which is excretory products from skin
 Best known Indian goat milch breed – jamnapari  The waviness of wool is known as crimp, fine wool will have more crimps
 Crude Fibre utilization - Goat>sheep>buffaloes>cows  Mutton – Pale pinkish
 In sheep flushing is practiced 2-3 weeks before mating  Chevon – dark red with coarse texture
 Age of ram for breeding purpose – 2 yrs
 Sheep tends to survive best in drier climates Floor space per animal (Sq.ft)
 At 20 wks of age, 16 hrs of lighting is required
Type of animal Covered area Open area
 Air movement should not exceed 30 ft (9.2m)/min
 For production of 1ml of milk 400-500ml of blood must pass through the udder Cows 20-30(3.5 m2) 80-100(7 m2)
 Major elements (Ca, P , K , Cl , and Na) cannot be changed by altering the levels of these
elements in the ration of a cow Buffaloes 25-35 80-100
 STH,ACTH,TSH and Oxytocin exert their effect in maintaining the normal lactation Young stock 15-20 50-60
curve
 Galactophore - a milk duct Pregnant cows 100-120 180-200
 Galactosidase - enzyme which catalyses the splitting of lactose into glucose + Galactose 2
Bulls 120-140(12 m ) 200-250(120 m2)
 Galactopoiesis – maintenance of lactation
 Lactogenesis – initiation of milk secretion Ram /Buck 3.4 m2
 Concentrate feeding – 0.35 kg per lit of milk
 Colostrums also known as Beesting Ewe /Doe 1 m2
 Best time for castration is 8-10 weeks for cattle Boar 9 m2 9 m2
 Deworming – with piperazine adipate with in 3rd to 7th day, repeat it once in a month
upto 6th month of age VIII POULTRY SCIENCE
 Calf mortality - below 8%
 BREED: group of individuals with in the species having distinct physical & productive
 Adult mortality – below 3% characteristics, which are efficiently transmitted to decendents
 Chemical used for shearing in sheep – Cyclophospamide  variety: subdivision of breed mostly decided by type of comb, colour of plumage
 Limiting amino acid of sheep - Methionine  Strain: population of small number of individuals in variety reproducing with well
 Dry matter requirement of sheep - 2.5 – 3 kg /head / day established common characteristics
 The only milk producing sheep breed (goat like sheep) – Sonadi Breeds
 Fineness of wool – expressed in terms of spinning counts (s)  Mediterranean class (Egg type): M L A (Minorca ,Leghorn , Ancona)
 Ratio of secondary to primary follicle in Fine wool breeds – 20 : 1  English class(Meat type): C O S A(Cornish, Orphington, Sussex, Australop)
Carpet wool breeds– 1:1 to 3:1  American class (Dual type): R P N W(Rhode islandred, Plymouthrock, New Hampshire,
 Diameter of Wool fiber – 15 – 50 µ Wyandotte) –
 Diameter of Kemp fiber – 100 – 200 µ  Asiatic class – Brhaman, Cochin,Langsharn
 Hair - Medulla is present Duck
Egg layers: Khaki Campbell, Indian runner
Type of wool Diameter S unit Meat ducks: white pekin, Aylsburry, Muscovy, Rouven
Sex ratio : Male:Female 1:15-16 - Replacement pullets
Fine wool <25 µ 64s to 80s 1:10-12 - broiler breeders
Family selection is useful in low heritability characteristics
Medium wool 25 – 40 µ 50s to 62s
 Low heritability characters egg production , fertility and viability
Coarse wool >40 µ <50s  Pedigree selection is used for sex limited traits
 Individual selection adopted for traits of high heritability,
 highly heritable characters – egg weight, shell quality, sexual maturity, growth rate, confirmation
75 76
Dhruv N Desai Dhruv N Desai
 Selection of birds for Layer Line – 10-14 weeks of age Intensive system – 10000-25000birds/ha
Meat Type Line – 8 weeks of age  Foul-patch – the ground immediately surrounding the houses- more danger of infection
 NAFED – National Agricultural Co-Operative Marketing Federation of India  Depth of litter – 5cm for chicks,7 -10cm for growers and layers
 In marketing of eggs, state level government organizations like MAFCO,TAPCO,POMFCO,NECC  The relative humidity in the deep litter system should be around 40%
and NAFED are making considerable efforts for marketing and sale promotion of eggs  The moisture content of litter should not be less than 18% and should not exceed >24%
 NECC – National egg coordination committee – fixes the prices for the eggs  The ammonia level produced by litter should not exceed 25ppm
 India – 3rd largest egg producer next to china & USA  Orientation of poultry houses – East-West direction
 Fertile egg – nucleus is called as Germ disc, infertile egg it is called as Germ spot
 Oviposition – act of laying, due to the release of Arginine and vasotocin Floor space requirement
 Brown color of egg shell is due to the pigment Porphyrin
 Blue shelled eggs – pigment Oocyanin Layers Broilers
 The normal depth of air cell is 4 to 8 mm
Age Space/bird(cm sq) Age (weeks) Space/bird(cm sq)
 Shell from outside covered by a layer of cuticle which is Bacteriostatic
(weeks)
 Shell membranes – 0.001 – 0.02 mm thick
0-7 650-675 0-4 450-470
 Shell – 11 % of total egg weight Deep litter
 Albumen – 58 % of total egg weight 8-11 900-925 5-7 750-850
 Yolk – 31 % of total egg weight
12-19 1800-2000
 Ovomucin – responsible for firmness of thick albumen
0-8 200-250
 Oviduct Cage system
 Infundibulum - fertilization of ovum, the yolk stays for about 15 min 9-20 275-300
 Magnum – major qty of thick albumen secreted here , materials stay about 3 hours
20 & above 337-375
 Isthmus – 1.25 hrs, egg white,2 shell membranes, some salt and water is added to egg
 Uterus – major role in egg formation, hard calcareous shell, shell pigment, some minerals& water
along with cuticle deposited, egg spends max time 21 hrs at this place  Restricted feeding- increases the size of initial eggs laid and is an important factor to regulate the
 Vagina – egg just passes without spending time size of eggs
 24 – 26 hrs required for formation of an egg  Egg-borne transmission (Trans ovarian diseases) - Salmonellosis , Mycoplasmosis, Avianleucosis
 Haugh unit(HU) – Evaluating albumen quality, the HU of good quality egg – 70 complex, Ranikhet disease , Infectious Bronchitis, Avian Encephalomyelitis,avian Adeno virus
 Temperature Egg holding room 18 – 20°c infection, IBH(inclusion body hepatitis), EDS-76, Fowl typhoid
 Physiological Zero – to arrest the development of embryo before setting at 75-80 % humidity  Mottled yolk – Due to coccidiostat, hot weather, gossypol poisoning
 Fumigation – 1x – 40ml of formalin with 20g of Kmno4/2.80m3  Blood spot – Vitamin A deficiency
 Incubator - temp-37.5- 37.8°C ,65-70% humidity  Brooder pneumonia – Aspergillus fumigates
 Hatcher – temp- 36.5 – 36.8°C, 75 – 80% humidity  Gape worm (Forked worms)– Syngamus trachea
 Incubation period – 20-21 days  Vaccine - Drinking water administration – For 10 liters of water 1kg of ice and 60g of skimmed
 Brooding management – up to 4 weeks – broilers, 6-8 wks – layers milk powder is used
 Brooding space – 50-66 cm2/chick, temperature – 33°C during first week,2.6°C reduced every week  The RH of poultry house should range from 45-75%
till reaches 21°C  Hatch weight of broiler chick - 35 – 40 g
 Debeaking - generally done twice in egg type chicken – Day old & Around 9th day or at 3-4 weeks  Chicks must remain in continuous lighting up to 8 wks of age
of age
 Toe-clipping – breeding males – 6-9 days of age NUTRITION
 Dubbing – removal of comb , around 7-8 weeks of age  Supplemental nitrogen: sulphur necessary in the ratio of 10:1
 Cropping - removal of wattles  Cereal grains are deficient in lysine and tryptophan
 The average stocking density of adult birds  Fish meal is rich source of lysine, tryptophan and methionine
Free range - 250birds/ha  Piglets highly susceptible to iron and cobalt deficiency
Semi intensive – 750 birds/ha  Piglet anaemia(thumps)
77 78
Dhruv N Desai Dhruv N Desai
 symptoms - Pale in the region of ears and belly, Listlessness, Rapid breathing, often 11. The bacteria that is used to evaluate the phenol coefficient using Rideal Walker method Salmonella
Diarrhoea .
 RX - 100-150 mg of iron in the form of iron dextron – 3 days after birth if necessary a second typhi
inj.-3 weeks later
12. Mastitis causing Str.agalactiae and Str. dysgalactiae are classified as group - B and C --- and group--
 Weight at weaning age is proof of efficient growth and also an indication of the milking ability of
the gilt A is S. pyogenes respectively as per Lancefield classification.
 Creep feeding given from 3rd week onwards (25-30% CP)
 Yellow maize is rich in cryptoxanthine 13. The substance present normally in spores at high levels,but decreases during the favourable
 Restricted feeding produces better quality meat
condition. Calcium Dipicolinate
 Cotton seed meal is known for its efficiency to produce hard and firm meat
 Feeds like ground nut, maize, rice bran, vegetable oils, etc when fed in liberal will result in soft 14. Greyish-white medusa head type of colony is shown by Bacillus anthracis in which medium ? .
pork
 Maize as a cereal deficient in calcium Nutrient Agar
 Choline and methionine are needed to supplement to counteract the toxicity resulting from tannin
15. Chinese letter arrangement and metachromatin granules are features of Corynebacteria
 Rice polish – rich in thiamine and higher in niacin and riboflavin
 Wheat bran – 12% fibrer rich in phosphorus and poor in calcium 16. Agent that causes Summer Mastitis Corynebacteriapyogenes
 Blood meal - >80% protein
 Meat meal – 50-55 CP 17. Growth of E.rhusiopathiae is favoured by which aminoacid ? Tryptophan
 Blood meal is deficient isoleucine
18. Tuberculous lesions are prominent in digestive tract rather than in respiratory tract in Poultry
 Ground nut cake 40-50% protein
 Feather meal 5% inclusion level 19. Etiological agent of Calf Diphtheria Fusobacterium necrophorus

20. Characteristic features of abortion in cattle caused by B.abortus Necrotic placentitis and

Leatheryplacenta

MICROBIOLOGY 21. Type of vaccines used against brucellosis in calves & cows. Strain 19 (living) and Strain

45/20(killed) respectively

1. Koch‘s postulates was derived by using which bacterium ?Bacillus anthracis 22. Kennel Cough in dogs caused by .. Bordetella bronchiseptica

2. Kanagawa reaction is exhibited by...Vibrio parahaemolyticum 23. Pasteurella, Yersinia and Listeria have one thing in common as part of their staining character.

3. Father of Microbiology Louis Pasteur Bipolar staining

4. In presence of specific antibody, Streptococcus pneumoniae shows --- Quellungreaction reaction 24. Etiological agent of fowl coryza Haemophilus gallinarum

5. Father of Bacteriology Robert Koch 25. Classification of Pasteurella species Robert's and Carter's serotyping

6. Small pox vaccine was developed by --- Edward Jenner -----in the year1796. 26. The best medium for an enhanced growth of Campylobacter Thiol medium

7. A polymer of glycerol phosphate that is present only in G+ bacteria cell wall Teichoic acid 27. In Mc Konkeys agar, E.coli produces - Pink colonies whereas Salmonella produces -Colourless

8. Rabies vaccine was first done on Joseph Meister 28. Ringer and Gillespie medium is used for the growth of .. Leptospira

9. Loeffler and Frosch shares the credit of discovery of---- FMD Virus -- 29. ‗Symptomatic anthrax is the synonym for.... Black Quarter

10. The only anti TB drug that has the ability to destroy the acid fastness of Mycobacterium. Izoniazid 30. Para anthrax in pigs is caused by .Clostridium septicum

79 80
Dhruv N Desai Dhruv N Desai
31. Gaint cells of Langhans are absent in T.B affecting which species Canines and Felines 1. Enzymes involved in hatching of Ascarid egg: Chitinase and esterases

32. In H&E staining T.B calcification appear as... Blue color. 2. McLean counting system is devised for : Ascaris suum

33. Epitheloid cells fuse to form syncytia and it enters ..Symplasma....stage in Johne‘s diseases. 3. An ascarid without somatic migration: Toxascaris leonina (A. galli belongs to Family heterakidae)

34. Among domestic species ... Sheep. is most susceptible to anthrax. 4. Herring worm: Anisakis

35. Condition in sheeps under 1year of age, affected by Cl.septicum due to toxaemia Braxy or Bradsot 5. Cod fish worm: Phoconema

36. Pulmonary Adenomatosis in sheeps by retrovirus is ........ whereas Cl.botulinum type D infection in 6. Hourglass shaped esophagus in : Oxyuris equi

cattle is ..... Jaagsiekte ; Lamsiekte 7. Hourglass shaped buccal capsule found in : Oxyspirura mansoni

37. Dunkop and Dikkop are forms of ...... African Horse Sickness. 8. Funnel shaped pharynx: Haebronema megastoma

38. Diagnostic test for E.I.A Coggin's Test 9. Cup shaped buccal capsule with cusp shaped teeth: Stephanurus dentatus

39. Inclusion bodies in Fowl pox is ...... and in cow pox is.... Bollinger bodies ; Guarnieri bodies 10. Parasite responsible for ―ungroomed rat tail appearance‖ in horse: Oxyuris equi

40. Instrument used to perform the Polymerase Chain Reaction Thermocycler 11. Caecal worm of poultry: Heterakis gallinae

41. Ulcerative enteritis in poultry caused by ........Clostridium colinum 12. Nematodes with ―H‖ shaped excretory system: Rhabditidae

42. Infectious encephalomyelitis caused by Flavi virus transmitted by ixodes ricinus. Louping ill 13. Characteristic ―ear‖ shaped (dorsal) tooth in : Strongylus vulgaris

43. Granules present within the Guarnieri body. Paschen's granules 14. ―Morocco leather‖ appearance associated with: Ostertagia ostertagi

44. Synonym for Infectious bulbar paralysis caused by Herpes. Psuedorabies/Mad Itch/Aujezky's 15. ―Ring worm like lesions‖ associated with: Trichostrongylus spp

disease 16. ―Horse shoe‖ shaped ovary: Echinococcus granulosus

45. Two medium used for the growth of mycoplasma. PPLO Agar and Frey's medium 17. ―Boot‖ shaped spicule: Dictyocaulus filariae

46. In McFaydean reaction ,color of organism and capsule Blue ; Pink 18. ―Heart‖ shaped spicule: Nematodirus baltus

47. Bursitis in horse caused by Brucella abortus Poll Evil and Fistulous Withers 19. ―Lancet‖ shaped spicule: Nematodirus fillicolis

48. The etiological agent of ‗Struck‘ in sheep Clostridium perferinges Type C 20. ―Spoon‖ shaped spicule: Nematodirus spathiger

49. Bottle brush appearance in Gelatin stab is growth feature of .. . Erysipelothrix rhusiopathiae and 21. ―Y‖ shaped dorsal ray: Haemonchus contortus

Clostridium perferinges 22. Recurved spicules: Gaigeria pachyscelis

50. Visna/Maedi in sheep is caused by......... Retro virus 23. No spicule: Trichinella spiralis

24. Black scours worm:Trichostrongylus colubriformis

25. Barber‘s pole worm/ wire worm/ large stomach worm/ twisted stomach worm: Haemonchus
PARASITOLOGY
contortus

26. Eyeworm of poultry: Oxyspirura mansoni

81 82
Dhruv N Desai Dhruv N Desai
27. Brown stomach worm: Ostertagia ostertagi 53. Radia with ―procruscula‖: Fasciola spp

28. Red stomach worm of pig: Hyostrongylus rubidus 54. Nematodes with flame cells: Class Acanthocephala (Macracanthorrhyncus hirudinaceus)

29. Fox hook worm: Uncinaria stenocephala 55. Halzoun syndrome associated with: Fasciolosis and spirometrosis

30. Pig hook worm: Globocephalus (G. urosubulatus, G. longimucornatus) 56. Cestode with ―sickle‖ shaped hooks: Taenia spp

31. Elephant hook worm: Barthomostomus (B. sangeri), Grammocephalus clatheratus 57. Cestode with ―rosethorn‖ shaped hooks: Dipylidium caninum

32. Lungworm of dog: Filaroides osleri 58. Cestode with ―Hammer‖ shaped hooks: Davinia spp

33. Lungworm of cat: Aleurostrongylus spp 59. Cestode with ―bunch of grape‖ ovary: Dipylidium caninum

34. Lungworm of rat: Angiostrongylus cantonensis- causes ―eosinophilic meningio encephalitis‖ in 60. Metacestode tetrathyridium is seen in: Family Mesocestoides

man 61. Metacestode strobilocercus seen in: Taenia taeniformis (as Cysticercus fasciolaris)

35. Eddy worm: Class Turbellaria 62. ―Lapets‖ present in: Anoplocephala perfoliata

36. Larva with ―S‖ shaped tail: Filaroides osleri 63. ―Dumbbell‖ shaped uterus: Stilasia hepatica

37. Nurse cells characteristic of Trichinella spiralis 64. Fringed tape worm: Thysanosoma actinoides

38. ―Stichosomes‖ are characteristic of: Trichurid esophagus 65. Gravid uterus is replaced by egg capsule in: Family Linstowiidae

39. ―Cordons‖ in: Ascaridae 66. Cooked rice grain appearance: monezia gravid segments

40. ―Bosses‖ in: Gongylonema 67. Cucumber shaped segments: gravid segments of Dipylidium caninum

41. Cuticle extended posteriorly beyond the tail of worm: Physaloptera spp

42. Bursa strengthened with chitinous plate: Protostrongylus spp


Following diseases/conditions are associated with parasites
43. L1 with characteristic cuticular knob: Dictyocaulus filariae
1. Milk spots: Ascaris suum
44. L1 with button hook tail: Dipetelonema dracunculoides
2. Balling up in horse:Parascaris equorum
45. Anterior helmet seen in: Dracunculus medenensis
3. Mud colour faeces: Toxocara vitulorum
46. Definitive host of Dioctophyma renale: mink
4. Rat tail appearance: Oxyuris equi
47. The dish ―Fessikhs‖ is associated with: Heterophyses heterophyses
5. Black head: Heterakis gallinae (Egg carrier of Histomonas meleagridis)
48. The dish ―Marrara‖ is associated with: Sparganosis
6. Parasitic otitis: Rabditis bovis
49. Phenomenon of ―progenesis‖ is associated with: Family Plagyorchidae
7. Black scours: Trichostrongylus worms
50. ―Furcocercus cercaria‖: Schistosomes
8. Villous atrophy: Trochostrongylus and Nematodirus
51. ―Microcercus cercaria‖: Paragonimidae
9. Ringworm lesions: Trichostrogylus
52. ―cercaria vitrina‖: Dicrocelium dendriticum
10. Morocco leather: Ostertagia ostertagi
83 84
Dhruv N Desai Dhruv N Desai
11. Pulpy kidney disease (withCl. welchi): nematodirus 38. Macrocytic / pernicious anemia: Diphyllobothrium latum

12. Swimmer‘s itch: Schistosoma spp (non human) 39. LD bodies: Leishmaniosis

13. Foot rot in sheep: Strongyloides papillosus 40. KB bodies: Theileriosis

14. Pimply gut: Oesophagostomum spp 41. Visceral Leishmaniosis: Leishmania donovani, L. chagasi, L. infantum

15. Colic in horse: Cythiostomum tetracanthum 42. PKDL: L. donovani

16. Haemorrhagic warts (in tracheal bifurcation): Filaroides osleri 43. Kala azar: L. donovani

17. Fistulous whither: Onchocerca cervicalis 44. American kala azar: L. chagasi

18. Eosinophilic meningeo encephalitis in man: Angiostrongylus cantonensis 45. Rural zoonotic leishmaniasis: L. major

19. Wahi /kaseri/ summer mange: Onchocerca spp 46. Chiclero ulcer/ bay sore: L. mexicana mexicana

20. Bursati / granular dermatitis/ summer sore: Habronema (cutaneous habronemiasis) 47. Classical espundya: L. braziliensis braziliensis

21. Arteritis in horse: strongylidae family 48. Uta: L. peruviana

22. Oesophageal tumour: Spirocerca lupi 49. Nagana: Trypanosoma brucei, T. congolensi, T. vivax

23. Gastric tumour: Gnathostoma spinigerumand Habronema megastoma (Draschia megastoma) 50. Souma: T. vivax in cattle

24. Cholangiocarcinoma: Clonorchis sinensis(oriental liver fluke/Chinese liver fluke) 51. African sleeping sickness: T. brucei gambiensi, T. brucei rhodasiensi

25. Urinary bladder carcinoma: Schistosoma haematobium 52. Surra: T. evansi

26. ―Swine fever‖ and epizootic pneumonia: Metastongylus spp 53. Tibarsa /Gufar: T. evansi in camel

27. Husk or hoose: Dictyocaulus viviparous 54. Mal de Cadares: T. equinum

28. Enzootic cerebrospinal nematodiasis: Setaria digitata 55. Dourine / equine syphilis: T. equiperdum

29. Hump sore: Stephanofilaria assamensis 56. Dollar spots: T. equiperdum

30. Ear sore: Stephanofilaria zaheeri 57. Yellow buttons: Trichomonas gallinae (Avian trichomonosis)

31. Fatal hemorrhagic enteritis in mink: Euryhelmis squamula 58. Saucer shaped ulcer in tissue: Histomonas meleagridis

32. Rot dropsy: Fasciola spp 59. Suphur yellow faeces: Histomonas meleagridis

33. Snoring in cattle: Schostosoma nasalis 60. Travellers diarrhea: Giardia lamblia

34. Nodular taeniasis in poultry: Reilettina echinobothrida 61. Flask shaped ulcer: Entamoeba histolytica (in intestine)

35. Hepatitis cysticercosa: Cysticercus tenuicollis 62. Red dysentery: Eimeria zuernii in cattle

36. Gid/Staggers: Coenurus cerebralis (of Taenia multiceps) 63. Rectal coccidiosis: Eimeria burnetti

37. False gid: Oestrus ovis (larva) 64. Ladder lesions in duodenum: Eimeria acervulina

85 86
Dhruv N Desai Dhruv N Desai
65. Signet ring: Plasmodium spp 15. Vitamin C is used as an antidote to ---Nitrate-----poisoning

66. Texas fever/Red water fever/Bovine pyroplasmosis: Babesia spp in cattle 16. Oseltamivir is the drug of choice against- Bird flu in humans

67. Equine biliary fever: Babesia equi (now as Theilaria equi) 17. The species of animal in which Ivermectin crosses the BBB- Equines

68. Tropical bovine theileriosis: Theileria annulata 18. The only anaesthetic agent known to be carcinogenic- fluoroxene

69. Benign tropical thieileriosis: T. mutans 19. The drugs contraindicated in parakeets & cats respectively are- procaine & morphine

70. East coast fever/ January disease: T. parva 20. The species most sensitive to the ill-effects of xylazine - cattle

71. Buffalo disease/Corridor disease: T. lawrensi 21. A pantropic virus that commonly affects Canines- Canine distemper virus

72. Malignant theileriosis: T. hirci 22. Symplasma stage in submucosa is seen in- Johne's Disease

73. Gall sickness: Anaplasma marginale 23. Oestrogenic mycotoxin causing reproductive disorders in swine is- Zearalenone

24. 'Blue eye' or Rubarth's Disease is the synonym for- Infectious Canine Hepatitis

25. Which is the most potent opiate analgesic? carfentanil

ICAR VET REFRSHER 26. Name a benzodiazepine antagonist- flumazenil

27. Ketamine is contraindicated in head injuries as it reduces -Intra cranial pressure

1. Bacterial disease in which Pasteur's vaccine used- Anthrax 28. The subtype of Avian flu virus causing human casualties around the world- H5N1

2. 'Para anthrax' in pigs caused by- Cl. septicum 29. Mad cow disease is caused by- Prions

3. Foot rot in sheep is caused by - Bacterioides nodosus 30. Cold enrichment procedure is done for the isolation of - Listeria

4. Duck Plague is the synonym for –Duck Viral Enteritis 31. 'J' chain is present in immunoglobulins- IgA and IgM

5. The indistinct margins of a radiograph due to a large focal spot of X-ray beam -penumbra. 32. In the body, Chloral hydrate is converted to - tricholoroethanol

6. The optimum temperature of developing and fixing solutions in radiography is -68F 33. Name one anaesthetic agent which is steroid in nature- Althesin

7. Avian influenza virus has 8 gene segments 34. Dose of Anthrax vaccine- 1ml s/c

8. M. leprae can be cultivated in vivo only in- Armadillo 35. Anaphylatoxins are- C3a and C5a

9. State of unresponsiveness towards an antigen is - Anergy 36. The specific antidote of Morphine is- Nalorphine

10. The cholinergic drug that cannot be hydrolyzed by AchE is -carbachol 37. Dunkop (pulmonary) and Dikkop (cardiac) are two forms of - African Horse Sickness

11. The penetrating power of X-ray beam depends on - kvp 38. 'Wire loop' lesions in the glomerular basement membrane seen in- SLE

12. Anaesthesia produced by combination of drugs is termed as-Balanced anaesthesia 39. The drug used to stabilize mast cells- Cromolyn sodium

13. Antigen-Antibody complexes present in dentrites of Dentritic cells- Iccosomes 40. Hyaluronidase-- enzyme used with local anaesthetics to promote its diffusion and absorption

14. BCG is an attenuated form of - Mycobacterium bovis 41. Milbemycin oxime is obtained from-Streptomyces hygroscopicus

87 88
Dhruv N Desai Dhruv N Desai
42. Name a suture material that glows in darkness easy to handle in poor light- Flurofil 69. Periarticular lymphoid sheath (PALS) is populated by T lymphocytes are seen in-Spleen

43. Cardinal Signs of Inflammation was propounded by- Cornelius Celsus 70. Habel's testing (mouse test) is done for the diagnostic assessment of- Rabies

44. FMD virus multiplies in the ------Str.spinosum--------- layer of epidermis 71. Most immunogenic viral polypeptide of FMD virus is - VP1

45. Father of Immunology is- Edward Jenner 72. Rabbits show genetic tolerance towards- Atropine

46. Lamsiekte in cattle and sheep is caused by - Cl.botulinum type D. 73. The diuretic that cannot be given along with aminoglycoside antibiotics- Furosemide

47. Vaccine strain of B.anthracis is - Aviurlent and Non capsulated 74. The antibiotic Gentamicin is obtained from- Micromonosporum purpureum

48. Monsell‘s suture technique is used in - Enteroanastomosis 75. Ochratoxin primarily causing renal impairment is produced by- Aspergillus ochraceus

49. Ethylene oxide is a gaseous agent used for sterilization kills microorganisms by-alkylation.. 76. Name the smallest animal virus- Porcine circovirus (17-20nm)., FMDV is of 28-30nm size.

50. VitC is necessary for hydroxylation of proline & lysine in synthesis of -collagen 77. Pyometra in bitches is caused by –E.coli

51. 'Bomb burst' or 'Umbrella like' colonies are characteristic of - Listeria 78. Strawberry foot rot is caused by-Dermatophilus congolensis

52. Influenza virus is typed based on Matrix and Nucleocapsid antigen 79. Drug of choice for mycoplasmosis- Tylosin

53. The conversion of DHFA to THFA is blocked by- Trimethoprim 80. 'Tennis racket' shaped spores are present in- Cl. chauvoei

54. Examples of pencillinase resistant pencillins- Methicillin and Cloxacillin 81. Psittacosis or Ornithosis in birds is caused by- Chlamydophila psittaci

55. First immunoglobulin to be synthesized in Neonates- IgM. 82. The proton pump inhibitor used to treat Zollinger-Ellison syndrome-Omeprazole

56. 'Abortion storms' in sheep is caused by- Campylobacter foetus 83. Rabies virus (bullet shaped) belongs to the genus- Lyssa virus

57. Vaccine strain used for prophylaxis of IBD infection-Georgia 84. 'Bull Nose' in pigs is caused by- Fusobacterium necrophorus

58. Which is the only benzimidazole drug that wont inhibit Fumarate reductase- mebendazole 85. DNA virus that codes for reverse transcriptase enzyme- Hepadna virus

59. Name an anthelminthic which was previously used to treat human gout- piperazine 86. Chronic Respiratory Disease in birds is caused by - Mycoplasma gallisepticum

60. The experimental animal used for FMD research is - Guinea pig 87. Immediate precursor of all sex steroids- Pregnenalone

61. The no: of Capsomers in Adenoviruses is- 252 88. In avian tuberculosis the lesion are confined to - GI tract

62. The source of Vero cell line is - African Green Monkey (Kidney) 89. Name a commonly used AchE reactivator- Pralidoxime.

63. Heat resistant ability of sporulated bacterium is due to the compound-Ca dipicholinate 90. Piperonyl butoxide is used as a synergist along with -Pyrethroids

64. The generation of monoclonal antibodies involves-Salvage pathway 91. HVT is the vaccine strain used against- Marek's Disease

65. Antibiotic which can also function as an anthelminthic agent is- Hygromycin B 92. In B.abortus, the ratio between LPS antigens A and M is - 20:1

66. Ivermectin toxicity can be reversed by using - Picrotoxin 93. Kume and Page Scheme is used for the classification of- Haemophilus

67. Cucurbitin, an active principle in Pumpkin seed is used for the treatment for-Cestodiasis 94. 'Reverse Genetics' is nowadays used for the production of vaccine strains of - Avian flu virus

68. Name a live vaccine previously used against rabies- Flury’s LEP & HEP 95. Drug of choice against Theileriosis is -Buparvaquone

89 90
Dhruv N Desai Dhruv N Desai
96. With H&E staining the calcified tissue appears -Blue 123. Bacillus anthracis have medusa head like colonies in -Nutrient agar

97. The toxic level of Aflatoxin in ducklings is- 0.03ppm 124. Clostridium perfringens type A in animals causes- Gas gangrene

98. Summer Mastitis in cattle is caused by -C. pyogenes 125. The class of immunoglobulin that first appears in primary immune response- IgM

99. Heart Water disease is caused by- Cowdria ruminantium 126. Polypeptide chain called secretory component is present in- IgA

100. EMJH medium is commonly used for the cultivation of -Leptospira 127. Docking in dogs can predispose to a conditon called- Perineal Hernia

101. Hoti's test is used for detecting -Str. agalactiae mastitis 128. Hoflund's syndrome is the synonym for- Vagal indigestion

102. Infectious Bulbar Paralysis is the synonym for- Psuedorabies 129. Vitamin K dependent clotting factors are- 2, 7, 9 and 10.

103. Sulphonamides are metabolized by acetylation in ruminantsand glucuronidation in canines. 130. Limber neck in poultry is caused by- Clostridium botulinum type C

104. Scythe shaped spleen is seen in -Horses 131. The amino acid tryptophan act as growth promoter for the microbe- Erysepelothrix

105. The most important symptom of anthrax in dogs is -Gastroenteritis. 132. The most potent Aflatoxin is- B1

106. Braxy in sheep is caused by -Cl. septicum. 133. The cytolytic product of CTL cells that forms transmembrane pores in target cells- Perforins

107. The microbes that have the ability to survive pasteurization temp is -Listeria and Coxiella 134. Autoimmune disease in which Ig's are formed against Ach receptors- Myasthenia gravis

108. Techoic acid is present in the cell wall of - Gram + bacteria 135. World's first veterinary school in - 1762, Lyons, Paris

109. Route of inoculation in chicken embryos is intravenous for Blue tongue virus 136. The dog breed that has genetic predisposition for skin tumor- Boxer

110. 'Darling disease' is caused by - Histoplasma 137. Gavard's muscle is the synonym of -Int.obl. muscle layer of stomach.

111. New Jersey, Indiana and Trinidad are strains of - Vesicular stomatitis virus 138. Ventral bending (concave) of the spinal column is called- Lordosis

112. Diene staining is used for - Mycoplasma 139. A and M antigens are absent in- B.canis & B.ovis

113. Aspergillus flavus in SDA produces-Yellowish green colonies 140. Foothill abortion / BEA in cattle is caused by- Chlamydophila psittaci

114. Cork screw motility is shown by -Campylobacter 141. The antibacterial system naturally present in milk is- Lactoperoxidase system

115. Turkey coryza which is highly contagious is caused by - Bordetella avium 142. Marsupialization in bitches is done usually to treat- Pyometra

116. Castanida and Machiavello staining are employed for detecting- Chlamydia 143. Dose of Ivermectin is – 200 micrograms/Kg body weight

117. The antibodies used against the Rh antigen to prevent Erythroblastosis foetalis- Rhogam 144. The breed of cattle which has strong predisposition for Eye Cancer- Hereford

118. The immunoglobulin known as 'Reagin'- IgE 145. Male dog urinates like bitch in - Cystitis

119. Macrophages present in Kidney are known as- Mesangial cells 146. Knott test is done to detect Dirofilariosis in dogs

120. Interleukin that suppresses the immune response is - IL-10 147. Half life of IgG is - 20-21 days

121. Etiological agent of silage disease is – Listeria. 148. Benign tumor of gingiva is called - Epulis

122. Smallest living organism of individual existence- Mycoplasma 149. The drug used to contract gall bladder (cholecystokinetic)- Ceruletide

91 92
Dhruv N Desai Dhruv N Desai
150. The type of paralysis produced by the drug piperazine on worms is – flaccid 177. ―Paple‖ shaped abdomen is diagnositic of – Vagus Indigestion

151. Cart wheel shape chromatin is seen – Plasma cells 178. Tenesmus followed by bloody dysentery in calves is characteristic of – Eimeria zuernii

152. The most abundant buffer system in plasma- bicarbonate buffer 179. The coccidial organism commonly found in felines and canines – Isospora (Eimeria absent)

153. The smallest animal cell has a diameter of – 2 microns 180. ―Slime balls‖ ie., cercarial aggregation seen in- Dicrocoelium infection.

154. The Fc fragment of Ig's can be recovered by digestion with the enzyme- Papain 181. ―Grunt‖ on applying pressure on xiphoid region in cattle is indicative of- TRP

155. Most common respiratory pathogen in canines - Bordetella bronchiseptica (kennel cough) 182. Salmon poisoning in dogs caused by -Neorickettsia helmintheca

156. Rabbit ileal loop assay is commonly done for the detection of - ET E.coli 183. Antibody having least half life is - IgE

157. Haemo-lymphnodes are commonly seen in - Ruminants 184. Genetically mutant mice lacking NK cells is called- Biege (Athymic mice- 'Nude')

158. Clonal selection theory of antibody production was proposed by- Burnet 185. Salivary cyst found in sublingual duct is- Ranula

159. Biphasic fever in dogs is indicative of - Canine distemper 186. Membrane bound IgM is a - Monomer

160. Ramstedt's surgical procedure is performed to correct- Pyloric stenosis 187. Class II MHC restricted cells are- T helper cells

161. The immediate precursor of thrombocytes are- Megakaryocytes 188. Lyme disease is caused by – Spirochete, Borrelia burgdoeferi, (IH- Ixodes)

162. Rouleaux formation in blood smear is a common finding in – Felines 189. Antidote for warfarin toxicity is – Vitamin K

163. Substance required for platelet aggregation is – Thromboxane 190. The drug that intensifies the toxic effects of Warfarin is – Phenyl Butazone

164. The interleukin commonly referred as chemokine is - IL-8 191. The amino acid which is deficient in cats is - Taurine

165. Rectal pinch test is done for the diagnosis of - Johne's disease 192. Complement activation is predominantly mediated by - IgM

166. Surgical cat gut is often sterilized by- Isopropyl alcohol or ethylene oxide 193. The compound used as gastric sedative in dogs is - Chloretone

167. Giant kidney worm of Dogs is - Dioctophyme renale 194. The receptor for co-stimulating B7 molecules on APC is- CD28

168. The sedimentation coefficient of IgG is 7S and IgM is 19S 195. Enteroplication is the surgical technique to correct- Intussusception

169. Principal metabolic pathway in RBC is- Glycolysis 196. The antibody that can exist as monomer, dimer, trimer and tetramer- IgA.

170. Tyzzer‘s disease in foals and lab animals is caused by – Bacillus piliformis 197. Insulin like growth factor –I is also called as – Somatomedins

171. Avian spirochetosis is caused by – Borrelia anserina 198. The diuretic with Aldosterone antagonistic action is – Spironolactone

172. Colopexy is used to treat- Recurrent rectal prolapse 199. The drug used to experimentally induce diabetes in dogs- Alloxan

173. Mouse ascites method is used to production of - Monoclonal antibodies 200. Allopurinol is the drug of choice for the treatment of – Gout

174. The dog breed having genetic predisposition to cardiac hypertropy- Grey hound 201. In passive HA, chemical used to coat antigens in RBC- Tannic acid or chromium chloride.

175. Suture technique used for uterine stump closure- Parker kerr method 202. Infectious RNA molecule of low molecular weight comes under the category- Viroids

176. In humans MHC is referred to as HLA complex whereas in mice it is H-2 complex 203. Pink eye is caused by Moraxella bovis and Summer pink eye is caused by IBR virus

93 94
Dhruv N Desai Dhruv N Desai
204. 'Dew drop' colonies and satellite phenomenon is exhibited by- Haemophilus 231. The dose of Heparin used as an anticoagulant is- 10-20 IU/ml

205. 9R is the vaccine strain of - Salmonella Gallinarum 232. The media used to select the myeloma cells in MAB technique is- HAT medium

206. Glaucoma, a condition of increased intra ocular pressure is treated using- Acetazolamide 233. The site present in an antibody to which an antigen binds is called- Paratope

207. Canrenone is the metabolite of – Spironolactone 234. The vector for Reoviral Blue tongue in sheeps is- Culicoides spp

208. Kanagawa reaction is shown by – Vibrio parahaemolyticum 235. ―Pizzle rot‖ (Ovine Posthitis) in sheeps is caused by- Corynebacterium renale

209. SMEDI in pigs is caused by – Parvo Virus 236. Gid, Sturdy, or Staggers is caused by-Coenurus cerebralis ( T. multiceps)

210. ―White Spotted‖ kidney is a sequelae to – Leptospirosis and E.coli infections 237. The most commonly used serological test ELISA was developed by- Engvall and Perlman

211. Infarcts in kidney of Swine is characteristic of – Erysipelas 238. The gene for virulence in ND virus and AI virus are-Fusion gene and HA gene (respectively)

212. In GI tract, the antigen transport is carried out by specialized cells called- 'M' cells 239. 'Tigroid heart' in calves is characteristic of- FMD

213. Infectious protein particles causing 'Scrapie' in sheep are called- Prions 240. Vascularization of Cornea is known as - Pannus

214. The penicillin which is effective against pseudomonas infection- Carbenicillin 241. The lesion in eyes of horses due to leptospiral infection - Periodic Ophthalmia

215. Burton's line in gums is indicative of - Lead poisoning 242. ―Blue eye‖ is the synonym for- Infectious Canine Hepatitis (adenovirus)

216. The drug that can replace Ivermectin in sensitive dogs- Milbemycin oxime 243. ―Hot Spot‖ (pyotraumatic dermatitis) is caused by- Staphylococcus aureus & S. intermedius.

217. Jaagsiekte ― Driving Sickness‖ in sheeps is caused by Retro virus 244. The interleukin often referred to as B cell growth factor is- IL-6

218. Jaagsiekte is Pulmonary adenomatosis and Visna-Maedi (Retro) is Progressive pneumonia 245. FMD virus belongs to the genus- Aphthovirus

219. Farmer‘s Lung in cattle is caused by - Micropolyspora faeni 246. 'Dropped Sole' in horses is a condition due to- Chronic Laminitis

220. The virus having a unique 'double capsid'- Reovirus 247. Feline pan leucopenia is caused by- Parvo virus

221. Viral etiology of neoplasms were first reported by- Ellerman and Bang 248. Avian Influenza is caused by Influenza A virus of the family- Orthomyxoviridae

222. The group specific antigen of ALV which is commonly used for COFAL test is -p27 249. The principal source of Interferon β is - Fibroblasts

223. Recombinant DNA technology was first developed by- Cohen and Boyer 250. SAT-1,2 and 3 are types of FMDV originated from- Africa

224. Ephemeral fever (3 day sickness) is caused by - Rhabdovirus 251. The interleukin responsible for class switching of IgM to IgG is - IL-4

225. ―Facial Eczema‖ in cattle is caused by Pithomyces chartarum (fungus with toxin sporidesmin) 252. The animal species that serves as ― mixing vessel‖ for Avian and Mammalian flu is – Pig

226. ―Gall Sickness‖ is the synonym for- Anaplasmosis 253. Disinfectant of choice against Anthrax bacilli- Gluteraldehyde (2%)

227. Antigenic variation in avian influenza virus is largely due to- Genetic Shift 254. HVT, the vaccine strain used against MDV (serotype I) belongs to- serotype III

228. MAB technique for producing monoclonal Ig's was devised by- Kohler and Milstein 255. In birds, Avulavirus is the causative agent of - Newcastle disease

229. Bence Jones proteins (light chains of Ig) are present in urine in -Multiple myeloma 256. The cell surface marker of memory T cells is- CD45RO

230. Rose-Waaler test is used for the detection of -Rheumatoid factor (IgM) 257. In ALV infection, the tumor development is due to activation of oncogene called- c-myc

95 96
Dhruv N Desai Dhruv N Desai
258. Disinfectant of choice against FMD virus- Sodium Carbonate (4%) 21. Antidote of choice in copper poisoning is ------------------- d-penicillamine

259. Staphylorraphy and Uranoplasty are techniques used to correct- Cleft palate 22. Adrenaline at the rate of 1:1000 is used for the treatment of ------------------- Shock

260. Negri bodies for diagnosis of rabies can be detected using- Seller's stain 23. Drug of choice in heart block is ------------------- Isoproterenol

261. Malignant tumor of mesenchymal cells is referred as –Sarcoma 24. Total no: of carbon atom in hemoglobin molecule is -------------------64

25. Neostygmine is the drug of choice in ------------------- Myasthenia gravis

ICAR -SET 26. Homatropine is a synthetic derivative containing tropine and ------------------ Mandelic acid

1. Pseudo glanders is caused by-------------------Histoplasma farciminosum 27. Etiological agent of "Strawberry Footrot" is ------------------- Dermatophilus congolensis

2. Gestation period of lion/tiger is -------------------. 100-105 days 28. The dose of Yohimbine and 4-AP in Xylazine reversal is -------------------0.05-.25mg/Kg

3. Eye worm of poultry is -------------------Oxyspirura mansoni 29. Antibiotic that has the ability to bind with calcium is ------------------- Gentamicin (all

4. Bubo is the abscess of -------------------lymph node aminoglycosides)

5. The etiology of Crazy chick disease is ------------------- hypovitaminosis-E 30. Xanthine Oxidase inhibitor that is used to treat Gout is ------------------- Allopurinol

6. A pet animal which is an induced ovulator is -------------------Cat 31. Neuromuscular blocker which is kept on ice to prevent hydrolysis is -------------------succinyl choline

7. The amino acid that is essential for the synthesis of haemoglobin. glycine 32. Refractive power of eye lens is -------------------59D

8. One gram of haemoglobin carry ------------------- ml of Oxygen. 1.34ml 33. -------------------connects the lateral and third ventricles in brain foramen of monroe

9. One gram of haemoglobin on degradation will produce---------mg of bilirubin. 35mg 34. The breed of dog sensitive to thiopentone is -------------------grey hound

10. .Micro organisms that survive pasteurisation temperature are------------------- listeria and coxiella 35. Local anesthetic with antifungal and antibiotic action is -------------------dorsacaine

11. Summer mastitis is caused by -------------------Corynebacterium pyogens 36. Drug of choice for tape worms in poultry is -------------------di butyl tin di laurrate

12. Both intranuclear and intracytoplasmic inclusion bodies are present in the infections caused by ------- 37. Commonly used anesthetic that by-passes stage II of anesthesia. Barbiturates

--- morbilli virus (paramyxo viridae) 38. Antibody with shortest half life is -------------------IgE

13. Normal intraocular pressure is -------------------20mm of Hg 39. The cells lining the ventricles of brain -------------------ependymal cells

14. Enzyme responsible for the production of Ketone bodies is ------------------- HMGCOA lyase 40. Bierbeck granules are present in -------------------dendritic cells

15. Plasma protein precipitated last in salting out is ------------------- Albumin 41. -------------------% of oxygen consumed is used by brain Eight

16. The ratio of Calcium and Magnesium in blood is -------------------6:1 42. Most potent local anesthetic is ------------------- Bupivacaine

17. Quellung reaction is shown by ------------------- Streptococcus pneumoniae 43. Fibrosis of yoke gall in cattle is also known as ------------------- Tumor neck

18. Drug of choice for Theileriosis is -------------------buparvaquone(BUTALEX) 44. -------------------is the condition seen in horses due to pressure of collar on neck Sit fast

19. Kanagawa reaction is shown by ------------------- Vibrio parahaemolyticum 45. Malignant hyperthermia in swine is caused by-------------------anesthesia Halothane

20. The cerebrospinal fluid pressure is ------------------- 8-12 mm of Hg 46. Cart wheel chromatin and Russel bodies are seen in ------------------- Plasma cells

97 98
Dhruv N Desai Dhruv N Desai
47. The ratio of systolic, diastolic and pulse pressure is -------------------3:2:1 79. ----------30---------% of milk produced in the world is used for cheese making
48. Mineral toxicity that leads to fracture of pes is ------------------- Fluorosis 80. Cheese made from buffalo milk - Mozarella
81. Comb, wattle and earlobes of poultry are developed from .....Dermis......
49. The immunoglobulin with longest hinge region is ------------------- IgD 82. The sternum of poultry is ....................... shaped-- Boat
83. Chicken kidney has ....................... lobes -- Three
50. The antagonist for Etorphine. Naltrexone
84. Brooding instinct is governed by ....Prolactin...................
51. Remnant of yolk sac in birds is called ------------------- Meckel's diverticulum 85. Gallus varius is also called ....Green jungle plexus......
86. The network of nerves in submucosa of intestine…..Meissner’s plexus
52. ------------------- litre of gas is produced per minute in ruminants 0.5 - 1.0
87. Brush border of intestinal mucosa is constituted by .........Microvilli.......
53. Central lacteal is absent in the villi of ------------------- Birds 88. Capacity of rumen in cattle- 250 litres
89. Ileo-caecal, Ileo-colic and Ileo-caeco-colic junction are seen in ...Horse..., ....Dog... and
54. Nostrils are most dilatable in ------------------- and rigid in ------------------- Horses, Pigs
.........Cattle..... respectively
55. When cattle lie down the rate of respiration ------------------- Increased 90. Ascending colon is replaced by large colon in ....Horse.......
91. Among birds, ....Pigeon........ drinks by suction
56. Inner surface area of lungs is ------------------- times the surface area of body 125
92. Starch on hydrolysis yield .....Maltose..................
57. During panting, the tidal volume is ------------------- Constant 93. Cholecystokinin .......Delays................ gastric emptying
94. Increased functional activity of colon leads to ....Constipation..........
58. Gas exchange in birds takes place in ------------------- Lungs
95. Cattle secrete .......100-200................litre of saliva per day
59. Diving ducks have respiratory centre sensitive to ------------------- Posture 96. Among domestic animals, amylase is highest in the saliva of . Swine.....
97. Saliva constitute .........80..............% of water entering rumen
60. Di palmityl lecithin is a -------------- Respiratory Surfactant 98. Rennin converts casein to ..Para casein.....................
99. Opening of bile duct to duodenum is guarded by ...Sphincter of Oddi.......
61. Homogenizer valve is made up of -------------------Stellite
100. In horse ....75....% of energy comes from VFA.
62. At vacuum, milk boils at a temperature of -------------------50-550C
63. Normal acidity of milk is -------------------0.14 %
64. Predominant bacteria of milk is -------------------Micrococci
65. Lactic acid bacteria are ------------------- in nature - Aerotolerent anaerobes
66. Flavour producing compounds found in curd are -------------------,-------------------and -------------------
PHARMACOLOGY
Diacetyl, Acetyl methyl carbinol, Acetalimide
67. Sweet curdling is due to -------------------like enzymes-Renine
68. Bitty cream is produced by the enzyme -------------------Lecithinase
1. Plasma protein to which majority of drugs bind is ---------Albumin
69. Milk fat contains generally ------------ type of fatty acids-- Saturated ,Shortchain
70. Fe content of Khoa is -------------------100ppm 2. ----------- is the principal metabolic pathway for sulfonamide compounds Acetylation
71. Balance wheel of dairy industry is -------------------Butter
72. Butter contains --------80-----------% of fat 3. Precursor of endogenous catecholamines in the body is--------phenylalanine
73. Bacteria used for ripening of cream…Streptococcus lactis and S. diacetylactis 4. Drug of choice in acute anaphylactic shock is ------epinephrine
74. Clarified butter fat is the synonym for --------Ghee-----------
75. Cheese made from whey---Ricotta cheese 5. OP compound that interact with both esteratic and anionic site of acetylcholine esterase is -----.
76. ------------------- is a low fat cheese--Sapsago Echothiophate
77. Sodium alginate is obtained from -------------Macrocystis pyrifera------
78. Commonly used packaging material for milk is -------LDPE food grade------------ 6. A racemic mixture of d-hyoscyamine and l-hyoscyamine is --------Atropine

99 100
Dhruv N Desai Dhruv N Desai
7. What are soporifics? sleep inducers 33. Old Hen Test is used to detect ------------ potential of Organophoshate compounds Organo

8. The term "Anaesthesia" was coined by ------Oliver Wendell Holmes Phosphorous Induced Delayed Neurotoxicity

9. Precursor of serotonin is ------Tryptophan 34. Bright blindness ----- is a type of retinal degeneration caused by Bracken Fern Poisoning

10. What are endorphins?Endogenous analgesics 35. The specific antidote for Copper poisoning is ------------D-penicillamine

11. Species that require more amount of anaesthetics is-------Horse 36. Violent Dyspnoea "Thumping" is seen in pigs as a result of -----------Gossypol poisoning

12. The avian species in which procaine is contraindicated is -----Parakeet 37. Animal species to which Benzene Hexa Chloride is highly toxic is -------cat

13. In which breed of dogs thiobarbiturates are contraindicated? Grey Hounds 38. Highly potent Organophosporous compound is ---------. Parathion

14. Paralysis of ------ is a complication encountered in anaesthesia of Horse facial nerve 39. The synergists that is added to pyrethroid compounds in order to enhance its effect is ------Piperonyl

15. Which stage of anaesthesia is bypassed by barbiturates?Stage 2 Butoxide

16. Laryngyospasm during induction of anaesthesia is more common in -------cats 40. What are Burton's Lines? Blue line in gums in lead poisoning

17. Specific treatment for malignant hyperthermia in Pigs caused by halothane is-----Dantrolene 41. Specific antidote for Nitrate poisoning is ----------Methylene Blue

18. Oxidation of chloroform to phosgene can be prevented by adding -----1% ethanol 42. Gentamicin, the aminoglycoside drug is obtained from --------- Micromonosporum purpureum

19. Barbiturates are derivatives of -----------Malonyl Urea 43. The Fungal toxin that causes reproductive problems in sows is --------Zearalenone

20. Oxytetracycline is obtained from ------------Streptomyces rimosus 44. The anti BP drug Atenolol belongs to which group of Antiarrythmic Agents ?Class 2

21. --------------- is a benzimidazole with antifungal property Thiabendazole 45. Most potent H2 Blocker is ----------Famotidine

22. Primary mechanism of action of Mebendazole is inhibition of --------- by wormsGlucose uptake 46. Most potent Local anaesthetic is---------Bupivacaine

23. Wormicidal drug that can be given as immunomodulator at lower doses is ------- Levamisole 47. What does " Utectic Mixture " contain? Prilocaine and Lidocaine

24. Two chemical components seen in Ivermectin are ----- and --------B1a and B1b 48. The most potent of all the Aflatoxins is------B1

25. Drug active against immature stages of Fascioa hepatica is -----------Diamfenetide 49. Father of Pharmacology is ------Rudolf Bucheim

26. Antidote for Cyanide poisoning was discovered by ---------K K Chen 50. Izoniazid and Ethambutol are the drugs used in the treatment of --------Tuberculosis

27. Organochlorine compound which does not accumulates in the body is ----------Endosulfan

28. The enzyme in haeme synthesis which is inhibited by lead poisoning is ---------Aminolevulinic acid

dehydratase

29. Conium maculatum is better known as ---------Hemlock

30. Dose of BAL in Arsenic poisoning in Large animals is -----3mg/kg @ 4hr interval deep i/m
PATHOLOGY
31. --------- is the metabolic product of Procaine PABA

32. Give an example of a specific COX-2 inhibitor Cefocoxib

101 102
Dhruv N Desai Dhruv N Desai
1. 'Punched out ulcers' in abomasum- pathognomonic lesion of ?Theileriosis 28. Special stain for demonstrating Calcium in tissues. Van Kossa's Silver nitrate

2. Negri bodies in cattle with rabies seen in ....Cerebellum 29. 'Tophi' is related to which disease ? Articular gout

3. Father of pathological anatomy Antonio Benevieni 30. Condition characterized by green refrigence of Congo red stained sections under polarizing

4. Father of cellular pathology Rudolph Virchow microscope. Amyloidosis

5. Lysosome first demonstrated by.....Novikoff 31. Name the anaphylatoxins C3a and C5a

6. Most reactive free radical in inducing cell damageHydroxyl radical 32. Chemical mediators from arachidonic acid metabolism via cyclooxygenase pathway.Thromboxane

7. Removal of damaged organelle during cell injury is called as .....Autophagy A2 and Prostaglandins

8. Component of cytoskeleton useful in tumor diagnosis Intermediate filaments 33. 'Triple response' in tissue inflammation was formulated by .....Sir Thomas Lewis

9. Eosinophilic, intracytoplasmic inclusion in liver in alcoholic liver diseaseMallory body 34. Colloidal carbon technique is used in identifying .....Leaking vessels in inflammation

10. Condition in which impairment of phagocytic property of WBC occurs. Chediak Higashi syndrome 35. Name some SRS-A( slow reacting substances of anaphylaxis)Leukotrienes like LTC4, LTD4, LTE4

11. Best fixative for glycogen Non-aqueous fixatives(methyl alcohol) 36. Cationic proteins produced by eosinophils toxic to parasites.Major Basic Proteins

12. Stains for glycogen (any two) Best carmine & PAS 37. Suppurative inflammation of hair follicles caused by Staph. AureusBoils

13. Macrophage laden with lipids in atherosclerosis called as....Foam cell 38. Diffuse spreading suppurative inflammation of connective tissuesCellulitis

14. Russel bodies seen in ....?Plasma cells 39. Modified macrophages in case of granuloma are called ...Epitheloid cells

15. Partial or complete loss of melanocytes in the epidermis...Vitiligo 40. Granulation tissue is a hallmark of ......Healing

16. Pigments causing 'Brown Atrophy' Lipofuscin 41. Adhesive glycoproteins of Extra-cellular matrix.Fibronectin & Laminins

17. Aggregates of ferritin micelles called... Hemosiderin 42. Condition in which cardiac sclerosis/ cardiac cirrhosis occurs.Chronic general passive hyperemia

18. Heart failure cells are ......Alveolar macrophage laden with Hemosiderin 43. Alteration from a less specialized cell type to more specialized ones.Metaplasia

19. Unconjugated hyperbilirubinemia is indicative of .............jaundice. Prehepatic jaundice 44. A malignant tumor which doesn't metastasizeBasal Cell Carcinoma

20. Direct Van den berg reaction is indicative of ........jaundice Obstructive jaundice 45. Oncogenes discovered by.....Michael Bishop& Harold Varmus

21. Color of faeces in obstructive jaundice Grey / Clay color 46. 'Sticker tumor' discovered by Novinsky is better known as ....Canine Transmissible Venereal

22. Hyperkeratosis in cattle common in which poisoning ?Chlorinated naphthalene poisoning Tumor

23. Type of necrosis involved in hypoxic cell death in the CNS Liquefactive necrosis 47. Reed Sternberg Cells are typical of ...Hodgkin's disease

24. Necrosis in which architectural details persist but cellular details are lost. Coagulative necrosis. 48. Horn cancer is most commonly seen in Bull or Bullocks ?Bullock

25. Enzymes important in apoptosis Caspases 49. Black tongue/ canine pellagra is caused by......Niacin deficiency

26. Conditions in which PM clotting of blood doesn't occur. Anthrax & Sweet clover poisoning 50. Rodent ulcer is better known as ...... Basal cell carcinoma

27. Pathological calcification without derangement in blood calcium levels. Dystrophic calcification

103 104
Dhruv N Desai Dhruv N Desai
BACTERIOLOGY & MYCOLOGY 25. North American blastomycosis‘ is caused by- Blastomyces dermatitidis

BACTERIOLOGY AND MYCOLOGY 26. ‗European blastomycosis‘ is caused by- Cryptococcus neoformans
1. The most commonly used endospore stain- Schaeffer-Fulton stain
27. Asexual spores produced by fragmentation of hyphae is – Arthrospore
2. Capsule of B. anthracis is made of – D-glutamic acid
28. Staining used for detection of fungi in pathological specimens- Periodic acid
3. Teichoic acid in Gram-positive bacterial cell wall has- Glycerol/ribitol
Schiff stain
4. Archaebacterial cell walls usually consists of – Proteins and polysaccharides
29. Media used for stimulating production of chlamydospores of C. albicans- Corn
5. Metachromatin granules serves as a reserve of – Inorganic phosphate
meal agar.
6. The chemi-osmotic mechanism of ATP synthesis proposed by- Peter Mitchell
30. Method used for observing the development of spores and hyphae of fungus is
7. Reducing media for anaerobic bacteria contains- Na-Thioglycolate
called- Block-slide culture technique.
8. Mycobacterium leprae is usually isolated in- Armadillos
31. Mucor is distinguished from other members of Phycomycetes by- Absence of
9. Selective media for S.typhi is – Bismuth sulphite agar
rhizoids
10. Selective media for Staphylococcus aureus is- Mannitol Salt Agar
32. Asexual spores of Ascomycetes are known as- Conidiospores
11. Paraffin and mineral oils are often sterilized by- Hot air oven
33. ‗Bottle bacillus‘ is the synonym for- Pityrosporum ovale
12. ฀ Sterilizing agent used in space crafts is- Ethylene oxide
34. ‗Pseudoglanders‘ or ‗Japanese Farcy‘ caused by Histoplasma capsulatum
13. Test used to find the carcinogenicity of mutagens is- Ames test
35. ‗Fluffy colonies‘ , ‗Powdery colonies‘ and ‗Lemon yellow colonies‘ are produced
14. Three kingdom classification of bacteria proposed by- Carl Woere
by- Microsporum distortum , M. gypseum and M. canis respectively.
15. Pyrogen induced by endotoxins is – IL-1
36. Most potent toxin producing aflatoxicosis is – B1
16. Lab test to detect the presence of endotoxins in preparations- Limulus assay
37. ‗Facial eczema‘ in cattle is caused by - Pithomyces chartarum
17. Resoloving power of a compound microscope is – 0.2ì m
38. ―Farmer‘s Lung‖ is caused by Micropolyspora faeni
18. Heat resistance of spores is due to presence of – Dipicolinic acid
39. ฀ % of Agar used for the preparation of solid media is- 1.5%
19. Target site of polymyxin is- Bacterial cell membrane
40. Microbial population can be maintained in the exponential growth and at constant
20. Thickness of Gram+ve and Gram-ve cell walls are- 20-80nm and 2-7nm
biomass for extended periods by using- Chemostat/Turbidostat
respectively
41. The phenomenon by which bacteria monitor their own population density through
21. Most abundant membrane protein of Gram-ve bacterial outer membrane- Braun’s
sensing the level of signal molecules- Quorum assay
lipoprotein
42. Indicator bacterium used in autoclaving is- Bacillus stearothermophilus (spores)
22. Non-motile asexual sporangiospores of phycomycetes is – Aplanospores
43. The commonly used agent for ‗cold sterilization is - Gamma radiation
23. Exogenously borne sexual spores are known as – Basidiospores
44. The disinfectant action of chlorine is due to the formation of- Hypochlorous acid
24. Psuedohyphae is characteristic of – Candida albicans
105 106
Dhruv N Desai Dhruv N Desai
45. Linear chromosomal DNA is found in- Borrelia burgdoferi vulgaris X strain

46. The topoisomerase that removes the supercoiling during replication- DNA gyrase. 65. Swarming growth and fishy odour of colonies is characteristic of- Proteus

47. The negative stain used in electron microscopy is – Phosphotungstic acid vulgaris

48. Bacterium that exhibits directed swimming in response to Earth‘s magnetic field 66. Kauffman-White scheme is used for serotypic differentiation of Salmonella

or to local magnetic fields- Aquaspirillum 67. VR (Venkatraman-Ramakrishnan) medium is used as a transport media for-

49. The structure that remain after the treatment of a gram –ve bacteria with Vibrio cholerae

lysozyme/penicillin is – Spheroplast 68. Epidemic typhus‘ (Brill-Zinsser disease) is caused by Rickettsia prowazaki

50. An inorganic solidifying agent used for autotrophic bacteria- Silica gel 69. ‗Scrub typhus‘ is caused by Rickettsia tsutsugamushi

51. The procedure for staining the capsular material of B. anthracis is- MacFadyean 70. Rickettsial disease that are not arthropod borne are Q-fever and Trench fever

reaction.

52. Botulism due to which type of toxin occurs in relation with phosphorous

deficiency- Type D

53. Vole‘s Bacillus is the synonym for - Mycobacterium murinum EPIDEMIOLOGY

54. Specific media used for isolation of M. bovis is Stonebrink‘s medium containing
1. Which is the first veterinary school? Lyon, France 1762
Sodium pyruvate
2. Quarantine was first introduced by------ Lancisi, physician to Pope Clement XI from Rinderpest
55. ‗Stormy clot‘ reaction is characteristic of - Clostridium welchi
3. First animal virus ------- and was identified by ------- & ------- FMD, Loeffler and Frosch
56. The characteristic ‗earthy odour‘ of the cultures is a feature of- Psuedomonas
4. Disease which do not produce any overt clinical sign. Subclinical infection
psuedomallei
5. Study of outbreaks in avian population is known as------- Epornitics
57. ‗Flying sea gull‘ appearance is characteristic of- Campylobacter
6. ------- Epidemiology involves observing and recording disease and possible causal factors.
58. Vaginal mucus agglutination test in cattle is employed for the diagnosis of-
Descriptive
Campylobacter infection.
7. ------- is the study of cause, distribution and control of disease in related individual and of inherited
59. Rolling disease in mice is caused by- Mycoplasma neurolyticum.
defects. Genetic epidemiology
60. Eaton‘s agent is the synonym of - Mycoplasma pneumoniae
8. ------- is an examination of aggregation of units. Survey
61. Dick‘s test is used for the id entification of- Streptococcus pyogens
9. Survey records events occurring at a particular point of time. Cross sectional survey
62. Chinese letter arrangement is seen for- Corynebacterium
10. Unit of an epidemiologist Population
63. Fried lander‘s bacillus is the synonym for Klebsiella pneumoniae
11. ------- is the identification of undiagnosed cases of disease using rapid tests. Screening
64. Weil-Felix reaction is used for diagnosis of Rickettsial infections using Proteus

107 108
Dhruv N Desai Dhruv N Desai
12. -------is the making of routine observation on health, production and environmental factors and 32. Map where line joining equal morbidity rate is ------- and mortality rate is------- Isomorbs, isomorts

recording and dissemination of these observations. Monitoring 33. ------- is any characteristic that affects the health of a population. Determinant

13. ------- investigate relationship between disease and hypothetic causal factors in specified 34. Epidemiological triad‘s are-------, ------- &------- Host, gent and environment

population.Cross sectional study 35. Ability of organism to cause disease in a particular host, in terms of severity is known as-------

14. ------- is comparison of exposed group with non exposed group to the factors with respect to Virulence

development of disease. Cohort study 36. ------- is quality of disease induction. Pathogenicity

15. -------is any observable event that can vary. Variable 37. Sites within genome that frequently mutate. Hot spot

16. Survey records events occurring for a long period of time. Longitudinal study 38. Infection of susceptible host without overt clinical sign. Inapparent infection

17. ------- Factors are associated with the definite onset of disease. Precipitating factors 39. ------- is any animal sheds an agent without clinical sign. Carrier

18. Constant occurrence of disease in a population or usual frequency of occurrence of disease is known 40. Animal which excrete agents during incubation period is known as------- Incubatory carrier

as------- Endemic 41. . ------- Climate comprises of normal component weather to which animal are exposed. Macro

19. Sudden unpredictable number of cases in a population. Epidemic 42. Infection transmitted from one segment of population to the another segment of the population is

20. Widespread epidemic Pandemic known as-------- Horizontal transmission

21. Irregularly and haphazardly occurring diseases are known as -------Sporadic 43. Infection transmitted from one generation to next generation is known as------- Vertical transmission

22. Amount of disease in a population is given by------- Morbidity 44. Host in which agents are transmitted mechanically. Paratenic host

23. Amount of death in a population is given by------- Mortality 45. Host in which multiplication of agents takes place. Amplifier host

24. Time of occurrence of a disease constitute-------distribution Temporal 46. Inanimate vectors are called as------- Fomits

25. Place of occurrence of disease constitute -------distribution Spatial 47. Cyclopropagative transmission is a combination of-------&------- Developmental and propagative

26. -------is the number of instance of disease or related attribute in a known population at designated 48. Example for stercorarian transmission. T.cruzi

time, without distinction of new and old cases. Prevalence 49. ------- is the period between infection and maximum infectiousness. Generation time

27. -------is the number of new cases occur in a known population over a specified period of time. 50. Time between infection and availability of agent in an arthropod vector is known as-------Extrinsic

Incidence incubation period

28. 28. -------is the proportion of cases of a contagious disease that develop as a result of contact with 51. ------ relates the amount of organism required to initiate an infection. Infectivity

primary cases. Secondary attack rate 52. The length of time for which an organism can remain infective outside its host is known as the -------

29. P α I X------- D(duration) Stability

30. ------- is more intensive form of data recording. Surveillance 53. Switch from virulence to non-virulence Phase variation

31. Total mortality rate of all disease is known as------- Death rate 54. Transmission of disease from one generation to another via egg is known as. Trans-ovarian

109 110
Dhruv N Desai Dhruv N Desai
55. Transmission from one developmental stage to another is known as------- Transtadial 77. Culling of infected animals during epidemic is often accompanies by the slaughter of animals that

56. Rain forest are described as------- where as deciduous forest is ------- Megatherms and mesotherms may have been exposed to infection and therefore be at risk of developing disease is known as -------

57. ------- is the natural restriction where animal can roam. Home range Pre emptive slaughtering

58. Part of the animals home range that it defend aggressively from invaders is known as------- Territory 78. Proportion of animals that are resistant to infection or disease in population. Herd immunity

59. According to Wynne Edward hypothesis population control was the main purpose of -------Group

behaviour

60. Which is the functional position of an animal in an ecosystem? Niche

61. Avoidance of competition is usually in -------animals. Sympatric animals


PHYSIOLOGY
62. Which is the smallest spatial unit providing uniform condition for life? Biotope

63. Collection of all living organism in a biotope is known as------- Biocenosis 1. Rumen gas largely consists of Carbon dioxide and methane in the proportion of ….65:35.....

64. Man made ecosystem Anthropurgic 2. Urea is recycled in the rumen through ....Saliva.......and .diffusion across rumen wall......

65. Junction of two ecosystem is known as------- Ecological interface 3. If acetic : propionic ratio in the rumen goes down below.. 3 : 1..... the fat content decreases in the cow
milk
66. ------- is the modified patch of vegetation, created by man, with in a biome that has reached in a
4. The pH of ileal fluid ranges from ......7....to ....8..
climax. Ecological mosaic
5. The rumen protozoa store carbohydrate in the form of ... amylopectin
67. Study of disease in relation to ecosystem in which they are found is known as------- Landscape
6. The testosterone released from tunica interna is converted to estradiol, a female sex hormone under the
epidemiology influence of ... FSH.....

68. Foci of infection. Nidi 7. The estrogen produced by mammalian ovary or placenta normally are ......, .......and .......( Estrone,
estradiol and estriol)
69. An area that has ecological, social, and environmental condition that can support a disease is known
8. In ruminants, the placenta is of ...................... type( Epitheliochorial)
as------- Nosogenic area
9. Grayish coloured corpus luteum present in. Ewe (species)
70. ------- is a nosogenic territory in which a particular disease is present. Nosoarea
10. PRL (prolactin) promote the secretion of progesterone by the corpora lutea in .................and
71. If all animal in a population are surveyed then it is known as------- Census ........................(species)Rat and mice

72. If relative risk is more than one it denotes-------Positive statistical association between factor and 11. Bruce effect in mice involve blockage of Prolactinsecretion which is necessary for maintenance of CL

disease 12. the blood supply to udder is maintained by .......Pudental......artery

73. ------- is the decrease in mortality and morbidity. Control 13. ..fructose...............is the main source of energy in bull semen

74. Examples for primary prevention Vaccination and quarantine 14. Collapse of alveoli is known as .........Atelectasis.............

75. Animal which excrete agents during recovery period is known as------- Convalescent carriers 15. one gram of hemoglobin can bind with maximum of ......1.34..........ml oxygen
76. Extinction of an agent Eradication 16. Expiration if regulated by ...pneumotaxic........centre of the brain

111 112
Dhruv N Desai Dhruv N Desai
17. Rate of diffusion of carbon dioxide through respiratory membrane is about ......20..........times rapid than 39. Central frozen semen production and training institute is located at ..Hessarghata........
oxygen diffusion
40. Credit of first birth of a buffalo calf through AI in India goes to...Allahabad Agriculture Institute
18. The volume of air which remains in the lungs after forceful expiration is called Residual volume...
41. Electro ejaculation was first adopted by Batteli......
19. Threshold level for low oxygen in air for sheep, goat and cattle is about ........250….meters above sea
level 42. .....Estrogen......hormone is very important for the duct growth of mammary gland

20.The critical environmental temperature at which the increase in respiration rate become marked is 60oF 43. The lactose content in milk is ...........4.6............% and is the most consistent component of milk
for HF,70o F for Jersey and .......800F...............for Brahman cattle
44. It is estimated that about .400-500....ml of blood must pass through udder for the production of one ml of
21.The most potent compound which can stimulate the closure of esophageal/reticular groove in cattle even milk
up to two years of age is.........NaCl...................
45. As per work physiology, contraction of ......Spleen..............(visceral organ) increase no of erythrocyte in
22.......... Acetic (C2) > Propionic (C3) > Iso & N-butyric (C4) > Iso & N-valeric (C5) > Methyl butyric the body.
acid (C5).......is the usual order of concentration of individual acids present in the rumen
46. Exercise result in increased cardiac output to meet the increased demand of working muscles for oxygen.
23.Enzymes responsible for metabolism are located in the ..Mid piece..........................(part) of spermatozoa The cardiac output =stroke volume X ...Heartrate.......................

24.The estrous cycle of ewe is of .......17...............................days duration 47. During strenuous exercise, cardiac output increases upto .......8............fold in horse

25.The bull spermatozoa can travel ........60................cms per 30 minutes in the female genital tract 48. Race horses are most susceptible to deficiency of ........Thiamine....(B vitamin)

26.Sexual receptivity in case of ewe in heat requires the presence of.....small amount of 49. Proteolytic bacteria represent about .......12-38...........% (range) of the total ruminal bacteria
progesetrone.....(hormone)
50. ..Pulmonarycompliance...is a measurement of the distensibility of the lungs and thorax and is
27..........80.............% of carbon dioxide transport occur in the form of Bicarbonate ion determined by measurement of the lung volume change for each unit of pressure change.

28.Utiliozation coefficient for oxygen consumption is...50.....% for birds and ...25.....%for mammals

29.The nerve network Meissners plexus controls secretions of epithelial cell where as Auerbachs plexus
control ............GI movements......... ENDOCRINOLOGY

30.Coiled colon (Ansa Spiralis) present in ...pig.....and......ruminants.....(species)


1) A meal rich in proteins but low in 3) After intravenous administration of a large dose
31.In GI system, contractile waves that travel short distance is termed as....Segmentation..and that travel carbohydrates does not cause hypoglycaemia of insulin, the return of a low blood sugar level to
longer distance is........Peristalsis..... because normal is delayed by

32.Daily production of saliva in cows comes around .....100-200........litres a. glucagon secretion is stimulated by meals b. the a. thyrotoxicosis b. glucagon deficiency
meal causes compensatory increase in T4 c. diabetes d. parathormone deficiency
33.The first hormone ever discovered was .........Secretin....................... secretion
c. cortisol in circulation prevents glucose from Ans. b
34.Horse obtains around ...........75.............% of its energy requirements from large intestinal absorption of entering the muscles d. the amino acids in the
meal are converted to glucose 4 ) Insulin increases entry of glucose into
volatile fatty acids
Ans. a a. renal tubule b. the mucosa of the small intestine
35.Protozoan count of rumen is about ......106..ml of rumen content and it account for ...20......%of rumen
metabolism c. neurons of motor cortex d. skeletal muscle cells
2) Which of the following is incorrectly paired
36. .........Prolactin......hormone induces gene expression in mammary tissue for casein synthesis a. beta cells-insulin b. F cells- gastrin Ans. d
c. delta cells- somatostatin d. alpha cells-
37. Herbivores donot have .....Cephalic........... phase of gastric stimulation glucagon 5) Glucagon is not normally found in the

38. ...Cervix.................part of female reproductive tract is known as "neck of the womb" Ans. b
113 114
Dhruv N Desai Dhruv N Desai
a. brain b. pancreas a. kidneys b. skin 18) Angiotensin increases blood pressure by a. hyperthyroidism b. pregnancy
c. git d. adrenal glands c. lungs d. intestine acting on the following EXCEPT c. parents treated with glucocorticoids d. parents
treated with estrogens
Ans. d Ans. c a. aldosteron secretion b. vascular smooth muscle
c. parasympathetic nervous system d. sympathetic Ans. a
6 ) Which of the following is NOT produced by 12) Ca++ plays an important role in following nervous system
physiological amounts of glucocorticoids biological processes except 25) In starvation which of the following is
Ans. c reduced
a. maintenance of normal vascular reactivity b. a. oxygen utilization b. contraction of cardiac
inhibition of inflammatory response muscle 19) Erythropoietin a. plasma T4 b. plasma T3
c. increased excretion of a water load d. inhibition c. contraction of skeletal muscle d. blood c. reverse tri-iodothyroxine d. D thyroxine
of ACTH secretion coagulation a. contains iron b. has no effect on WBC count
c. stimulates renin secretion d. increases half life Ans. b
Ans. b Ans. a of RBC
26) Hypothyroidism is associated with increased
7) Cortisol increases blood glucose level by 13) Epiphyseal closure is regulated by Ans. b levels of

a. increasing lipolysis b. increasing protein a. calcitonin b. somatomedins 20) Somatostatin a. cholesterol b. albumin
synthesis in muscles c. 1,25 dihydroxy cholecalciferol d. thyroxine c. TBG d. iodine
c. increasing gluconeogenesis d.increasing growth a. inhibits insulin and glucagoon release b.
hormone secretion Ans. b stimulates insulin and glucagon release Ans. a
c. stimulator of glucagon release d. acts as obesity
Ans. c 14) Which of the following pituitary hormones is hormone 27) The metabolic rate is least affected by an
a polypeptide increase in the plasma levels of
8) Epinephrine and norepinephrine Ans. a
a. MSH b. ACTh a. TSH b. TRH
a. are amino acids b. are both secreted by neurons c. beta - endorphin d. growth hormone 21) Thyroid hormone stored in the lumen of c. TBG d. none of the above
in the autonomic nervous system follicles is in the form of
c. are polypeptides d. both activate alpha and beta Ans. c Ans. c
adrenergic receptors a. free T3 b. free T4
15) Growth hormone acts directly on c. attached to thyroglobulin in the gland d. 28) The coupling of mono iodotyrosine and di-
Ans. d attached to thyroid binding globulin iodotyrosine and the iodination of thyroglobin is
a. stimulation of protein synthesis b. stimulation blocked by
9) A decrease in extracellular volume is expected of cartilage formation Ans. c
to cause increased secretion of all except c. elevation of BSL d. stimulation of bone a. TSH b. TRH
formation 22) Secretion of growth hormone c. iodine d. thiocarbamides such as
a. vasopressin b. renin propylthiouracil
c. thyroxin d. ACTH Ans. c a. increases during REM sleep b. increases during
exercise Ans. d
Ans. c 16) Hypopituitarism is characterized by c. increases during starvation d. increases during
NREM sleep 29) Parathyroid hormone
10) A patient with parathyroid deficiency 10 days a. infertility b. intolerance to heat
after thyroidectomy will show c. weight gain d. excessive growth of the soft Ans. d a. decreases Ca++ mobilization of bone b.
tissue increases Ca++ mobilization from bone
a. a low plasma phosphate and Ca++ levels and 23) Atrial natriuretic peptide brings c. decreases circulating levels of free Ca++ d.
tetanus b. a low plasma Ca++ levels, increased Ans. a increases urinary excretion of Ca++
muscular excitability and Trousseaus sign a. afferent arteriolar constriction in kidney b.
c. high plasma phosphate and Ca++ and bone 17) Excessive growth hormone secretion in adults efferent arteriolar consrtiction in kidney Ans. b
demineralization d. increased muscular causes c. increases renin secretion d. constriction of
excitability, high plasma Ca++ and bone mesangial cells 30) Thyrocalcitonin
demineralization a. acromegaly b. gigantism
c. increased entry of glucose in muscles d. Ans. b a. is secreted by thyroid b. is secreted by
Ans. b hypothyroidism hypothalamus
24) Thyroid binding globulins are normal in c. is secreted by parathyroid d. increases Ca++
11) Which of the following is not involved in Ans. a absorption by stomach
regulation of plasma Ca++ levels
115 116
Dhruv N Desai Dhruv N Desai
Ans. a (A) Antidiuretic hormone _____12 As the Na-K pump function in a neuron _____20. The eukaryotic plasma membrane is a
(B) Cholecystokinin membrane, _____ Na ions is/are pumped for basically bilayer of:
31) Growth hormone causes hyperglycemia. It is a (C) Calcitriol every ____ K ions A) protein C) phospholipids
result of (D) Gastrin pumped in: D) glycopeptide D) phosphopeptides
A) 1 . . . . .2 C) 3 . . . . .2 _____21 The function of the nucleolus is to
(A) Decreased peripheral utilization of 33) Hormones B) 2 . . . . 3 D) 2 . . . . 1 A) produce ribosomal subunits
glucose _____13. The inner ear of mammals also contains B) orient the cellular poles during cytokines
(B) Decreased hepatic production via (A) Act as coenzyme the apparatus for balance and equilibrium. C) store MRNA before transport to the ER
gluconeogenesis (B) Act as enzyme Changes of the D) process, packages, and stores lipid and portion
(C) Increased glycolysis in muscle (C) Influence synthesis of enzymes position of the head with respect to gravity, as in from the nuclear membrane.
(D) Decrersed lipolysis (D) Belong to B-complex group bending forward, are detected by hair cells i9n the ______22. The following structures are all part of
chamber known as the: a continuous network of membranous tubules,
32) Hormone that bind to cell surface receptor and A) semicircular canals C) statocyst flattened sacs and
require the second messenger camp is B) vestibular canal D) saccules channels found in eukaryote cells
_____14. the cilia of sensory hair on the basilar A) Golgi apparatus, vesicle, Rough ER,
membrane are embedded in the: peroxisome
A) round window C) oval window B) RER, SER, Golgi apparatus
B) tectorial membrane D) vestibular canal C) ER, lysosomes, mitochondria, Golgi complex
PHYSIOLOGY _____15 Which of the following sequence D) nuclear membrane pore, vesicle, mitochondria
Cell, Cardio-Vascular System, Nervous System and Muscles describe the blood clotting process correctly ______23. Which of the following molecules is
_____ 1. The Pumping chambers of the heart are _____6 the pulmonary circuit of a mammalian A) damaged platelets + Ca - enzyme - thought to block myosin cross bridge) binding site
called the: circulatory system connects which chamber of the prothrombin – enzyme - fibrinogen – fibrin on actin when
A) Atria C) ventricles heart? B) damaged platelets + Ca – prothrombin – a muscle is not contracting
C) pacemaker D) cardiac muscle A) RV to LA C) LA to LV fibrinogen – fibrin A) calcium C) tropomyosin
_____ 2. Why is there no passive tension B) LV to RA D) RA to RV C) damaged platelets + Ca – enzyme – B) troponin D) ATP
developed when the muscle is shortened to less _____7 Which of the following tissue or organs is prothrombin – enzyme – fibrin – fibrinogen ______ 24. Regeneration of axons:
than its resting length? best adapted fro anaerobic respiration D) damaged platelet + Na – thromboplastin – A) occurs in the segmental distal to the damage
A) passive tension is developed, but it is obscured A) skeletal muscle C) cardiac muscle thrombin – fibrinogen B) is independent of the survival of the perikaryon
by the greater active tension B) brain D) smooth muscle _____16. One of the simplest kinds of behavior is C) includes a decrease in volume of the
B) muscle contraction cannot occur when the _____8. Which of the following statement the knee jerk, in which a tap below the knee perikaryon
muscle is shorter than its resting length correctly describe a fully contracted sarcomere to causes the legs to D) is dependent on proliferating of Schwann cells.
C) tension cannot develop at wall if muscle is at one that is relaxed? jerk up. This behavior require as a minimum ______25. The amount of tension that whole
less than resting length A) the A band remain the same length which of the following combination of structure? muscle can produce is greatest in which of the
D) there is no stretch of connective tissue and B) The H zone remains the same length A) a motor neuron and a muscle following situations?
elastic fiber C) The I band remains the same length B) a receptor and at least two segment of the A) in the single twitch response
_____.3. When a muscle contract, tension D) The Z line remains equidistant from each other spinal cord B) when ext5racellular Ca is decreased
develops because of: _____9. Which of the following sensory or motor C) a receptor neuron, a motor neuron, and a C) when extracellular Mg increased
A) interaction between the actin and myosin tissue would most likely have electrical synapse? muscle D) during maximal complete tetanus
filament A) cardiac muscle C) retina D) an intact spinal cord and brain _____ 26.During the relative refractory period
B) the overlapping arrangement of the actin and B) skeletal muscle D) pressure receptor _____17. The vertebrate immune system response A) the rate of depolarization is decreased
myosin filament _____10. when an action potential moves along involves the following different types of cells B) the rate or repolarization is increased
C) a slackening within the connective tissue skeletal muscle fiber, calcium ions are released which respond C) the threshold for eliciting an action potential is
element from SR. The in a highly specific manner decreased
D) the length-tension relationship calcium ion binds with what molecule component A) erythrocyte and phagocyte C) reticulocyte ands D) the conductance of potassium is decreased
_____4 The initiation of the heart beat normally of the thin filament? phagocyte _____27. The sodium gradient across the nerve
originate from the A) actin C) myosin B) T cell and B cells D) Both A and B cell membrane is
A) AV node C) CNS B) troponin complex D) tropomyosin _____18. The Y-shaped antibody molecule is A) a result of donnan equilibrium
B) SA Node D) thyroid gland _____11. the openings through which Na and K produced by B) significantly changed during an AP
_____5 In an Adult mammalian heart, the ion move to creates an AP are known __________ A) infected cells C) B cells C) used as source of energy for the transport pf
pulmonary artery is carrying channels. B) T cells D) phagocyte other ions
A) oxygen rich blood to the lungs A) potential C) ion _____19 cell mediated immunity is the function of D) maintained by an Na-Ca exchanger
B) oxygen rich blood from the lungs B) electrochemical D) voltage-gated A) T cells C) phagocyte _____ 28. The highest blood flow per gram of the
C) oxygen poor blood from the lungs B) B cell D) reticulocyte left ventricular myocardium would occur
D) oxygen poor blood to the lungs A) when aortic pressure is highest
117 118
Dhruv N Desai Dhruv N Desai
B) when left ventricular pressure is highest C) arterial pressure B) olfactory cortex D) nasal mucosa D) Norepinephrine D) Isoproterenol
C) at the beginning of isovolumic contraction D) hematocrit _____ 47. Which of the following statements may _____ 55.All of the following statements
D) at the beginning of diastole _____38. Cerebral blood flow may be increased accurately characterize the properties cardiac regarding systemic homodynamic are true EXCEP
______29. During ventricular ejection, the by increasing muscle? that the:
pressure difference smallest in magnitude is A) ventilation C) pH A) The T – tubule system is located at the Z-lines A) greatest cross-sectional area is within the
between the B) arterial blood pressure D) carbon dioxide B) a transient influx of extracellular calcium ions capillaries rather than small veins.
A) pulmonary artery and left atrium C) left tension contribute to contraction B) greatest percentage of blood volume is in the
ventricle and aorta _____39. The greatest percentage of blood C) individual cells are electrically coupled small veins and the least is in the
B) right ventricle and right atrium D) left ventricle volume is found in the D) all of them are correct arterioles
and left atrium A) heart C) distributing arteries and arterioles _____ 48. Which of the following statements may C) velocity of blood flow is lowest in the
______30 . Closure of the aortic valve occurs at B) capillaries D) venules and veins correctly describe the coupling of excitation and capillaries
the onset of which of the cardiac cycle? _____40. Turbulence is more likely to occur in a contraction in the heart? D) compliance of the venous circulation is less
A) isovolumic contraction C) protodiastole blood vessel if A) increase in extracellular k ions may cause than the arterial circulation
B) rapid ejection D) isovolumetric relaxation A) the velocity of blood within the vessel cardiac arrest in diastole _____ 56. The most important component in the
______31 Which of the following will be greater increases B) absence of Na ions prevent the heart from formation of hemostatic plug is
during the plateau phase of the ventricular AP B) the viscosity of blood within the vessel beating A)RBC C) lymphocyte
than at increases C) free intercellular ca ions is primarily B) fibrin D) platelets
rest C) the diameter of the vessels decreases responsible for the state of myocardial _____ 57. The portion of the electrocardiogram
A) sodium conductance C) potassium conductance D) the length of the vessel increases contractility with which it most closely associated with
B) total membrane conductance D) calcium _____41. Which one of the following would NOT D) All of them are correct ventricular
conductance contribute to local hemostasis? _____ 49. All of the following are true for the repolarization.
_____32. Stroke volume can be decreased by A) exposure of platelets to collagen smooth muscle, cardiac muscle, skeletal muscle A) P wave C) ST segment
A) increasing ventricular contractility C) B) the conversion of prothrombin to thrombin and B) QRS complex D) T wave
increasing central venous pressure C) the conversion of plasminogen to plasmin macrophage EXCEPT that they: _____58. Which of the following encapsulated
B) increasing heart rate D) decreasing total D) the release of thromboxane A2 A) contain actin and myosin C) have cytoskeleton receptors is the largest and most widely
peripheral resistance _____42. Following the loss of blood, the LEAST B) respond to nervous stimuli D) use ATP for distributed?
_____33 The electrocardiogram (ECG) is least likely event is an increase in contraction A) tactile corpuscle of meissner C) pacinian
effective in detecting abnormalities in A) heart rate C) stroke volume _____ 50. The ATPase activity require for muscle corpuscle
A) the position of the heart in the chest C) cardiac B) sympathetic activity D) total peripheral contraction is located in: B) end bulbs D) neuromuscular spindle
rhythm resistance A) myosin C) sarcoplasmic reticulum _____ 59. Under normal conditions, the major
B) atrioventricular conduction D) cardiac _____43. As a result of reduced stretch of the B) troponin D) actin mechanism of body heat loss is:
contractility carotid baroreceptors, all the following would _____ 51. All of the following are important A) radiation C) perspiration
_____34. Which one of the following would cause increase compensatory mechanism on hemorrhagic shock B) evaporation D) insensible perspiration
a reduction in arterial pressure? EXCEPT except: _____ 60. A neuronal soma has a resting
A) a decrease in arterial compliance C) a decrease A) cardiac output C) total peripheral resistance A) tachycardia and vasoconstriction C) decreased membrane potential of -65 mV. Opening
in venous compliance B) heart rate D) parasympathetic nerve activity peripheral vascular resistance potassium channels in
B) a decrease in blood volume D) an increase in _____44. which of the following changes in B) absorption of fluid from interstitial space D) the neuronal membrane will most likely causes:
central venous pressure perfusion of an organ system is an example of formation of angiotensin II A) depolarization to about -30 mV C) initiation of
_____35. The increase in skeletal muscle blood autoregulation? _____ 52. Long term regulation of arterial blood an action potential
flow that occurs during vigorous exercise A) The decrease in renal blood floe during pressure is primarily a function of: B) hyperpolarization to about -85mV D) no
A) causes an increase in total peripheral resistance hemorrhage A) the CNS C) peripheral receptor change in membrane potential
B) causes an increase in blood pressure B) The decrease in Blood flow to the skin during B) the sympathetic NS D) urine output and fluid _____ 61. Numerous ion channels are involved in
C) is primarily due to parasympathetic and exposure to cold environment intake the generation of the cardiac action potential. The
sympathetic activity C) The increase in cerebral blood flow during _____ 53. Which of the following is present only ion
D) is primarily the result of the accumulation of hypercapnia in the intrinsic pathway of clotting? channel ,most closely associated with the plateau
vasoactive metabolites D) None of the Above A) fibrinogen 9factor I) C) prothrombin (Factor phase of the cardiac action potential is:
_____36. A reduction in carotid sinus pressure _____45. In which of the following organs will II) A) voltage-gated sodium channels C) calcium-
would cause a decrease in the rate of blood flow change the LEAST during C) accelerin (Factor V) D) anti-hemophilic factor gated potassium channels
A) heart rate C) venous capacitance exercise (Factor VIII) B) voltage-gated potassium channels D) voltage-
B) myocardial contractility D) cardiac output A) skin C) intestine _____ 54. In animal suffering from severe gated calcium channels
_____37. Blood flow through an organ would be D) brain D) kidney anaphylactic shock, the drug of choice for _____ 62. The relationship between sodium
increased by decreasing _____ 46. The olfactory receptors are located in restoring circulation excretion and blood pressure is an example of a:
A) the diameter of the arterial vessels the: and relaxing bronchial smooth muscle is: A) hormonally mediated events C) counter-current
B) the number of open arterial vessels A) olfactory bulb C) olfactory tract A) epinephrine C) dopamine system
119 120
Dhruv N Desai Dhruv N Desai
B) negative feedback system D) positive feedback _____ 68. During a voluntary movement, the This wave reaches a second mass of nodal tissue, An AP is generated by rapid, differential diffusion
mechanism Golgi tendon organ provides the CNS with AV node, which is then stimulated to contract. of ions across the membrane, thereby temporarily
_____ 63. The hypothalamus is LEAST involve in information This contraction is transmitted to all part of the reversing polarity across the membrane. The
the regulation of: about ventricle causing them to contact as a unit. channel through which first sodium and then K
A) intake of water C) temperature and osmolarity A) the length of the muscle being moved 5. The Answer is D moves during this process are regulated by
of urine B) the change in joint able produces by the Since arteries always carry blood away from the changes in membrane voltage, hence the name
B) emotional behavior D) respiration movement heart and the blood going to the lungs from the ―voltage-gated‖. The term found in other choices
_____ 64. All the following neurotransmitters are C) the velocity of the movement heart is oxygen – poor. Blood returning to the have no reference to membrane channel.
inactivated when diffused out of the cleft or D) the tension developed by the muscle being heart from the lungs will be carried in veins 12. The Answer is C
pumped moved (pulmonary veins) and will be oxygen-rich. The Na-K pump moves Na from the inside of a
into the presynaptic nerve ending EXCEPT: _____ 69. Thrombocytopenia is a reduction in the 6. The Answer is A cell to the outside and K ion from the outside to
A) serotonin C) norepinephrine number of circulating blood platelets. Which of The RV pumps oxygen-poor blood to the lungs the inside. The ―pump‖ is actually a protein
B) dopamine D) acetylcholine the via the pulmonary artery and the left atrium embedded in the cell membrane. Conformational
_____ 65. Norepinephrine will cause contraction following would most likely occur in receives oxygen-rich blood from the lungs via the changes requires energy in the form of ATP and
of the smooth muscle in the thrombocytopenia? pulmonary veins. The LV to RA connection is result in Na ion being pumped out and K ion
A) bronchioles C) intestine A) decreased vascular permeability made via the systemic component of mammalian pumped into the cell in a ration of approximate 3
B) pupils and ciliary body D) arterioles B) failure of initiation of blood clotting cascade circulation. The other two choices describe two to 2 during each cycle.
_____ 66. Which of the following statements C) failure of conversion of fibrinogen to fibrin correct sequences of heart chamber in normal 13. The Answer is D
about the cerebrospinal fluid (CSF) is true? D) absence of plasmin circulation, but neither sequences includes Of the 3 areas of the inner ear or labyrinth –
A) it is absorbed by the choroids plexus _____ 70. Erythrocytes may have abnormal pulmonary circulation vestibule, semicircular canals and cochlea – the
B) it circulates in the epidural space shapes and sizes in certain diseases. In iron 7. The Answer is A vestibule has two interconnected sacs called the
C) it has a higher protein concentration than deficiency you Skeletal muscle is the best adapted fro this process utricle and saccules. These sacs have receptors
plasma would expect to see and can function some time under very low that are sensitive to straight line movement of the
D) it has a lower glucose concentration than A) microcytic, hypochromatic anemia with oxygen tension. Brain cells are among the most head and gravity. The 3 semicircular canals are
plasma smaller mature erythrocyte sensitive to lack of oxygen while cardiac and also located in the labyrinth, but they respond to
_____ 67. Which of the following statements B) macrocytic, hyperchromatic anemia woith smooth muscle are less so, but will not as well rotational movement of the head; the vestibular
about the hair cells of the cochlea is true? fewer, larger mature erythrocytes adapted as skeletal muscle. canal or scala vestibule, is the fluid filled upper
A) they protect the lower airways from large C) poikilocytosis (shape change) and more fragile 8. The Answer is A chamber in the cochlea, separated from the lower
particulate matter erythrocytes the sarcomere contracts as the thick myosin slide chamber (scala tympani) except fro a narrow
B) they support the basilar membrane D) spherical rather than biconcave erythrocytes. by the thin actin filament. This is accomplished as connection at the apex called helicotrema.
C) they are vestigial organs without function cross bridges from the myosin ―pull‖ the actin Statocyst is mechanoreceptor of invertebrates that
D) they are contained in the macula filament. During the contraction of a sarcomere, functions as organ of equilibrium.
passive tension does not develop. Answer B and C the Z lines move closer to each other, the I-band 14. The Answer is B
Answers and Explanations: Cell, CV, NS, are incorrect because muscle contracts and thus, of each sarcomere shorten and the H zone The cilia of sensory hair cells located on the
Muscle tension development can occur at shorter muscle gradually disappear., The A band remain virtually basilar membrane of the cochlea are embedded in
1. The Answer is B length. the same length since it corresponds to the tectorial membrane. Distortion of these
The ventricles are the two lower chambers of the 3. The Answer is A the length of the myosin filament. sensory hairs causes nerve impulses to be sent to
heart and are responsible for the pumping of the Tension development in a muscle is a function of 9. The Answer is A specific region of the cerebral cortex where they
blood: The right ventricle pumps blood through the intersection which occurs between actin and Synapses are two types: electrical and chemical. are interpreted as sound of a particular frequency.
the pulmonary artery into the lungs, and the myosin filaments. Answer (B) is incorrect because Electrical synapse occurs between cells that are The structure indicated by the other choices for
left ventricle pumps blood to the aorta into the a physical interaction must occur not merely in joined by gap junction, which are found in both this question are involved in hearing but are not in
systemic circulation. The atria are the two upper overlapping of the filament in order for cardiac and smooth muscle direct contact with the sensory hair cilia.
chambers of the heart responsible for the receiving contraction to occur. Answer C is wrong because 10. The Answer B 15. The Answer is A
of blood from the body. Pacemaker initiates the a slackening would not contribute to tension As an AP moves along a muscle cell, calcium ions Blood clotting is initiated by damaged platelets +
beating of the heart. Cardiac muscles make up the development at all. Answer D explains the released from the SR bind with one of two calcium to liberate thromboplastin which acts on
wall of all four chambers of the heart. differences in tension development at different regulatory proteins found as part of the thin prothrombin and converts it to thrombin.,
2. The Answer is D muscle length, but, not only tension develops filament. The protein which calcium bind is thrombin initiate the conversion of fibrinogen to
Passive tension develops when connective tissue 4. The Answer is B known as the troponin complex. This binding fibrin to produce fibrin clot, an insoluble fibrin
and elastic element with in the muscle are The SA node is a small strip of specialized muscle cause a slight conformational change in the other 16. The Answer is C
stretched, as opposed to active tension which is in the wall of the heart RA. This node has the regulatory protein known as tropomyosin because Three kinds of responses do not involve the brain,
produced by the actual muscle contraction. Thus, contractile properties of muscle and can transmit of this change the myosin cross bridges can then for conscious input is unnecessary and would slow
there is no passive tension when muscle shortened impulses like a nerve. The SA nodegenerated the attach to specific sites on the actin and the the response. Instead, the Impulse is carried from
to less than its resting length because the muscle is rhythmic self-exciting impulse which causes a contraction can proceed. the receptor neuron directly to the spinal cord and
not being stretched. Answer A is wrong because wave of contraction across the wall of the atria. 11. The Answer is D then back pout through a motor neuron to the
121 122
Dhruv N Desai Dhruv N Desai
appropriate muscle. Thus, the receptor neuron, the regeneration since it remains in continuity with uses the energy in the sodium gradient to help pressure, the mitral and tricuspid valves and rapid
motor neuron, and the muscles are the only perikaryon. Chromatolysis is the first step in the maintain the low intracellular calcium required fro filling of the ventricles begins.
structure necessary for thereflex action to occur. regeneration process in which there is breakdown normal cell function. Although sodium enters the 30. The Answer is D
17. The Answer is B of the Nissl substance, swelling of the perikaryon, cell during AP, the quantity of Na is so small that During the plateau phase of the cardiac AP, k
T and B lymphocytes response to specific and lateral migration of the nucleus of the neuron. no significant change in the intracellular sodium conductance decreases below its resting value
invading organism. Erythrocytes are red blood Regeneration is dependent on the proliferation of concentration occurs. Because the Na while calcium conductance is greater than it is at
cells which contain oxygen-transporting Schwann cells, which serve to guide sprouting transferences are so low, the Na equilibrium rest. However, the decrease in K conductance is
hemoglobin. Phagocytes are scavenge cells and axons from the proximal segment toward the potential is not important determinant of the greater than the increase in Ca conductance, so
are not responsible for the immune process. target organ that is being reinnervated. This resting membrane potential. total membrane conductance decreases. The N
Reticulocytes are young RBC just after loss of process is referred to Wallerian regeneration. 28. The Answer is D channels inactivate during the plateau phase,
their nuclei and mature into erythrocyte. Degeneration of perikarya and neuron processes Blood flow through the coronary vessels of the returning Na conductance to its resting value.
18. The Answer is C occurs when there is extensive neuronal damage. left ventricle is determined by the ratio 32. The Answer is B
B cells or b lymphocytes produce antibodies after Transnueronal degeneration only occurs when ofperfusion pressure to vascular resistance. The Stroke volume is determined by preload,
stimulated by a specific antigen. T cells or T there is a single input (synapse) with another perfusion pressure is directly treated to the afterload, and contractility. Increasing preload by
lymphocytes directly attack foreign cells or neuron, In the presence of multiple inputs, aorticpressure at the opening of the coronaries. increasing central venous pressure will increase
substances in the cell mediated immune response. Transnueronal degeneration does not occur. Myocardial vascular resistance is significantly stroke volume. Similarly, decreasing afterload by
Phagocytes are scavenger cells. 25. The Answer is D influenced by the contractile activity of the decreasing total peripheral resistance or systemic
19. The Answer is A The single muscle twitch generates only a single, ventricle. During systole, when the ventricle is blood pressure will cause an increase in stroke
T cells or T lymphocytes participate in cell sudden contraction. During summation, individual contracting, vascular resistance increases volume. Increasing contractility will also increase
mediated immunity – attacking foreign cells or muscle twitches are added together to make strong substantially. Flow is highest just at the beginning stroke volume. Cardiac output equals stroke
substances. B cells produce specific antibody muscle movements. Indeed, the tension developed of diastole because during this phase of the volume times heart rate. If the heart rate increases
20. The Answer is C during summation is much greater than during the cardiac cycle, aortic pressure is still relatively and CO does not change, stroke volume will
The plasma or cell membrane in eukaryotes single muscle twitch. When a muscle is stimulated high and vascular resistance is low due to the fact decrease.
consists of a phospholipids bilayer, a double layer at progressively greater frequencies, activation of that the coronary vessels are no longer being 33. The Answer is D
of phospholipids with their relatively hydrophilic the contractile mechanism occurs repeatedly squeezed by the contracting myocardium The ECG records the conduction of the AP
(polar) head on the outside and their hydrophobic before any relaxation has occurred and the 29. The Answer is C through the heart. Changes in the rate, rhythm or
(non-0polar) fatty acid tail pointed inward. successive contraction fuses into one continuous The pressure gradient between regions of the CV conduction pathway are recorded. Changes in the
Different protein and carbohydrates may or may contraction. Such response is called tetanus. system is directly proportional to the resistance of position of the heart in the chest will change the
not be present, depending on the cell type. During complete tetanus, there is no relaxation the intervening structures. During ventricular size and shape of the ECG recorded by various
21. The Answer is C between stimuli; during incomplete tetanus there ejection the aortic valves are open and do not offer leads. Local; areas of ischemia caused by changes
The nucleolus is the region of the mRNA are period of incomplete relaxation between the any significant resistance to blood floe. Therefore in coronary blood flow will, cause changes in the
synthesis and the assembly of ribosomal unit. It is summated stimuli. The tension developed during there is very little, if any, pressure difference AP that will be reflected in the shape of the ECG
formed around the nucleolus organizing region on complete tetanus is about four times he developed between the LV and the aorta. Since tricuspid recording. The ECG is unable to detect any
a particular chromosome, the region of DNA by the individual twitch contractions. valve is closed during ventricular ejection, there is changes in the ability of the heart to develop
which is codon to mRNA. 26. The Answer is A an appreciable pressure difference between the force.
22. The Answer is B During the relative refractory period, an action RV and the AA. Although pulmonary vascular 34. The Answer is B
The rough and smooth ER is continuum with the potential can still be elicited, but the stimulus resistance is relatively small compared with The two major factors that influence pulse
Golgi complex. A vesicle is a membrane bound must be stronger than normal. The larger the systemic vascular resistance, it nonetheless pressure are stroke volume and arterial
sac. The peroxisome is a type of vesicle which stimulus is required because the threshold is produces a pressure drop between the RV and the compliance. Decreasing stroke volume reduces
contains peroxide forming and destroying increased owing to the increase in potassium LA. Since most of the resistance in the systemic pulse pressure, whereas decreasing arterial
enzymes. Lysosomes are vesicle which contain conductance and sodium inactivation that occur vasculature occurs at the level of the arteriole, compliance increases pulse pressure. A decrease
hydrolytic enzyme,, microtubules are part of the during the AP. These changes in membrane there is a large pressure gradient between the aorta in venous compliance would cause an increase in
cytoskeleton. Mitochondria are double membrane permeability are also responsible for causing the and the capillaries central venous pressure, which would tend to
organelle. decreases in the refractory period. The decrease in 30. The Answer is D increase stroke volume. An increase in myocardial
23. The Answer is C the overshoot potential causes a decrease in the Closure of the Semilunar valves (aortic and contractility would also tend to increase stroke
tropomyosin, a linear protein attaches to the actin, number of K channels that open during AP. Thus pulmonic vales) marks the beginning of the volume and, therefore, a decrease in pulse
and covers the myosin binding site when the the repolarization phase of the AP is slower than isovolumetric relaxation phase of the cardiac pressure.
muscle is not contracting. normal. cycle. During this brief period (approximately 35. The Answer is D
24. The Answer is D 27. The Answer is C 0.06sec.), the ventricles are closed and myocardial During vigorous aerobic exercise there is a
Axonal regeneration occurs in neurons if the The Na-K pump uses the energy contained in ATP relaxation, which began during protodiastole, pronounced decrease in vascular resistance in
perikarya survive following damage. The to maintain the sodium gradients across the continues. Intraventricular pressure falls rapidly, skeletal muscle, which lowers total peripheral
segmental distal to the wound, including the membrane. The sodium gradient, I n turn, is used although ventricular volume changes little. When resistance. It it were not for the increase in CO
myelin, is phagocytosed and removed by to transport pother substances across the intraventricular pressure falls below atrial that occurs with this kind of exercise, blood
macrophages. The proximal segment is capable of membrane. For example, the Na-Ca exchanger pressure would decrease. The primary factor that
123 124
Dhruv N Desai Dhruv N Desai
contributes to the increase in blood flow to The total circulating blood volume is correct the problem completely, stroke volume circulation keep blood flow in the brain from
exercising skseletal muscle is the local approximately 70 ml/kg; about 2/3 is found in the would at best be returned to control value. changing.
accumulation of vasoactive metabolites. systemic veins and venules. A significant volume 43. The Answer is D 46. The Answer is D
Stimulation of sympathetic nerve fibers that of blood (15%) is found in the pulmonary The Baroreceptor reflex decreases blood pressure The olfactory receptor cells and peripheral
innervates blood vessels within exercising skeletal circulation. Smaller quantities are found in the when the mean arterial pressure suddenly rises [processes are located in and developed from the
muscle would tend to increase vascular resistance heart (5%), the arterial system (11%), and the and increases blood pressure when the mean epithelium of the nasal mucosa. The central
and decrease flow. Local metabolites overpower capillaries (5%). The large volume of nboo0d arterial blood pressure suddenly falls. Stretch pouf processes of receptor cell pass through the
the effects of sympathetic stimulation so that flow found on the venous side of the circulation is used the carotid sinus baroreceptors is reduced when cribriform plate of the ethmoid bone to terminate
can increase despite high levels f sympathetic to adjust circulating blood volume. For example, there is a decrease in blood pressure, and in the olfactory bulb. Fibers from the bulb run
activity. during hemorrhage, contraction of the veins and therefore, the reflex responses to a reduced stretch through the olfactory tract to the olfactory cortex.
36. The Answer is C venules of the skin increases the amount of blood of the carotid sinus baroreceptors all tend to 47. The Answer is D
A reduction in carotid sinus pressure due to a available for perfusion of the heart and brain. increase blood pressure. These reflex responses In cardiac muscle, T system is located at the Z
decrease in mean blood pressure due to a decrease 40. The Answer is A include an increase in sympathetic nervous system lines. This is in contrast to skeletal muscle, in
in mean blood pressure would elicit a The critical factors affecting the flow of blood of activity and decrees in activity of the vagal fibers which the T system is found at the junction of the
Baroreceptor reflex tending to restore blood incompressible fluid pipes were described by the that innervate the heart. Increasing sympathetic A Band and I band. The long duration of cardiac
pressure to normal. The reflex response include an English physicist Osborne Reynolds. He nervous activity increases heart rate and AP is largely due to slow, inward calcium current
increase in sympathetic nervous system activity, discovered that the point at which flow changes contractility, which; leads to increase in CO. that is expressed during the plateaus phase of the
which would cause an increase in heart rate and from laminar smooth) to turbulent is a function of Increased sympathetic activity also increases AP. The calcium that enters during this phase
myocardial contractility, both of which would fluid density, viscosity, and velocity and of the arteriolar tone, which increases total peripheral contributes to the contractile response.
tend to increase CO. sympathetic stimulation diameter of the vessels. Increasing the length of resistance and blood pressure. The reduced vagal 48. The Answer is D
would also cause constriction of both arterioles the vessel may indirectly decrease the likelihood nerve activity allows the heart rate to increase and The concentration of cation affects cardiac
and venous vessels. Arteriolar constriction would of turbulence by increasing vascular resistance thus contributes to the increase cardiac output and function. Ultimately, the ionic basis of coupling of
cause an increase in total peripheral resistance. and thus decreasing blood velocity. blood pressure following a decreased stretch of excitation and contraction resides in free
Sympathetic stimulation of the venous vessels 41. The Answer is C the carotid sinus baroreceptors. intercellular calcium. However, superphysiologic
would cause a decrease in venous The activation of platelets and formation of fibrin 44. The Answer is D levels of k ions may arrest the heart diastole, and
capacitance. strands contribute to blood clotting sand Autoregulation is the maintenance of a constant lack of Na ions will prevent an isolated perused
37. The Answer is D hemostasis. Exposure to platelets collagen, blood flow in the presence of a change in arterial heart from beating. Contraction occurs as a result
Blood flow through an organ is increased by thrombin, and thromboxane A2 causes activation pressure. Two mechanisms have been used to of entry of calcium ions from the interstitial fluid
either increasing the perfusion pressure across the of the explain autoregulation, the myogenic and (especially T-tubules) and relaxation occurs by the
organ or by decreasing the vascular resistance. A platelets. The conversion of fibrinogen to fibrin is metabolic theories. The myogenic theory proposes removal of calcium ions from them myoplasm by
decrease in the arterial [pressure would decrease essential for the production of fibrin strands to that an increase 0or decrease) min perfusion the SR.
the perfusion pressure. Decreasing the diameter of trap blood component in the forming clot. Plasmin pressure causes a contraction (or relaxation) of the 49. The Answer is B
the arterial or venous vessels or decreasing the is an enzyme that contributes to the lysis of blood arteriolar muscle, thus reducing 9or increasing) All contractile cells contain actin and myosin. In
number of open arterial vessels would contribute clots. The injection of lytic drugs has become an blood flow toward normal. The metabolic theory non-muscle cells, such as macrophages,the
to increasing vascular resistance. If the hematocrit important clinical tool in the prevention of proposes that blood flow is adjusted to keep the contractile elements are important for mobility
is decreased, the viscosity of the blood is myocardial infarction. concentration of metabolic by products at a and shape changes. The mechanics ofcontraction
decreased resulting in a decrease in resistance and, 42. The Answer is C constant level. The changes in blood flow in seem to be similar, using ATP hydrolysis as a
therefore, an increase in blood flow through an Following the loss of blood there is a reduction in response to overall homeostasis (e.g. regulation of driving force. In all cell types, thecytoskeletons
organ, preload, which results in a decrease in stroke temperature or blood pressure during are composed of contractile filament. Unlike
38. The Answer is D volume. The direct consequence of the decrease in hemorrhage_ 09r specific tissue needs (e.g., the muscle cells, the contraction of nonmusclecells
Cerebral blood flow is under local metabolic stroke volume is a reduction in CO and dilation of coronary arteries when energy does not seem to be governed by nervous
control. The increase in H, carbon dioxide, and K secondarily a decrease in blood pressure. This requirements of the heart increase during exercise) stimulation.
that accompanies activity causes increases in reduction in blood pressure would be detected are not classified as autroregulatory processes. 50. The Answer is A
cerebral blood flow. Hyperventilation causes a bythe baroreceptors, leading to an increased 45. The Answer is B Myosin contains the ATPase activity that
respiratory alkalosis, which, by decreasing brain activity of the sympathetic nervous During exercise, metabolism and cardiac output hydrolyzes ATP and allows contraction to
H ions concentration (increasing pH), decreases system.Sympathetic stimulation would cause an increase, Blood flow to the skin increases to aid in proceed. The binding of actin to myosin enhances
cerebral blood flow. Increasing central venous increase in heart rate and total peripheral the dissipation of heat while blood flow to the the ATPase activity of myosin. In fact, actin
pressure decreases the perfusion pressure across resistance. Sympathetic stimulation would also heart increases to provide adequate oxygen and alternatively binds to myosin and is released from
the brain vasculature and thus impedes cerebral lead to an increase in myocardial contractility. An nutrients and to remove wastes. During exercise, myosin as ATP is hydrolyzed. Although troponin
blood flow. The brain is protected from an increase in ejection fraction could result from the systemic resistance falls because of the extensive is not directly involved in the ATPase reaction, it
increase in blood flow during hypertension by increase in contractility and reduced afterload. All vasodilation in the exercising muscles. Blood flow binds calcium released from the SR and in doing
autroregulatory mechanism. the reflex compensations described above help to to the intestine and kidney decreases in order to so allows conformational changes in tropomyosin
39. The Answer is D return blood pressure to normal when stroke maintain adequate blood pressure. and action to occur, permitting contraction.
volume is reduced. Even if compensation were to Autroregulatory mechanisms within the cerebral
125 126
Dhruv N Desai Dhruv N Desai
Myokinase catalyzes the formation of ATP and and beta effects. The alpha and beta effects the circulation and as one substance that platelets relationship between sodium excretion and blood
AMP from two molecules of ADP. constrict the smaller arterioles and precapillary initially stick to when they from a hemostatic pressure. Answer C is incorrect because counter-
51. The Answer is C sphincter, thereby markedly reducing cutaneous plug. current system generally refer to fluid exchange
Although metabolic acidosis may occur and H+ blood flow. Veins and large arteries also respond 57. The Answer is D systems, in which fluid and solutes exchange
increases, the initial compensatory response to to epinephrine. The Beta effects of epinephrine The depolarization observed in the p wave signals rapidly between parallel streams flow.
hemorrhage results in large increase in total cause relaxation of the bronchial smooth muscle the onset of atrial contraction, whereas the QRS Answer D is the opposite of negative feedback,
peripheral vascular resistance. The loss of blood and induce a powerful bronchodilatation which is complex is associated with the initiation of and in effect it promotes disequilibrium.
initially decrease cardiac output, but baroreceptor- most evident when the bronchial muscle is ventricular contraction. .the sustained 63. The Answer is D
mediated sympathetic drive causes contracted, as in anaphylactic shock. Neither depolarization of the plateau is represented by the The hypothalamus contains osmoreceptors
vasoconstriction. Thus, vascular resistance Norepinephrine nor dopamine would be the drug ST interval (which is not normally associated with responsible for detecting increases in extracellular
increases, heart rate increases, and blood pressure of choice, since neither has action on the beta2 any voltage deflection). Finally, the T wave is osmolarity. These osmoreceptors produce the
return toward normal. Slightly later, the kidney receptor, and therefore, would not cause the associated with the onset of ventricular sensation of thirst, increase drinking, and cause
may secrete renin and the product of angiotensin bronchodilatation needed for treating anaphylactic repolarization. the release of ADH. Thermoreceptors in the
II via converting enzyme activity ultimately shock. Isoproterenol has a powerful action on all 58. The Answer is C anterioe hypothalamus measure core temperature.
ensues. Fluid also will shift from the interstitial beta receptor but almost no action on alpha The tactile corpuscles are found in the dermal Other hypothalamic neurons are involved in the
compartment to the vascular space, helping to receptor, so vasodilation instead of papillae. The end bulbs vary greatly in dimension initiation and coordination of heatconserving and
restore cardiac output. Other humoral agent, vasoconstriction would produce. and have a wide distribution. The neuromuscular heat-losing mechanisms. The hypothalamus also
including epinephrine, vasopressin, and 55. The Answer is D spindles are widely scattered in the fleshy bellies serves as a component of the limbic system, which
Glucocorticoids may also be released to further Although the capillaries are the smallest vessels, of skeletal muscle. The pacinian corpuscles are is responsible, in part, for mediating emotional
compensate for the cardiovascular effects of by virtue of the large number and the largest and most widely distributed. behavior. Respiration is controlled by respiratory
hemorrhage. parallelexistence, their effective cross-sectional 59. The Answer is A centers of the brain stem.
52. The Answer is D area is very large. Since velocity is inversely A wide variety of environmental conditions 64. The Answer is D
Although short-term regulation of arterial blood related tocross-sectional are, the velocity in the provoked several mechanisms to come into play to the action of acetylcholine is terminated by
pressure is primarily affected by in the integrated capillaries is very low. The large surface area and maintain body temperature by balancing heat acetylcholinesterase, which hydrolyzes Ach
response of peripheral baroreceptors and the CNS lowvelocity promote exchange of substance production and heat loss. The loss of heat via to acetate and choline. The choline is pumped into
and sympathetic NS, the primary determinant of between blood and tissue. Resistance to blood radiation accounts for more than 60% of the the nerve terminal and used in the re-synthesis
regulation of blood pressure is the long run in the flowprimarily occurs in arterioles with smooth normal heat loss. Conduction of heat to objects or of new Ach. All other transmitters are inactivated
relationship of urine output to fluid intake. This muscle, and thus, this is the site of the larges to air (i.e., convection) accounts for 15%, and by re-uptake into the nerve terminal.
system is normally capable of returning blood pressure edrop. Blood volume is greatest in small evaporation accounts for about 25%. Sweating is 65. The Answer is D
pressure to normal level, which is different from vein by nature of their high compliance. an important form of heat loss and is regulated by The catecholamine Norepinephrine and
the short-term nervous regulation. By adjusting 56. The Answer is D various mechanisms. Insensible perspiration epinephrine will activate both alpha- and
ECF and blood volume, renal-body fluid The simplest form of blood clot at a site of injury through the skin and lungs although important, betaadrenergic receptors. . When alpha1
mechanisms alter venous return. The total is a hemostatic plug. It is composed of an remain relatively constant despite environmental adrenergic receptors are stimulated it increases
peripheral vascular resistance is thus altered by aggregate of platelets with a web of fibrin, which changes and thus does not provide major muscle contraction. Alpha1 adrenergic receptors
these mechanisms rather being the variable that prevents leakage of blood into the intravascular mechanisms to regulate body temperature. predominate on arteriolar muscle, sp these
directly determine AP. spaces. Platelets are the most component in the 60. The Answer is B muscles contract when stimulated with
53. The Answer is D formation of this plug. When the blood vessel is Increasing the membrane‘s conductance to K will norepinephrine. The bronchiolar, papillary and
The activation of factor X is the final reaction of injures, cells and plasma start to leak out, but result in the membrane potential approaching the ciliary smooth muscle all contains beta receptor,
both the extrinsic and intrinsic pathways of platelets are immediately attracted to the site of value dictated by the K Nernst Potential, which is which cause smooth muscle relaxation. The
clotting. Activated factor X proteolitycally cleaves injury. They accumulate, pile up, and stop the about -85mV. intestinal muscle relaxation is initiated by an
prothrombin to thrombin, which in turn cleaves leakage. They also release tissue thromboplastin, 61. The Answer is D alpha2 adrenergic receptor.
fibrinogen to fibrin. Accelerin stimulates the which activates the intrinsic blood coagulation The prolonged depolarization of the plateau phase 66. The Answer is D
activities of factor X and fibrin stabilizing factor pathways, causing the fibrin mesh to form. The of the cardiac AP is attributed to slowly CSF, which is in osmotic equilibrium with the
(Factor XII) stabilizes the clot by cross-linking fibrin tightens the plug and traps other cells, inactivating voltage-gated calcium channels. ECF of the brain and spinal cord, is formed
fibrin. All of these factors are parts of the strengthening the platelets plug and forming a 62. The Answer is B primarily in the choriod plexus by an active
common pathway. The defect in hemophilia is more permanent plug. RBC and lymphocytes are To answer this question, the concept of a feedback secretory process. It circulates through the
deficiency in Factor VIII or anti-hemophilic seen in hemostatic plug as they are trapped from system must be understood. Negative feedback subarachnoid space between the dura mater and
factor. This factor acts at the last step of the the circulating blood by the aggregation of occurs when a change in a variable sets in motion pia mater and is absorbed into the circulation by
extrinsic pathway. Factor VIII acts in concert with platelets and fibrin. They act as filler materials in a series of events which are designed to restore the the arachnoid villi. The epidural space, which lies
Factor IX, a proteolytic enzyme to activate Factor the plug and have no other defined role in the variable to its original condition. In this case, outside the dura mater, may be used clinically for
X formation of hemostatic plug. Collagen is when blood pressure increases, sodium excretion instillation of anesthetic. CSF protein and glucose
54. The Answer is A important In the initiation of hemostasis, as when will increase until blood pressure decreases back concentration are much lower than those plasma.
Epinephrine is the drug of choice for treating blood vessels are damaged. The collagen fibrils in to its original level. Answer A is incorrect because Changes in those concentration in the CSF are
severe anaphylactic shock, since it has both alpha the endothelial wall of the vessels are exposed to hormones are not directly involved in the helpful in detecting pathologic processes, e.g.
127 128
Dhruv N Desai Dhruv N Desai
tumor or infection, in which the blood-brain fibrin through the actin of platelet phospholipids. 14. Urethral glands are found in ? - Man
barrier is disrupted. Thrombin is also involved in this conversion, but
67. The Answer is D it is a plasma protein, not a platelet secretory 15. Fructose and citric acid are secreted from which accessory gland – Seminalvesicles.
The cochlear hair cells are the functioning factor. Platelets are not required for the initiation 16. High content of ergothionine and inositol in vesicular glands is characteristics of which species .Boar
auditory receptor. Neural pathways from the hair of the blood clotting cascade, but they are required 17. Nerve supplying sensory fibres to vagina , vulva and clitoris.- Pudic
cells synapse with the auditory cortex. The hair for the adherence and normal formation of a clot.
cells are contained in the macula 9otolith organ Plasmin is not secreted by platelets but is formed 18. Sex cords of female are called...Medullarycords
and overlaid by the otolithic membrane. by the conversion of plasma-derived plasminogen 19. In females --Mullerian--- ducts develop into gonadal system while in male-Wolffian-ducts develop.
68. The Answer is D under the influence of plasminogen activator
20. Vestibule arises from----------Urogenitalsinus
The Golgi tendon organ is located in the tendon of secreted by endothelial cells..Plasmin is involved
skeletal muscles and therefore is in series with the in dissolution, not formation, of blood clots. 21. The endocrine cells of ovary originate from—Ovarianmedulla---------
muscle. Each time the muscle contracts, the Thrombocytopenia is a reduction in thenumber of 22. Oocytes surrounded by one layer of flattened cells –PrimordialCells-------
tension developer by the muscle causes the GTO platelets. Under this condition, fibrinogen will not
23. Ovulation generally occurs in response to –LHsurge
to be stretched... the afferent fibers, which be converted to fibrin in sufficient quantity to
innervate the GTO, fire in proportion to the allow normal clotting. The absence of platelets 24. Follicular development is enhanced / suppressed in ovary containing corpus luteum ?
amount of GTO stretch, and therefore their firing aggregation interferes with the normal 25. Second polar body is formed at the time of ---Fertilization-------
rate provides the CNS with information about the maintenance and repair of endothelial injury. The
amount of tension developed by the muscle. The endothelium becomes increasingly leaky and 26. At ovulation ova of cattle, sheep and swine contain ---one-------- polar body.
muscle length and speed of eventually may permit thrombocytopenia purpura 27. At ovulation ova of horse, dog and fox are in --Firstmaturation---division.
shortening is sent to the CNS by afferents that with seepage of blood from the vessel. 28. At ovulation the oocyte liberated in cattle is --Seconary----------
innervate the intrafusal fibers within muscle 70. The answer is A
spindles. In iron deficiency, anemia results with the 29. At ovulation the oocyte liberated in equines is -------Primary---
69. The Answer is C presence of smaller, pale-staining erythrocyte 30. Primary spermatocyte gives rise to -----Four-------- spermatozoa.
Platelets (thromboytes0 are cell fragments (microcytic, hypochromatic).In hemolytic anemia 31. Primary oocyte fives rise to --one----------- egg.
obtained by the break up of megakaryocytes. there is excessive destruction of RBC in the
These cell fragments contain a number of spleen. Hypochromic, macrocytic anemia results 32. The regression of corpus lutea begins by day --------15-16------ in cattle.
important substances as well as cytoskeletal from vitamin B12 deficiency. The presence of 33. Mature corpus luteum is smaller than mature graffian follicle in the-.---Mare----------
elements involved in biologic processes such as a spherical rather than biconcave erythrocytes is
34. Corpus luteum lysis is ---Estrogen-------- induced in cattle and sheep.
clot retraction. Platelets function in aggregation, associated with spherocytes, which often results in
coagulation, clot re traction, and removal. They hemolysis. 35. Intrauterine injection of -------Indomethacin----------- blocks estrogen induced corpus luteum lysis in cattle
are involved in the conversion of fibrinogen to 36. The functional segments of oviductfimbriae, Infundibulum ,Ampulla, Isthmus
37. PGE3 has a -Relaxing--------------- effect on oviduct .
38. Uterus of cow, ewe and mare is --Bipartite---
Animal Reproduction 39. Uterus of sow is---Bicornuate-------
40. Oviduct is supplied blood by ---Utero - ovarian-------
1. Desirable concentration of actively motile spermatozoa per dose of frozen bull semen. -10-15 million
41. Blastokinin, a protein which influences blastocyst formation is secreted by uterus of---Rabbits-------
2. Commonly used model of AV for bulls.- Danish
42. Fern pattern of cervical mucus is associated with high ----Chloride---------- content.
3. Temperature time protocol needed for destroying spermicidal factor in milk.-92-95°C for 10-12 minutes
43. pH of vaginal secretion is favourable / unfavourable to spermatozoa ?
4. Spermicidal factor present in fresh milk.-Lactanin
44. Gartner‘s ducts are remnants of ---Wolffianduct…
5. Volume of semen dependent upon the secretions from seminal vesicles.-Seminalvesicles
6. -------present in goat seminal plasma causes coagulation when sodium citrate is added.- Lysolecithin 45. Depleted secretory cells of oviductal musculature…Pegcells
7. Freezing point depression of bull semen- (– 0.55°C) 46. Cervix possesses / doesnotpossess glands ?
8. Dose of penicillin G sodium per ml of extended semen.-500-1000 IU 47. FSH and LH are chemically ----Glycoproteins…
9. Distance between grill and straw rack during semen freezing- 4 cm 48 ------------ causes crop milk production in pigeons. – Prolactin
10.Which is better ? rapid / slow freezing. 49. The long half life of PMSG is due to -------Sialicacid----
11. Dose of dihydrostreptomycin sulphate per ml in extended semen.-500-1000µg 50. PMSG is formed by endometrial cups which are of ----Foetal--------- origin.
12. ---------------ovary is physiologically more active. -Right
13. Shape of non-pregnant uterus in mare- Cruciform A. OBJECTIVE QUESTIONS

129 130
Dhruv N Desai Dhruv N Desai
1. Animal showing external evidence of pro-oestrus with vulval oedema, hyperemia & sanguinous 18. An animal in which pheromones are secreted in saliva foam
vulval discharge is a) Boar b) Bull c) Stallion d) Ram
a) Cattle b) Bitch c) Doe d) Ewe and Mare
19. In the testes , testosterone secreting cells are -----------
2. Mammary gland duct system growth is under the influence of a) Germinal epithelium b) Leydigcells c) Sertoli cells d) Blood testes barrier
a) Estrogen b) Progesterone c) Prolactin d) Prostaglandins
20. First scientific research in A.I in domestic animals was conducted by Italian physiologist -------------- in
3. Mammary gland alveolar growth is under the influence of 1780.
a) Estrogen b) Progesterone c) Prolactin d) Prostaglandins a) Leeuwenhoek b) G. Amantea c) L.Spallanzani d) Rapiquet

4. Exogenous oxytocin has luteolytic action in ---------- 21. ---------------- (1963) developed special cotton plug consisted of polyvinyl alcohol powder for sealing
a) Bitch b) Cow & Ewe c) Cattle & Sow d) cow & Doe straws.
a) Sorenson b) Nishikawa c) Van Demark d) Cassou
5. Upto secondary spermatocyte stage-------------- hormone acts, after which testosterone regulates
spermatogenesis
a) Growth hormone b) F.S.H c) I.C.S.H d) Insulin e) Androgens

6. In birds & reptiles, ------------- is important in contraction of shell glands & vagina to induce
oviposition
a) Oxytocin only b) F.S.H & L.H c) Prolactin & vasopressin d) vasotocin

7. Predominate Ig in follicular fluid is ---------


a) Ig A b) Ig M c) Ig G d) Ig E

8. One primary spermatocyte produces -----------


a) 4 Spermatids b) 64 Spermatids c) 1 Spermatid d) 16 Spermatids

9. One spermatogonia produces--------------spermatids


a) 4 b) 1 c) 64 d) 16

10. B-Spermatogonia is formed after -----------------stage


a) A 4 b) Intermediatespermatogonia c) Primary spermatocyte d) Secondary Spermatocyte

11. 4 - Cell stage embryo is transported from site of fertilization to uterus in


a) Sow b) Mare c) Ewe d) Cattle

12. Transuterine migration of embryo is absent in -------


a) Bitch b) Cattle c) Sow d) Both a and c

13. Maternal Recognition of Pregnancy (M.R.P) is responsible for


a) Fetal growth b) Implantation c) Maternal circulation d) Fertilization e) Parturition

14. First maturation division / meiotic division is not completed at the time of ovulation in
a) Sow b) Cattle & Buffalo c) Ewe & Doe d) Mare&Bitch

15. Hippomanes are usually found in ---------


a) Yolk sac b) Amniotic fluid c) Allantoicfluid d) All of these

16. Steroid hormones have receptors in ---------


a) Cytoplasm b) Nucleus c) Cell membrane d) Both a & c

17. An anabolic hormone --------


a) Insulin b) Estrogen c) Testosterone d) Allofthese
131 132
Dhruv N Desai Dhruv N Desai
B. FILL IN THE BLANKS Answer of B –fill in the blanks
1) Acrosome reaction is an indicator of completed------------------- 1) Capacitation 16) ―Balling Up‖
2) Birds & Reptiles 17) Onanism
2) Polyspermy is common in---------and------------- 3) Mare 18) Pedarasty (Rectal Copulation)
3) The C.L persists through out pregnancy in all farm animals except---------- 4) Equine , PMSG / eCG 19) Rabies, Dourine
4) Endometrial cups are a unique feature of ---------- placenta which secrete---------- hormone. 5) Glucose 20) Horse , Pig
6) Fructose 21) Inhibin
5) --------- is the major metabolic fuel for foetus
7) 17-α- hydroxylase 22) HIOMT( Hydroxy Indole –O-
6) Although --------- comprise 70-80% of sugar in fetal ungulates (sheep, goat, cattle) and does not cross 8) Secundus , Second cleansing Methyl Transferase)
placental barrier. 9) Right , opposite 23) Stewart
10) Tom (Male Cat) 24) Perks
7) Fetal cortisol act on placenta to induce ----------- enzyme which convert progesterone to estrogen to have
11) Dizygotic , monozygotic 25) Glass Ampule
role in parturition. 12) Cerebellar , brain stem lesions 26) Buck
8) Lochia , the post partum uterine discharge is also known as -------- or ----------- 13) Horses 27) Glycerol
9) Rate of ovulation is more in ---------- ovary of cattle but ---------- in mare 14) Multipara (Dog & Cat) 28) Sorenson
15) Boar , Tom ( Male Cat)
10) Glans penis is absent in -----------(species)
11) --------- twins are much more common than --------- twins
12) Doddlers are due to a pair of autosomal recessive genes causing ------------ or other------------- C. MATCH THE FOLLOWING
13) Congenital valvular defects are common in --------------(species) 1)
14) Super fecundation is more common in ---------------- A. Homosexual behaviour ------ 1. Cat
15) Scrotum of domestic animals is located between a thigh except ---------- and------- in which scrotum is B. Frequent urination in heat ------- 2. Buffalo
C. I.V.R.I Crystoscope ------- 3. Early Embryonic Mortality
located caudal to thighs.
D. Irregular long estrous cycle ----- 4. White Side Test
16) Boars masturbate by inserting their penis inside the preputial diverticulum & ejaculate , the condition is E. Endometritis ----- 5. Cattle
termed as --------------- F. Rage Reaction -- ---- 6. Fern Pattern
17) Masturbation in animals is also termed as---------------
a) 2, 5, 6, 1, 3, 4
18) Young boars in artificial insemination studs are separated to prevent ------------- b) 5, 2, 4, 3, 6, 1
19) Paralysis and paraphimosis of penis is seen bulls diseased with--------- & in horses in late stage of ----- - c) 5, 1, 4, 2, 6, 3
--- (Disease) d) 5, 2, 6, 3, 4, 1
20) Inguinal hernia is considered a common breeding defect in ------ & ----- (Species)
21) The hormone produced by Sertoli cells in male & granulosa cells in female is---- 2) A. Pseudo pregnancy ----- 1.Dog & Cat
22) The enzyme involved in melatonin synthesis & found only in Pineal gland is ------------ B. Prostaglandin antagonist ----- 2. Ruminants
23) ------------ (1951) reported the birth of first calf from insemination with frozen semen in cooperation C. Endotheliochorial placenta ---- 3.Carazolol
D. Epitheliochorial placenta ----- 4. Bromocriptine
with Polge & Smith.
E. Accelerating parturition ----- 5 .Horse & Pig
24) ----------- (1955), first time used pellets as packaging material. F. Synepitheliochorial placenta ---- 6. Indomethacin
25) Macpherson , Van Demark & Kinnoy (1954) developed freezing of semen in --------(Packaging
material ) a) 4, 3, 6, 1, 5, 2
b) 6, 3, 5, 2, 4, 1
26) Egg yolk coagulating enzyme (EYCE) / Coagulase / phospholipase /Tricyl glycerol lipase is found in c) 4, 6, 1, 5, 3, 2
bulbourethral secretion of ----------- semen, which interacts with milk constituents in milk diluents & inhibit d) 6, 1, 5, 2, 4, 3
motility of spermatozoa.
27) -------------- is the most commonly used cryoprotectant for freezing of semen. 3) A. Post partum vaginal discharge ---- 1. Glycoprotein
28) --------------- (1948) in Denmark for the first time used large sized straws made of polyvinyl chloride. B. Pseudo pregnancy ----- 2. Secundus
C. Cervical mucus ---- 3. Embryo Transfer,1890
D. Embryonic estrogen ----- 4. Clenbuterol
E. Heape ----- 5. M.R.P. in Pig
F. Delaying Parturition ----- 6. Cabergoline
d[Type text] Page 133 d[Type text] Page 134
ANSWERS of C matches:
a) 2, 6, 1, 5, 3, 4 1 d
b) 2, 4, 5, 1, 3, 6 2 c
c) 6, 5, 1, 2, 4, 3 3 a
d) 5, 3, 2, 1, 4, 6 4 b
5 c
6 a
4) A. Zero semen volume ------ 1. Azoospermia 7 d
B. Zero sperm concentration ----- 2. Teratozoospermia
C. Reduced sperm motility ----- 3. Asthenozoospermia
D. Reduced sperm concentration ----- 4.Hypospermia
E. Abnormal sperm ----- 5. Aspermia ANIMAL REPRODUCTION – II
F. Reduced semen volume ------ 6. Oligozoospermia

a) 1, 5, 4, 3, 6, 2 A. MULTIPLE CHOICE QUESTIONS


b) 5, 1, 3, 6, 2, 4
c) 1, 4, 2, 5, 3, 6 1. Unfertilized ovum remains for months in the oviduct of -------species
d) 5, 1, 6, 3, 2, 4 a) sow b) mare c) ewe and doe d) bitch
2. In sows, maternal recognition of pregnancy is mainly due to the action of
a) Interferon tau b) Oxytocin c) Estrogen d) Prostaglandins
5) A. Mature C.L. ----- 1. Mesonephric duct 3. * Ovulation of ―primary oocyte‖ occurs in
B. Female genitalia ------ 2. 40 - 90 days of gestation a) mareandbitch b) sow c) cow and doe d) all of the above
C. Slipping of fetal membranes ----- 3. 80 - 120 days of gestation 4. Centric type of nidation or implantation occurs in
D. Male genitalia ----- 4. Liver fluke like consistancy a) rodents b) primates c) ruminants d) none of the above
E. Fremitus ----- 5. Paramesonephric duct 5. Chemical structure of GnRH, a decapeptide, was determined by
F. Cuboni Test ----- 6. 150-290 days of gestation a) Green and Harris b) Cole and Hart c) Gorski d) ShalleyandGuellemin
6. Endometrial cups are formed from
a) 4, 1, 3, 5, 2, 6 a) chorionicgirdle (fetalorigin) b) maternal caruncles c) endometrium (maternal origin) d) none of the
b) 4, 1, 6, 5, 3, 2 above
c) 4, 5, 2, 1, 3, 6 7. In sow, the villi near the endometrial glands are enlarged and specialized to form structures called
d) 4, 5, 3, 1, 6, 2 a) Hippomanes b) amniotic plaques c) areolae d) placentomes
6) A. Chediak Higashi syndrome ---- 1. Swine 8. Most of the developmental anomalies occur during
B. Polycystic Kidneys ---- 2. Hopping gait a) periodofembryo b) period of ovum c) period of fetus d) during birth
C. Anury ---- 3. Ghost Pattern 9. Low land abortion or Marsh land abortion is due to
D. Twinning ---- 4. Boars a) Fescue poisoning b) Leptospirosis c) nitratepoisoning d)None of the above
E. "Balling Up" ---- 5. Iodine deficiency 10. Transformation of secondary spermatocytes to spermatids
F. Prolonged gestation ---- 6.Equine abortion a) spermatocytegenesis b) spermateliosis c) spermiogenesis d) spermiation
11. Attachment of sperm to the ovum occurs initially at -------segment of sperm head
a) 3, 1, 2, 6, 4, 5 a) apical b) post-acrosomal c) principal d) equitorial
b) 2, 5, 3, 1, 6, 4 12. Diffuse arm like structure of microtubules in the sperm tail are made of proteins
c) 6, 3, 1, 5, 2, 4 a) flactin b) tubulin c) spermosin d) dynein
d) 5, 3, 6, 2, 1, 4 13. In boars, seminal vesicles produces ----------which acts as chief osmotic pressure regulator in the semen
7) A. Refrigeration of semen ---- 1. Glycerol a) ergothionine b) citrate c) inocitol d) fructose
B. Cryopreservation of semen ---- 2. 20 times more Zn than blood 14. **pH of TRIS extender is
C. Buffalo semen ---- 3. Egg yolk a) slightlyacidic b) slightly alkaline c) neutral d) alkaline
D. Sperm membrane integrity --- 4. I.V.T dilutor
E. Preservation of semen at room temp. ----5. D2 Extender • * ovulation of secondary oocyte in other species
F. Dog semen --- 6. H.O.S.T • ** pH 6.8

a) 1, 3, 2, 5, 4, 6
b) 1, 5, 6, 3, 4, 2
c) 3, 1, 2, 6, 4, 5
d) 3, 1, 5, 6, 4, 2 B. FILL IN THE BLANKS 1) Ovary of mare is ----------shaped

135 136
Dhruv N Desai Dhruv N Desai
2) Cervix is poorly defined in ---------------species 21) Failure to expel the second polar body 40) Glyceryl phosphoryl choline, carnitine and 48) Examples for non-penetrating or extracellular
resulting in triploid zygote --------- sialic acid in semen are secreted from------ cryoprotectants are---------
3) Progesterone concentration at oestrus fluctuates
below---------ng/ml 22) ***In cow, mare and ewe, new CL is 41) Dag defect is more common in -----sp. and is 49) ---------gland is the source of antiagglutinin in
refractory for -----days of ovulation associated with high level of---------metal sperm
4) Irregular long oestrous cycles are mainly due to
------------- 23) Antimicrobial constituent of semen 42) Optimum temperature for preservation of boar 50) Semen freezes at -------temperature
semen is-------------
5) In-----------species, ovulation occurs in 24) Sigmoid flexure is pre-scrotal in ---------sp. 51) Level of ascorbic acid in semen----------
metoestrus 43) -----percentage of sodium citrate dehydrate is
25) In Yolkmedia for buck semen, seminal plasma isotonic to semen 52) Trichomonas abortion is more common in ----
6) *PgF2α has local effect on ovary in all species must be removed to prevent yolk coagulation due -trimester of pregnancy
except----------- to the action of ------ 44) First A.I was done by --------- in beagle bitch
53) -------is the most widely used extender for
7) Fertile life span of stallion spermatozoa---------- 26) Generally semen of ------sp. doesn‘t respond 45) First A.I in India was done by--------in Mysore frozen semen
to freezing Palace Dairy farm
8) Abnormal fertilization in which only male 54) High catalase activity, reduced fructose and
pronucleus develops------ 27) From oogenesis onwards diplotene nucleus of 46) Osmotic pressure of semen ranges from-------- high pH in semen are indicative of --------
oocyte remains in resting stage called---- ---
9) In rabbits, -------substance plays role in 55) -----------is a β-blocking agent used to shorten
embryonic nutrition 28) Growth of follicle upto the stage of antrum is - 47) Examples for penetrating or intracellular parturition
----------- cryoprotectants are---------
10) ----------is a polypeptide hormone produced by 56) ---------is a β-adrenergic agent used to delay
ovary 29) The cell layer of trophoectoderm covering the parturition
inner cellmass--------
11) hCG is produced by ------------cells of the
placenta 30) Split oestrus is common in ----------

12) **Number of carbon atoms in estrogen--------- 31) Percentage of spermatozoa in semen -----------
--- ---

13) ------------- is a unique species in which 32) ---------- is the most important maternal cause
epididymis can produce testosterone of dystocia in ewe

14) Ultrasonography for pregnancy diagnosis is 33) ---------- is the most important maternal cause
based on ----phenomenon of dystocia in sow

15) Maintenance of CL and Progesterone from CL 34) First successful embryo transfer in cow was
are necessary throughout the gestation in --------- done by………….

16) In cows, metoestral bleeding is associated 35) --------ions are necessary for optimum sperm
with withdrawal of -------- hormone motility

17) Fertile life of ova in bitch ------------ 36) Most important spermicidal heavy
metals………..
18) Shape of CL in mare----------
37) Normal fructolysis index of semen ranges
19) Cystic follicles are common in--------- from----------

20) Potato soup pyometra or post service 38) ------cells in are more common in severe
pyometra is characteristic of ------------infection testicular hypoplasia

39) Sperm specific LDH localized in midpiece

137 138
Dhruv N Desai Dhruv N Desai
ANSWERS: 25) phospholipase or triacyl 50) -0.53°C BIOCHEMISTRY
glycerol lipase
1) Kidney shaped 51) 3 to 8 mg/ 100ml
26) Boar semen 1. The network of interrelated catabolic and anabolic pathways in cells is referred to as ______________
2) bitch 52) first trimester
27) Dictyate stage 2. A system that exchanges both energy and material with its surrounding is said to be ______________
3) 1 ng/ml 53) Yolk citrate
28) Gonadotropin independent 3. ________________ is a type of weak interaction that stabilizes the native conformation of a biomolecule
4) early embryonic mortality 54) Seminal vesiculitis or supramolecular complex.
29) Rauber cells
5) cattle and buffaloes 55) Carazolol 4. The monomeric subunits of ________ are ribonucleotides.
30) Mare
6) Mare (*In mare, PgF2α 56) clenbuterol 5. The stretching and breaking of bonds that occurs during the conversion of a reactant to a product creates a
has systemic action ) 31) 10% ____________ state.

7) 70 to 120 hours 32) ring womb 6. ____________ is a measure of randomness.

8) androgenesis 33) uterine inertia 7. Enzymes enhance the rate of chemical reactions by lowering the __________ energy that constitutes an
energy barrier between reactants and products.
9) Blastokinin or uteroglobulin 34) Willet in 1951
8. mRNA molecules with two or more attached ribosomes are called ____________
10) relaxin 35) Potassium
9. ______________ is a component of eukaryotic cells consisting of microtubules, actin filaments, and
11) syncytiotrophoblastic cells 36) Cu and Fe intermediate filaments.

12) 18 carbon atoms 37) 1.4 to 2 mg/hr 10. _____________ and _______________ are the two groups of extant prokaryotes.
(**Testosterone-19C steroid
and progesterone-21C steroid) 38) medussa cells and giant 11. The role of _____________ is to produce large number of ribosomes needed by the cell and have DNA
cells that contain many copies of ribosomal RNA coding genes.
13) stallion
39) LDH-X 12. ________________ helps in the condensation of DNA molecule.
14) Doppler phenomenon
40) Epididymis 13. ____________, _______________ and __________ are three classes of cytoskeletal proteins.
15) swine
41) Danish Jersey, Zn 14. ________________ is a complex of RNA and protein.
16) estrogen
42) 15 to 18°C 15. ______________ are molecular complexes of DNA plus associated histone and nonhistone proteins.
17) 4 to 8 days
43) 2.94% 16. _____________ are compounds having electron-deficient functional groups; they tend to bond to
18) cauliflower shaped electron-rich sites.
44) Lazzaro Spallanzani
19) sow 17. ______________ are steroisomers that cannot be superimposed.
45) Dr.Sampathkumaran
20) Trichomonas infection 18. ________________ are a pair of stereoisomers that are not mirror images of each other.
46) 280 to 300 milliOsmol
21) polygyny 19. ___________ is the energy or heat content of a system.
47) Glyserol, DMSO and
22) 3 to 5 days (*** 11 to 12 Ethylene glycol 20. Henderson-Hasselbalch equation = ____________________.
days in sow)
48) Raffinose, sucrose, PVP 21. The glycan portion of glycoprotein is known as a ___________ group.
23) seminal plasmin and glycine
22. A covalent bond between two adjacent cysteines in a polypeptide chain is a __________ bond.
24) boar 49) Prostate
d[Type text] Page 139 d[Type text] Page 140
23. All stereoisomers must have at least one __________ centre. 45. The metabolic intermediate _________________ binds to haemoglobin with a stoichiometry of 1:1 and
promotes the release of oxygen.
24. ________________ procedure provides information about a protein‘s primary structure.
46. A helper T cell can signal nearby lymphocytes by secretion of a signal protein called ______________
25. The whole assortment of proteins in an organism.
47. The contribution of lactic acid in muscle tissue contributes to the _______ effect, which explains the link
26. ______________ are cellular agents that assist in protein folding at elevated temperatures. between lactate production and an increased release of oxygen from haemoglobin.

27. _____________ is stable arrangement of few secondary structures. 48. RBCs transport carbon dioxide produced by respiring tissues in two forms: as bicarbonate ions and as
_____________
28. ______________ is an amino acid which can either accept protons or donate them at a pH that is close to
physiological pH values. 49. ___________ are small molecules covalently attached to large proteins in the laboratory in order to elicit
an immune response.
29. _______________ interactions are thought to be the driving force behind the formation of ―molten
globule‖ during protein folding. 50. _____________ is a particular molecular structure within antigen that binds an individual antibody.

30. Individual amino acids in a protein is called a ____________. 51. Michaelis-Menten equation = ________________

31. ______________ refers to the portion of a protein that is often composed of noncontiguous amino acid 52. kcat is known as the ________________ number. At saturating substrate concentrations kcat=Vmax/Et.
sequences and is usually defined on the basis of its contribution to protein function.
53. ______________ inhibitor alters the Km of an enzyme without altering Vmax.
32. _________ is a type of secondary protein structure that extends 0.15 nm per amino acid residue.
54. An enzyme without a prosthetic group is called _____________.
33. _________ is a type of secondary protein structure that extends 0.35nm per amino acid residue.
55. The common structural motifs recognized by specific protein kinases are known as ______________
34. Disrupting the hydrophobic interactions of a single subunit protein would have the greatest effect on the sequences.
____________ structure of that protein.
56. _______________ is the enzyme that contains Ni2+ as a cofactor and was the first enzyme crystallized
35. Proteins that belong to a ___________ have related structural features though they are unrelated based by Sumner. It enhances the rate of the reaction by ____.
on their amino acid sequences.
57. A molecule essential to the functioning of an enzyme, but not part of the enzyme protein itself is called
36. The alpha-beta subunits in hemoglobin comprise a single__________; the intact haemoglobin tetramer ____________
contains two of these.
58. ______________ inhibitor binds only to the ES complex and does not bind to the substrate-binding site.
37. The saddle conformation is a __________________ structure.
59. A specific, rare type of mixed inhibitor that alters Vmax without affecting Km is __________________
38. alpha-Keratin is referred to as a _______________ ____________ of protein subunits; haemoglobin inhibitor.
with only four subunits is referred to as a(n) ____________
60. ______________ of a substrate occurs when hydrogen bonds between a substrate molecule and water
39. Beta turn is an example of ___________ structure. are replaced by noncovalent interactions between the substrate molecule and an enzyme.

40. ______________ occurs when the binding of one ligand increases or decreases the binding of additional 61. ___________ is an allosteric enzyme whose activity is regulated by a modulator other than its substrate
ligands. whereas _______________ is an allosteric enzyme whose substrate is also a modulator of activity.

41. The _______________ immune system protects against bacterial infections. 62. When the K‘eq=one, ∆G‘°=___________

42. ______________ has a hyperbolic oxygen binding curve, no quarternary structure and serves as an 63. Inhibitors that rreversibly bind to an enzyme are known as ____________ inactivators.
oxygen ―reservoir‖ in muscle cells.
64. The regulation of enzyme activity by the reversible binding of a phosphoryl group is an example of
43. ______________ has a sigmoid oxygen binding curve and has a quaternary structure. regulation by ___________ modification.

44. ______________ is also called programmed cell death.

141 142
Dhruv N Desai Dhruv N Desai
65. Allosteric enzymes__________(do/do not) follow Michaelis-Menten kinetics and some show 84. Match the following :
____________ kinetic behaviour in the velocity versus substrate concentration plot which reflects
cooperativity. A B
1. Testosterone and cortisol a. fatty acid derivatives that act on the
66. The plot of an enzyme kinetic reaction eventually plateaus as the active site is saturated with substrate. tissue in which they are produced.
[T or F] 2. Phosphatidylinositol and its derivatives b. isoprenoids that must be obtained from
the diet
67. Six membered ring form of sugars are called _____________ and five-membered ring form of sugars are
3. Eicosanoids c. intracellular messengers that are
called ____________
components of the plasma membrane.
68. Lectins are proteins that bind to specific ______________ 4.Vitamin K and Vitamin E d. steroid hormones that are produced in a
tissue and carried via blood stream to target
69. An isomer that differs at only one of two or more chiral centres are called _________ tissues.

70. The process that interconverts isomers of pyranoses 85. ____________ are lipids stored in adipocytes

71. __________ DNA is the dehydrated compact form of DNA.

72. _______ DNA is a structure containing polypurine tracts and mirror repeats and forms a triple helix.
ANSWER – 16) electrophiles 33) beta pleated sheet/beta
73. __________ pairing or Non-Watson-Crick pairing allows the formation of triplex DNAs. conformation
1) Metabolism 17) enantiomers
74. ____ of purine and ____ of pyrimidines is linked to C1 of ribose. 34) tertiary
2) Open 18) diastermers
75. The increase in UV light absorption when double-stranded DNA is denatured is referred to as the 35) superfamily
___________ effect. 3) non-covalent interaction 19) enthalpy
36) protomer
76. Purine or pyrimidine base covalently bound to furanose through and ___________ 4) RNA 20) pH=pKa+log[proton
acceptor]/[proton donor] 37) supersecondary
77. ______________ bonds are covalent bonds tht link the individual nucleotide residues in DNA and RNA. 5) Transition
21) prosthetic 38) supramolecular complex;
78. The deamination product of : 6) Entropy oligomer
22) disulfide
a) Cytosine = ____________ 7) Activation 39) secondary
23) chiral
b) Guanine = _____________ 8) Polysomes 40) cooperativity
24) Edman degradation
c) 5-methyl cytosine =___________ 9) Cytoskeleton 41) humoral
25) Proteome
d) Adenine = ___________ 10) archaebacteria and 42) myoglobin
eubacteria 26) Chaperones
79. _____________ is an extremely hydrophobic isoprenoid compound that anchors sugars to cell 43) haemoglobin
membranes. 11) nucleolus 27) Motif
44) apoptosis
80. The polar head group of cholesterol is ____________ group. 12) nucleosome 28) Histidine
45) 2,3-bisphosphoglycerate
81. ______________________ is a lipid seen in beeswax. 13) actin/microfilament; 29) Hydrophobic
microtubules;intermediate 46) Interleukin
82. Lignoceric acid is a/an _____________ free fatty acid with _____ carbon atoms. filaments 30) Residue
47) Bohr
83. The fatty acid 20:4(∆5,8,11,14) is commonly called ____________ which is a precursor of 14) ribosome 31) Domain
____________ 48) Carbaminohaemoglobin
15) chromatin 32) alpha helix
143 144
Dhruv N Desai Dhruv N Desai
49) Hapten 62) zero 75) Hyperchromic 11. ------ and ------- are two recessive traits that are inherited.

50) Epitope 63) suicide 76) N-β-glycosidic bond 12. What is the ratio due to double recessive epistasis?

51) Vo = Vmax + [S]/ Km + 64) covalent 77) Phosphodiester 13. Linkage was first observed by--------- in sweet pea.
[S]
65) do not; sigmoid 78) a. uracil; b. xanthine; c. 14. Crossing over occurs in the --------- stage of meiosis.
52) Turnover thymine; d. Hypoxanthine
66) T 15. Chromosome number in fowl is ---------
53) Competitive 79) Dolichols
67) pyranoses; furanoses 16. Epistasis works at------ level where as Dominance always work at the ------level
54) apoenzyme 80) Hydroxyl
68) oligosaccharides 17. What is penetrance ?
55) Consensus 81) Triacontanylpalmitate
69) epimers 18. The degree to which a genotype is expressed phenotypically is called
56) urease; 10^14 82) saturated/unsaturated;
70) mutarotation saturated;24 19. Name the scientist who first discovered chromosomes
57) cofactor
71) A-DNA 83) arachidonic acid; 20. Who coined the term Chromosomes?
58) uncompetitive eicosanoids – an example:
72) H-DNA prostaglandins 21. The organelle from which the r-RNA is synthesized
59) noncompetitive
73) Hoogsteen 84) 1.d; 2.c;3.a;4.b 22. Metacentric chromosomes assume which shape?
60) desolvation
74) N9;N1 85) Triacylglycerol 23. Pairing of the homologous chromosomes takes place at ------- stage
61) heterotropic/homotropic
24. Coiled filament that runs throughout the length of the chromosome is called

25. Darkly stained regions of the chromosomes at prophase is called ----

GENETICS 26. Sex chromatin are rich in -----------------

27. Where are Lampbrush chromosomes found?


1. Scientist who coined the term Genetics
28. The Octate structure in the nucleosome consists of ----
2. What is the contribution of Wilhem Johanssen to Genetics?
29. Balbiani rings or Chromosomal puffing are present in-----------
3. Theory of pangenesis was proposed by ------------
30. Interphase of the cell cycle consists of ---------
4. The concept of Genotype and Phenotype was introduced by -----
31. Complete synaptonemal complex is found in which stage?
5. Chromosome theory of heredity was proposed by
32. Sythesis of DNA is completed in -----stage of Meiosis
6. Germ plasm theory was put forward by
33. The unit representing a map unit between the linked gene……
7. The nationality of Gregor Mendel who is regarded as ―father of genetics‖
34. Phenomenon by which crossing over in one region suppresses crossing over in adjacent region----
8. In 1900, Mendel‘s work were rediscovered by -------
35. What is coefficient of coincidence ?
9. Law of Segregation is also known as-------
36. ----- is measured using coefficient of coincidence ?
10. Human blood group type is an example of -----
37. In fowl females are heterogametic T / F
145 146
Dhruv N Desai Dhruv N Desai
38. Barred plumage in poultry is a type of ---- 48.Polyploidy 50.Karyotype
49.Giemsa
39. Genic balance theory of sex determination was proposed by -------

40. ----- confirms the presence of barr body in female somatic cells. PART.A: ANIMAL GENETICS - OBJECTIVE

41. -------syndrome which is caused by deletion of short arm of chromosome no 5

42. Point mutation was first noticed by ------- in Ancon sheep? 1. Germ Plasm theory was postulated by

43. World‘s first chromosome map was produced by------- a) Lamark b) Weisman c) Kolliker d) Hertwig

44. The no . of chromosomes in Drosophila is -------- 2. The ability of a given gene or gene combination to be expressed phenotypically to any degree

45. Haploid-Diploid type of sex determination is seen in --------- a) Penetrance b) expressivity c) pleiotropism d) prepotency

46. In Drosophila sex is determined by the ratio of X chromosome to -------- 3) Linkage group in swamp buffalo

47. Polyploids created by chromosome duplication is called -------- a) 25 b) 30 c) 24 d) 26

48. The presence of extra chromosome sets in a cell is called as ------------- 4. Total number of genotypes in human ABO blood group system

49. which is the stain used for G banding---- a) 3 b) 12 c) 6 d) 4

50. The chart of images of chromosomes is called ---------- 5. Eye colour in drosophila is an example of

ANSWERS :- 24.chromonema a) Sexlinkedinheritance b) sex limited inheritance c) sex influenced inheritance d) incomplete
25.heterochromatin dominance
1.William Bateson 26.Heterochromatin
2.Coined 'allele' and 'genes' 27. Primary oocytes of amphibians and 6. If the X/A ratio is 0.5, the individual will be
3.Darwin spermatocyte of Drosophila.
4.W Johanssen 28. Two molecules each of H2A, H2B, H3 and a) Intersex b) normal female c) super female d) normalmale
5.W S Sutton H4.
6.Weismann 29.Salivary gland cells of Drosophila. 7) Total number of barr bodies in an individual with Turner‘s syndrome
7.Austria 30.G1, S and G2
8.Hugo de Vries, Carl Correns Eric von 31.Zygotene a) 0 b) 1 c) 2 d) none of the above
Tschermack 32.Zygotene
9.Law of purity of gametes 33.CentiMorgan 8. In translocation, the exchange of chromosomal segments occurs
10.Multiple alleles 34.Interference
11.Alkaptonuria and Phenylketonuria 35.% of observed cross overs/ % of expected a) With in the same chromosome b) between homologous chromosomes c)
12.9:7 crossovers (RATIO) betweennonhomologouschromosomes d) both b and c
13.Bateson and Punnet 36. Degree of interference
14.Pachytene 37.T 9. If a woman heterozygous for colour blindness marries a colour blind man, what is the probability that
15.78 38.Sex linked character their first child will be colour blind daughter?
16.Intergenic, Intragenic 39.C.B.Bridges
17. % of individuals with a given genotypes which 40.Lyon's hypothesis a) 50% b) 25 % c) 75% c) 100%
exhibits the related phenotypes. 41.Cridu-chat syndrome
18.Expressivity 42.Seth Wright 10. If the centromere is located close to the end, and giving a very short arm and an exceptionally long arm,
19.Strassburger 43.Alfred Sturtevant the chromosome is called
20.Waldeyer 44.8
21.Nucleolus 45.Honey bees and wasps a) acrocentric b) telocentric c)submetacentric d)none of the above
22.― V ― shape 46.Autosomes
23. Zygotene 47.Autopolyploid 11. Shortest phase in mitosis is
147 148
Dhruv N Desai Dhruv N Desai
a) prophase b) anaphase c) metaphase d) telophase 25. Epistasis is ………… type of interaction

12. Soluble RNA (sRNA) is a) inter allelic b) intra allelic c) allelic genetic d) both a and c

a) tRNA b) rRNA c)mRNA d)mitochondrial RNA 26. Sickle cell anemia is due to point mutation, in which
a) glu. is replaced by val. b) val. is replaced by glu. c) his. is replaced by val. d) val. is replaced by
13. The theory of epigenesis was proposed by his.
(Note: val-valine,glu-glutamic acid)
a) K.F. Wolf b) Charles Darwin c) Swammerdam d) Kolliker
27. MN blood group system in humans is an example of
14. Phenotypic F2 ratio of duplicative recessive epistasis (complementary gene interaction) is
a) multiple allelism b) co dominance c) incomplete dominance d) pleiotropism
a) 12:3:1 b) 9:7 c) 9:6:1 d) 9:3:3:1
28. If the number multiple alleles for a trait is 5, then what is the total number of genotypes
15. Source of energy in nucleus is
a) 9 b) 16 c) 12 d) 15
a) glycolysisonly b) TCA cycle only c) both a and b d) none of the above
29. Chromosome theory of linkage was proposed by
16. Crossing over taking place at
a) Beadle and Tatum b) Bateson and Punnett c) Morgan and Castle d)Sutton and
a) mitosis b) meiosis ll c) meiosis l d)all of the above Bovery

17. Xanthine is deaminated product of 30. Barred condition in poultry is an example of

a) adenine b) thymine c) cytosine d) guanine a) sex linked inheritance b) sex limited inheritance c) multiple allelism d) sex influenced
inheritance
18. The types of phenotypes in F2 generation is

a) 2n b) 3n c) n2 d) n3 PART.B: ANIMAL BREEDING

19. Sum total of genes in a population is 1. Quantitative traits are controlled by


a) major genes b) minor genes c) poly genes d ) both b and c
a) genotype b) karyotype c) genepool d) gene frequency
2. Breeding value (BV) is
20. Rho factor is required for the termination of a) equal to the TA b) twice the TA c) 1/2 of TA d) 1/4 of TA

a) Replication b) transcription c) translation d) transversion 3. Heritability in narrow sense is


a) VA/VP b) VE/VP c) VG/VP d) VA+VG/VP
21. The types of histone proteins present in eukaryotic chromosome are
4. Repeatability value set in …………of heritability
a) 4 b) 5 c) 6 d) 7 a) lower limit of h2 b) upperlimitofh2 c ) intermediate of h2 d) both a and b

22. The classical test cross ratio in dihybrid is 5. The most effective method of selection
a) mass selection b) pedigree selection c) progeny testing d) selection index
a) 7:1:1:7 b) 1:7:7:1 c) 1:1:1:1 d) both a and b
6. Quantitative traits shows
23. Chromatin consists of a)continuousvariation b)discontinuous variation c)both a and b d)none of the above

a) DNA & protein only b) RNA & protein only c) RNA & protein only d) DNA,RNA&protein 7. Genotypic frequency of progeny depends on
a) genotypic freq. of parent b) genotypic freq. of population c) genefreq.
24. Chromosome number of domestic pig is ofparent d) both a and c

a) 38 b) 78 c) 62 d) 54
149 150
Dhruv N Desai Dhruv N Desai
8. In a population sex linked genes are carried by homogametic sex is ………..of the total sex linked genes 24. In MN blood group system, genotypes are MM = 153, MN = 260, NN = 87, then the gene frequencies
in the population of M and N alleles are
a) 1/2 b) 1/4 c) 1/3 d) 2/3 a) 0.64, 0.36 b) 0.5, 0.5 c) 0.566, 0.434 d) 0.518, 0.482

9. Type of gene action in general combining ability (GCA) is 25. A new breed can be evolved by
a) additive b) non additive c) both a and b d) none of the above a) out crossing b) cross breeding c) grading up d) none of the above
2
10.Range of h and fitness is 26. If the coefficient of selection is 0.25, then the fitness is
a) 0 to 1 b) -1 to +1 c) 0 to infinity d) 0 to xn a) 1 b ) 0.25 c) 0.75 d)0.5

11. The proportion of population which shows genetic death is 27. Father of modern animal breeding
a) genetic sterility b) genetic linkage c) geneticload d) genetic drift a) Robert Bakewell b) S.Wright c)J.L. Lush d)Gauss

12. For inbreeding, mated individuals should have common ancestors with in 28. Reproductive traits in animals usually follow …………… type h 2

a) 2-3 generations b) 4-6 generations c) 6-7 generations d) more than 10 a) High b) medium c) low d) none of the above
generations
29. The goat breed, toggenberg originated from
13. Inbreeding coefficients of progenies produced by self mating, parent offspring mating, full sibs and half a) Spain b) Denmark c) Switzerland d) France
sibs are in the order
a) 1,0.5,0.25,0.125 b) 0.5,0.5,0.25,0.125 c) 1,0.5,0.5,0.25 d) 0.5,0.25,0.25,0.125 30. Grading up produces pure breed in how many generations
a) 2-4 b) 4-5 c) 10-12 d) 7-8
14. Hissardale is the cross of
a) Merino ewe x Bikaneri ram b) Lincoln ram x Rambouillet ewe c) Merino ram x Bikaneri
ewe d) Lincoln ewe x Rambouillet ram MEAT SCIENCE

15. In the second generation, hybrid vigor will be


a) same as the first b) doubled the first c) half of the first d)none of the above 1. Muscle fibers of meat animals with diameters of 50 microns contains ----- no. of Myofibrils

16. Crossing of two inbred lines of the same breed is 2. The unit of myofibril between two adjacent Z discs is called -----
a) Incrossing c) Incross breeding c) top incrossing d) top incross breeding
3.A typical mammalian muscle at rest has a sarcomere length of -------
17. The intensity of selection depends with number traits considered ‗n‘ is
a) 1/n b) 1/n2 c) n d) 1/√n 4. Actin molecule has a --------- shape

18. Non additive gene action include, 5. Myosin constitutes approx --------% of myofibrillar proteins
a) epistasis b) dominance c) interaction d) alloftheabove
6. --------- is the most abundant protein in animal body
19. Mildest form of out breeding is
a) Cross breeding b) close breeding c) outcrossing d) rotational crossing 7. --------- is the most abundant amino acid of collagen

20. The dispersive process mainly occurs in small population in which, 8. Glycine constitute about -----% of amino acids of collagen
a) only direction can be predicted not amount b) onlyamountcanbepredictednotdirection
c) both amount and direction can be predicted c) direction and amount cannot be predicted 9. ------------ is the structural unit of collagen fibril

21. The contribution of offspring to the next generation is called 10. The cervical ligament of neck is made of --------fibers
a) fitness b) adaptive value c) selective value d) alloftheabove
11. --------- is the amino acid present in the greatest quantity in elastin
22. Genotypic correlation is due to
a) polymorphism b) linkage c) pleiotropism d) both b and c 12. -------- & --------- are two unique amino acids present in elastin

23. Carcass quality and quantity is an example of 13. The color of brown fat is due to high content of --------- in mitochondria
a) non additive gene interaction b) additive gene interaction c) both a &b d)none of
the above 14. A primary muscle contains approx ------ number of muscle fibers

151 152
Dhruv N Desai Dhruv N Desai
15. Intramuscular fat is called ---------- of meat 41. The bright pink color characteristic of cured meat is due to ---------

16. Intermuscular fat is also called --------- fat 42. The amount of nitrite permitted in finished products by US meat inspection regulation is ----ppm

17. The element which constitutes maximum % of animal body weight is-------- 43. The sodium salts of ------- or -------- acids are most widely used cure accelerators.

18. ----------- is the most abundant fatty acid in animal body 44. The greening of cured meat pigment by excessive use of nitrites-----------

19. The most abundant carbohydrate in muscles----------- 45. Large fat particles coalesce at the end of the sausages to form -------

20. Average protein percentage of mammalian skeletal muscles----- 46. -----flavor develops due to lipid oxidation in pre-cooked frozen meat.

21. A genetic condition of cattle causing unusually thick bulging muscles. 47. The heat resistance of microrganisms is usually expressed as ---------

22. Excessive fat infiltration in muscle fibers is called-------- 48. To stabilize meat products, a radiation dosage of ----- megarads is used.

23. An action potential enters the interior of a muscle fibers along --------- 49, Loss of tenderness occuring in the first few hours postmortem is called--------toughning.

24.Only about --------% of total blood volume can be removed via exsanguination. 50. Cooked testicle of lambs, calves and turkeys are commonly called ------

25. The range of ultimate pH of meat is -------- ANSWERS - 17. Oxygen-65%

26. The period of time during which the muscle is extensible and elastic is called---phase of rigor mortis. 1. 1000-2000 18. Oleic acid

27. ATP complexed with --------is required for a muscle to maintain a relaxed state 2. Sarcomere 19. Glycogen

28. The decrease in tension with time is described as ------- of rigor mortis. 3. 2.5 microns 20. 18.5%

29. Holding carcass at refrigeration temperature after initial chilling is called--------in US &-----in other 4. Globular 21.Double Muscling
countries
5. 45 22. Steatosis
30. In------- condition of meat, there is lowered processing yield, increased cooking loss and reduced
juiciness. 6. Collagen 23. T-tubules

31. Cold shortening develops when muscle is chilled below --------before onset of rigor mortis. 7. Glycine 24. 50%

32. Thaw rigor shortening is approx. ------- % of original length of muscles. 8. 33 25. 5.3-5.7

33.Marked shortening and early onset of rigor induced by maintaining muscles at high temp is called------- 9. Tropocollagen 26. Delay

34. Lipid oxidation in muscles is measured as -----values. 10. Elastin 27. Mg2+

35. Loss of weight during storage of meat is called ------------ 11. Glycine 28. Resolution

36. Lack of space for water molecules within protein structures is known as -------effects. 12. Desmosine and Isodesmosine 29. Aging , conditioning

37. In well bled muscles, Myoglobin constitutes ------% of the total pigments. 13. Cytochrome 30. PSE

38. The typical color of meat from pork is ------- 14. 20 to 40 31. 15-16 degrees

39. The bright red color development of meat is due to oxymyoglobin is called 15. Marbling 32. 60%

40. Oxidized myoglobin is called------ 16. Seam 33. Heat Rigor


153 154
Dhruv N Desai Dhruv N Desai
34. Thiobarbituric Acid 43. Ascorbic or Erythorbic 17. Pasteuization refers to the process of heating every particle of milk to at least 63ºCfor 30 min or heating
to 72ºC for 15 sec.
35. Shrinkage 44. Nitrite Burn 18. The time-temperature combinations used for producing sterilized milk are 145ºC and 3 sec.
19. The low temperature storage of raw milk prior to processing is likely to increase Psychrotrophic counts.
36.Steric 45. Fat Caps
20. The spores of B. stearothermophilus are known to withstand UHT treatment.
37. 80-90% 46. Warmed Over
21. Sterilizing effect refers to the number of decimal reductions that the heat treatment is able to effect in
38. Grayish Pink 47.Thermal death time milk.
22. For aseptic packaging of UHT milk, Tetra pack, Tetra-Brick etc are used as packaging materials
39. Bloom 48. 4.5
23. The common groups of post-pasteurization contaminants include Coliforms and psychrotrophs
40. Metmyoglobin 49. Actomyosin 24. The thermal destruction of bacteria in milk is based on the principle of Protein denaturation
41. Nitrosyl Haemochromogen 50. Mountain Oysters 25. Pseudomonas putrefaciens causes surface taints in butter.
26. The gas producing organisms may enter milk chiefly from soil and manure.
42. 200
27. The blue discolouration in milk is caused by the associative action of Ps. Syncyanea and S. lactis.
28. Slime production in milk is mainly caused by Leuconostoc genus of lactic acid bacteria.
29. Ropiness in milk is mainly caused by Alcaligenes viscolactis
DAIRY SCIENCE
30. Coliforms cause Early blowing in cheese.
1. Cottage cheese is a soft, unripened cheese usually made from Skim milk 31. The two types of materials responsible for ropiness are Gums, and Mucins
2. Operation flood was started in the year 1970 32. The three types of rancidity in milk are Hydrolytic, oxidative and ketonic
3. Plastic cream contains 65-85 per cent milk fat 33. The fruity aroma of milk produced by Ps. fragi is due to the production of Esters;
4. According to PFA Rules the milk fat content of khoa should not be less than 20 per cent of finished 34. Malty flavour produced by S. lactis var. maltigenes in milk is due to the production of Aldehydes .
product.
35. Unclean flavour in milk may be due to microbial production of Dimethyl sulfide by Gram-negative
5. The average specific gravity of skim milk ranges from 1.035 to 1.037 psychrotrophic bacteria.
6. According to the PFA Rules the mixed milk should contain minimum per cent of milk fat and milk SNF 36. Faecal coliforms in dairy products are detected by Eijkman test test.
respectively 4.5, 8.5
37. The common indicator organisms used for determining faecal contamination in frozen and thermized
7. The chairman of NDDB, has become the first Indian to be elected to the board of the International Dairy foods are Enterococci
Federation (IDF) Dr. (Ms) Amrita Patel 38. The tentative standards for bacterial count of environment in butter section are 300 cfu/m3.

8. Daily per capita milk consumption recommended by the Medical Authorities is 280g 39. The aflatoxin B1 in dairy animal feed is transformed into aflatoxin M1 and is secreted into milk.

9. According to the PFA Rules chhana should not contain more than 70 per cent moisture 40. The efficiency of dairy sanitizers is determined by Capacity and suspension tests.

10. The acidity in mastitic milk is Lower than normal milk 41. Food-borne intoxications through dairy products are mainly caused by Staph.aureus

11. The starter organisms for yoghurt are Streptococcus thermophilus and Lactobacillus delbruekii 42. Widal test is used for the detection of Salmonella in dairy products.
subsp. bulgaricus 43. E.coli forms typical Dark centered with green metallic sheen colonies on Eosin Methylene Blue
12. Natural acidity of milk is due to casein,acid phosphates and citrates (EMB) agar.

13. Temperature and time of flash pasteurization 720Cfor 15 second 44. Indian Standards Institution (ISI) has been renamed as Bureau of Indian Standards

14. Pizza is prepared from mozzarella cheese 45. ICMSF stands of International Commission on Microbiological Specifications for Foods;

15. National Dairy Development Board, Anand ,Gujarat was set-up in the year1965 46. Indole is produced from tryptophan by the action of Tryptophanase enzyme of micro-organisms during
IMViC test.
16. Soft ice cream is usually drawn from the freezer at around - 8 to -7°C. The overrun may be in the range
of 30 to 50 per cent. 47. Milk with titratable acidity more than 0.17 % (LA) gives a positive COB test.

155 156
Dhruv N Desai Dhruv N Desai
48. A special pipette called Breed‟s pipette is used for performing direct microscopic count (DMC). 21. laying hens most efficiently utilized phosphorus from which of the following sources (a) phytate
49. For staining milk smear during direct microscopic count, a special stain, a special stain called phosphorus (b) phosphorus of cereal grains (c) disodium phosphate (d) dicalcium phosphate
Newman‟s stain is used. 22. which of the fowl has a single medium wattle (a) red jungle fowl (b) ceylon jungle fowl (c) grey jungle
50. The oxidation-reduction potential of resazurin is 0.34 V whereas it is 0.1 V or les for dihydroresorufin. fowl (d) javan jungle fowl
23. white leghorn are white because (a) no colour gene (b) a dominant gene which inhibits color (c)
recessive white gene (d) they have silver gene
24. the best breed for using as male line in broiler production is (a) white rock (b) Cornish (c) New
Hampshire (d) Australorp
25. the wild ............is the ancestor of all domestic duck breeds
26. immature ducks up to age of 8-11 weeks are called.......
27. the black and white barring in barred Plymouth rock is due to ............barring gene
28. in a sex-linked cross involving barring, the female parent is a ...............
POULTRY SCIENCE
29. in a sex-linked cross involving silver and gold, the silver gene carrying ...........parent is used
1. Fibrous proteins contains the ............which are the main proteins of ............. 30. in a sex-linked cross involving silver and gold, the gold gene carrying ........parent is used
2. The chemical name of vitamin D2 is ........whereas D3 is .......... 31. in a sex-linked cross involving feathering gene, a late feathering .......parent is used
3. .............concerned with night vision is a ..........pigment 32. which one is sex linked (a) dwarfism (b) nakedness (c) albinism (d) rapid feathering
4. The enzyme like .........breakdown fat into ...........and.............. 33. egg shell treatment is done to reduce the rate of ............loss
5. The inorganic element present in the arginase is .............. It splits arginine into .............and .................. 34. shank length and width is a good indicator of ........
6. Antivitamin K activity is exhibited by (a) biotin (b) dicumarol (c) sulfanilamide (d) caproic acid 35. blood meal is deficient in essential aminoacid...........
7. Gossypol of cotton seed meal react with (a) zinc (b) manganese (c) iron (d) selenium 36. maximum level of molasses which can be included in chick feed.........
8. Nutritional roup is due to deficiency of (a) vit A (b) vit B6 (c) vit K (d) vit E 37. metablizable energy value of maize grain.....
9. Pastures are classified in (a) silage (b) roughage (c) additives (d) succulent forages 38. the best protein source among the plant protein sources........
10. A calorie is the amount of heat required to raise the temperature of 1g water from (a) 12.5 to 13.5 (b) 39. poultry need one more essential aminoacid ........than cattle
14.5 to 15.5 (c) 15.7 to 16.7 (d) 10.2 to 11.2 40. eggs are pasteurized primarily to destroy bacteria pathogenic to humans especially..........
11. For determination of metabolizable energy instrument used is (a) metabolizable energy meter (b) bomb 41. eggs act as ...........agent in baked foods
calorimeter (c) barometer (d) energy thermometer 42. hens egg contains about.....grams of protein
12. Keratin are proteins of (a) arteries (b) DNA (c) hairs (d) connective tissue 43. a component of egg white having antibacterial activity
13. Protamines are basic proteins associated with nucleic acids are rich in (a) tyrosine (b) tryptophan (c) 44. compared to red meats, poultry meat contain a higher proportion of ............(saturated/unsaturated fatty
methionine (d) arginine acids)
14. weight gain per unit weight of protein consumed refers to (a) biological value (b) gross protein value (c) 45. which of the following is not a glucan (a) starch (b) inulin (c) cellulose (d) dextrins
protein efficiency ratio (d) essential amino acid index 46. which of the following was considered lately as an essential mineral for poultry (a) molybdenum (b) zinc
15. denaturation of proteins in chicken occurs in (a) oesophagus (b) proventriculus and gizzard (c) crop and (c) selenium (d) chromium
pancreas (d) small and large intestine 47. weight loss of broiler between farm and processing plant is ........%
16. vitamin E was discovered by (a) Funk (b) Hopkins (c) Evans and Bishop (d) Mc Collum and Davis 48. darkening of egg yolk in hard boiled eggs is due to .......formation
17. Maintenance of normal cerebrospinal fluid pressure is physiological function of (a) riboflavin (b) 49. ...............is done for recycling of birds to get another cycle of egg production
pyrodoxin (c) retinol (d) folic acid 50. conalbumen complex with ........
18. selenium is an essential component of enzyme (a) coenzyme A (b) D aminoacid oxidase (c) glutathione 51. ...........is the trypsin inhibitor in egg
peroxidase (d) choline esterase 52. avidin complexes with.........in egg
19. "clubbed down condition" occur due to deficiency of (a) pterylglutamic acid (b) riboflavin (c) 53 one molecule of avidin complexe with.........molecules of biotin
menaquinone (d) cholecalciferol 54. hen become sexually active at the age of ......weeks
20. vitamin H is the old name of (a) nicotinic acid (b) folic acid (c) tocoferol (d) biotin 55. length of ovulatory cycle in birds is ..........
157 158
Dhruv N Desai Dhruv N Desai
56. within a clutch, the interval from oviposition to the following ovulation averages about......... 49. forced/induced moulting 60. ovotestis
57. abolishing 'bearing down reflex' results in .............. 50. iron 61. uterovaginal junction
58. minimum time from spermatocyte stage to the production of mature spermatozoa is............... 51. ovomucoid 62. glucose
59. the daily turn over of calcium in the normal laying hens is ..........% of her total body calcium 52. biotin 63. gastrula
60. if the left ovary of a 15 days old chick is removed, the right ovary will become an ............... 53. 4 (avidin has 4 subunits and each subunit 64. connective tissue septa
61. the sperm host glands of avian oviduct are located at ..........of oviduct complex with one molecule of biotin) 65. photo receptors
62. chicken sperms are able to utilize...........for energy purpose 54. 18-20 66. 40.6-41.7 degrees Celsius
63. the fertilized chicken egg when laid contains an embryo at .........stage 55. 25-26 hrs 67. 47 degrees Celsius
64. the avian testes are soft because they lack ..............commonly found in mammals 56. 35 minutes
68. 70-80%
65. extra retinal .............present in birds produce the effects of light in blinded birds 57. delay in oviposition
66. Body temperature of fowl is............... 58. 12 days 69. 9-10 hours

67. Upper lethal temperature of fowl is .................. 59. 10 70. 14 hours


IMPS for Poultry
68. Fresh poultry excreta contains..........% water
Present day production potential of commercial broilers and layers
69. Threshold photoperiod for poultry................... commercial broiler
70. For maximum egg production, the photoperiod is .................. body wt at 42 days of age- 2 kg
ANSWERS - 24. Cornish feed intake to 4 days age 4.2 kg
25. mallard livability- 95%
1. collagens, connective tissue 26. green duck
commercial layer
2. ergocalciferol, cholecalciferol 27. sex linked
livability upto 18 weeks- 95%
3. rhodopsin, bright red 28. barred Plymouth rock
no of eggs- hen housed- 310
4. lipase, fatty acids and glycerol 29. female
body wt at 76 weeks age- 1.6 kg
5. magnesium, ornithine and urea 30. male
feed intake during laying period- 48-52g
6. dicumarol 31. female
livability (18-76wk)- 92%
7. iron 32. rapid feathering
8. vitamin A 33. carbon dioxide
9. succualnt forage 34. skeleton size BIS Requirements of chicken feeds IS 1374:1992 (fourth revision)
10. 14.5 to 15.5 35. isoleucine characteristic requirements for (to be declared (on dry matter basis)
11. bomb calorimeter 36. 5% broiler broiler chick growing laying chicken breeder
12. hairs 37. 3300 kcal/kg starter finisher feed (0-8 chicken feed feed layer feed
13. arginine 38. soybean meal (0-5 (after 5 wk) (8-20 wk)
wk) wk)
14. protein efficiency ratio 39. glycine moisture perent by 11 11 11 11 11 11
15. proventriculus and gizzard 40. salmonella mass, max.
16. Evans and Bishop 41. leavening agents crude protein (Nx6.25) 23 20 20 16 18 18
percent by mass, Min
17. Retinol 42. 6-7
crude fibre, percent by 6 6 7 8 8 8
18. glutatione peroxidase 43. lysozyme
mass, Max.
19. riboflavin 44. unsaturated acid insoluble ash, 3 3 4 4 4 4
20. biotin 45. inulin percent by mass, max.
21. disodium phosphate 46. chromium salt (as NaCl), percent 0.6 0.6 0.6 0.6 0.6 0.6
by mass, Max.
22. Javan jungle fowl 47. 5-10 calcium percent by 1.2 1.2 1.0 1.0 3.0 3.0
23. dominant gene that inhibits color 48. ferrous sulfide mass, min.
159 160
Dhruv N Desai Dhruv N Desai
available phosphorus, 0.5 0.5 0.5 0.5 0.5 0.5 33. ฀ ฀ Jerusalem antichoke contains the main reserve carbohydrate- Inulin
percent by mass, min. 34. ฀ ฀ The term―protein‖ is coined by-Mulder
35. ฀ ฀ Who introduced the balance and thermometer in to nutrition studies for the first
metabolizable energy, 2800 2900 2600 2500 2600 2600 36. time?-Antoine Lavoisier
Kcal/kg, Min 37. ฀ ฀ Pinnaglobulin contains Manganese and hemocyanin contains copper
38. ฀ ฀ Legumes are exceptionally rich in- Calcium
39. ฀ ฀ Germinating Barley contains a starch digesting enzyme called -Diastase
40. ฀ ฀ Skim milk is the feed ingredient which can said to be rich in both- Calcium and
41. Phosphorus
42. ฀ ฀ First accurate respiration calorimeter was constructed by-Rubner
43. ฀ ฀ Starch equivalent system was designed by-Kellner
44. ฀ ฀ According to NRC, the ME=DE x 0.85
45. ฀ ฀ Physiological fuel values were devised by-Atwater
46. ฀ ฀ Heat increment consists of-Heat of fermentation and Heat of nutrient metabolism
47. ฀ ฀ Feces is the main route of phosphorus excretion in herbivores and urine is in case
48. of carnivores.
49. ฀ ฀ Citrate ,lactate ,pyruvate ,ascorbate etc enhance the absorption of-Iron
50. ฀ ฀ RUMENSIN, MONENSIN modifies rumen fermentation by-promoting
51. propionate producing microbes
ANIMAL NUTRITION 52. ฀ ฀ In hibernating animals the RQ is- less than 0.7
53. ฀ ฀ Whole blood contains from 35-45mg % phosphorus
1. ฀ ฀ Father of nutrition- Antoine Lavoisier 54. ฀ ฀ Glucosyl transferase needed in mucopoly sacharide synthesis depend on-
2. ฀ ฀ The yeast variety commonly known as ―fodder yeast‖-Torulopsis utilis 55. Manganese
3. ฀ ฀ Water content in the body of new bone calf is-80% 56. ฀ ฀ ‗Degnala disease‘ is caused by-Selenium toxicity
4. ฀ ฀ In Van Soest method of feed estimation the ADF comprises of----cellulose and 57. ฀ ฀ Net gain of ATP while one mole of glucose is oxidized completely-36
5. lignin 58. ฀ ฀ Scandinavian feeding system based on barley as the standard is introduced by-
6. ฀ ฀ The only true ketogenicamino acid-leucine 59. Hanssen
7. ฀ ฀ Fat contains-----% carbon-77 60. ฀ ฀ Urea toxicity results when the rumen ammonia level exceeds-80mg/100ml
8. ฀ ฀ A dietry excess of Tyrosine cause-eye lesions 61. ฀ ฀ Leaves of plants containgalactolipid as the major lipid
9. ฀ ฀ Dietry excess of Methionine produces-inhibition of ATP synthesis
10. ฀ ฀ Zinc forms an integral part of enzyme- Carbonic anhydrase
11. ฀ ฀ About 96% of plasma copperis bound to an alpha-2 globulin called-
12. Ceruloplasmin
13. ฀ ฀ Organic acids promotes the absorption of calcium I.Indicate True or False
14. ฀ ฀ Curled toe paralysis is caused by the deficiency of-Riboflavin
15. ฀ ฀ One IU of vitamin A is equivalent to 0.6mcg of beta-carotene 1. Albumins are not soluble in water.- F
16. ฀ ฀ Vitamin A promotesmuco-polysaccharide synthesis by- activating sulphate 2. Elastins are fibrous proteins. - T
17. molecule 3. Triglycerides are known as fat.- T
18. ฀ ฀ ‗Ito cells‘ in the liver is the storage site of-Vitamin E 4. G.E. content of fat is about 4 kcal/g.- F
19. ฀ ฀ Vitamin E is involved in the synthesis of-Ascorbic acid and ubiquinine 5. Net yield of ATP per mole of glycerol is 21.--T
20. ฀ ฀ Ascorbic acid was first isolated by-Szent Gyorgi 6. Thaer developed the first feeding standard.--T
21. ฀ ฀ A dermin or vitamin H is-Pyridoxine 7. Sucrose is sweetest of all the sugars. - F
22. ฀ ฀ Niacin requirements can be compensated with-Tryptophan 8. Maltose is a reducing sugar. - T
23. ฀ ฀ The entire process of citric acid cycle take place in side mitochondria-under 9. Starch equivalent of wheat bran is 45. - T
24. aerobic condition 10. Antibiotics are essential for large ruminants in feed. -- F
25. ฀ ฀ Branching enzyme in glycogen synthesis is- Glycosyl 4,6 transferase 11. NFE is determined by analysis. - F
26. ฀ ฀ Rate limiting step in glycogen synthesis is-addition of activated glycosyl units 12. BMR declines about 8% per year of age.T
27. ฀ ฀ Apart from liver cells which other body tissue is capable of producing glucoseintestinal 13. Vitamin E deficiency causes crazy chick disease.-- T
28. cells 14. VanSoest system of feed analysis was proposed in 1967.-- T
29. ฀ ฀ ―Alkali disease‖ or ―blind staggers‖ is caused by the toxicity of-Selenium 15. Activity increment of cattle, sheep and swine is less when compared to poultry. - T
30. ฀ ฀ Glutathione and insulin contains-Sulfur 16. R.Q. for carbohydrate is 0.7. -- F
31. ฀ ฀ Chromium deficiency may lead to- Impaired glucose tolerance
17. Blood meal is deficient in isoleucine but rich in lysine. -T
32. ฀ ฀ Nickel is essential for urease activity of rumen microbes.
18. Soybean meal is rich in methionine.F
161 162
Dhruv N Desai Dhruv N Desai
19. The pH of silage in A. I. V. method is kept below 4.T 7. Ether extract in solvent extracted cake is ............................%.
20. For guinea pig the Vitamin C requirement is 222 mg/kg DM of diet. - T 8. .............................are the main proteins of connective tissue.
21. Zone of thermal neutrality for pig is 20-260C. --T 9. Toxic amino acid present in subabul ....................................
22. Chief route of phosphorus excretion in ruminants is urine.-- F 10. Cereals are deficient in the amino acid .......................................
23. Molybdenum deficiency in chicken causes femoral head necrosis.---T 11. 1 calorie =.................................J
24. Plasma calcium level is 4-5 mg/dL in most species…. F 12. Methane contains .............................kcal energy /g.
25. Chromium deficiency causes impaired glucose tolerance.-- T 13. Chief VFA in rumen is ..........................
26. Diammonium phosphate contains 18% Nitrogen and 20% Phosphorus.--T 14. The practice of feeding extra concentrate in last 6-8 weeks of pregnancy is called ........................
27. Iron requirement for pig is 80mg/kg diet.--T 16. DCP requirement for milk production in goats is ...............................
28. 1 IU of Vitamin E is equal to 1 mg α tocopherol acetate.--T 17. Urea can replace about ................................% of DCP requirement.
29. Menadione is both water-soluble and fat-soluble.--T 18. Optimum DM content of silage premix is ..............................
30. Fibrobacter succinogens is the chief fibre degrading bacteria in the rumen.--T 19. The mineral associated with the enzyme tyrosinase is ...............................
31. Fungal count in rumen is 103 to 105/ml of rumen liquor.---T 20. Safe upper limit of fluoride in water is ...................ppm.
32. Specific function of rumen fungi is substrate penetration.---T 21. Dissecting aneurysm in chicken is due to deficiency of ...........................
33. Sequestration is function of holotrich protozoa.---T 22. Aflatoxin content in the feed of duck should not exceed ......................ppm.
34. Microbe with highest protease activity in rumen is bacteria--- F 23. Minimum CP content of BIS Type I cattle feed is .......................%.
35. Majority of rumen bacteria are Gram positive.---F 24. .................... is the only VFA found in appreciable quantities in peripheral circulation.
36. Butyrivibrio fibrisolvens is a hemicellulose degrading bacteria.--T 25. Vitamin required in propionic acid metabolism is ...............................
37. Defaunation causes increase in bacterial and fungal biomass.--- T 26. Pica is due to deficiency of .......................................
38. Yeast is a probiotic.--T 27. An adult elephant requires .............................kg green per day.
39. Trypsin acts on the peptide linkage involving aromatic amino acids.--F 28. ............................ is a measure of amount of water soluble steam volatile fatty acids.
40. Secretion of Brunners gland is alkaline.--T 29. Calcium deficiency in bitches results in .................................
41. Amino peptidase and di peptidase is secreted from small intestine.--T 30. Grass staggers is due to deficiency of ...........................
42. Monensin supplementation increases methane production in ruminants.--F 31. N S P present in wheat is ..............................
43. The chief end product of purine metabolism in ruminants is allantoin.--T 32. Thumps in piglets is due to deficiency of .................................
44. Prehensile organ of cattle is lip.--F 33. Father of the science of Nutrition is ......................................
45. MFN has no relationship with feed intake.--F 34. Parakeratosis in swine is due to deficiency of .........................
46. Maintenance requirement of dogs is 132 kcal/kgW0.75. --T 35. Optimum pH of silage is ...........................
47. Haecker showed that nutritive requirements varied with quality and quantity of milk produced in 36. ATP produced from 1 mole of propionate is .............................
dairy cattle.--T 37. The enzyme Alcohol dehydrogenase has the mineral ..............................
48. Microbial digestion in rabbits takes place in proximal colon and caecum.--T 38. The enzyme Arginase contains the mineral ..............................
49. Armsby developed surface area law.--F 39. Specific function of rumen bacteria is ....................................
50. As per NRC, protein content in hamster diet should be 15%.--T 40. Protozoa used for the evaluation of protein quality in feeds is ......................
51. Methane is the chief rumen gas.--F
52. Struvite is Magnesium Ammonium Phosphate.--T
53. In dairy cows grazing resulted in a maintenance requirement that was 40% greater than when they ANSWER :- 13. Acetic acid 28. Reichert-Meissl
were fed in the barn.--T 14. Steaming up Number
54. Terpenes yield isoprene moiety on degradation.--T 1. Feline Central 15. 2.5-3 29. Eclampsia
55. Lymph draining the intestine is always milky in ruminants.--T Retinal Degeneration (FCRD) 16. 70 g/kg milk 30. Magnesium
56. Availability of calcium is 45%.--T 2. Cellulose produced 31. Arabinoxylan
57. β oxidation of fat takes place in endoplasmic reticulum.-- F 3. Abomasum 17. 30 % 32. Iron
58. 1000 ppm TDS is ideal for water.--T 4. β Carotene 18. 35 % 33. Lavoisier
59. Cats are very sensitive to deficiency of arginine.--T dioxygenase 19. Copper 34. Zinc
60. Metabolic water comprises 20-25% of total water intake of domestic animals.--F 5. Glycogen 20. 2 ppm 35. 3.8-4.2
6. 15% 21. Copper 36. 17 ATP
II. Fill in the blanks 7. 0.5-1 22. 0.03 ppm 37. Zinc
1. Taurine deficiency in cats results in ............................... 8. Collagen 23. 22% 38. Manganese
2. Lignin is associated with .............................. in plants. 9. Mimosine 24. Acetate 39. Methanogenesis
3. The true stomach of ruminants is ................................................ 10. Lysine 25. Vitamin B12 40. Tetrahymena
4. Cat cannot convert β Carotene to Vitamin A as it lacks the enzyme............................. 11. 4.184 26. Phosphorus pyriformis
5. Animal starch is ............................................ 12. 13.34 27. 200 kg
6. Hay cannot be stored if the moisture content is above .....................................
163 164
Dhruv N Desai Dhruv N Desai
IMPS for NUTRITION
23. The oil seed crop that is produced in 37. Alkaloid in legume which predispose
the largest amount in the world is? bloat?
1. Chairman of the scientific panel set 12. Beneficial effect of condensed tannin Soybean Saponin
up for the development of the first in legumes is attributed to their ability
edition of feeding standard published to? 24. Domesticated avian species having 38. If no green grass is fed to ruminants
by ICAR in 1985? Protect protein high requirement for Niacin? the concentrate mixture should have
N. D. Kehar Duck Vitamin A at the rate of ……..?
13. Name two tannin complexing agents? 5000 IU/Kg
2. A scientist from KAU, who was a Polyethylene glycol (PEG) and 25. Mineral which is present in glucose
member of the sub-committee for Polyvinylpyrrolidone (PVP) tolerant factor? 39. Other than HMP shunt, the conversion
drafting ICAR feeding standards for Chromium of …..to ……… is a source of NADP
goats? 14. 3,4 DHP (dihydroxypyridone) and 2, 3 in non ruminants?
M. Shivaraman DHP are the break down products of 26. Central Research Institute for Dry Malate to Pyruvate
the antinutritional factor ………….? land Agricuture (CRIDA) is located at?
3. Feeding standards in U. K. is Mimosin Hyderabad 40. Phosphorus content of bran?
developed by? 1.2-1.5%
ARC 15. Chief endproduct of purine 27. Tree loppings or prunings available as
metabolism in ruminants? feed in silvipastoral system is termed? 41. Carprice reaction is concerned with
4. Starch digestibility in rumen ranges Allantoin Top feeds the estimation of?
from? Vitamin A
63-70% 16. A naturally occurring fatty acid found 28. Name a selenium accumulator plant?
in ruminant products which has Astragalus 42. Antimetabolite of folic acid?
5. Chief cellulose degrading bacteria of beneficial health attributes like Aminopterine
rumen? anticarcinogenic activity, anti obesity 29. N : S ratio of wool?
Fibrobacter succinogens and anti atherogenic activity? 5:1 43. Fatal syncope in calves and pigs is due
Conjugated linoleic acid (CLA) to deficiency of?
6. Only VFA present in appreciable 30. The pathway of propionate production Vitamin E
quantity in peripheral blood as an 17. Plants belonging to genus Brassica has in animal consuming high fibrous diet?
important energy source? the antinutritional factor…………? Succinate pathway 44. First discovered amino acid?
Acetate Glucosinolates Aspargine
31. Term metabolizability denotes?
7. Berseem is a plant from? 18. Slobber syndrome and facial eczema ME/GE 45. Chief acid of silage is?
Egypt in cattle is caused by the consumption Lactic acid
of….? 32. Vitamin C requirement for guinea pig
8. A I V method of silage making uses Mycotoxins (slaframine and diet? 46. Silo with minimum spoilage is?
the acids? swainsonine) 200 mg/kg feed Upright silo
Sulphuric acid and Hydrochloric
acid 19. Hydrated sodium calcium 33. Fodder feed is? 47. Scotopsin is rich in the amino acid?
aluminosilicates (HSCAS) are added in Cow pea Lysine
9. Flieg index is a commonly used feed for?
method for evaluation of? Binding mycotoxins 34. VFA having maximum absorption rate 48. Meskawi is a common variety of the
Silage quality is? plant?
20. Maximum permitted level of aflatoxin Butyrate Berseeem
10. Silo-fillers disease is an illness of farm in animal feeds (as per Prevention of
workers that is caused by inhalation of food adulteration act)? 35. Zinc deficiency causes infertility in 49. Domesticated ruminant with highest
the oxides of? 30 ppb (0.03 ppm) males because it is a component of the BMR?
Nitrogen enzyme……..? Goat
21. BT cotton has the gene from the Thymidine kinase
11. ―Vana Mahotsava‖ the annual festival bacterium? 50. Deficiency disease in which ceroid
of trees was inaugurated in? Bacillus thuringiensis 36. Preferred source of enzyme for pigment is accumulated in adipose
1950 estimating degradability of protein in tissue of cats?
22. Plant which is named Biodiesel? French PDI system?
Jatropha S. griseus (protease)
165 166
Dhruv N Desai Dhruv N Desai
Yellow fat b. Tannin b. 18%
disease/Pansteatitis (Vitamin E 59. First two enzymes of urea cycle is c. Glucosinolate c. 73%
deficiency) located in? d. Mimosine d. 36%
Mitochondria
51. Colour of pure vitamin A? 7. Colour of ruminant bile? 15. Urea treatment of straw increases?
Colourless 60. Cell organelle involved in initial steps a. Green a. CP and DCP
of alkoxy-phospholipid biosynthesis b. Golden yellow b. TDN
52. Reference standard in a Bomb which leads to the production of c. Orange c. Dry matter digestibility and feed
calorimeter? plasmalogens? d. Colourless intake
Benzoic acid Peroxisomes d. All the above
8. DCP% is highest for?
53. Brouwer equation is used to estimate? 61. Aminoacid required for the production a. Lucerne hay 16. Pregnancy toxemia is seen in?
Heat production of carnitine? b. Berseem hay a. Sheep and Goat
Lysine c. Oat hay b. Sheep and Rat
54. A fungal enzyme added in poultry d. Wheat straw c. Sheep and Guinea pig
feeds containing barley? 62. ………..% of the nitrogen of milk is d. Sheep and Rabbit
ß glucanase NPN?
5% 9. Which of the following is required for 17. Taurine requirement of cats is ………..mg/kg
55. Ruminant which is most prone to both Ubiquinone synthesis? DM in diet?
cobalt deficiency and copper toxicity? 63. Hammer mill works on the principle a. Vitamin A and Copper a. 200
Sheep of/ b. Vitamin E and Selenium b. 500
Impact grinding c. Vitamin E and Copper c. 800
56. Biological value of microbial protein? d. Vitamin C and Selenium d. 1000
80 64. Major pathway for ATP synthesis in
tissues lacking mitochondria like RBC, 10. The order of toxicity is? 18. Amino acid precursor of lignin?
57. (DCP + DTP)/2 is ? cornea and lens? a. Tyrosin>Threonine>Methionine a. Phenylalanine
Protein Equivalent Glycolysis b. Methionine>Threonine>Tyrosine b. Tyrosine
c. Threonine>Tyrosine>Methionine c. Alanine
58. The pathway occurring in plants 65. Metals inhibiting pyruvate d. Methionine>Tyrosine>Threonine d. Glycine
which is responsible for the conversion dehydrogenase complex?
of fat to carbohydrate? Arsenic and Mercury 11. Which of the following is most important in 19. Mineral needed for acetate incorporation in
Glyoxylate cycle inhibiting the digestibility of paddy straw? cholesterol biosynthesis?
a. Lignin a. Calcium
Multiple Choice Questions c. Barley b. Silica b. Copper
d. Jowar c. Hemicellulose c. Manganese
1. Efficiency of conversion of ß carotene to d. Oxalate d. Magnesium
vitamin A is in the order? 4. Experimental animals for determining GPV of a
a. Rat>Ruminants> Pig> Poultry feed? 12. Concentration of Ammonia and Total VFA in 20. Microbe in rumen capable of breaking
b. Rat>Poultry>Pig>Ruminants a. Rats rumen is highest for? lignocellulosic bond?
c. Rat>Poultry>Ruminants>Pig b. Rabbit a. Goat a. Bacteria
d. Pig>Poultry>Ruminants>Rat c. Guinea pig b. Buffalo b. Protozoa
d. Chick c. Sheep c. Fungi
2. Rumen degradable protein content is highest d. Cattle d. None
for? 5. Order of the efficiency of conversion of
a. Soybean meal Tryptophan to Niacin? 13. Most promising initial symptom of Vitamin A 21. Protease activity in rumen is highest for?
b. Coconut cake a. Pig>Chicken>Duck>Cat deficiency in cows and horses? a. Bacteria
c. Groundnut cake b. Cat>Chicken>Duck>Pig a. Copius lacrymation b. Protozoa
d. Fish meal c. Chicken>Pig>Duck>Cat b. Copius salivation c. Fungi
d. Duck>Chicken>Pig>Cat c. Xeropthalmia d. Bacteriophage
3. The feed which is fed ―whole‖ to poultry but d. Night blindness
―crushed‖ to cattle and pig? 6. Which of the following is common to salseed, 22. Naturally occurring fatty acid has
a. Pearl Millet sorghum and jowar? 14. ……….% NDF in total ration is critical for ………..configuration?
b. Great Millet a. Mucilage maintenance of normal milk fat? a. Cis
a. 66% b. Trans
167 168
Dhruv N Desai Dhruv N Desai
c. Both 29. Which of the following is used as energy 3. Major complement component present in serum is- C3
d. None source (not protein source)? 4. PCR technique was developed by – Kary. B. Mullis
a. Linseed meal 5. Major DNA polymerase involved in replication in prokaryotes is- DNAP III
23. Which of the following is common in nature? b. Salseed meal 6. Most abundant polysaccharide among living system- Cellulose
a. D sugars and D amino acids c. Mustard cake 7. Recombinant DNA technology developed by – Cohen and Boyer
b. L sugars and L amino acids d. Sunflower cake 8. No: of assymetrical carbon atoms in Ribulose- 2
c. D sugars and L amino acids 9. Semi-conservative replication of DNA was proved by- Meselson and Stahl
d. L sugars and D amino acids 30. All reactions in TCA cycle are reversible 10. Protein part of an enzyme is termed as- Apoenzyme
except the formation of? 11. During replication, the enzyme that prevents torsion by breaking DNA
24. The order of salt tolerance? a. Succinyl CoA 12. strands- Topoisomerase.
a. Sheep>Cattle>Pig>Poultry b. Succinate 13. Eukaryotic DNAP for mitochondrial DNA replication is- DNAP-gamma.
b. Sheep>Pig>Cattle>Poultry c. α keto glutarate 14. Monoclonal antibody technique developed by- Kohler and Milstein
c. Cattle>Sheep>Pig>Poultry d. Fumarate 15. The most stable form of DNA and RNA seen under physiological condition is-
d. Pig>Cattle>Sheep>Poultry 16. B-DNA and A-RNA respectively
31. Which of the following cereal has more lysine 17. Type II restriction enzymes were discovered by – Hamilton Smith (1970)
25. Urea supplementation is not recommended if content? 18. In prokaryotes, the DNA polymerase having 5‘-3‘ exonuclease activity-
CP content of ruminant diet is above? a. Rice 19. DNAP I.
a. 18% b. Wheat 20. Concept of Transformation was proved by- Griffith
b. 25% c. Corn 21. During replication of DNA the separation of double strands is done by-
c. 7% d. Oats 22. Helicases.
d. 13% 23. DNA replication takes place from 5‘-3‘ direction.
32. β oxidation can occur in? 24. Cracking of genetic code was performed by- Nirenberg and Mathaei.
26. Which of the following deficiency contribute a.Mitochondria 25. Nucleotide sequence within a gene that is transcribed into RNA but excised
to perosis? b. Peroxisomes 26. before translation in called- Introns.
a. Manganese and Choline c. Both 27. Jumping genes or transposons were first reported by – Barbara McClintock.
b. Biotin and Folic acid d. Endoplasmic reticulum 28. One gene-One Enzyme hypothesis was proposed by- Beadle and Tatum.
c. Thiamine, Manganese, Choline, 29. Operon concept was proposed by- Jacob and Monod.
Biotin and Folic acid 33. Glycosphingolipids and glycoproteins are 30. The major form of super coiling found in chromatin is- Solenoidal.
d. Vitamin B12, Manganese, synthesized in? 31. Phenomenon of Conjugation was put forth by- Lederberg and Tatum.
Choline, Biotin and Folic acid a. Golgi body 32. Histones are rich in amino acids arginine and lysine.
b. Mitochondria 33. Wobble hypothesis was proposed by- Francis Crick
27. Order of tolerance of aflatoxin? c. Endoplasmic reticulum 34. Bacterial DNA is compacted in a structure called- Nucleoid.
a. Chicken>Guinea fowl>Duck d. Glyoxysomes 35. Transfer RNA is produced by - RNApolymerase III.
b. Duck>Guinea fowl>Chicken 36. Chemical method of DNA sequencing was developed by- Maxam and Gilbert.
c. Guinea fowl>Chicken>Duck 34. Rate limiting enzyme in cholesterol 37. ‗Molecular beacons‘ are probes used in detection system for - Real Time PCR.
d. Chicken>Duck>Guinea fowl biosynthesis? 38. Reverse transcriptase was first discovered by- Temin and Baltimore.
a. α 1-4 glucosidase 39. The enzyme employed for amplification of specific genes in PCR technique is-
28. Arrange the susceptibility to aflatoxin by b. HMG CoA reductase 40. Taq DNA polymerase.
domestic animals in descending order? c. Squalene synthetase 41. In Agarose gel electrophoresis, the movement of proteins is based on-
a. Rabbit> Pig> d. 7 α hydroxylase 42. Charge:Mass ratio.
Cattle>Sheep>Chicken 43. Phenomenon of transduction was proposed by- Zinder and Lederberg.
b. Pig>Rabbit>Sheep>Chicken>Cattl 35. For fatty acid synthesis, Acetyl CoA comes 44. The medium used for selecting myeloma cells in hybridoma technology is-
e from mitochondria to cytoplasm as? 45. HAT medium.
c. Chicken>Rabbit>Pig>Sheep>Cattl a. Carnitine 46. Amino acid that does not exhibit optical activity is- Glycine.
e b. Malate 47. In nucleotides, both types of pentoses are in beta-furanose form.
d. Cattle>Sheep>Rabbit>Pig>Chicke c. Citrate 48. In alkaline conditions, RNA is rapidly hydrolyzed due to the presence of 2‘ -
n d. Oxaloacetate 49. OH group.
50. Hinge region of IgG is rich in - Proline.
51. Imidazole group is present in the amino acid- Histidine.
52. In SDS-PAGE, the movement of proteins is based on- Mass.
BIOTECHNOLOGY 53. Separation of proteins in iso-electric focusing is based on- Isoelectric point of
54. the particular protein.
1. Chemical synthesis of DNA was devised by- H. G. Khorana 55. The reagent developed by Sanger to identify the amino terminal amino acid is-
2. Most commonly used type of restriction enzymes are of- Type II 56. 1-fluoro-2,4- Dinitrobenzene.
169 170
Dhruv N Desai Dhruv N Desai
57. ‗Beta turn‘ is a secondary structure of protein. 6. The following have branching except:
58. The most abundant amino acid present in collagen is- Glycine. o Actinomycetes; Nocardia; Mycobacterium; Listeria
59. Hershey and Chase first reported that DNA is the genetic material. 7. The zoonotic disease involving birds playing an important role in the transmission:
60. In reversible competitive inhibition of an enzymatic reaction, Vmax remains o Salmonellosis; Campylobacterosis; Influenza; All
61. same but Km increases. 8. GnRH is secreted from:
62. Co-factor for Glutathione peroxidase is – Selenium. o Hypothalamus; Hypophysis; Ovary; Uterus
63. In Agarose gel electrophoresis the DNA is visualized using- Ethidium 9. Thawing is done at:
64. bromide. o 300C-30s; 370C-30s; 400C-30s; 250C-20s
65. Megaloblastic anemia often occurs due to deficiency of -Folic acid. 10. Hjarre‘s disease in poultry is due to
66. The prosthetic group present in amino transferases is- Pyridoxal phosphate. o E coli; Shigella; Salmonella; Proteus
67. Reverse transcriptases are present in – Retroviruses and Hepadna viruses. 11. Type of lenses in electron microscope:
68. A diploid cell line of human origin is- HeLa. o Glass; Electrostatic; Quartz; None
69. Vero cell lines are obtained from -African green monkey. 12. Average volume of semen ejaculate in boar (ml) is:
70. Cell lines are commonly preserved in- Liquid Nitrogen. o 10; 100; 250; 500
71. Viruses commonly used for production of vector vaccines are- Fowl pox virus, 13. The anaesthesia which facilitates the examination of penis and prepuce
72. Retrovirus and Herpesvirus. o Epidural; Pudental; Paravertebral; High Epidural
73. Size of a prokaryotic cell generally ranges from- 1-10 microns. 14. Brcella ovis infection in ram is causes
o Posthitis; Epididymitis; Orchitis; Prostatitis
15. Calcitonin is secreted by
o Parathyroid; Adrenal; Thyroid; Ovary
16. Type of WBC most numerous in cows is
o Eosinophils; Lymphocytes; Neutrophils; Monocytes
17. Duration of spermatogenesis (days) in buffalo bulls:
o 64; 54; 48; 40
18. The antibiotic doesn‘t have dose dependent antibacterial action
o OTC; Amikacin; Enrofloxacin; Sulfadiazine
19. The estrogen antagonist used to treat mammary and endometrial carcinoma in bitch
o Megestral acetate; Tamoxifen citrate; Estradiol cypionate; Mitotane
20. Cyclozoonosis is related to:
o Brucellosis; Echinococcosis; Leishmaniosis; None
21. Subacute glomerulonephritis is groslly described as
o White Spotted Kidney; Large White Kidney; Small Granular Contracted Kidney; Flea Bitten Kidney
22. Vagus nerve is:
o Sensory Nerve; Motor Nerve; Mixed Nerve; Spinal Nerve
23. Ovulation takes place at the end of estrus period in:
o Canine; Bovine; Ovine; Caprine
24. The following is to be injected prior to any major surgery/ wound management in horses
o Antibiotics; Styptics; NSAIDs; Tetanus toxoid
25. The following produces Aflatoxin:
o Penicillum notatum; Penicillium rubri; Aspergillus fumigates; Trichophyton sp.
26. Acute gangrenous myositis is characteristic pathological lesion of:
o Anthrax; BQ; Leptospirosis; Pasteurellosis
Solve it by own 27. Type of Nucleic acid present in virus:
o DNA; RNA; Both; Either
1. Which of the following inhibits aggregation of platelets 28. World environment day falls on:
o Aspirin; Thromboxane A2; Urokinase; Streptokinase o February 12; April 8; June 5; October 4
2. The longest muscle in animal body is: 29. Lobulation of the lungs is distinct in:
o Biceps femoris; Longissimus dorsi; Longissimus costarum; Levator costarum o Cow; Horse; Dog; Fowl
3. Epithelial pearls are seen in 30. The number of Lumbar vertebrae in dog is:
o Basal cell carcinoma; Adenocarcinoma; Trichoepithelioma; Squamous cell Carcinoma o 6; 5; 7; 8
4. Motility of bacteria is due to 31. Mode of hook worm infection is mainly through
o Plasmid; Flagella; Pili; None o Oral; Skin Penetration; Lactogenic; All
5. The organ needs to be examined for Trichinella spiralis in routine PM examination 32. The nucleated thrombocytes are present in blood of:
o Lungs; Diaphragm; Spleen; Intestine o Horse; Camel; Fowl; Cow
171 172
Dhruv N Desai Dhruv N Desai
33. The disease not produced by Mycoplasma 60. Garlic like odour of gastrointestinal contents is suggestive of poisoning with
o CRD; CBPP; CCPP; BSE o Nitrate ; HCN; Alkali; Phosphorus
34. The following species not affected by FMD 61. The extracellular parasite
o Elephant; Gaur; Rhino; Wild Boar o Babesia; Theileria; Anaplasma; Trypanosome
35. Type of animals equines are: 62. Occupational radiation hazards can be prevented by wearing an apron of
o Polyestrus; Seasonally Polyestrus; Monoestrus; None o Aluminium; Copper; Lead; Silver
36. Bitterness of milk is due to 63. The largest deer found in india
o Proteolysis; Lipolysis; Autolysis; All o Sambar; Nilgai; Spotted Deer; Barasingah
37. The important vitamin that inactivates free radicals 64. The state bird of Gujarat
o Vitamin A; Vitamin B; Vitamin D; Vitamin E o King Vulture; Saras Crane; Pea Fowl; Flamingo
38. Which of the following diseases in poultry is not vertically transmitted? 65. Campylobacterosis is diagnosed by
o EDS 76; Mycoplasmosis; Lymphoid Leucosis; New Castle Disease o Milk Ring Test; HA; Intradermal Inoculation; Vaginal Mucous Agglutination Test
39. The ingredient of blister is: 66. Cubonis test is used to diagnose pregnancy in
o Mag sulph; Bin Iodide of mercury; Copper sulph; Iodine o Bitch; Mare; Sow; Cow
40. The vector through which Trypanosomes are transmitted 67. Programmed cell death is called
o Tabanus; Anopheles; Culicoides; Boophilus o Phagocytosis; Mytosis; Necrosis; Apoptosis
41. Camel is 68. Rodent control is very much effective in control of
o Spontaneous Ovulator; Induced Ovulator; Silent Ovulator; None o Leptospirosis; Plague; Salmonellosis; All
42. The largest immunoglobulin 69. The term epsilon is associated with
o Ig G; Ig M; Ig A; Ig D o Brucellosis; Enterotoxaemia; Marek‘s Disease; Erysipelas
43. An example of long duration local anaesthetic 70. Blow gun rifle is fairly accurate for the target up to the distance of
o Bupivacaine; Lignocaine; Lidocaine; Paracaine o 40 metres; 80 feets; 80 metres; 40 feets
44. The chemical used to control snail population 71. A live vaccine among the following
o Copper sulph; Pot hydroxide; Carbon tetrachloride; None o HS; Brucella S19; BQ; Rabies
45. The infective stage of Schistosoma spps. 72. A well established protozoal disease transmitted by way of milk
o Eggs; Sporocyst; Cercaria; Metacercaria o Toxoplasmosis; Giardiosis; Cryptosporidiosis; None
46. Reserpine is obtained from 73. Cells spermatids are
o Ocimum sanctum; Adhatoda vasica; Leptadena Reticulare; Rauwolffia serpentine o Haploids; Diploids; Tetraploids; Triploids
47. Soil erosion is due to: 74. Navicular bone in horses
o Deforestation; Soil Formation; Soil Conservation; All o Patella; Proximal Sesamoids; Febella; Distal Sesamoids
48. Diffuse suppuration in the sub cutaneous tissue is 75. Length of gestation in mares
o Pustule; Phlegmon; Acne; Furuncle o 9 months 9 days; 8 months 8 days; 10 months 10 days; 11 months 11 days
49. During second stage of parturition there is a release of an extra amount of 76. Ingestion of Lantana foliage causes
o Oestrogen; Progesterone; Oxytocin; PGF2 alfa o Hepatotoxicity And Secondary Photosensitization; Acute Enteritis; Pulmonary Haemorrhage;
50. Brucella organisms multiply in the presence of the alcohol Nephrotoxicity
o Glucose; Galactose; Erythritiol; Fructose 77. Parasite of pulmonary artery
51. Domestic sewage contains the following o Sarcoptes; Cysticercus; Toxoplasma; Dirofilaria immitis
o Chemicals; Organic Matter; Highly Toxic Substances; All 78. Irritant and non isotonic drug solution are injected by which route
52. Gasping is a symptom in: o Intravenous; Intramuscular; Sub Cutaneous; Intraperitoneal
o ILT; Avian Influenza; Avian Leucosis; Ranikhet Disease 79. Deaths among clinically affected animals indicates
53. The sporadic disease is: o Incident Rates; Morbidity Rate; Fatality Rate; Prevalence Rate
o HS; Tetanus; FMD; Avian Influenza 80. The stomach fluke disease is caused in cattle due to
54. The leucocytic granules more toxic to parasites o Paramphistomum cervi; Moniezia expansa; Fasciola hepatica; Neoascaris vitulorum
o Eosinophils; neutrophils; basophils; lymphocytes 81. In paraffin block making technique fat/lipid is dissolved by
55. The target organ of shock in dogs is o Formaline; Xylene; Paraffin; Alcohol
o Liver; Lungs; Intestine; Heart 82. Electron microscope was invented by
56. Electrical stunning is widely used in o Leewenhock; Pastuer; Knoll and Ruska; Elford
o Cattle, Poultry; Pig, Poultry; Buffalo, Poultry; Sheep 83. The characteristic lesion in brain of cow affected by mad cow disease
57. The vector for Leishmania is o Neuronal Degeneration; Neuronal Vacuolation; Inclusion Bodies in Neurons; Encephalitis
o Phlebotomus; Culicoides; Tabanus; Musca 84. Antihypertensive drug with angiotensin converting enzyme inhibiting action
58. The presentation of fetus in breech presentation is o Prazosin; Verapamil; Frusemide; Captopril
o Anterio Longitudinal; Posterior Longitudinal; Dorso Transverse; Ventro Transverse 85. Apex of bovine heart is attached by
59. During recent outbreak of Avian Influenza in South East countries, subtype has been identified o Cardio thoracic ligament; Pericardio sternal ligament; Cardiac phrenic ligament; Coronary ligament
o H5N1; H5N2; H2N9; H1N5 86. Purkinjee cells are noted in the
173 174
Dhruv N Desai Dhruv N Desai
o Myocardium; Cerebellum; Cerebrum; Myometrium 112. Teat surgery is more successful during which stage
87. Michael Bishop and Harold Varmus were awarded Nobel Prize in 1989 for their work on o Lactating Stage; Dry Stage; Post Pubertal Stage; None
o Monoclonal Antibodies; Proto Oncogenes; Chemical Carcinogens; Apoptosis 113. Pipe stem liver condition is seen in which of the following infection
88. Bioterrorism is associated with o Fasciola hepatica; Moneizia expansa; Dicrocelium dentriticum; None
o Echinococcosis; Anthrax; Leishmaniosis; Tuberculosis 114. The diabetes insipidus develops due to deficiency of
89. Caecal coccidiosis is caused by o ADH; Glucagon; Insulin; Aldosterone
o E acervulina; E magna; E tenella; E necatrix 115. An important source of biofuel is (Ethanol)
90. Name the drug of choice for treatment of Thieleriosis o Jowar; Oat; Sugarcane; Rice
o Suramin; Buparvaquon; Nitrothiozol; Clopidol 116. Main immunoglobulin protecting mucosal surface is:
91. An antibiotic that interferes with bacterial cell wall synthesis o Ig M; Ig A; Ig G; All
o Gentamicin; Penicillin; Sulphonamide; None 117. Surgical removal of stones from the urinary bladder is known as
92. Fundamental germ layer o Nephrectomy; Cystotomy; Penectomy; Nephrotomy
o Ectoderm; Mesoderm; Endoderm; Mesenchymal Cells 118. The reference test for diagnosis of rabies
93. Agar is composed of o FAT; AGPT; Agglutination; ELISA
o Protein; Lipids; Carbohydrates; Mixture of all three 119. Toxic principle present in cotton seed is
94. Rabies virus is o Sinigrin; Gossypol; Tannin; Mimosin
o Viscerotropic; Neurotropic; Dermotropic; None 120. Microglia cells are present in
95. Reverse transcriptase enzyme is present in the virus family of o Blood; Bone Marrow; Pancreas; Brain
o Pox; Adeno; Retro; Irido 121. Suturing of the uterus after the Caesarean section starts from
96. The drug active against cestodes o Ovarian end; Cervical end; Middle of the uterus; Either of end
o Pyrantel; Thiophanate; Hexachlorophene; Praziquantel 122. Anaesthesia is produced when the blood concentration of chloroform reaches to level of
97. The desirable limit of fluoride (mg/l) in human drinking water is o 0.035%; 0.35%; 0.053%; 1.035%
o 1; 3; 5; 7 123. Surgical operation for providing drainage from middle ear is known as
98. The inflammation of hoof of horse is called o Zepps Operation; Hyovertebrotomy; Ventriculectomy; Bulla osteotomy
o Synovitis; Bursitis; Naviculitis; Laminitis 124. T lymphocytes get maturity in organ
99. The brachicephalic breed of dog o Liver; Thymus; Spleen; Bursa
o Collie; Pug; Doberman; German Shepherd 125. Antibacterial drug associated with nephrotoxicity is
100. Warfarin poisoning is treated by administration of Vitamin o Tetracycline; Chloramphenicol; Streptomycin; Levofloxacin
o K; E; A; C 126. Kohler and Mihlstein are known for the achievement in
101. Lemberts pattern is not used for sutured o Hybridoma; Nucleotide sequencing; Viral Culture; Prion discovery
o Uterus; Urinary Bladder; Oesophagus; Rumen 127. Bronze discoloration of liver is characteristic feature of
102. The smallest virus o Pullorum Disease; Fowl Cholera; Fowl Typhoid; Spirochaetosis
o Fowlpox; FMD; Ranikhet Disease; Avian Leukosis 128. Double stranded RNA is found in
103. Atropine : o Retro Virus; Reo Virus; Pox Virus; Parvo Virus
o Reduces metabolic rate; 129. The total dry matter requirement of cow
o Reduces salivary, gastric and bronchial secretion; o 3% of body wt; 3% of metabolic body wt; 5% of body wt; 1% of body wt
o Reduces body temperature 130. Raw egg feeding in dog may produce deficiency of
o Decrease intestinal motility o Biotin; Cholin; Niacin; Pantothenic acid
104. The larva that causes VLM 131. Catgut is prepared from the intestine of
o Toxocara canis; Ascaris suum; Ancylostoma caninum; Dirofilaria immitis o Rabbit; Sheep; Pig; Cat
105. The microchromosomes are seen in 132. Sodium calcium EDTA is used as antidote in poisoning of
o Cattle; Horse; Poultry; Dog o Arsenic; Mercury; Lead; Copper
106. Othaematoma is the haematoma involoving 133. The common infectious disease affecting snake
o Eye and Ear; Ear; Eye; None o Brucellosis; Pasteurellosis; Salmonellosis; Tuberculosis
107. Punched ulcers in abomassum is caused by 134. WTO is related with
o Babesia bigemina; Theileria annulata; Anaplasma marginale; Babesia bovis o Environment; Biodiversity; International tourism; International trade
108. Death of animal suffering from rabies occurs due to 135. Nervous sign in ketosis is due to
o Neuritis; Gastritis; Asphyxia; Paralysis o Hypocalcemia; Hypoproteinemia; Hypoglycemia; Hypophosphatemia
109. Cattle genome is made up of how many organic bases 136. Amputation of horn in goats can be done by blocking of
o 2.9-3.1 trillion; 2.9-3.1 billion; 2.9-3.1 million; 2.9-3.1 lakh o Cornual nerve; Infraorbital; Cornual and Infraorbital; None
110. Brooder pneumonia is caused by 137. The range of pH of rumen liquor
o Aspergillus flavus; Aspergillus ochoreceal; Aspergillus parasiticus; Aspergillus fumigatus o 2-3; 5-7; 7-8; 3-5
111. Paralysis of hind quarter is termed as 138. Highly toxic poison has oral LD50 value of
o Hemiplegia; Diplegia; Quadriplegia; Paraplegia o <1 mg/kg; 1-50 mg/kg; 50-100 mg/kg; 1-50 mg/kg
175 176
Dhruv N Desai Dhruv N Desai
139. The molecules is an endogenous antigen (i) Meloxicam (A) Xylazine antagonist
o MHC Type I; MHC Type II; MHC Type III; MHC Type E (ii) Congenital (B) Equithesin
140. Thin membranous partition between lateral ventricles of brain (iii) Yohimbine (C) Recto vaginal fistula
o Tapetum lucidum; Septum lucidum; Intradorsal septum; Inter ventricular septum (iv) Pudental nerve block (D) NSAID
141. Whales and dolphins breathe through (v) Anaesthesia for horse (E) Ischio rectal fossa
o Gills; Spiracles; Body surface; Lungs
142. The following characteristic palpable through the rectal examination for the pregnancy
diagnosis in 35 days in cattle DRUGS RELATED TERMS
o Asymmetry of uterine horn; CL on ovary; Slipping of foetal membrane; All of above
143. To relieve the right side uterine torsion, animal should be cast in (i) Xylazine (A) Inhalant Anaesthesia
o Left side; Right side; Dorsal recumbency; Sternal recumbency (ii) Largactil (B) Muscle Relaxant
144. The ligament surgically cut for correction of subluxation of patella in bovine (iii) Ether (C) Osmotic Diuretic
o Dorsal; Ventral; Middle; Medial (iv) Mephenesin (D) Sedative
145. Dilated pupils and fish eye appearance is observed in which stage of anaesthesia (v) Mannitol (E) Chlorpromazine HCl
o Stage 3; Stage 1; Stage 2; Stage 4
ITEMS RELATED TERMS

(i) Gastrulation (A) Ear Ossicle


(ii) Malleus (B) Prostate
(iii) Sex gland male dog (C) Cock
(iv)Synsarcosis (D) Trilaminar Embryo
(v) Spur (E) Muscular Joint-Forelimb
Cross matching type questions.
ORGANISM SELECTIVE MEDIUM

(i) Salmonella (A) Mannitol salt Agar


DISEASES RELATED TERMS (ii) Staph. aureus (B) LJ Medium
(iii) Hemophilus (C) Chocolate Agar
(i) Mastitis (A) Wart hog disease (iv) E coli (D) Brilliant Green Agar
(ii) Strangles (B) Multiceps multiceps (v) Mycobacterium tuberculosis (E) EMB Agar
(iii) African swine fever (C) BTB Test
(iv) Gid (D) Blue eye SPECIES SPERM CONCENTRATION
(v) Infectious Canine Hepatitis (E) Equine Distemper
(i) Buffalo bull (A) 3000 million/ml
(ii) Ram (B) 3600 million/ml
DISEASES ETIOLOGY (iii) Stallion (C) 250 million/ml
(iv) Cock (D) 1000 million/ml
(i) Hard pad disease (A) Borrelia anserina (v) Boar (E) 150 million/ml
(ii) Wool sorter disease (B) Bacillus anthracis
(iii) IBR (C) Coxiella burnetti SOURCE ANTIBACTERIAL AGENT
(iv) Q fever (D) Bovine Herpes Virus
(v) Fowl Spirochaetosis (E) Canine Distemper virus (i) Strept. venezulae (A) Polymyxin / Colistin
(ii) Micromonospora purpurea (B) Neomycin
ITEMS RELATED TERMS (iii) Bacillus colistinus (C) Gentamicin
(iv)Bacillus subtilis (D) Bacitracin
(i) Brucellosis control (A) Progesterone (v) Strept. fradiae (E) Chloramphenicol
(ii) Campylobacterosis (B) Metronidazole
(iii) Trichomoniosis (C) Estrogen DISEASES RELATED TERMS
(iv) Follicale (D) Vaccine S 19
(i) Shipping fever (A) Brucellosis
(v) Corpus Luteum (E) StreptoPenicillin
(ii) Tick fever (B) Pasteurellosis
(iii) Spleenic fever (C) Salmonellosis
ITEMS RELATED TERMS (iv)Malta fever (D) Ehrlichiosis
(v) Enteric fever (E) Anthrax
177 178
Dhruv N Desai Dhruv N Desai
b) Swine and poultry 22. Type of necrosis in which cellular details
c) Dog and cattle are lost and architectural details are
SET- 1 8. Which of the following is resistant cell d) Dog and horse preserved-
against irritant- 16. Amylodoisis of spleen is met in a) Caseative
1. Study of tumors is called as- a) Hepatic cell
a) Etiology tuberculosis in- b) Liquifactive
b) Renal cell a) Poultry c) Fat necrosis
b) Oncology c) Fibroblast
c) Pathology b) Dog d) Coagulative
d) Brain cell c) Cattle 23. In which of the following liquifactive
d) None of the above. 9. Cloudy swelling less likely to appear in
2. Which of the following is not a anomaly d) Horse necrosis is not due to the action of
which cells- 17. Diffuse and focal amyloid infiltration of autolytic enzymes -
related to reproductive system- a) Cardiac muscle
a) Freemartin spleen is termed as- a) Abscess
b) Tubular cells of kidney a) Bacon spleen and sago spleen b) Thiamine deficiency
b) Pseudo hermaphrodite c) Brain
c) Monster respectively c) Cyanide toxicity
d) Liver cells. b) Sago spleen and bacon spleen d) Crazy chick disease.
d) Hermaphrodite. 10. Pox and FMD are characterized by-
3. Which of the following disease is not respectively 24. In which organ autolytic changes are slow-
a) Paranchymatous degeneration c) Ham spleen and sago spleen a) Brain
transmitted by droplet infection- b) Hyaline degeneration
a) CBPP respectively b) Kidney
c) Amyloid degeneration d) Bacon spleen and ham spleen c) Liver
b) Tuberculosis d) Hydropic degeneration.
c) Glanders respectively d) Bone marrow
11. A condition of cells and connective tissue 18. Gout occurs in poultry due to deficiency 25. Action of alkali causes-
d) Tetanus. which become converted into a
4. In which disease mechanical carrier of- a) Dry gangrene
homogeneous, glassy material is called as- a) Vitamin C b) Gas gangrene
doesn‘t play role in its spread- a) Paranchymatous degeneration
a) Anthrax b) Vitamin D c) Moist gangrene
b) Hyaline degeneration c) Vitamin A d) All of the above.
b) Surra c) Amyloid degeneration
c) Tuberculosis d) Vitamin K 26. Which of the following statement
d) Hydropic degeneration. 19. Which of the following are lipotropic regarding gangrene is incorrect-
d) All of the above. 12. Mucin takes which colour in PAS stain-
5. Indiscriminate use of drugs or factors whose imbalance leads to fatty a) In moist gangrene demarcation
a) Blue changes in liver- between live and dead tissue is visible
overmedication favors the spreading of b) Purplish red
which disease in poultry- a) Serine and choline b) In dry gangrene demarcation between
c) Black b) Choline and methionine live and dead tissue is visible
a) Coccidiosis d) Green
b) Fowl typhoid c) Serine and methionine c) Gas gangrene is mainly caused by
13. Cystadenoma of ovary contains a viscid d) Methionine and glysine. anaerobes
c) HPS material, called as-
d) IBD. 20. Ketone bodies can be demonstrated in d) Crackling sound is evident in gangrene
a) Mucin urine by- produced by anaerobic bacteria.
6. Who termed the plasma membrane as b) Serous fluid
‗Unit Membrane‘- a) Hunter test 27. In anthrax clotting of blood is absent
a) Robertson
c) Pseudomucin b) Rothra‘s test because-
d) Fibrin c) Benedict test a) Hypostatic congestion occurs
b) Rudolph Virchow 14. Which of the following also termed as
c) Metchnikoff d) Gmelins test. b) Polypeptide capsule is present in
waxy degeneration- 21. In caseous necrosis cracks on nucleus bacteria
d) John hunter a) Paranchymatous degeneration
7. ER is not evident in- appears and its fragments scattered, is c) Polysaccharide capsule is present in
b) Hyaline degeneration called as- bacteria
a) Liver cells c) Amyloid degeneration
b) Brain cells a) Karyolysis d) Fibrinolysin produced by bacteria
d) Hydropic degeneration. b) Pyknosis causes lysis of fibrin
c) Kidney cells
d) Muscle cells. c) Chromatolysis 28. During circulatory changes in
15. Amyloid degeneration is most commonly d) Karyoschisis inflammation axial stream contains-
seen in- a) Plasma
a) Cattle and swine b) RBC‘s and WBC‘s
179 180
Dhruv N Desai Dhruv N Desai
c) Blood constituents except plasma 36. The animal in which serum is particularly b) Mg d) In hyperplasia cells size increases and
d) Platelets only rich in antienzyme, poor in leucocytes and c) Cu there is disruption of normal
suppurative conditions is not commonly d) Zn architecture
29. Which of the chemical mediator only seen, is- 44. Which of the following regenerates by 50. Transformation of one type of cell into
present in mast cell of rat and mouse- a) Fowl fibrous tissue proliferation- another is called as-
a) 5-HT b) Hamster a) Smooth muscle a) Metaplasia
b) Histamine c) Guinea pig b) Skeletal muscle b) Dysplasia
c) Kinin d) Rabbit c) Cardiac muscle c) Hyperplasia
d) Globulin permeability factor 37. Small suppurative inflammation in the skin d) None of the above. d) Anaplasia
30. Which cells are referred as microphages of which involves a hair follicle is called as- 45. The peripheral nerve undergoes a series of 51. Alteration in the size, shape and
Metchinkoff- a) Sinus retrogressive changes known as- orientation of adult cells is called as-
a) Macrophages b) Boil a) Zenker‘s degeneration a) Metaplasia
b) Lymphocytes c) Pustule b) Hyaline degeneration b) Dysplasia
c) Basophils d) Phlegmon c) Wallerian degeneration c) Hyperplasia
d) Neutrophils 38. In which disease inclusion bodies are d) None of the above d) Anaplasia
31. Phagocytic cells of body are- intranuclear only- 46. Which of the following cell doesn‟t 52. Local deficiency of arterial blood in an
a) Macrophages and lymphocytes a) Pox produce any endogenous pyrogen- organ is called as-
b) Macrophages and neutrophils b) Rabies a) Lymphocyte and macrophage a) Hyperemia
c) Neutrophils and lymphocytes c) Canine distemper b) Neutrophil b) Congestion
d) Macrophages, neutrophils and d) Infectious canine hepatitis c) Monocyte c) Ischemia
lymphocytes 39. Which cells are included in labile cells- d) Lymphocyte and eosinophil d) Induration
32. Which sentence regarding plasma cell is a) Fibroblast 47. In fever which stage is termed as 53. Thrombosis of spleen is seen in-
incorrect- b) Nerve cells ‗Fastigium‘- a) Strangles
a) They are non-phagocytic c) Muscle cells a) Cold stage b) Glanders
b) Possess cart wheel shape nucleus d) Skin cells b) Hot stage c) Swine fever
c) Present in blood and tissue both 40. Which is a feature of viral inflammation- c) Sweating stage d) Infectious bovine rhinotracheitis
d) Present in tissue only. a) Suppuration d) None of the above. 54. Thrombi found in auricles of heart is
33. Infectious feline enteritis and malignant b) Presence of neutrophils 48. Hypoplasia means- called as-
catarrhal fever are characterized by- c) Presence of lymphocytes a) Organ has a beginning but due to some a) Mural thrombi
a) Catarrhal inflammation d) All of the above. reason it had failed to develop b) Valvular thrombi
b) Serous inflammation 41. Autolysis of dead tissue by enzymes b) Organ fail to develop to their full c) Saddle thrombi
c) Fibrinous inflammation derived from inflammatory leucocytes is normal size though there was a d) Ball thrombi
d) Suppurative inflammation called as- beginning 55. When Occlusive thrombi develop a
34. Croupous and diphtheric membrane are a) Autolysis c) Decrease in number of cells in a tissue passage through which partial blood
seen in- b) Chromatolysis d) Complete absence of organ and there is supply is maintained, is called as-
a) Catarrhal inflammation c) Heterolysis no beginning a) Laminated thrombi
b) Serous inflammation d) Lysis 49. Which of the following statement is b) Saddle thrombi
c) Fibrinous inflammation 42. Connective tissue continues to grow under correct- c) Canalized thrombi
d) Suppurative inflammation the scar even after the epithelium covers it, a) In hypertrophy cells size increases and d) Propagating thrombi
35. When membrane is not easily peeled away is called as- there is disruption of normal 56. Thrombi at bifurcation of an artery is
from underlying tissue and is firm, is a) Proud flesh architecture called as-
called as- b) Trephone b) In hyperplasia cells number increases a) Laminated thrombi
a) Peudo membrane c) Keloid and there is no disruption of normal b) Saddle thrombi
b) Croupous membrane d) Granulation tissue architecture c) Canalized thrombi
c) Diphtheric membrane 43. Which is necessary in diet for early wound c) In hypertrophy cells size increases and d) Propagating thrombi
d) False membrane healing- there is no disruption of normal 57. Arterial thrombi is seen in-
a) Mn architecture a) Strongylus vulgaris infection
181 182
Dhruv N Desai Dhruv N Desai
b) Spirocercalupi infection c) Obstruction of lymph vessels d) African swine fever 79. Bran like scales on skin due to vitamin A
c) Onchocercaarmillata infection d) Increase permeability of capillary deficiency is called as-
d) All of the above endothelium 72. Allotriophagy in animals is due to a) Keratomalacia
58. Which of the following statement is true- deficiency of- b) Pityriais
a) Lines of zahn is seen in post-mortem a) Ca c) Hyperkeratosis
clot 65. Most radiosensitive organ among the b) P d) Parakeratosis
b) Post mortem clot is friable, dry and following is- c) Mg 80. Battery sickness in poultry which causes
crumbles when pressed a) Liver d) Se paralysis and atrophy of striated muscles is
c) Surface of thrombus is smooth and b) Bone 73. Which of the following regarding piglet caused by-
glistening c) Brain anemia is correct- a) Vitamin C
d) Thrombus is firmly attached to d) Bone marrow a) It occurs in baby pigs which are reared b) Vitamin D
endothelium and may fills the vessel. 66. Most susceptible animals for salt toxicity- on soil because it is deficient in Fe c) Vitamin E
59. Emboli are rarely present in- a) Fowl and pigs b) It is a macrocytic hypochromic type of d) Vitamin K
a) Veins b) Fowl and horses anemia 81. Polyneuritis and star grazing attitude in
b) Arteries c) Fowl and cattle c) Piglets reared on cement floors easily poultry is due to deficiency of-
c) Capillaries d) Fowl and cats fulfill their Fe requirement and not a) Vitamin B1
d) None of the above 67. Calcification which is due to necrosis or suffers from anemia b) Vitamin B2
60. A cat is met with an accident and there is degeneration of cells and calcium level of d) It is microcytic hypochromic type of c) Vitamin B6
crushing injury to the femur and tibia- blood is normal, is called as- anemia d) Vitamin B12
fibula bones, the emboli most likely to a) Metastatic calcification‘ 74. Which of the following is not due to 82. Black tongue or Stuttgart disease in
occur in circulation will be- b) Dystrophic calcification copper deficiency- canines is due to deficiency of-
a) Air emboli c) Pathological calcification a) Sway back a) Pantothenic acid
b) Paradoxical emboli d) None of the above b) Enzootic marasmus b) Niacin
c) Fat emboli 68. In south africa ‗Lamsiekte‘ disease which c) Fatal syncope in cattle c) Riboflavin
d) Lymphatic emboli means lame sickness is caused due to d) Achromotrichia d) Thiamine
61. The emboli that passes directly into the left deficiency of- 75. Which of the following pair is incorrectly 83. Goose stepping gait in pigs is due to
auricle from the right auricle through a a) Ca matched- deficiency of-
patent foramen ovale, is called as- b) P a) Steely wool- Zinc a) Pantothenic acid
a) Amniotic emboli c) Mg b) Stiff lamb disease- Vitamin E b) Niacin
b) Paradoxical emboli d) Fe c) Exudative diathesis- Vitamin E c) Riboflavin
c) Air emboli 69. Grass tetany is caused due to deficiency d) Nutritional roup- Vitamin A d) Thiamine
d) Parasitic emboli of- 76. Shifting lameness is seen in toxicity of- 84. In poultry development of feathers are
62. Infraction is an area of- a) Ca a) Ergot alkaloids retarded and dermatitis and broken
a) Caseative necrosis b) Cu b) Zinc feathers are seen in deficiency of-
b) Liquifactive necrosis c) Mg c) Fluorine a) Pantothenic acid
c) Coagulative necrosis d) Mn d) Iodine b) Niacin
d) Fat necrosis 70. Perosis or slipped tendon in fowl is caused 77. Chronic selenium toxicity produces- c) Riboflavin
63. Infracts of brain will produce- due to deficiency of- a) Alkali disease d) Thiamine
a) Apoplectic cyst a) Mg b) Sway back disease 85. Folic acid deficiency causes-
b) Dentigerous cyst b) Fe c) White muscle disease a) Macrocytic anemia
c) Hydatid cyst c) Mn d) Blind staggers b) Microcytic anemia
d) All of the above d) Cu 78. Peat scour is caused by- c) Normocytic anemia
64. Which of the following is not a cause of 71. Which of the following is a not a disease a) Molybdenum toxicity d) None of the above.
edema- of swine- b) Copper deficiency
a) Increase in hydrostatic pressure of a) Hog cholera c) Both of the above
blood b) Diamond skin disease d) Copper toxicity
b) Increase in osmotic pressure of blood c) Glanders 86. Poikilocytosis means-
183 184
Dhruv N Desai Dhruv N Desai
a) Variation in shape of RBC c) Silverosis 1. b 48. b
b) Variation in size of RBC d) Argyria 2. c 49. c
c) Variation in number of RBC 95. Common term for deposition of exogenous 3. d 50. a
d) None of the above pigmentation in body is called as- 4. c 51. b
87. Pregnancy toxemia in ewes is due to a) Anthracosis 5. a 52. c
deficiency of- b) Plumbism 6. a 53. c
a) Folic acid c) Argyria 7. c 54. d
b) Choline d) Pneumoconiosis 8. c 55. c
c) Biotin 96. Which of the following statement is true- 9. c 56. b
d) Riboflavin a) Metastasis occurs in benign tumors 10. d 57. d
88. Chastek paralysis is caused due to b) Necrosis is more evident in malignant 11. b 58. d
deficiency of- tumors 12. b 59. a
a) Niacin c) Rhabdomyoma is benign tumor of 13. c 60. c
b) Thiamine smooth muscle 14. c 61. b
c) Mn d) Sarcoma always refers to benign 15. d 62. c
d) Vitamin K tumors 16. a 63. a
89. Hydrocele means accumulation of fluid in- 97. Neural lymphomatosis is also called as- 17. a 64. b
a) Thorax a) Avian leucosis complex 18. c 65. d
b) Tunica vaginalis b) Infectious bursal disease 19. b 66. a
c) Oviduct c) Fowl typhoid 20. b 67. b
d) Peritoneal cavity d) Marek‘s disease 21. d 68. b
90. Starling‘s hypothesis is related with- 98. Which is incorrect regarding avian 22. d 69. c
a) Thrombosis leucosis complex- 23. a 70. c
b) Infraction a) Caused by RNA virus 24. a 71. c
c) Edema b) Nodular tumors in bursa of fabricus are 25. c 72. b
d) Shock seen 26. a 73. d
91. Which of the following is not included in c) Nerves are commonly affected 27. d 74. b
endogenous pigmentation- d) Occular lesions are not present 28. c 75. a
a) Plumbism 99. Epithelial pearls are seen in- 29. a 76. c
b) Melanosis a) Basal cell carcinoma 30. d 77. a
c) Hemosiderosis b) Seminoma 31. b 78. c
d) None of the above c) Cholangiocellular carcinoma 32. c 79. b
92. Van den bergh test is indirect in- d) Squamous cell carcinoma 33. c 80. a
a) Hemolytic jaundice 100. Which of the following is a tumor of 34. c 81. a
b) Obstructive jaundice adrenal medulla- 35. c 82. b
c) Toxic jaundice a) Dysgerminoma 36. d 83. a
d) None of the above b) Arrhenoblastoma 37. b 84. a
93. Which substance play major role in c) Pheochromocytoma 38. d 85. a
brucellosis- d) Interstitial cell adenoma. 39. d 86. a
a) Proline 40. c 87. b
b) Erythriotol ****************************************
41. c 88. b
c) Hyaluronic acid **************************************
42. c 89. b
d) Estrogen 43. d 90. c
94. Deposition of silver particles in body is 44. b 91. a
called as- 45. c 92. a
a) Siderosis ANSWERS 46. d 93. b
b) Anthracosis 47. b 94. d
185 186
Dhruv N Desai Dhruv N Desai
95. d d) Euploidy a) Avian leucosis complex a) Arteritis
96. b 5. Porphyria is most commonly seen in- b) Ranikhet disease b) Aneurysm
97. d a) Cattle and swine c) Marek‘s disease c) Thromboangitis
98. c b) Sheep and goats d) Infectious bursal disease d) Polyarteritisnodusa
99. d c) Laboratory animals 14. Which disease is also called as ‗Big Liver 22. Type of aneurysm in which a pouch is
100. c d) Horses and dogs Disease‘- formed on one side of the wall, is-
6. Papilliferouscystadenoma of bile duct a) Avian leucosis complex a) Fusiform aneurysm
**************************************** epithelium in rabbits is caused by- b) Ranikhet disease b) Circoid aneurysm
************************************** a) Gongylonemaneoplasticum c) Marek‘s disease c) Saccular aneurysm
b) Eimeriastiedae d) Infectious bursal disease d) Arteriovenous aneurysm
c) Spirocercalupi 15. Sciatic nerve is affected unilaterally in-
d) Cysticercusfasciolaris a) Avian leucosis complex
7. Model virus for tumor research work is- b) Ranikhet disease
a) Polyoma virus c) Marek‘s disease 23. Stagnation of blood in the dilated veins
b) Rous sarcoma virus d) Infectious bursal disease causes pain is called as-
c) Shope papilloma virus 16. Tumors of parathyroid gland produces a) Phlebolith
d) Reo virus condition called as- b) Phlebitis
8. Tumor originating from all germinal layers a) Lumpy jaw c) Aneurysm
is- b) Rubber jaw d) Varicose veins
a) Neurofibromata c) Lock jaw 24. Aneurysm of minute arteries is called as-
SET-2 b) Dermoid cyst d) Bottle jaw a) Berry aneurysm
c) Teratoma 17. A pathological condition in which a shunt b) Polyarteritisnodusa
1. Condition in which local loss of pigment d) Sertolicell tumor connects the pulmonary artery and the c) Arteriovenous aneurysm
occurs, is called as- 9. In myxomatosis of rabbit inclusion bodies aorta is called as- d) None of the above
a) Albinism are- a) Intraventricular foramina 25. Lymphadenitis is inflammation of-
b) Hemosiderosis a) Intranuclear b) Tetrology of fallot a) Lymph node
c) Leucoderma b) Intracytoplasmic c) Patent foramen ovale b) Lymph vessel
d) Acanthosisnigricans c) Both d) Patent ductusarteriosus c) Both of the above
2. Large quantity of hemosiderin is deposited d) None of the above 18. A pathological condition in which right d) None of the above
in liver and kidneys in- 10. Which tumor is called as gaint cell tumor- atrium communicates with left atrium is 26. Erythropoiesis is intravascular in-
a) Babesiosis a) Osteoma called as- a) Swine
b) Leptospirosis b) Osteosarcoma a) Intraventricular foramina b) Cattle
c) Equine infectious anemia c) Osteoclastoma b) Tetrology of fallot c) Dog
d) Dicroceliumdendriticum d) Osteomyxoma c) Patent foramen ovale d) Fowl
3. In vitamin E deficient animals, a peculiar 11. Dilatation of liver sinusoids is called as- d) Patent ductusarteriosus 27. Erythrocytes that have a narrow rim of
wax-like acidfast material accumulates in a) Hamartomas 19. Macrophages laden with lipoids seen in hemoglobin surrounding a large pale area
uterine muscle fibers, ovary and testes, is- b) Telangietasis atherosclerosis are called as- are called as-
a) Amyloid c) Recklinghausen‘s disease a) Gitter cells a) Cabot rings
b) Byssinosis d) Myxomatosis b) Limmocytes b) Anisocytosis
c) Hematoidin 12. Astrocytoma is single in all animal species c) Foam cells c) Annulocytes
d) Ceroid EXCEPT- d) Microglial cells d) Leptocytes
4. Transport of pigment particles by a) Pig 20. Term used for hardening of artery is- 28. Bluish thread like rings in RBC‘s and
macrophages to the connective tissue in b) Cattle a) Atherosclerosis which are nuclear remnants are called as-
the corium is called as- c) Fowl b) Arteriosclerosis a) Crenation
a) Phagocytosis d) Horse c) Arteritis b) Pessary cells
b) Mosaicism 13. Which disease is also called as ‗Avian d) None of the above c) Cabot rings
c) Tatooing Reticulitis‘- 21. Local dilatation of artery is called as- d) Basophilic stippling
187 188
Dhruv N Desai Dhruv N Desai
29. Abnormal notching of the erythrocytes is d) Sweet clover toxicity b) Spirochetosis 52. The type of pneumonia in which alveolar
called as- 37. Which of the following pair is correctly c) Both of the above septa are affected is called as-
a) Crenation matched- d) None of the above a) Bronchopneumonia
b) Pessary cells a) Excess onion- Haemorragic anemia 45. Excessive phagocytic activity of the spleen b) Interstitial pneumonia
c) Cabot rings b) Aplastic anemia- Ionising radiation is called as- c) Verminous pneumonia
d) Basophilic stippling c) Deficiency of folic acid- Microcytic a) Hypersplenism d) Mycotic pneumonia
30. Refractile inclusions found in RBC‘s of anemia b) Splenomegaly 53. Psittacosis organism in birds causes-
horses that undergo phenothiazine therapy d) Defecieny of iron- Macrocytic anemia c) Any term can be used a) Bronchopneumonia
are called as- 38. Active toxic principle in onion which d) None of the above b) Metastatic suppurative pneumonia
a) Howell-Jolly bodies causes anemia is- 46. Chronic suppurative inflammation of c) Pulmonary adenomatosis
b) Annulocytes a) n-propyl disulphide young pigs caused by d) Interstitial pneumonia
c) Drepanocytes b) Ricin Spherophorusnecrophorusis called as- 54. Asteroid body in lungs is seen in-
d) Heinz-bodies c) Both of the above a) Rhinitis a) Bronchopneumonia
31. Thin erythrocytes with larger surface d) None of the above b) Nasal granuloma b) Pulmonary adenomatosis
without increase in volume is called as- 39. In hemorrhagic anemia the type of anemia c) Rhinohyperplasia c) Verminous pneumonia
a) Drepanocytes will be- d) Atrophic rhinitis d) Mycotic pneumonia
b) Leptocytes a) Macrocytic normochromic 47. Infectious sinusitis caused by Mycoplasma 55. The condition in which exudate is absent
c) Annulocytes b) Microcytic hypochromic sp. is most commonly seen in- in alveoli is seen in-
d) Microcytes c) Normocytic normochromic a) Chicken a) Bronchopneumonia
32. Crescent shaped RBC‘s characteristic of d) Microcytic normochromic b) Dog b) Pulmonary adenomatosis
sickle cell anemia are called as- 40. Hemophilia is found in both sexes and not c) Turkey c) Verminous pneumonia
a) Leptocytes sex linked in d) Sheep d) Mycotic pneumonia
b) Annulocytes a) Fowl 48. Which condition is termed as ‗Broken- 56. Which of the following statement is
c) Drepanocytes b) Swine wind‘ or ‗Heaves‘ in horses- incorrect-
d) Macrocytes c) Horse a) Chronic alveolar emphysema a) Inclusion bodies are present in Maedi
33. Ovalocytes(elliptical RBC‘s) are present d) Cattle b) Acute alveolar emphysema affected lung
in- 41. Ruptured immature neutrophils are called c) Chronic bronchitis b) Absence of rhinitis is seen in
a) Fowl as- d) Pulmonary hemorrhage pulmonary adenomatosis
b) Camel a) Pessary cells 49. Which of the following statement is true- c) Lymph nodes are affected in Jaagsiekte
c) Cattle b) Dust cells a) Pneumonitis specifically means d) The course of disease in Maedi is
d) Dog c) Basket cells inflammation of alveolar wall longer.
34. Anemia which is due to failure of d) Dohle‘s bodies b) Pneumonia specifically means 57. Inflammation of pleura is called as-
erythrocyte maturation factor is called as- 42. Which of the following doesn‘t have inflammation of alveolar lumen a) Pleuritis
a) Dyshemopoietic anemia lymph nodes- c) In animals lobular pneumonia is b) Pleurisy
b) Macrocytic hypochromic anemia a) Fowl common c) Both terms are correct
c) Aplastic anemia b) Camel d) All of the above d) None of the above
d) Hemolytic anemia c) Swine 50. Pneumonia in rarely met in- 58. Glossitis is inflammation of-
35. Anemia which is due to replacement of d) All of the above a) Swine a) Gums
bone marrow by other tissues is called as 43. Granulomatous lymphadenitis with b) Cattle b) Palate
a) Dyshemopoietic anemia caseation and calcification occurs in all of c) Cat c) Lips
b) Macrocytic hypochromic anemia the following EXCEPT- d) Dog d) Tongue
c) Aplastic anemia a) Tuberculosis 51. Verminous pneumonia in horse is caused 59. Inflammation of palate is called as-
d) Myelophthisic anemia b) Glanders by- a) Cheilitis
36. Acute hemorrhagic anemia is caused by- c) Johne‘s disease a) Dictyocaulusfilaria b) Ranula
a) Anaplasmosis d) Strangles b) Dictyocaulus viviparous c) Lampas
b) Snake venom 44. Splenomegaly in fowl is seen in- c) Dictyocaulusarnifieldi d) Glossitis
c) Deficiency of folic acid a) ALC d) All of the above 60. Epulis is fibroblastic tumor of-
189 190
Dhruv N Desai Dhruv N Desai
a) Gums c) Colon c) Fibrosis 85. High specific gravity of urine will be
b) Lips d) All of the above d) Hypertrophy observed in-
c) Palate 69. Telescoping of a portion of intestine into 77. Glycosuria in sheep is sequlae of- a) Acute glomerulonephritis
d) Tongue another is called as- a) Enterotoxaemia b) Subacute glomerulonephritis
61. Pathologic processes are very rarely found a) Volvulus b) Braxy c) Chronic glomerulonephritis
in- b) Intussusception c) Anthrax d) Interstitial nephritis
a) Salivary glands c) Incarceration d) Fluke infestation 86. Which type of crystals are seen in acidic
b) Brain d) Torsion 78. Chronic venous congestion is urine-
c) Liver 70. Hernia of intestine is common in- characterized by- a) Carbonate
d) Kidney a) Pigs and horses a) Biliary cirrhosis b) Phosphate
62. Salivary calculi is very common in- b) Dogs and cats b) Cardiac cirrhosis c) Oxalate
a) Cattle c) Cattle and horses c) Portal cirrhosis d) All of the above
b) Horse d) Pigs and buffalo d) Multinodular cirrhosis 87. Presence of calculi in ducts of Bellini in
c) Dog 71. Enteritis term usually applied for 79. Appearance of fresh blood in stools is kidney is called as-
d) Swine inflammation of- called as- a) Piloconcretions
63. Dilatation of esophagus is termed as- a) Small intestine a) Melana b) Microconcretions
a) Ranula b) Large intestine b) Hematuria c) Urinary calculi
b) Epulis c) Both of the above c) Hemophilia d) Nephrolithiasis
c) Acheresia d) Stomach d) Haematochezia 88. Presence of calculi in pelvis of kidney is
d) Ectasia 72. Enteroliths are formed of- 80. Choleliths are more common seen in- called as-
64. Diphthericingluvitis is seen in- a) Triple phosphate a) Cattle a) Piloconcretions
a) Fowl pox b) Calcium carbonate b) Horse b) Microconcretions
b) Fowl cholera c) Calcium oxalate c) Pig c) Urinary calculi
c) Fowl typhoid d) None of the above d) Fowl d) Nephrolithiasis
d) Fowl pest 73. Enteroliths are more commonly seen in- 81. Degenerative renal lesions are known as- 89. Presence of cast in urine is called as-
65. Gastritis in which stomach mucosa thrown a) Small intestine in horses a) Nephritis a) Cylindruria
into polypoid folds seen in- b) Small intestine in pigs b) Nephrocalcinosis b) Calculi
a) Bacterial gastritis c) Large intestine in horses c) Nephrosis c) Cylindroids
b) Parasitic gastritis d) Large intestine in pigs d) None of the above d) Any of the above
c) Viral gastritis 74. Difference between lesions of coli- 82. Inflammation of all parts of kidney 90. Gliosis refers to-
d) Fungal gastritis granuloma and tuberculosis is- involving the pelvis and parenchyma is a) Increase in astrocytes
66. Twisting of intestine on its own axis is a) Coli granuloma lesions are not single known as- b) Increase in microglial cells
called as- b) Coli granuloma lesions are found in a) Embolic nephritis c) Increase in oligodendrocytes
a) Volvulus spleen and bones b) Pyelonephritis d) All of the above
b) Intussusception c) Tuberculosis lesions are found in c) Interstitial nephritis 91. Microglia after engulfing become foamy
c) Incarceration spleen and bones d) Glomerulonephritis containing lipids, are called as-
d) Torsion d) All of the above 83. White spotted kidney is main lesion seen a) Gitter cells
67. Twisting of bowel on itself when it passes 75. Which type of necrosis of liver is seen in in- b) Gemiocytes
through a tear in the mesentery, is called phosphorous poisoning- a) Focal interstitial nephritis c) Leptocytes
as- a) Periportal necrosis b) Diffuse interstitial nephritis d) Plasma cells
a) Volvulus b) Mid zonal necrosis c) Glomerulonephritis 92. Exencephalus means-
b) Intussusception c) Paracentral necrosis d) None of the above a) Protrusion of meninges
c) Incarceration d) Centrilobular necrosis 84. Large white kidney is seen in- b) Absence of cranial vault exposing the
d) Torsion 76. Which of the following is absent in a) Acute glomerulonephritis brain
68. In cattle torsion is common in- cirrhosis- b) Subacute glomerulonephritis c) An abnormal small brain
a) Abomasum a) Hyperplasia c) Chronic glomerulonephritis d) Absence of most of the brain
b) Caecum b) Degeneration d) Interstitial nephritis 93. Rachicele is hernia of-
191 192
Dhruv N Desai Dhruv N Desai
a) Spinal cord **************************************** 43. d 90. a
b) Brain ************************************** 44. c 91. a
c) Abomasum 45. a 92. b
d) Intestine ANSWERS 46. c 93. a
94. Abnormal accumulation of CSF in and 47. c 94. b
around the brain is called as- 1. c 48. a 95. a
a) Hydronephrosis 2. c 49. d 96. b
b) Hydrocephalus 3. d 50. c 97. b
c) Hydropericardium 4. c 51. c 98. b
d) Cerebellar hypoplasia 5. a 52. b 99. a
95. Calcification is more commonly seen in- 6. b 53. d 100. b
a) Meninges 7. a 54. d
b) Brain 8. c ****************************************
55. b
c) Spinal cord 9. b **************************
56. c
d) All of the above 10. c 57. c
96. Inflammation of duramater is called as- 11. b 58. d
a) Encephalomyelitis 12. c 59. c
b) Pachymeningitis 13. c 60. a
c) Leptomeningitis 14. a 61. a SET-3
d) Meningoencephalomyelitis 15. c 62. b
16. b 1. The degenerative changes of cells depends
97. In rabies negri bodies are seen in- 63. d
17. d on which factors
a) Hippocampus and cerebrum 64. a
18. c a) Kinds of cells
b) Hippocampus and cerebellum 65. b
19. c b) Quality of injurious agent
c) Medulla and hippocampus 66. d
20. b c) Quantity of injurious agent
d) Cerebrum and medulla 67. a
21. b d) All of the above
98. Which of the following statement is 68. b
22. c 2. Most common cause of cloudy swelling
incorrect- 69. b
23. d a) Hypoxia
a) Neuritis refers to inflammation of 70. a
24. a b) Alteration in the physical state of the
peripheral nerves 71. a
25. a protein
b) Wallerian degeneration occurs in 72. a
26. d c) Toxins
muscle fibers 73. c
27. c d) None of the above
c) Wallerian degeneration occurs in nerve 74. c
28. c 3. ―Ground glass‖ appearance of the
fibers 75. a
29. a cytoplasm of cell seen in
d) Inflammation of uterus is termed as 76. d
30. d a) Parenchymatous degeneration
metritis 77. a
31. b b) Cloudy swelling
99. Presence of blood in tunica vaginalis is 78. b
32. c c) Albuminous degeneration
called as- 79. d
33. b d) All of the above
a) Hematocele 80. a
34. a 4. Vesicles form in Pox diseases is example
b) Hydrocele 81. c
35. d of
c) Rachicele 82. b
36. d a) Cloudy swelling
d) Pachicele 83. a
37. b b) Hydropic degeneration
100. Brucellosis in rams causes- 84. b
38. a c) Hyline degeneration
a) Orchitis 85. a
39. c d) Amyloid infiltration
b) Epididymitis 86. c
40. b 5. Which ion is present in extra cellular fluid
c) Keratitis 87. b
41. c in high concentration?
d) None of the above 88. d
42. a a) Potassium
89. a
193 194
Dhruv N Desai Dhruv N Desai
b) Bicarbonate 14. Which one of the following is made up of a) Urea b) Only cellular details are maintained.
c) Sodium branching fibres? b) Uric acid c) Only architectural details are
d) None of the above a) Galactose c) Ammonia maintained.
6. In which diseases, the hydropic b) Maltose d) All of the above d) Cellular details and architectural
degeneration occurs most commonly? c) Amyloid 22. Which is glucogenic fatty acid? details both are lost.
a) Pox disease d) Amylopectin a) Acetic aced 29. Tuberculosis is the example of which
b) Foot and Mouth disease 15. Focal accumulation of amyloid in spleen b) Butyric acid necrosis?
c) Carbon tetrachloride poisoning called c) Propionic acid a) Coagulative necrosis
d) All of the above a) Bacon spleen d) None of the above b) Caseative necrosis
7. Which is/ are the common example/s of b) Sago spleen 23. Local death of cells or tissue in a living c) Fat necrosis
hyaline degeneration? c) Both body? d) Liquefactive necrosis
a) Zenker‘s degeneration of muscle d) None of the above a) Necrosis 30. Which is the good example for
b) Equine azoturia 16. Diffuse accumulation of amyloid in spleen b) Gangrene liquefactive necrosis?
c) White muscle disease in calves called c) Somatic death a) White muscle disease
d) All of the above a) Bacon spleen d) None of the above b) Tuberculosis
8. How muscle looks in hyaline b) Sago spleen c) Abscess
degeneration? c) Both 24. The death of the body as a whole is called d) Adipose tissue trauma
a) Fish-flesh resemble d) None of the above a) Physiological death
b) Fibrillar appearance b) Somatic death 31. Which organ is most commonly affected
c) Cooked meat appearance c) Both to liquefactive necrosis?
d) All of the above 17. Special staining technique is used for d) None of the above a) Kidney
9. Hyperkeratosis seen in deficiency of amyloid is 25. Nucleus becomes smaller, rounded and b) Intestine
a) Vitamin K a) Oil red O condensed is called c) Lung
b) Vitamin C b) Sudan I a) Karyolysis d) Brain
c) Vitamin A c) Congo red b) Karyorrhexis 32. Death of tissues with putrefaction by
d) None of the above d) All of the above c) Pyknosis saprophytic bacteria is called
10. Hyperkeratosis seen in which following 18. Isolated deposition of amyloid material in d) Chromatolysis a) Autolysis
hyaline degeneration? a neoplastic form in the heart is called 26. Here names of fatty acids are given. b) Gangrene
a) Connective tissue hyaline degeneration a) Sago spleen I) Acetic acid c) Autopsy
b) Epithelial hyaoine degeneration b) Amyloidoma II) Butyric acid d) Necrosis
c) Muscle hyaline degeneration c) Bacon spleen III) Propionic acid 33. Which is the most common site for dry
d) All of the above d) None of the above Which are ketogenic fatty acids in gangrene?
11. Chemically, what is mucin or mucous? 19. End product/s which excreted by human in ruminant? a) Internal organs
a) Lipid urine? a) I and II b) Muscles
b) Glycogen a) Urea b) II and III c) Extremities
c) Protein only b) Uric acid c) I and III d) All of the above
d) Glycoprotein c) Ammonia d) I, II and III 34. Which compound gives greenish or
12. Which organelle is produced mucin? d) All of the above 27. Which cells are more resistant to blackish colour to the gangrenous area?
a) Mitochondria 20. End product/s which excreted by bird in ischemia? a) Hydrogen peroxide
b) Golgi apparatus urine? a) Tubular epithelial cells b) Hydrogen sulphide
c) Endoplasmic reticulum a) Urea b) Connective tissue cells c) hydrogen chloride
d) Lysosome b) Uric acid c) Brain cells d) sulfur oxide
13. Waxy degeneration is also known as c) Ammonia d) All of the above 35. Which type of gangrene is most dangerous
a) Cloudy swelling d) All of the above 28. Which statement is correct for coagulative among all type of gangrene?
b) Hydropic degeneration 21. End product/s which excreted by fish in necrosis? a) Moist gangrene
c) Amyloid infiltration urine? a) Cellular details and architectural b) Dry gangrene
d) All of the above details both are maintained. c) Gas gangrene
195 196
Dhruv N Desai Dhruv N Desai
d) None of the above a) Pavementing 52. The crystalline proteins derived from the d) Plasma cell
36. Which compounds give foul smelling b) Emigration nucleus of eosinophils seen in the sputum 60. Which cell is knows as microphage
affected by moist gangrene? c) Extravasation of man is called as a) Monocyte
a) Indol d) Redistribution a) Reed Stenberg cell b) Neutrophil
b) Skatol 45. Escape of RBC from intact wall of the b) Touton giant cell c) Basophil
c) Hydrogen sulphide blood vessels is called as c) Charcot leydon cell d) Macrophage
d) Both (a) and (b) a) Diapedesis d) None of the above 61. Suppurative inflammation of hair follicle
37. Which is the example of gas gangrene? b) Rhexis 53. In tissue mast cell is derived from or sebaceous gland in the skin is called
a) Black quarter c) Echymotic a) Monocyte a) Furuncle
b) Black disease d) Petechie b) Neutrophil b) Pustule
c) Malignant edema 46. First line of cellular defense in c) Lymphocyte c) Papule
d) All of the above inflammation is d) Basophil d) Dermatitis
38. Which organism is cause gas gangrene? a) Monocyte 62. Eosinophillia is seen in
a) Aerobic b) Macrophage a) Allergic condition
b) Anaerobic c) Lymphocyte b) Parasitic condition
c) Capnophilic d) Neutrophil 54. Perivascular cuffing is seen in poisoning c) Hypersensitivity reaction
d) All of the above 47. An increase in the number of leucocytes in of d) All of the above
39. Algor mortis means the blood is a) Ammonia 63. The granulation tissue continues getting
a) Cooling of the body a) Leukocytopenia b) Salt produced and present in abnormally large
b) Stiffening of the body b) Leukophillia c) Cyanide amounts called as
c) Softening of the body c) Leukocytosis d) Urea a) Proud flesh
d) None of the above d) Lymphocytosis 55. Plasma cell is derived from b) Keloid
40. Black quarter is characterized by 48. The granulopoietin control the production a) B-lymphocyte c) Scar
a) Crackling noise of neutrophils called as b) T-lymphocyte d) All of the above
b) Metallic seen of affected muscle a) Leukopoetin c) Basophil 64. Mass of proliferating connective tissue is
c) Necrotizing myositis b) Leukocytosis inducing factor d) Monocyte called as
d) All of the above c) Colony stimulating factor 56. Which lymphocyte is responsible for cell a) Proud flesh
41. In postmortem clotting of blood, WBC clot d) All of the above mediated immunological reaction? b) Keloid
is called as 49. Release of neutrophils from the bone a) B-lymphocyte c) Scar
a) Current jelly marrow is promoted by a factor in the b) T-lymphocyte d) All of the above
b) Chicken fat plasma is called as c) Both of above 65. The organs fail to develop to their full
c) Chicken jelly a) Leukopoetin d) None of the above normal size though there was a beginning
d) All of the above b) Leukocytosis inducing factor 57. Plasma cell found in is called as
42. Who discovered the fifth cardinal sign of c) Colony stimulating factor a) Blood a) Aplasia
inflammation? d) All of the above b) Urine b) Hypoplasia
a) Celsus 50. What is the normal value of neutrophil in c) Saliva c) Atrophy
b) Galen dog in percentage? d) Tissue d) None of the above
c) Rudolf wirchow a) 55-70 58. Which is the second line of defense in 66. The decrease in the size of a tissue after it
d) None of the above b) 30-45 inflammation? has attained its full growth is called
43. The adherence of the leucocytes to the c) 20-40 a) Histiocyte a) Aplasia
wall is called d) 70-85 b) Macrophage b) Hypoplasia
a) Pavementing 51. Normal range of neutrophil in cattle in c) Resting wandering cell c) Atrophy
b) Extravasation percentage? d) All of the above d) None of the above
c) Emigration a) 50-75 59. Macrophage is derived from 67. The transformation of one type of tissue
d) All of the above b) 25-45 a) Monocyte into another is called as
44. The process of leucocytes moving outside c) 15-25 b) Lymphocyte a) Anaplasia
the vessels is called as d) 70-85 c) Neutrophil b) Metaplasia
197 198
Dhruv N Desai Dhruv N Desai
c) Dysplasia a) Brown induration b) Rhexis
d) Hyperplasia b) Pneumoconiosis c) Petechiae
68. Which vitamin deficiency leads c) Consolidation d) Diapedesis 92. Grass tetany is caused by deficiency of
conversion of simple columnar to d) None of the above 85. The mechanism by which foreign material a) Magnesium
squamous stratified cell? 76. Nut meg liver is seen in is transported through the circulatory b) Manganese
a) Vitamin A a) Rupture of aorta system is called as c) Sulphur
b) Vitamin C b) Passive venous congestion a) Embolism d) Iron
c) Vitamin E c) Right side heart failure b) Thrombus 93. Steely wool is caused by
d) All of the above d) Dilatation of coronary artery c) Oedema a) Deficiency of copper
77. Bleeding from nose is known as d) Shock b) Deficiency of molybdenum
a) Epistaxis 86. Which type of necrosis occurs in c) Toxicity of molybdenum
69. The enlargement of tissue due to increased b) Hemoptysis infarction? d) Both (a) and (c)
size of the individual cell is called as c) Melena a) Liquefactive necrosis 94. Parakeratosis is caused by
a) Hypertrophy d) Apoplexy b) Caseative necrosis a) Excessive eating of calcium and
b) Hyperplasia 78. Blood in vomit is c) Fat necrosis phosphorus
c) Neoplasm a) Hemoptysis d) Coagulative necrosis b) Deficiency of fatty acid
d) Hypoplasia b) Hematesis 87. Generalized subcutaneous edema is knows c) Deficiency of zinc
70. The enlargement of tissue due to increased c) Melena as d) All of the above
number of the individual cell is called as d) Apoplexy a) Dropsy 95. Find out which is not correctly match.
a) Hypertrophy 79. Blood in sputum is b) Hydrops a) Calcium – Vitamin D
b) Hyperplasia a) Epistaxis c) Anasarca b) Phosphorus – Parathormon
c) Neoplasm b) Hemoptysis d) Ascites c) Calcium – Calcitonin
d) Hypoplasia c) Melena 88. Hydrocele means d) Selenium – Vitamin E
71. The dilatation of alveoli is called as d) Apoplexy a) Oedema of brain 96. Where is vitamin A stored?
a) Emphysema 80. Blood in stool is b) Oedema of cerebellum a) Eye
b) Atelectasis a) Epistaxis c) Oedema of tunica vaginalis b) Liver
c) Pneumonia b) Hemoptysis d) Oedema of oviduct c) Intestine
d) Pneumoconiosis c) Melena 89. Which shock is occurring in excessive d) Adipose tissue
72. The coalence of alveoli is called as d) Apoplexy anaesthesia? 97. Which vitamin knows as anti-infection
a) Emphysema 81. Hemorrhage into the brain is called as a) Hypovolumic shock vitamin?
b) Atelectasis a) Epistaxis b) Neurogenic shock a) Vitamin A
c) Pneumonia b) Hemoptysis c) Septic shock b) Vitamin D
d) Pneumoconiosis c) Melena d) All of the above c) Vitamin C
73. Iron particle can be stained by d) Apoplexy 90. Which condition occurs due to deficiency d) Vitamin K
a) Masons trichrome method 82. Small pin point hemorrhage is known as of calcium? 98. Which vitamin knows as Vitamin A
b) Van kossa method a) Petechiae a) Parturient paresis sparer?
c) Perl‘s Prussian blue method b) Ecchymoses b) Pica a) Vitamin A
d) None of the above c) Rhexis c) Allotriophagy b) Vitamin E
74. Hemosiderin ladein macrophage is called d) Diapedesis d) Enzootic marasmus c) Vitamin D
as 83. Extensive hemorrhages on the surface 91. Which disease is caused by phosphorus d) Vitamin C
a) Heart failure cell a) Rhexis deficiency? 99. Which diseases occur by Vitamin E
b) Dust cell b) Petechiae a) Big head disease deficiency?
c) Mast cell c) Echymoses b) Miller‘s disease a) Crazy-chick disease
d) All of the above d) Diapedesis c) Bran disease b) Exudative diathesis
75. The pigment gives a brownish colour 84. Extensive hemorrhages in the tissue is d) All of the above c) Nutritional muscular dystrophy
caused by chronic venous congestion is called as d) All of the above
called as a) Extravasation
199 200
Dhruv N Desai Dhruv N Desai
100. Which disease is caused by niacin 30. c 77. a 94. d
deficiency? 31. d 78. b 95. b
a) Canine pellagra 32. b 79. b 96. b
b) Encephalomalacia 33. c 80. c 97. a
c) Allotriphagy 34. b 81. d 98. b
d) Enzootic marasmus 35. a 82. a 99. d
101. Vitamin K is required for the formation of 36. d 83. c 100. a
a) Prothrombin 37. d 84. a 101. d
b) Factor IX 38. b 85. a
c) Factor X 39. a 86. d ****************************************
d) All of the above 40. d 87. c **************************************
41. b 88. c
****************************************
42. c 89. b
**************************************
43. a 90. a
44. b 91. d
SET-4
45. a 92. a
ANSWERS 46. d 93. d
1. d 47. c (D) Sqamous cell carcinoma.
2. a 48. c 1. Bronze discolouration of Liver in poultry 6. Haemoglobinuria is seen in-
3. d 49. b is caused by- (A) Theileriosis
4. b 50. a (A) Pasteurellosis (B) Leptospirosis
5. c 51. b (B) IBD Virus (C) Salmonellosis
6. d 52. c (C) Leptospirosis (D) Pasturellosis.
7. d 53. d (D) Salmonellosis 7. Who is the father of Cellular Pathology?
8. a 54. b 2. Fragmentation of nucleus in a cell is (A) John Hunter
9. c 55. a termed as- (B) Robert Koch
10. b 56. b (A) Pyknosis (C) Rudolph Virchow
11. d 57. d (B) Karyorrhexis (D) K.Cohnhiem.
12. a 58. d (C) Karyolysis 8. Pseudo Rabies is caused by-
13. c 59. a (D) Chromatolysis (A) Lyssa virus
14. d 60. b 3. Infectious Necrotic Hepatitis in sheep is (B) Picorna virus
15. b 61. a caused by- (C) Paramyxo virus
16. a 62. d (A) Leptospira sp. (D) Herpes virus.
17. c 63. a (B) Fasciola hepatica 9. Pulpy Kidney Disease is caused by-
18. b 64. b (C) Clostridium sp. (A) Clostridium perfringens
19. a 65. b (D)Heterakisgallinarum (B) Clostridium septicum
20. b 66. c 4. The animal susceptible to Atherosclerosis (C) Clostridium novyi
21. c 67. b is- (D) Clostridium tetani
22. c 68. a (A) Cattle 10. Turkey Egg Kidney is seen in-
23. a 69. a (B) Swine (A) Swine Pox
24. b 70. b (C) Rabbit (B) Swine Influenza
25. c 71. a (D) Poultry. (C) Swine Fever
26. a 72. b 5. Tumour which Metastasize in different (D) Swine Erysipelas.
27. b 73. c organ is- 11. Poll evil in Horse is caused by-
28. c 74. a (A) Lipoma (A) Clostridium tetani
29. b 75. a (B) Angioma (B) Actinomycesbovis
76. b (C) Seminoma (C) Brucellaabortus
201 202
Dhruv N Desai Dhruv N Desai
(D) Both (B) and (C). (B) Inhalation 29. Diamond skin disease is primarily a (A) Pox diseases
12. Nutritional roup in Poultry is caused due (C) Both disease of- (B) Herpes virus infection
to deficiency of- (D) None of the above. (A) Horse (C) Adeno virus infection
(A) Vitamin B 21. Section size in tissue microtome is- (B) Lion (D) Lyssa virus infection
(B) Vitamin C (A) 25-30 micron (C) Sow 38. Negri bodies are seen in Rabies which
(C) Vitamin E (B) 4-6 micron (D) Turkey are-
(D) Vitamin A. (C) 15-20 micron 30. In Johne‘s disease, corrugation is not the (A) Intranuclear
13. Epithelial Pearls are seen in- (D) 8-10 micron feature in- (B) Intracytoplasmic
(A) Sebaceous cell Adenoma 22. Collagen fiber gives which colour in (A) Cattle (C) Both
(B) Squamous cell carcinoma Masson‘s tricome stain- (B) Sheep (D) May be intranuclear or intracytoplasmic
(C) Melanoma (A) Black (C) Horse 39. Enlargement of Bursa of fabricius in
(D) Venereal granuloma. (B) Red (D) Dog poultry is seen in-
14. Blue Tongue in sheep is caused by- (C) Blue 31. Most susceptible species for Hemorrhagic (A) CRD
(A) Herpes virus (D) Green septicemia- (B) IB
(B) Birna virus 23. Maedi is primarily a disease of- (A) Sheep (C) RD
(C) Picorna virus (A) Sheep affecting respiratory system (B) Buffalo (D) IBD
(D) Orbi virus. (B) Cattle affecting reproductive system (C) Cattle 40. Zebra marking is predominant feature of-
15. Most common Serotype of FMD virus in (C) Sheep affecting nervous system (D) Pig (A) Johne‘s disease
India is- (D) Cattle affecting nervous system. 32. Erythritol sugar plays important role in (B) Tuberculosis
(A) A 24. Equine Plague is also called as- pathogenesis of- (C) Rinder pest
(B) C (A) Equine viral arteritis (A) Clostridium spp. (D) Both (A) and (C)
(C) Asia-1 (B) Glanders (B) Brucella spp. 41. CBPP differs from CCPP in-
(D) O. (C) Strangles (C) Bacillus spp. (A) Both occur in same species
16. In which disease post mortem of carcass (D) African Horse sickness. (D) Corynebacterium spp. (B) Sequestra formation
is prohibited? 25. Sore mouth in cattle is seen in- 33. Substance responsible for increase (C) Mode of transmission
(A) Haemorragic septicemia (A) Blue tongue penetration of Lyssa virus- (D) Pathogenesis
(B) Rinder pest (B) Bovine malignant catarrh (A) Hyaluronidase 42. Which bacterium is predisposed by
(C) Anthrax (C) Rinder pest (B) Erythriol Fasiola hepaticainfestation-
(D) Brucellosis. (D) Vesicular Stomatitis (C) Protagen (A) Bacillus spp.
17. Mad itch is mostly a disease of- 26. Disease caused by Clostridium septicum (D) Amylase (B) Clostridium spp.
(A) Caprine is- 34. Crop mycosis in poultry is caused by- (C) Leptospira spp.
(B) Bovine (A) Black Quarter (A) Bacteria (D) Pasturella spp.
(C) Swine (B) Enterotoxaemia (B) Mycoplasma 43. Which is the most potent aflatoxin-
(D) Ovine (C) Braxy (C) Fungi (A) M1
18. Tigroid Heart is seen in cattle affected (D) Tetanus (D) Virus (B) M2
with- 27. Sulphur granules in yellowish pus is seen 35. Brooder‘s pneumonia in poultry is caused (C) B1
(A) Bovine malignant catarrhal in- by- (D) B2
(B) Botulism (A) Glanders (A) Candida albicans 44. Curled toe paralysis in chicken is due to
(C) Bovine viral diarrhea (B) Strangles (B) Aspergillusfumigatus deficiency of-
(D) Foot and mouth disease. (C) Staphylococcosis (C) Haemophilusparagallinarum (A) Vitamin B12
19. Wooden Tongue in cattle is seen in- (D) Actinomycosis (D) Pasturellamultocida (B) Vitamin B1
(A) Actinomycosis 28. Toxins of organism causes peripheral 36. Circling disease in cattle is caused by- (C) Niacin
(B) Botriomycosis nerve paralysis in cattle- (A) Listeria monocytogenes (D) Vitamin B2
(C) Haemorrhagic Septicemia (A) Botulism (B) Erysipelothrixrhusiopathiae 45. Mn deficiency is chicken will lead to-
(D) Actinobacillosis. (B) Tetanus (C) Streptococcus equi (A) Pica
20. Mode of transmission of IBR virus is- (C) Both (D) Chlamydia psittacii (B) Star grazing condition
(A) Venereal (D) None of the above 37. Intranuclear inclusion bodies are seen in- (C) Crazy chick disease
203 204
Dhruv N Desai Dhruv N Desai
(D) Slipped Tendon 53. Liquifactive necrosis is most commonly (D) Spermatic cord (A) Haemopericardium
46. Phosphorous deficiency in soil will seen in- 60. Which of the following is correct (B) Hydropericardium
predispose the cattle to- (A) Kidney regarding poultry diseases- (C) Myocarditis
(A) Hemorrhagic septicemia (B) Liver (A) In pullorum disease, green constant (D) Pneumopericardium
(B) Botulism (C) Heart diarrhea is seen 68. In Left side heart failure, the heart failure
(C) Anthrax (D) Brain (B) Face is swollen and edematous in cells are seen in-
(D) Mucosal disease 54. Which of the following is correctly Haemophilus infection (A) Lungs
47. Black head disease is predominately a matched- (C) Bloody mucous expelled from trachea in (B) Heart
disease of- (A) Picorna virus-Ranikhet disease Infectious Bronchitis (C) Kidney
(A) Cattle caused by Parasite (B) Lumpy skin disease- Pox virus (D) In pullorum disease, nervous signs are (D) Spleen
(B) Poultry caused by Parasite (C) Diamond skin disease-Herpes virus seen along with diarrhoea 69. Sway back condition is seen due to
(C) Horse caused by Virus (D) Paramyxo virus-FMD 61. In which outbreak at poultry farm deficiency of-
(D) Pig caused by Virus 55. Enlargement of Sciatic nerve is seen in- maximum mortality of birds will be (A) Cu
48. Which of the following is correctly (A) Ranikhet disease expected- (B) Co
matched- (B) Marek‘s disease (A) Ranikhet disease (C) Mn
(A) Tubercular lesions are calcified-Buffalo (C) Chronic respiratory disease (B) Infectious Bronchitis (D) Se
(B) Johne‘s disease-Foul smelling diarrhea (D) Infectious Coryza (C) Infectious Lanyngiotracheitis 70. Most pathogenic species/disease affecting
(C) Avian spp.- Dry pus (D) Avain Encephalomyelitis Snakes-
(D) Lamb dysentery- Clostridium 62. Dohle‘s bodies are toxic granules of- (A) Pasteurellosis
perfringenstype D (A) Macrophages (B) Histomoniasis
49. Post mortem of chick shows foul smelling (B) Eosinophils (C) Salmonellosis
yellow-brown watery Yolk, 56. Apennosis is- (C) Neutrophils (D) Listeriosis.
fibrinous (A) Intracellular edema of epidermis (D) Lymphocytes 71. Increase in number of immature lymphoid
perihepatitis and pericarditis, suspect the (B) Congenital lack of feathers in fowl 63. East coast fever is caused by- cells in blood is known as-
disease (C) Absence of pineal gland (A) Theleriaparva (A)Shift to left
(A)Infectious Coryza (D) Lack of cell differentiation during (B) Theleriaannulata (B) Shift to right
(B) Fowl Typhoid embryogenesis (C) Babesiabovis (C) Leukemia
(C) Coli Bacillosis 57. Which is incorrect about avian 64. Edema consisting of gelatinous material (D) Leukocytosis
(D) Infectious Bronchitis tuberculosis-? in neck and brisket region seen in 72. Oval and nucleated RBC‘s are present in-
50. Haemorrhages at the tip of the (A) Calcification absent cattle in- (A) Parrot
proventricular gland is the (B) Liver and bones are most commonly (A) Black Quarter (B) Cobra
pathognomic lesion seen in- affected (B) Deganala disease (C) Camel
(A)Ranikhet disease (C) Lungs are most commonly affected (C) Botulism (D) Both (A) and (B)
(B) Chronic respiratory disease (D) Intradermal test is performed on wattle. (D) HaemorrhagicSepticemia 73. Spleen with diffuse Amyloidosis is
(C) Infectious bursal disease 58. Post mortem of cattle reveals too much 65. Which is the main chemical mediator of known as-
(D) Avian influenza emaciated carcass,mucosa of inflammation-? (A) Sago spleen
51. Which of the following is not correctly intestine thrown (A) Serotonin (B) Bacon spleen
matched-? into corrugated folds, most probable cause (B) Bradykinin (C) Ham spleen
(A) Zn deficiency-Pig will be- (C) Histamin (D) Pulpy spleen
(B) Epidemic tremor-Virus (A) Rinder pest (D) Interleukin-1 74. Which of the following statement is
(C) Siderosis-Silica dust (B) Johne‘s disease 66. Big liver disease is also known as- incorrect-
(D) Alkali disease-Se (C) Tuberculosis (A) IBD (A) FMD doesn‘t occur in Elephant
52. Pachymeningitis is inflammation of- (D) Pasterellosis. (B) ALC (B) Star grazing in chicken is due to vitamin
(A) Piamater 59. Spondylitis is inflammation of- (C) Fowl cholera B1 deficiency
(B) Brain (A) Prepuce (D) Fowl typhoid (C) Actinomycosis mostly affects hard tissues
(C) Duramater (B) Vertebrae 67. In Angara disease, the pathological in animals
(D) Spinal cord (C) Bone finding is- (D) In Anthrax rigor mortis is absent.
205 206
Dhruv N Desai Dhruv N Desai
75. Which of the following pair is (B) Liquifective necrosis (A) Dysplasia (B) Salmonellosis
incorrectlymatched- (C) Caseative necrosis (B) Metaplasia (C) Pasteurellosis
(A) Haemoptysis- Blood in Vomit (D) Fat necrosis (C) Hyperplasia (D) Listeriosis
(B) Pyelonephritis- Suppuration in kidney 84..First change after death is- (D) Aplasia 100. Big head disease is caused by-
(C) White muscle disease- Vitamin E (A) Algor mortis 92. Capillary rupture and hemorrhage occurs (A) Heterakisgallinarum
(D) Glanders- Mallein test (B) Rigor mortis due to deficiency of: (B) Salmonella gallinarum
76. Which vitamin is act as antioxidant- (C) Formation of bloat (A) Vitamin B (C) Haemophilusparagallinarum
(A) Vit.B (D) Both (B) & (C) (B) Vitamin C (D) Excess of Phosporous feeding
(B) Vit. D 85..Inflammation of crop is- (C) Thyroxin
(C) Vit. E (A) Blephritis (D) Vitamin A
(D) Vit. A (B) Ingluvitis 93. Healing by first intention occurs in:
77. Localized loss of melanin is known as- (C) Typhlitis (A) Closed wound
(A) Vitiligo (D) Gonitis (B) Open wound
(B) Leucoderma 86.Cart wheel shape nucleus found in- (C) None of the above
(C) Acanthosisnigricans (A) Plasma cell (D) All of above
(D) All of above (B) Basophils 94. The mass of proliferating connective
78. Macrophage in spleen are known as- (C) Eosinophils tissue under scar is known as:
(A) Septal cell (D) Monocyte (A) Keloid
(B) Kuffer cell (B) Proud flesh
(C) Alveolar cell (C) Cyst ANSWERS
(D) All of above (D) Scar
79. Macrophage laden with haemosiderin 95. Pulmonary adenomatosis is characterized 1. D
pigment is- 87. Extreme elevation of leucocytes in by: 2. B
(A) Kuffer cell peripheral blood is known as- (A) Hyperplasia of epithelium 3. C
(B) Foam cell (A) Shift to left (B) Hypertrophy of epithelium 4. B
(C) Heart failure cell (B) Leukamoid reaction (C) Both of the above 5. D
(D) None of above (C) Right shift (D) None of above 6. B
(D) Both (A) & (C) 96. Nucleus becoming smaller and condensed 7. C
80. Toxic jaundis is also known as- 88.Blood in vomitus is known as- is called as- 8. D
(A) Post hepatic jaundice (A) Haematamiasis (A) Necrosis 9. A
(B) Hepatic jaundice (B) Haemoptysis (B) Pyknosis 10. C
(C) Pre hepatjc jaundice (C) Epistaxis (C) Chromatolysis 11. D
(D) Obstructive jaundice (D) Melena (D) Keratolysis 12. D
81. Siderosis means- 89. Bleeding from the oviduct is designated 97. Closure of lumen of hollow organ is 13. B
(A) Deposition of calcium in lung as: called: 14. D
(B) Deposition of iron in lung (A) Epitaxis (A) Fissure 15. D
(C) Deposition of silicon in lung (B) Hemosalpinx (B) Hypoplasia 16. C
(D) Deposition of silver particle in lung (C) Hematocele (C) Atresia 17. C
82. Van den Bergh test for obstructive (D) Hematemasis (D) Sinus 18. D
jaundice- 90. Condition which is hereditary and sex 98. Serum gives Direct Van Den Bergh test 19. D
(A) Direct linked in which clotting is delayed: in- 20. C
(B) Indirect (A) Apoplexy (A) Diabetes 21. B
(C) Biphasic (B) Hemophilia (B) Obstructive jaundice 22. C
(D) Both (B) & (C) (C) Brown induration (C) Hemorrhage 23. A
83. In abscess which type of necrosis is (D) Epistaxis (D) Toxic jaundice 24. D
seen? 91. Transformation of one type of cell into 99. Granulomatous inflammation is seen in- 25. D
(A) Coagulative necrosis another is known as: (A) Tuberculosis 26. C
207 208
Dhruv N Desai Dhruv N Desai
27. D 74. A a) Adeno virus c) Cellular swelling
28. B 75. A b) Retro virus d) Amyloid degeneration
29. C 76. C c) Herpes virus 12. Crop mycosis is caused by-
30. B 77. B d) Reo virus. a) Fungus
31. B 78. A 4. Epithelial pearls are seen in:- b) Bacteria
32. B 79. C a) Squamous cell carcinoma c) Virus
33. A 80. B b) Adenosarcoma d) Prion
34. C 81. B c) Adenoma 13. Haemmorage at tip of proventriculus is
35. B 82. A d) Cholangio-cellular carcinoma main lesion in-
36. A 83. B 5. Most common sites for metastatic a) Avain influenza
37. C 84. A spread of malignant tumors is:- b) IBD
38. B 85. B a) Liver and brain c) IB
39. D 86. A b) Lungs and brain d) Ranikhet disease
40. C 87. B c) Liver and kidney 14. Marek‟s disease is characterized by-
41. B 88. A d) Liver and lungs a) Enlagement of muscles
42. B 89. B 6. Which nephritis is due to antigen b) Enlargement of joints
43. C 90. B antibody reaction:- c) Enlargement of nerves
44. D 91. B a) Embolic nephritis d) Deformality in bones
45. D 92. B b) Glomerulonephritis 15. Equine sarcoid is caused by-
46. B 93. A c) Interstitial nephritis a) Paramyxovirus
47. B 94. A d) Pyelonephritis b) Reovirus
48. C 95. A 7. Twisting of the intestine on its c) Poxvirus
49. C 96. B mesenteric axis is called as:- d) Papillomavirus
50. A 97. C a) Volvulus 16. Coligranuloma is associated with-
51. C 98. B b) Torsion a) Salmonella spp.
52. C 99. A c) Intussuseption b) E. coli spp.
53. D 100. D d) Paralytic ileus c) Klebsiella spp.
54. B 8. Folliculitis is inflammation of:- d) Viral origin
55. B **************************************** a) Graffian follicle 17. Omarthritis is inflammation of-
56. B ************************************** b) Hair follicle a) Stifle joint
57. C c) Fat b) Tarsal joint
58. B d) Spinal cord c) Knee joint
59. B SET-5 9. Pathogenesis of which of the following d) Shoulder joint
60. B disease involves „protagon‟ substance:- 18. Which of the following is corrected
1. Heart failure cells are seen in:-
61. C a) Black quarter matched-
a) Left side heart failure
62. C b) Tetanus a) Anthracosis- liver
b) Right side heart failure
63. A c) Braxy b) Mad itch- Se deficiency
c) Both
64. D d) Enterotoxemia c) Free radical- unpaired electron in
d) None of the above.
65. C 10. Predominant cell in pus is:- inner orbit
2. Cholecystitis involves inflammatory
66. B a) Lymphocyte d) Hepatic coccidiosis- absent in
response in:-
67. B b) Monocyte poultry.
a) Bile duct only
68. A c) Neutrophil 19. In Zn deficiency parakeratosis is seen
b) Bile duct and gall bladder
69. A d) Eosinophil in-
c) Gall bladder only
70. C 11. Main pathological alteration in pox a) Swine
d) Bile duct, gall bladder and
71. C disease is- b) Birds
adjacent liver parenchyma.
72. D a) Hydropic degeneration c) Horse
3. Aujeszky‟s disease is caused by:-
73. B b) Hyaline degeneration d) Caprine
209 210
Dhruv N Desai Dhruv N Desai
20. Identify the incorrect match- a) ESR 38. Brooders pueumonia is caused by- b) Aplasia
a) Mn-perosis b) PCV a) Aspergillusnigar c) Metaplasia
b) Se- alkali disease c) DLC b) Aspergillusfumigatus d) Anaplasia
c) Elephant-FMD d) MCHC c) Candida albicans 47. Turkey egg kidney is found in-
d) Rhinoceros-FMD 30. Programmed cell death is called as- d) Brucellaabortus a) Pseudorabies
21. Blood in sputum is termed as- a) Necrosis 39. Bollinger bodies are feature of- b) Swine fever
a) Haemoptysis b) Autolysis a) Rabies c) Enterotoxemia
b) Haematuria c) Gangrene b) Pox d) Pasteurellosis
c) Epistaxis d) Apoptosis c) IBD 48. The large cells containing lipid material
d) Haemetemesis 31. Nodules of tuberculosis is example of- d) ICH and found in xanthomas are-
22. First line of defence in inflammation is- a) Liquifactive necrosis 40. Which of the following is not of a) Reed-stenberg cells
a) Lymphocyte b) Caeseative necrosis morbilivirus genus- b) Foreign body gaint cells
b) Monocyte c) Coagulative necrosis a) Measles c) Touton giant cells
c) Neutrophil d) Fat necrosis b) CD d) Tumour giant cells
d) Basophil 32. Which of the following organism is c) ICH 49. Blueeye in cattle is also known as-
23. Foul smelling diarrhea containing fat is detected by dark field microscopy of d) Rinder pest a) Morexellabovis
called as- urine- 41. Sway back condition is due to deficiency b) ICH
a) Ceratorrhea a) Leptospira spp. of- c) IBH
b) Steatorrhea b) Brucella spp. a) Se d) CD
c) Steatitis c) Corynebacterium spp. b) Cu 50. Blueeye in dog is also known as-
d) Keratitis d) Bacillus spp. c) Mg a) ICH
24. Necrotizing myositis is feature in- 33. Inflammation of crop is called as- d) Iodine b) CD
a) Black leg a) Cropitis 42. Omphalitis is caused by- c) Rubarth‟s disease
b) Black disease b) Ingluvitis a) Salmonella spp. d) Both a and c
c) Black Quarter c) Spondylitis b) E. coli spp. 51. In which of the following thrombi
d) Both a and c d) Lampas c) Campylobacter spp. partial blood flow is maintained-
25. Sequel of FMD is- 34. Lumpy skin disease is caused by- d) None. a) Lateral thrombi
a) Ulcer a) Poxvirus 43. Main feature of chronic inflammation b) Saddle thrombi
b) Hair loss b) Paramyxovirus is- c) Canalized thrombi
c) Panting c) Picornavirus a) Infiltration of gaint cells d) Propagating thrombi
d) Myositis d) Reovirus b) Infiltration of neutrophils 52. Lines of zahn is related with-
26. Which of the following is incorrect- 35. Blue tongue virus is transmitted by- c) Fibrosis a) Infraction
a) Bloody exudate- ILT a) Contact d) Both a and c b) Embolism
b) Soft pad-Canine distemper b) Fleas 44. Hypoxia causes- c) Thrombosis
c) Hyaluronidase-rabies c) Culex a) Dyspnoea d) Edema
d) Erythritol-Brucellosis d) Culicoides spp. b) Apnoea 53. Hydrosalpinx is edema of-
27. Sequel of canine distemper is- 36. Classical example of gas gangrene is- c) Polypnoea a) Oviduct
a) Paralysis a) Tetanus d) Hyperpnoea b) Ovary
b) Shivering b) Braxy 45. Vitamin C is synthesized in- c) Vagina
c) Chorea c) Enterotoxaemia a) Gall bladder d) Peritoneal cavity
d) Paraphymosis d) Black quarter b) Intestine 54. Wool sorter‟s disease in man is due to-
28. Haemoglobin is expressed in- 37. Intracytoplasmic and intranuclear c) Kidney a) Black quarter
a) % inclusions are present in- d) Skin b) Anthrax
b) gm % a) Rabies 46. The lack of differentiation and c) Avian influenza
c) Cubic mm b) Pox reversion to embroyonic type of cells is d) Schistosomiasis
d) millions/cumm c) Canine distemper known as- 55. Disease transmitted by vector is-
29. Haematocrit is also known as- d) ICH a) Hyperplasia a) Rabies
211 212
Dhruv N Desai Dhruv N Desai
b) Blue tongue a) More mucous than pus in 72. Most susceptible species for salt toxicity 80. Abnormal notching of erythrocytes is
c) FMD exudate is- called as-
d) Fowl typhoid b) More pus than mucous in a) Horses and swine a) Basophilia
56. Stain for amyloid in tissue is- exudate b) Fowls and cattle b) Crenation
a) Congo red c) Both are in equal quantity c) Fowls and swine c) Cabot rings
b) Best carmine d) Either a or b d) Sheep and swine d) Annulosytes
c) PAS 65. Which Vitamin is required in wound 73. Slipped tendon in fowl is due to 81. Increase in number of immature
d) Alcian blue healing- deficiency of- neutrophils is called as-
57. Fragmentation of nucleus is called as- a) K a) Mn a) Shift to left
a) Pyknosis b) C b) S b) Shift to right
b) Karyolysis c) E c) Co c) Neutrophilia
c) Karyorrhexis d) A d) Se d) None of the above.
d) Chromatolysis 66. Chronic irritation of a cell will lead to- 74. Curled toe paralysis in fowl is due to 82. Inflammation of lymph node is called
58. Necrosis in which architectural details a) Metaplasia deficiency of- as-
are preserved is- b) Hyperplasia a) Vitamin B1 a) Lyphangitis
a) Coagulative c) Hypoplacia b) Vitamin B2 b) Lymphadenitis
b) Caseative d) Anaplasia c) Vitamin A c) Both of the above
c) Liquefactive 67. Hypostatic congestion is more common d) Vitamin B12 d) Lymphocytosis
d) Fat necrosis in- 75. Which of the following is incorrect- 83. Dilatation of bronchus is termed as-
59. Cooling of body is called as- a) Liver a) Foul smelling diarrhea is absent a) Bronchitis
a) Cryo mortis b) Spleen in Johne‟s disease. b) Bronchostenosis
b) Algor mortis c) Lungs b) Fat is stained by Sudan IV c) Bronchiectasis
c) Rigor mortis d) Brain c) Deposition of iron dust in tissue d) Bronchopneumonia
d) Somatic death 68. Pin point haemorrhage are called as- is called as Siderosis 84. Erythritol sugar play important role in
60. Post mortem white clot is termed as- a) Extravasation d) In IB chick embryo inoculation pathogenesis of-
a) Chicken fat clot b) Ecchymoses lesions are absent. a) Pasteurellosis
b) Current jelly clot c) Petechiae 76. Neural lymphomatosis is main lesion in- b) Salmonellosis
c) Thrombus d) Apoplexy a) IBD c) Brucellosis
d) None of the above 69. Infraction is a area of- b) Marek‟s disease d) Campylobacteriosis
61. Macrophages of brain are- a) Only ischemia c) ALC 85. Which is example of direct zoonoses-
a) Epitheloid cells b) Ischemia with liquifactive d) Ranikhet disease a) Histoplasmosis
b) Microglial cells necrosis 77. Main target of virus is T cell in- b) Japanese encephalitis
c) Histiocytes c) Ischemia with coagulative a) IBD c) Rabies
d) Schwann cells necrosis b) Marek‟s disease d) Taeniasis
62. Which is absent in exudate- d) Only liquifactive necrosis c) ALC 86. Most common strain of FMD is-
a) Irritant 70. In poultry Intadermal test in T.B. is d) Ranikhet disease a) A
b) WBC performed at- 78. Rubber jaw syndrome is seen in- b) O
c) RBC a) Skin a) Tumors of parathyroid c) C
d) Platlets b) Comb b) Tumors of adrenal d) Asia-1
63. Diffuse spreading suppurative c) Wattle c) Tumors of pancreas 87. The most indicative symptom of
inflammation of connective tissue is d) All of the above d) Tumors of thyroid Hemorrhagicsepticemia is-
called as- 71. Most radioresistant organ is- 79. Localized dilatation of artery is called a) Hard edema of neck
a) Abscess a) Spleen as- b) Stop feeding
b) Furuncle b) Brain a) Arteritis c) Difficult breathing
c) Phlegmon c) Ovary b) Aneurysm d) High temperature
d) Pustule d) Skin c) Arthritis 88. Amyelia means absence of-
64. Mucopurulent exudates means- d) Lymphangitis a) Legs
213 214
Dhruv N Desai Dhruv N Desai
b) Lower jaw 96. Microabscesses in brain is b) Anthrax 113. Sequretra is Characteristic of
c) Cranium pathognomonic lesion in- c) Listeriosis a) CBPP
d) Spinal cord a) Gid d) Streptococcosis b) CCPP
89. Swollen face in poultry is seen in- b) Listeriosis 105.Moraxella bovis causes infection in- c) Both
a) Infectious bronchitis c) Ranikhet disease a) Ears d) None of the above
b) Infectious coryza d) Bovine spongiform b) Eyes 114. A disease in which cat acts as final host
c) IBD encephalopathy c) Foot and intermediate host both-
d) Ranikhet disease 97. Which of the following produces d) Skin a) Trichomoniasis
90. Mycoplasmal disease of poultry is- fibrinousperihepatitis in fowl- 106. Wooden tongue is caused by- b) Toxoplasmosis
a) Chronic respiratory disease a) E. coli a) Actinomycosis c) Besnoitiosis
b) Chicken infectious anemia b) Salmonella pullorum b) Actinobacillosis d) Sarcocytosis
c) Infectious coryza c) Pasteurellamultocida c) Spirochaetosis 115. East coast fever is caused by-
d) Avain encephalomyelitis d) IBD virus d) Avian Tuberculosis a) Theileriaannulata
91. Avain encephalomyelitis is also known 98. Vaccine given at the first day itself at 107. Equine infectious anaemia is also known b) Theileriaparva
as- hatchery is- as- c) Eimeriaacervulina
a) Bird flu a) Marek‟s disease a) Damp fever d) Isosporacanis
b) Fowl pest b) Ranikhet disease b) Malignant fever 116. Species of Eimeria causing coccidiosis in
c) Fowl plague c) Fowl cholera c) Swamp fever proximal part of small intestine-
d) Epidemic tremor d) None of the above d) Avain nephritis a) Eimeriaacervulina
92. Hemorrhages at pectoral muscles are 99. Telescoping of intestine is called at 108.In which disease virus causes b) Eimeriastaedie
seen in- a) Volvulus destruction of neurons diffusely throughout c) Eimeriacanis
a) Infectious bursal disease b) Torsion grey matter- d) EimeriaZuernie
b) Ranikhet disease c) Intusseption a) Equine viral arteritis 117. Rhexis means-
c) Infectious bronchitis d) Paralytic ileus b) Japanese B encephalitis a) Escape of blood from a vessel by
d) Pullorum disease 100. PPR is also called as- c) Louping ill rupture of vessel
93. Which of the following statement is a) Goat plague d) Bovine viral diarrhea b) Escape of blood from a vessel
incorrect- b) Cattle plague 109. Recent outbreak of avian influenza is through intact vessel
a) Horse is most susceptible to c) Swine plague by which strain- c) A type of emboli
tetanus d) Sheep plague a) H5N2 d) Postmortem congestion
b) Button ulcers in intestine are 101. Zebra markings in intestine is seen in- b) H7N1 118. Multiple linear mucosal haemorrhages in
seen in swine fever a) Rinder Pest c) H7N2 rectum is seen in-
c) Pulpy kidney is found in swine b) PPR d) H5N1 a) ALC
fever c) Botulism 110. Biphasic fever is characteristic of- b) Ranikhet disease
d) Overeating disease is d) Meliodosis a) Rabies c) Marek‟s disease
enterotoxaemia 102. AvainSpirochetosis is transmitted by- b) Canine distemper d) Infectious coryza
94. In which disease haemoglobinurea is not a) Mite c) Infectious canine hepatitis 119. Smallest RBC is found in-
a feature- b) Tick d) Bovine ephemeral fever a) Goat
a) Babesiosis c) Flies 111. Nasal granuloma in cattle is caused by- b) Elephant
b) Leptospirosis d) Culicoides spp. a) Schistosomaspindalis c) Rat
c) Theleriosis 103. Lymphoid leucosis is also called as- b) Schistosomanasalis d) Horse.
d) None of the above a) Big liver disease c) Sporothixschenckii 120.In which outbreak at poultry farm maximum
95. Vacuolation of neurons are seen in- b) Big kidney disease d) Rhinosporadium spp. mortality of birds will be expected?
a) Listeriosis c) Big spleen disease 112. Psittacosis is mainly a disease of a) Ranikhet disease
b) Mad cow disease d) Splenic fever a) Elephant b) Infectious Bronchitis
c) Rabies 104. Oedema factor play important role in b) Birds c) Infectious Lanyngiotracheitis
d) Avian encephalomyelitis pathogenesis of- c) Horse d) Avain Encephalomyelitis
a) Botulism d) Sheep 121. Dohle‘s bodies are toxic granules of-
215 216
Dhruv N Desai Dhruv N Desai
a) Macrophages a) FMD doesn‘t occur in Elephant a) Coagulative necrosis b) Vitamin C
b) Eosinophils b) Star grazing in chicken is due to b) Liquifective necrosis c) Thyroxine
c) Neutrophils vitamin B1 deficiency c) Caseative necrosis d) Vitamin A
d) Lymphocytes c) Actinomycosis mostly affects hard d) Fat necrosis 146.Healing by first intention occurs in:
122. Tropical fever is caused by- tissues in animals 138. First change after death is a) Closed wound
a) Theleriaparva d) In Anthrax rigor mortis is absent. a) Algor mortis b) Open wound
b) Theleriaannulata 130. Which of the following pair is incorrectly b) Rigor mortis c) None of the above
c) Babesiabovis matched-? c) Formation of bloat d) All of above
d) Anaplasmacentrale a) Haemoptysis- Blood in Vomit d) Both (b) & (c) 147. The mass of proliferating connective
123. Edema consisting of gelatinous material in b) Pyelonephritis- Suppuration in 139.Cart wheel appearance of nucleus found tissue under scar is known as:
neck and brisket region seen in cattle in- kidney in a) Keloid
a) Black Quarter c) White muscle disease- Vitamin E a) Plasma cell b) Proud flesh
b) Deganala disease d) Glanders- Mallein test b) Basophils c) Cyst
c) Botulism 131. Which vitamin is act as anti-oxidant c) Eosinophils d) Scar
d) Haemorrhagic Septicemia a) Vit.B d) Monocyte 148. Fibrin entraps the irritant and thus
124. Which is the main chemical mediator of b) Vit. D 140. Extreme elevation of leucocyte in facilitates:
inflammation-? c) Vit. E peripheral blood is known as a) Diapedesis
a) Serotonin d) Vit. A a) Shift to left b) Chemotaxis
b) Bradykinin 132. Macrophage in spleen are known as b) Leukamoid reaction c) Phagocytosis
c) Histamin a) Septal cell c) Right shift d) Rhexis.
d) Interleukin-1 b) Kuffer cell d) Both (a) & (c) 149. Black head disease is predominately a
125. In Angara disease, the pathological finding c) Alveolar cell 141.Blood in vomitus disease of-
is- d) All of above a) Haematemesis a) Cattle caused by Parasite
a) Haemopericardium 133. Macrophage laden with haemosiderin b) Haemoptysis b) Poultry caused by Parasite
b) Hydropericardium pigment c) Epistaxis c) Horse caused by Virus
c) Myocarditis a) Kuffer cell d) Melena d) Pig caused by Virus
d) Pneumopericardium b) Foam cell 142. Bleeding from the oviduct is designated 150. Which of the following is correctly
126. Most pathogenic species/disease affecting c) Heart failure cell as: matched-?
Snakes- d) None of above a) Epitaxis a) Picorna virus-Ranikhet disease
a) Pasteurellosis 134. Toxic jaundice is also known as b) Hemosalpinx b) Lumpy skin disease- Pox virus
b) Histomoniasis a) Post haepatic jaundice c) Hematocele c) Diamond skin disease-Herpes virus
c) Salmonellosis b) Haepatic jaundice d) Hematemasis d) Paramyxo virus-FMD
d) Listeriosis. c) Pre haepatjc jaundice 143. Condition which is hereditary and sex
127. Increase in number of immature lymphoid d) Obstructive jaundice linked in which clotting is delayed: ****************************************
cells in blood is known as- 135. Siderosis means a) Apoplexy **************************************
a) Shift to left a) Deposition of calcium in lung b) Hemophilia
b) Shift to right b) Deposition of iron dust in lung c) Brown induration
c) Leukemia c) Deposition of silicon in lung d) Epistaxis
d) Leukocytosis d) Deposition of silver particle in lung 144. Transformation of one type of cell into ANSWERS
128. Spleen with diffuse Amyloidosis is 136. Van den Bergh test for obstructive another is known as: 1. a
known as- jaundice a) Dysplasia 2. c
a) Sago spleen a) Direct b) Metaplasia 3. c
b) Bacon spleen b) Indirect c) Hyperplasia 4. a
c) Ham spleen c) Biphasic d) Aplasia 5. d
d) Pulpy spleen d) Both (b) & (c) 145. Capillary rupture and hemorrhage occurs 6. b
129. Which of the following statement is 137. In abscess which type of necrosis is due to deficiency of: 7. a
incorrect- seen? a) Vitamin B
217 218
Dhruv N Desai Dhruv N Desai
8. b 55. b 102. b 149. b
9. b 56. a 103. a 150. b
10. c 57. c 104. b ************************************
11. a 58. a 105. b *****************************
12. a 59. b 106. b *********
13. d 60. a 107. c
14. c 61. b 108. c
15. d 62. d 109. d SET-6
16. b 63. c 110. b
17. d 64. b 111. b 1. Genus salmonella is given by
18. d 65. b 112. b a) Robert Koch
19. a 66. a 113. a b) Daniel .E.salmon
20. d 67. c 114. b c) A.jenar
21. a 68. c 115. b d) R.vircho
22. c 69. c 116. a 2. Largest single reservoir of salmonella
23. b 70. c 117. a organisms
24. d 71. b 118. b a) Domestic poultry
25. c 72. c 119. a b) Cattel
26. b 73. a 120. c c) Camel
27. c 74. b 121. c d) Pig
28. b 75. d 122. b 3. Pullorum disease is also known as
29. b 76. b 123. d a) Bacillary white diarrhea
30. d 77. b 124. c b) Fowl cholera
31. b 78. a 125. b c) Fowl pox
32. a 79. b 126. c d) Avian influenza
33. b 80. b 127. c 4. Which form of pullorum disease is mainly
34. a 81. a 128. b seen in chicks
35. d 82. b 129. a a) Chronic
36. d 83. c 130. a b) Acute
37. c 84. c 131. c c) Sub acute
38. b 85. c 132. a d) Per acute
39. b 86. b 133. c 5. Which form of pullorum disease is mainly
40. c 87. a 134. b seen in adults
41. b 88. d 135. b a) Chronic
42. b 89. b 136. a b) Acute
43. d 90. a 137. b c) Sub acute
44. c 91. d 138. a d) Per acute
45. b 92. a 139. a 6. Pullorum disease is predominantly seen in
46. d 93. c 140. b a) Adults
47. b 94. c 141. a b) Chicks under 3 wk of age
48. c 95. b 142. b c) Chicks above 3 wk of age
49. a 96. b 143. b d) Grower
50. d 97. a 144. b 7. Post mortem lesion in adults in case of
51. c 98. a 145. b pullorum disease
52. c 99. c 146. a a) Misshapen, irregular, discoloured,
53. a 100. a 147. a cytic ova
54. b 101. a 148. c
219 220
Dhruv N Desai Dhruv N Desai
b) Hydropericardium b) Leg swollen a) Mycoplasma gallisepticum c) CRD
c) Discoloured liver c) Conjunctivitis b) Sal.gallinarum d) Aftosa
d) All of above d) Swollen wattles c) Sal. typhimurium 31. In case of putryfication of birds in fowl
8. Fowl typhoid is primarily disease of 16. Infectious coryza is caused by d) Sal.pullorum cholera, Which sample is important for
a) chicken a) Sal. typhimurium 24. Mycoplasma sp. was probably first demonstration of organium.
b) Turkey b) Sal. gallinarum encountered in chicks by a) Heart blood
c) Both of above c) Haemophilusparagallinarum a) Nelsen b) Bonemarrow impression
d) Pigeon d) Sal. pullorum b) Koch c) Both of above
9. Causative organisum of fowl typhoid 17. Hemophilusparagallinarum also known as c) Vircho d) Liver
a) Sal. Pullorum a: d) Jener 32. Which tick is transmit Borreliaanserin
b) Sal.gallinarum a) Avibacteriumparagallinarum 25. Mycoplasmosis is also known as a) Argus persicus
c) Sal.suis b) Salmonella paragallinarum a) CRD b) Ornithodorus
d) Sal.bovis c) Corynebacteriumparagallinarum b) Air sac disease c) Otobius
d) Bacillus paragallinarum c) Both of above d) None of above
18. Infectious coryza mainly affect : d) Aftosa 33. Pathognomic lesion of spirochetosis
10. Main Route of horizontal transmission of a) Gi tract 26. Which part of the poultry is affected in case a) Carcass is emaciated
fowl typhoid b) Urogenital tract of CRD b) Spleen is enlarged upto 6 times more
a) Orofaecal c) Respiratory tract a) Upper respiratory tract than normal
b) Respiratory d) Liver b) Lower respiratory tract c) Liver is frible
c) Genital c) Beak d) Intestinal haemorrage
d) None of above d) All of above 34. Which is the main sign of botulinum
11. Pathognomic lesion of fowl typhoid 27. Characteristics feature of fowl a) Paralysis of wings , legs,neck
a) Sciatic nerve become enlarge mycoplasmosis is : b) Swollen head
b) Swollen wattles and combs 19. Factor required for growth of haemophilus a) White fibrinous membrane on the c) Enlarged spleen
c) Haemorrhage in trachea a) Haemin surface of liver d) All of above
d) Liver show greenish bronze app b) NAD b) Pericarditis 35. Gangreneous dermatitis is mainly seen in
12. Feature of chronic fowl typhoid c) X and V factor c) Perihepatitis a) Broiler birds
a) Pericarditis d) All of above d) Air sacculitis b) Layer birds
b) Turbid yellow fluid in the pericardial 20. Pathogenicity of H.paragallinarum is c) Both of above
sac depends on d) None of them
c) Fibrin attached to the the surface of the a) Capsule 36. Characteristic sign of gangreneous
heart b) Specific haemagglutination antigens dermatitis is
d) All of above c) Both of above a) Discolouration and oedema of the skin
13. Which is intestinal carrier of paratyphoid d) None of them 28. Avian pasturellosis is also known as and s/c tissues of the body including
organism 21. Main source of infection of a) Fowl cholera wings.
a) Rat Haemophilusorganisum b) Fowl typhoid b) Wing rot
b) Mice a) Clinically affected carrier birds c) CRD c) Both of above
c) Both of above b) Water d) Aftosa d) None of them
d) Cattel c) Feed 29. Causative organisum of fowl cholera
14. Rat and mice carry paratyphoid organism d) All of above a) Pasturellamultocida
particularly 22. Sign of infectious coryza b) Pasturellahaemorragica
a) Sal.typhimurium a) Swollen heads c) Avibacteriumparagallinarum 37. Causative organisum of gangrenous
b) Sal. enteritidis b) Swollen wattles d) None of them dermatitis
c) Both of above c) Seromucoid discharge from nasal and 30. Facial oedema and swollen wattles is a sign a) Clostridium perfringens type A
d) Sal.suis ocular orifices of ______disease b) Clostridium septicum
15. Sign common in paratyphoid d) All of above a) Fowl cholera c) Staphylococcus aureus
a) Blindness due to opacity of the cornea 23. Avian mycoplasmosis is caused by b) Fowl typhoid d) All of above
221 222
Dhruv N Desai Dhruv N Desai
38. Gangreneous dermatitis is also occurs due d) All of above c) Dne day old a) Fusariotoxins
to nutritionl deficiency of d) All age b) Increased peristalsis in new layers.
a) Vit E 54. Post mortem lesion of coligranuloma c) Both of above
b) Se a) Hard, yellow nodular granuloma in the d) None of them
c) Both a&b mesentry 62. Vent Gleet is inflammation of:
d) None of them 46. Which Ag is exotoxin and liberated by b) Hard, yellow nodular granuloma in the a) Sub acute inflammatory condition of
39. Predisposing disease of gangreneous autolysis of somatic cells wall of caeca cloaca
dermatitis a) O Ag c) Both a and b b) Inflammation of crop
a) IBD b) K Ag d) None of them c) Inflammation of gizzard
b) Fowl cholera c) H Ag d) Inflammation of proventriculous
c) IlT d) All of above 63. Proventriculitis is :
d) All of above 47. Fibrinious pericarditis with thick pericardial a) Sub acute inflammatory condition of
40. Predisposing factor incase of necrotic sac is characteristic feature of cloaca
enteritis a) Colisepticemia 55. Sign of panopthalmitis b) Inflammation of crop
a) Coccidial infection b) Salmonellosis a) Torticollis c) Inflammation of gizzard
b) Dietary factor c) Pasturellosis b) Convulsion d) Inflammation of proventriculous
c) Both a and b d) Noneof them c) Blindness
d) None of the above 48. Internal laying is common in case of______ d) Not observable
41. Which type of diarrhoeaseen in case of a) Colisepticemia 56. Cause of panopthalmitis
necrotic enteritis. b) Egg peritonitis a) E.coli
a) Watery c) Fowl typhoid b) Salmonella 64. Smothering often occurs during
b) Disentry type d) Fowl cholera c) Pasturella a) Night
c) Dark coloured 49. Synonym of yolk sac infection. d) Corynebacterium b) Day
d) Reddish coloured a) Mushy chick disease 57. Complete destruction of retina seen in case c) Afternoon
42. Which part is mainly affected in case of b) Egg peritonitis of d) All of above
necrotic enteritis c) Colisepticemia a) Panopthalmitis 65. Sign of carbone monoxide poisoning in
a) Lower small intestine d) Colibacillosis b) Egg peritonitis poultry
b) Upper small intestine 50. Which bacterial disease is most common c) Colisepticemia a) Blood and lungs look like cheery red in
c) Large intestine cause of 1st wk after hatching d) Coligranuloma colour
d) Gizzard a) Egg peritonitis 58. Sequelae of colisepticemia b) Necrosis in lung
43. Test used for screening and identification of b) Colisepticemia a) Synovitis c) Gangrene in ovary
E.coli c) Fowl typhoid b) Panopthalmitis d) None of them
a) Eijkman‘s test d) Yolk sac disease c) Hepatitis 66. Blood seen cherry red in colour in poultry
b) Animal inoculation test 51. Synonym of Hajarre‘s disease d) Myositis in co poisoning due to
c) Milk ring test a) Egg peritonitis 59. Which orgen is affected in case of heat a) Formation of carbonedioxide
d) Dermal test b) Coli granuloma stock in poultry? b) Formation of formic acid
44. Which Ag is not correlated with c) Colisepticemia a) Liver c) Formation of carbone nitrite
pathogenicity of E.coli d) Unknown b) Kidney d) Formation of carboxyhaemoglobin
a) O Ag 52. Cause of coligranuloma c) Spleen 67. Which homeopathic drug use incase of
b) K Ag a) E.coli d) Brain prolapse of cloaca:
c) H Ag b) Salmonella 60. Which type of sign seen in brain incase of a) Fusariotoxins
d) All of above c) Pasturella heat stock in poultry? b) Utrogens
45. Which Ag is associated with virulence of d) Corynebacterium a) Congestion and haemorrage c) Titali
E.coli 53. Which age birds mainly affected by b) Necrosis d) Podophyllum
a) O Ag coligranuloma c) Grangrene 68. Some birds stricks their head on roof causes
b) K Ag a) Adults d) All of above death due to
c) H Ag b) Young 61. Cause of prolapse of cloaca in poultry is a) Anorexia
223 224
Dhruv N Desai Dhruv N Desai
b) Cerebral concussion d) Vit B12 d) Fungal disease a) Early mortality of embryos
c) Asphyxia 76. Pendulous crop or sour crops is cause by: 83. Howmuch concentration of ammonia in b) Late mortality of embryos
d) Oedema a) Inherited condition shed cause keratoconjunctivitis c) Stunting of embryos and soft bones
69. Infectious dermatitis also k/a b) Hot weather a) 10-20ppm d) Dwarfing of embryos
a) Red wing disease c) Moniliasis infections b) 20-30ppm
b) Blue wing disease d) All of above c) 30-40 ppm
c) Green wing disease 77. Which type of lesion seen in case of d) Above 50 ppm
d) Yellow wing disease visceral gout 84. Which type of toxicity is seen in case of
70. Primary cause of infectious dermatitis may a) Presence of whitish uric acid deposits Ochrotixin A in poultry 91. Which type of embryonic sign seen incase
be immunosupression by: or urates on surface of kidney, a) Nephrotoxic of Vit B2 deficiency
a) IBD , Reovirus pericardium b) Hepatotoxic a) Early mortality of embryos
b) Chicken anaemia agent b) Ureters are impacted and dilated with c) Both b) Late mortality of embryos
c) IBH urates d) Cardiotoxic c) Stunting of embryos and soft bones
d) All of above c) Both of above 85. Ochrotoxin A is produce by which d) Dwarfing of embryos
71. Primary cause of infectious dermatitis d) None of above organisum 92. Which type of embryonic sign seen incase
generally followed by: 78. Which vitamins deficiency may cause a) Species of Aspergillus of Vit E deficiency
a) Staph.aureus visceral gout b) Species of penicillium a) Early mortality of embryos
b) Pox virus a) Vit A c) Both b) Late mortality of embryos
c) Coccidiosis b) Vit B d) None of above c) Stunting of embryos and soft bones
d) All of above c) Vit C 86. Which type of toxicity is seen in case of d) Dwarfing of embryos
72. Xanthomatosis are more commonly seen in d) Vit D Citrinin in poultry 93. Coccidiosis is characterized by
: 79. Other name of Fatty liver kidney syndrome a) Nephrotoxic a) Necrosis of intestitis
a) Sexually mature birds is: b) Hepatotoxic b) Emphysema of lung
b) Young one a) Grey eye disease c) Both c) Cirrhosis of liver
c) Growers b) Pink disease d) Cardiotoxic d) Haemarragic enteritis
d) All of above c) Pink eye disease 87. Which type of lesion is seen incase of 94. One of the most parasitic disease of poultry
d) Grey disease Tricothecenes a) Coccidiosis
80. Lesion of fatty liver kidney syndrome: a) Necrosis in thymus, spleen and bursa b) Salmonellosis
a) Pale liver and kidney due to excess b) Lymphoid depletion in thymus, spleen c) Ascariosis
amount of fat. and bursa d) Histoplasmosis
73. In xanthomatosis which substance b) Haemorrhage in liver c) Decreased cell- mediated immune 95. Coccidial infections is spread by
accumulates in the skin c) Necrotic foci on liver response a) Oocysts
a) Cholesterol and fat d) Enlarged bursa d) All of above b) Larvae
b) Carbohydrate 81. Other name of Keratoconjunctivitis in 88. Which toxins produce by fusarium spp. c) Pupae
c) Protein poultry affect poultry d) Biting flies
d) All of above a) Pink eye a) T1 96. Essenteial condition s for the survivl of
74. Which is responsible for haemorrhagic b) Ammonia blindness of broiler b) T2 oocyst
disease c) Nitrite blindness c) T3 a) Moisture
a) Sulpha drugs as anticoccidialangents d) Formaline blindness d) T 4 b) Oxygen
b) Aflatoxin 89. Which type of embryonic sign seen incase c) Suitable temperature
c) Both of Vit A deficiency d) All of above
d) None of above a) Early mortality of embryos 97. Most coccidial infections are
75. Which vitamine deficiency may cause b) Late mortality of embryos a) Subclinical
Gizzard erosions occurs ? 82. Cause of keratoconjunctivitis in poultry c) Stunting of embryos and soft bones b) Clinical
a) Vit B2 a) Nitric acid fumes d) Dwarfing of embryos c) Both of them
b) Vit C b) Ammonia fumes 90. Which type of embryonic sign seen incase d) None of above
c) Vit A c) Formaline fumes of Vit D deficiency
225 226
Dhruv N Desai Dhruv N Desai
98. Infact most of the birds survive to the 20. c 67. d (C) Fowl Pox
coccidiosis due to 21. a 68. b (D) Ranikhet Disease.
a) Preimmunity 22. d 69. b 3. Bronze discolouration of Liver in poultry is
caused by-
b) Postimmunity 23. a 70. d
(A) Pasteurellosis
c) Both of them 24. a 71. a (B) IBD Virus
d) None of above 25. c 72. a (C) Leptospirosis
99. Species of eimeria affect the epithelium of 26. b 73. a (D) Salmonellosis
duodenal loop- 27. a 74. c 4. Fragmentation of nucleus in a cell is termed as-
a) E.nacatix 28. a 75. d (A) Pyknosis
b) E.maxima 29. a 76. d (B) Karyorrhexis
(C) Karyolysis
c) E. acevulina 30. a 77. c
(D) Chromatolysis
d) E.brunetti 31. c 78. a 5. Infectious Necrotic Hepatitis in sheep is caused
32. a 79. b by-
33. b 80. a (A) Leptospira sp.
100. Species of Eimeria affects the lower small 34. a 81. b (B) Fasciola hepatica
intestine , rectum , and proximal area of 35. a 82. b (C) Clostridium sp.
caeca- 36. c 83. d (D) Heterakis gallinarum
a) E. brunette 37. d 84. c
b) E.necatrix 38. c 85. c 6. The animal resistant to Atherosclerosis is-
c) E.acevulina 39. a 86. a (A) Cattle
d) E.maxima 40. c 87. d (B) Swine
41. c 88. b (C) Rabbit
**************************************** (D) Poultry.
42. a 89. a
************************************** 7. Tumour which Metastasize in different organ
43. a 90. c
is-
ANSWERS 44. c 91. d (A) Lipoma
45. b 92. b (B) Angioma
46. a 93. d (C) Seminoma
47. a 94. a (D) Sqamous cell carcinoma.
1. b 8. Haemoglobinuria is seen in-
48. b 95. a
2. a (A) Theileriosis
49. a 96. d
3. a (B) Leptospirosis
50. d 97. a
4. b (C) Salmonellosis
51. b 98. a (D) Pasturellosis.
5. a
52. a 99. c 9. Who is the father of Cellular Pathology?
6. b
53. a 100. a (A) John Hunter
7. d (B) Robert Koch
54. c
8. c **************************************** (C) Rudolph Virchow
55. c
9. b ************************** (D) K.Cohnhiem.
56. a
10. a 10. Pseudo Rabies is caused by-
57. a SET-7 (A) Lyssa virus
11. d
58. a (B) Picorna virus
12. d 1. Inflammation of lymph node is called as-
59. d (C) Paramyxo virus
13. c (A) Lymphangitis
60. a (D) Herpes virus.
14. c (B) Lymphadenitis
61. c (C) Typhilitis
15. a
62. a (D) Both (A) and (B).
16. c 11. Pulpy Kidney Disease is caused by-
63. d 2. Disease of poultry which is not caused by virus
17. a (A) Clostridium perfringens
64. a is- (B) Clostridium septicum
18. c (A) Chronic respiratory Disease
65. a (C) Clostridium novyi
19. d (B) Infectious Bronchitis
66. d (D) Clostridium tetani
227 228
Dhruv N Desai Dhruv N Desai
12. Turkey Egg Kidney is seen in- (B) Botriomycosis (A) Johne‘s disease
(A) Swine Pox (C) Haemorrhagic Septicemia 31. Most susceptible species for Haemorrhagic (B) Tuberculosis
(B) Swine Influenza (D) Actinobacillosis. septicemia- (C) Rinder pest
(C) Swine Fever 22. Mode of transmission of IBR virus is- (A) Sheep (D) Both (A) and (C)
(D) Swine Erysipelas. (A) Venereal (B) Buffalo
13. Poll evil in Horse is caused by- (B) Inhalation (C) Cattle
(A) Clostridium tetani (C) Both (D) Pig 41. CBPP differs from CCPP in-
(B) Actinomyces bovis (D) None of the above. 32. Erythritol sugar plays important role in (A) Both occur in same species
(C) Brucella abortus 23. Maedi is primarily a disease of- pathogenesis of- (B) Sequestra formation
(D) Both (B) and (C). (A) Sheep affecting respiratory system (A) Clostridium spp. (C) Mode of transmission
14. Nutritional roup in Poultry is caused due to (B) Cattle affecting reproductive system (B) Brucella spp. (D) Pathogenesis
deficiency of- (C) Sheep affecting nervous system (C) Bacillus spp. 42. Which bacterium is predisposed by Fasiola
(A) Vitamin B (D) Cattle affecting nervous system. (D) Corynebacterium spp. hepatica infestation-
(B) Vitamin C 24. Equine Plague is also called as- 33. Substance responsible for increase penetration (A) Bacillus spp.
(C) Vitamin E (A) Equine viral arteritis of Lyssa virus- (B) Clostridium spp.
(D) Vitamin A. (B) Glanders (A) Hyaluronidase (C) Leptospira spp.
15. Epithelial Pearls are seen in- (C) Strangles (B) Erythriol (D) Pasturella spp.
(A) Sebaceous cell Adenoma (D) African Horse sickness. (C) Protagen 43. Which is the most potent aflatoxin-
(B) Squamous cell carcinoma 25. Sore mouth in cattle is seen in- (D) Amylase (A) M1
(C) Melanoma (A) Blue tongue 34. Crop mycosis in poultry is caused by- (B) M2
(D) Venereal granuloma. (B) Bovine malignant catarrh (A) Bacteria (C) B1
(C) Rinder pest (B) Mycoplasma (D) B2
(D) Vesicular Stomatitis (C) Fungi 44. Curled toe paralysis in chicken is due to
16. Blue Tongue in sheep is caused by- (D) Virus deficiency of-
(A) Herpes virus 35. Brooder‘s pneumonia in poultry is caused by- (A) Vitamin B12
(B) Birna virus 26. Disease caused by Clostridium septicum is- (A) Candida albicans (B) Vitamin B1
(C) Picorna virus (A) Black Quarter (B) Aspergillus fumigatus (C) Niacin
(D) Orbi virus. (B) Enterotoxaemia (C) Haemophilus paragallinarum (D) Vitamin B2
17. Most common Serotype of FMD virus in India (C) Braxy (D) Pasturella multocida 45. Mn deficiency is chicken will lead to-
is- (D) Tetanus (A) Pica
(A) A 27. Sulphur granules in yellowish pus is seen in- (B) Star grazing condition
(B) C (A) Glanders 36. Circling disease in cattle is caused by- (C) Crazy chick disease
(C) Asia-1 (B) Strangles (A) Listeria monocytogenes (D) Slipped Tendon
(D) O. (C) Staphylococcosis (B) Erysipelothrix rhusiopathiae
18. In which disease post mortem of carcass is (D) Actinomycosis (C) Streptococcus equi
prohibited? 28. Toxins of organism causes peripheral nerve (D) Chlamydia psittacii 46. Phosphorous deficiency in soil will predispose
(A) Haemorragic septicemia paralysis in cattle- 37. Intranuclear inclusion bodies are seen in- the cattle to-
(B) Rinder pest (A) Botulism (A) Pox diseases (A) Haemorrhagic septicemia
(C) Anthrax (B) Tetanus (B) Herpes virus infection (B) Botulism
(D) Brucellosis. (C) Both (C) Adeno virus infection (C) Anthrax
19. Mad itch is mostly a disease of- (D) None of the above (D) Lyssa virus infection (D) Mucosal disease
(A) Caprine 29. Diamond skin disease is primarily a disease 38. Negri bodies are seen in Rabies which are- 47. Black head disease is predominately a disease
(B) Bovine of- (A) Intranuclear of-
(C) Swine (A) Horse (B) Intracytoplasmic (A) Cattle caused by Parasite
(D) Ovine (B) Lion (C) Both (B) Poultry caused by Parasite
20. Tigroid Heart is seen in cattle affected with- (C) Sow (D) May be intranuclear or (C) Horse caused by Virus
(A) Bovine malignant catarrhal (D) Turkey intracytoplasmic (D) Pig caused by Virus
(B) Botulism 30. In Johne‘s disease, corrugation is not the 39. Enlargement of Bursa of fabricius in poultry is 48. Which of the following is correctly matched-
(C) Bovine viral diarrhea feature in- seen in- (A) Tubercular lesions are calcified-
(D) Foot and mouth disease. (A) Cattle (A) CRD Buffalo
(B) Sheep (B) IB (B) Johne‘s disease-Foul smelling diarrhea
(C) Horse (C) RD (C) Avian spp.- Dry pus
21. Wooden Tongue in cattle is seen in- (D) Both (B) and (C) (D) IBD (D) Lamb dysentery- Clostridium
(A) Actinomycosis 40. Zebra marking is predominant feature of- perfringens type D
229 230
Dhruv N Desai Dhruv N Desai
49. Post mortem of chick shows foul smelling (D) Intradermal test is performed on (D) Interleukin-1 (D) In Anthrax rigor mortis is absent.
yellow-brown watery Yolk, fibrinous perihepatitis wattle. 75. Which of the following pair is incorrectly
and pericarditis, suspect the disease 58. Post mortem of cattle reveals too much 66. Big liver disease is also known as- matched-?
(A)Infectious Coryza emaciated carcass,mucosa of intestine thrown into (A) IBD (A) Haemoptysis- Blood in Vomit
(B) Fowl Typhoid corrugated folds, most probable cause will (B) CRD (B) Pyelonephritis- Suppuration in kidney
(C) Coli Bacillosis be- (C) Fowl cholera (C) White muscle disease- Vitamin E
(D) Infectious Bronchitis (A) Rinder pest (D) Fowl typhoid (D) Glanders- Mallein test
50. Haemorrhages at the tip of the proventricular (B) Johne‘s disease 67. In Angara disease, the pathological finding is-
gland is the pathognomic lesion seen in- (C) Tuberculosis (A) Haemopericardium
(A)Ranikhet disease (D) Pasterellosis. (B) Hydropericardium 76. Which vitamin is act as anti-oxidant
(B) Chronic respiratory disease 59. Spondylitis is inflammation of- (C) Myocarditis (A) Vit.B
(C) Infectious bursal disease (A) Prepuce (D) Pneumopericardium (B) Vit. D
(D) Avian influenza (B) Vertebrae 68. In Left side heart failure, the heart failure cells (C) Vit. E
(C) Bone are seen in- (D) Vit. A
51. Which of the following is not correctly (D) Spermatic cord (A) Lungs 77. Localized loss of melanin
matched-? 60. Which of the following is correct regarding (B) Heart (A) Vitiligo
(A) Zn deficiency-Pig poultry diseases-? (C) Kidney (B) Leucoderma
(B) Epidemic tremor-Virus (A) In pullorum disease, green constant (D) Spleen (C) Acanthosis nigricans
(C) Siderosis-silica dust diarrhea is seen 69. Sway back condition is seen due to deficiency (D) All of above
(D) Alkali disease-Se (B) Face is swollen and edematous in of- 78. Macrophage in spleen are known as
52. Pachymeningitis is inflammation of- Haemophilus infection (A) Cu (A) Septal cell
(A) Piamater (C) Bloody mucous expelled from trachea (B) Co (B) Kuffer cell
(B) Brain in Infectious Bronchitis (C) Mn (C) Alveolar cell
(C) Duramater (D) In pullorum disease, nervous signs are (D) Se (D) All of above
(D) Spinal cord seen along with diarrhoea 70. Most pathogenic species/disease affecting 79. Macrophage laden with haemosiderin pigment
53. Liquifactive necrosis is most commonly seen Snakes- (A) Kuffer cell
in- 61. In which outbreak at poultry farm maximum (A) Pasteurellosis (B) Foam cell
(A) Kidney mortality of birds will be expected? (B) Histomoniasis (C) Heart failure cell
(B) Liver (A) Ranikhet disease (C) Salmonellosis (D) None of above
(C) Heart (B) Infectious Bronchitis (D) Listeriosis. 80. Toxic jaundice is also known as
(D) Brain (C) Infectious Lanyngiotracheitis (A) Post haepatic jaundis
54. Which of the following is correctly matched-? (D) Avain Encephalomyelitis (B) Haepatic jaundis
(A) Picorna virus-Ranikhet disease 62. Dohle‘s bodies are toxic granules of- 71. Increase in number of immature lymphoid (C) Pre haepatjc jaundis
(B) Lumpy skin disease- Pox virus (A) Macrophages cells in blood is known as- (D) Obstructive jaundis
(C) Diamond skin disease-Herpes virus (B) Eosinophils (A)Shift to left
(D) Paramyxo virus-FMD (C) Neutrophils (B) Shift to right
55. Enlargement of Sciatic nerve is seen in- (D) Lymphocytes (C) Leukemia 81. Siderosis means
(A) Ranikhet disease 63. East coast fever is caused by- (D) Leukocytosis (A) Deposition of calcium in lung
(B) Marek‘s disease (A) Theleria parva 72. Oval and nucleated RBC‘s are present in- (B) Deposition of iron in lung
(C) Chronic respiratory disease (B) Theleria annulata (A) Parrot (C) Deposition of silicon in lung
(D) Infectious Coryza (C) Babesia bovis (B) Cobra (D Deposition of silver particle in lung
(D) Anaplasma centrale (C) Camel 82. Van den Bergh test for obstructive jaundice
64. Edema consisting of gelatinous material in (D) Both (A) and (B) (A) Direct
56. Apennosis is- neck and brisket region 73. Spleen with diffuse Amyloidosis is known as- (B) Indirect
(A) Intracellular edema of epidermis seen in cattle in- (A) Sago spleen (C) Biphasic
(B) Congenital lack of feathers in fowl (A) Black Quarter (B) Bacon spleen (D) Both (B) & (C)
(C) Absence of pineal gland (B) Deganala disease (C) Ham spleen 83. In abscess which type of necrosis is seen?
(D) Lack of cell differentiation during (C) Botulism (D) Pulpy spleen (A) Coagulative necrosis
embryogenesis (D) Haemorrhagic Septicemia 74. Which of the following statement is incorrect- (B) Liquifective necrosis
57. Which is incorrect about avain tuberculosis-? 65. Which is the main chemical mediator of (A) FMD doesn‘t occur in Elephant (C) Caseative necrosis
(A) Calcification absent inflammation-? (B) Star grazing in chicken is due to (D) Fat necrosis
(B) Liver and bones are most commonly (A) Serotonin vitamin B1 deficiency 84..First change after death is
affected (B) Bradykinin (C) Actinomycosis mostly affects hard (A) Alger mortis
(C) Lungs are most commonly affected (C) Histamin tissues in animals (B) Rigor mortis
231 232
Dhruv N Desai Dhruv N Desai
(C) Formation of bloat 93. Healing by first intention occurs in: 10. D 11. A 64. D 65. C
(D) Both (B) & (C) A) Closed wound 12. C 66. A
85..Inflammation of crop B) Open wound
(A) Blephritis C) None of the above 13. D 14. D 67. B 68. A
(B) Ingluvitis D) All of above 15. B 69. A
(C) Typhlitis 94. The mass of proliferating connective tissue
(D) Gonitis under scar is known as: 16. D 17. D 70. C 71. C
86.Cart wheel appearance of nuclease found in A) Keloid 18. C 72. D
(A) Plasma cell B) Proud flesh
(B) Basophils C) Cyst 19. C 20. D 73. B 74. A
(C) Eosinophils D) Scar 21. D 75. A
(D) Monocyte 95. Fibrin entraps the irritant and thus facilitates:
87..Extreme elevation of leucocyte in peripheral A) Diapedesis 22. C 23. A 76. C 77.B
blood is known as B) Chemotaxis 24. D 78. A
(A) Shift to left C) Phagocytosis
(B) Leukamoid reaction D) Rhexis 25. D 26. C 79. C 80. B
(C) Right shift 27. D 81. B
(D) Both (A) & (C)
88.Blood in vomitus 96. Biliary cirrhosis is also known as: 28. A 29. C 82. A 83. B
(A) Haematamiasis A) Multi nodular cirrhosis 30. D 84. A
(B) Haemoptysis B) Nodular cirrhosis
(C) Epistaxis C) Monolobular cirrhosis 31. B 32. B 85. B 86.A
(D) Melena D) All of above 33. A 87. B
89. Bleeding from the oviduct is designated as: 97. Pulmonary adenomatosis is characterized by:
A) Epitaxis A) Hyperplasia of epithelium 34. C 35. B 88. A 89. B
B) Hemosalpinx B) Hypertrophy of epithelium 36. A 90. B
C) Hematocele C) Both of the above
D) Hematemasis D) None of above 37. C 38. B 91. B 92. B
90. Condition which is hereditary and sex linked 98. Nucleus becoming smaller and condensed is 39. D 93. A
in which clotting is delayed: called
A) Apoplexy A) Necrosis 40. C 41. B 94. A 95. C
B) Hemophilia B) Pyknosis 42. B 96. C
C) Brown induration C) Chromatolysis
D) Epistaxis D) Keratolysis 43. C 44. D 97. C 98. B
99. Closure of lumen of hollow organ is called:
45. D 99. C
A) Fissure
91. Transformation of one type of cell into another B) Hypoplasia 46. B 47. B 100. B
is known as: C) Atresia
48. C
A) Dysplasia D) Sinus ****************************************
B) Metaplasia 100. Serum gives Direct Van Den Bergh test in 49. C 50. A *************************************
C) Hyperplasia A) Diabetes
51. C
D) Aplasia B) Obstructive jaundice SET-8
92. Capillary rupture and hemorrhage occurs due C) Hemorrhage 52. C 53. D
to deficiency of: D) Toxic jaundice
54. B 1. Other name of aspergillosis in poultry
A) Vitamin B
B) Vitamin C a) Brooder pneumonia
55. B 56. B b) Interstitial pneumonia
C) Thyroxine
57. C c) Suppurative pneumonia
D) Vitamin A
ANSWER KEY: VETRINARY PATHOLOGY 4. B 5. C 58. B 59. B d) None of above
6. A 60. B 2. Multiple yellow white pin head sized
1. B 2. A nodules scattered throughout the lung tissue
3. D 7. D 8. B 61. C 62. C is seen in:
9. C 63. A a) Flavus
233 234
Dhruv N Desai Dhruv N Desai
b) Aspergillosis a) Ascardiagalli c) By ingestion of grass hopper and
c) Moniliasis 10. Largest oocyst liberating coccidian is b) Hetrakisgallinae cockroaches
d) Histoplasmosis a) E.mivati c) Capillariaannulata d) All of above
3. Moniliasis is caused by b) E. maxima d) Capillariacontorta 26. Orgen of poultry in which Acuariidae
a) Moniliaalbicans c) E.burnetti 19. Which organisms spread the infection in the species inhabited
b) Candida albicans d) E.tenella poultry a) Gizzard
c) Both 11. Which Eimeria sp. cause heavy mortality a) Fly b) Proventriculus
d) None of above a) E.tenella b) Snail c) Duodenum
4. Turkish towel appearance in crop is seen in b) E.praecox c) Earthworm d) Crop
case of: c) E.mitis d) Ticks 27. Birds infected with Acuariidae sp. Often
a) Flavus d) All of above seen
b) Aspergillosis 12. Diptheretic enteritis is common in poultry a) Ulcer in the proventriculus
c) Moniliasis after recovery in case of occidiosis due to: b) Ulcer in the lung
d) Histoplasmosis a) E.necatrix 20. Capillaria species are stay in the which part c) Ulcer in the kidney
5. Cause of flavus b) E. burnetti of body of birds: d) None of them
a) Trichophytongallinae c) E. tenella a) Oesophagus 28. Parasites causes the condition of gapes in
b) Tichomonasgallinae d) E. maxima b) Crop birds
c) Moniliaalbicans 13. Sequalae of coccidiosis is: c) Both a) Syngamus tracheae
d) Candida albicans a) Fever d) None of the above b) Strongylus vulgaris
6. White powder like spots on the unfeathered b) Cough 21. Hair like worms of poultry: c) Schistosomanasale
parts of the head c) Devastating a) Ascardia sp. d) Ascardiagalli
a) Flavus d) Diarrhoea b) Hetarkis sp. 29. Tracheal worm of poultry is
b) Aspergillosis 14. Which chemical use for stored and c) Strongylus sp. a) Strongylusvalgaris
c) Moniliasis sporulation of oocyst in coccidiosis ? d) Capillaria sp. b) Schistosomanasale
d) Histoplasmosis a) Potassium dichromate at 100c 22. Which sp. Of capillaria is common in c) Syngamus tracheae
7. which parts of intestine is affected by b) Liquid N2 at 1960c pigeons: d) Ascardiagalli
E.necatrix c) Glycerin at 100c a) Capillariaannulata
a) Mid gut near the yolk sac diverticulum d) Fomaline at 100c b) Capillariacontorta
b) Duodenum 15. Which nematode parasite are important to c) Capillariaobsignata
c) Rectum poultry? d) All of above
d) Whole intestine a) Ascaridia 23. Bright red worms seen are seen in glandular 30. Who is acts as a transport hosts of
8. which parts of intestine is affected by b) Capillaria stomach of poultry is Syngymus tracheae
E.preacox c) Hetarkis a) Capillaria sp. a) Earth worm
a) Mid gut near the yolk sac d) All of above b) Ascardia sp. b) Grosshoppers
diverticulume 16. Which part of body affected by genus c) Hetarkis sp. c) Cockroaches
b) Upper third of the digestive tract tetrameriasis d) Tetrameriasis d) Snail
c) Rectum a) Gut 24. Intermediate hosts of tetramere Americana 31. Red nematode worm of trachea of poultry is
d) Whole intestine b) Respiratory tract are: a) Schistosomanasale
9. Smallest oocyst liberating coccidian is c) Gi tract a) Grasshopper b) Strongylusvalgaris
a) E.mivati d) Liver b) Cockroaches c) Syngamus trachea
b) E.maxima 17. Largest round worm of poultry that stay in c) None of them d) Hetrakisgallinae
c) E.burnetti the intestinal tract d) All of above 32. Trematodes of skin of poultry
d) E.tenella a) Ascardiagalli 25. How Fowl is infected with Tetramere a) Collyriclumfaba
b) Hetrakisgallinae Americana? b) Echinostomarevolutum
c) Caillariaannulata a) By ingestion of snail c) Prosthogonimusmacrorchis
d) Capillariacontorta b) By ingestion of food d) Philopthalmusgalli
18. Caecal worm of chicken : 33. Trematode of intestine of poultry
235 236
Dhruv N Desai Dhruv N Desai
a) Collyriclumfaba a) Collyriclumfaba 49. Which type of embryonic sign seen incase of a) Anaemic syndrome
b) Echinostomarevolutum b) Typhlocoelumcucumarinum Zinc deficiency b) Gizzard erosions
c) Prosthogonimusmacrorchis c) Philopthalmusgalli a) Skeletal deformities c) Nutritional croup
d) Philopthalmsgalli d) Billharziellapolonica b) Oedema of embryo d) Rickets or rachitic syndrome
34. Inter mediate hosts of 42. Nodular taeniasis is caused by c) Perosis and haemorrhage 57. Which condition is seen incase of Vit B12
Echinostomarevolutum is a) Railliteniaechinobothridia d) Development of faulty spine and limb deficiency syndromes
a) Snails b) Davaineaproglottina a) Anaemic syndrome
b) Fishes c) Philopthalmusgalli b) Gizzard erosions
c) Tad poles d) Billharziellapolonica c) Nutritional roup
d) All of above 43. Which type of intestinal lesion seen in case 50. Which type of embryonic sign seen incase of d) Rickets or rachitic syndrome
35. Prosthogonimus sp. is found in which orgen of Railliteniaechinobothridia infestation? Iodine deficiency 58. Which condition is seen incase of Vit A
of fowl a) Fibrosis and nodular appearance to the a) Skeletal deformities deficiency syndrome
a) Bursa, Oviduct intestine b) Oedema of embryo a) Anaemic syndrome
b) Liver, Bursa b) Haemorragic enteritis c) Perosis and haemorrhage b) Gizzard erosions
c) Kidney, Liver c) Haemorrage in ilium d) Enlarged thyroid glands c) Nutritional roup
d) Brain, Heart d) Rectal prolepse 51. Which condition seen incase of nicotinic d) Rickets or rachitic syndrome
36. Intermediate host of Prosthogoimus sp. 44. Which type of embryonic sign seen incase of acid deficiency 59. Which condition is seen incase of Vit D
a) 1st water snail and 2nd dragon fly Vit B2 deficiency a) Black tongue deficiency syndrome
b) 1st dragon fly and 2nd water snail a) Dwarfing of embryos b) Star gazing a) Anaemic syndrome
c) 1stcoccaroch and 2nd snail b) Embryos show oedema c) Curled toe paralysis b) Gizzard erosions
d) 1st snail and 2ndcoccaroch c) Clubbing down d) Twisted legs c) Nutritional croup
37. Trematode of kidney of poultry d) All of above 52. Which condition seen incase of Thiamine d) Rickets or rachitic syndrome
a) Collyriclumfaba 45. Which type of embryonic sign seen incase of deficiency
b) Echinostomarevolutum Pantothenic acid deficiency a) Black tongue
c) Eucotylenephritica a) Early mortality of embryos b) Star gazing
d) Philopthalmsgalli b) Late mortality of embryos c) Curled toe paralysis 60. Which condition is seen incase of Vit D
38. Trematode of Blood vessels of poultry c) Abnormal feathering development d) Twisted legs deficiency syndromes
a) Collyriclumfaba d) Dwarfing of embryos 53. Which condition is seen incase of Riboflavin a) Encephalomalacia
b) Echinostomarevolutum 46. Which type of embryonic sign seen incase of deficiency b) Exudative diathesis
c) Eucotylenephritica Biotin acid deficiency a) Black tongue c) Muscular dystrophy
d) Billharziellapolonica a) Skeletal deformities b) Star gazing d) All of above
39. Which vein of poultry affected by b) Oedema of embryo c) Curled toe paralysis 61. Which condition is seen incase of
Billharziellapolonica c) Perosis and haemorrhage d) Twisted legs Manganese deficiency syndromes
a) Abdominal veins d) Development of faulty spine and limb 54. Which condition is seen incase of a) Chodrodystrophy
b) Portal veins 47. Which type of embryonic sign seen incase of Pyridoxine deficiency b) Femoral head necrosis
c) Both Vit B12 deficiency a) Black tongue c) Round heart disease
d) Messenteric vein a) Skeletal deformities b) Star gazing d) Oil bird syndrome
b) Oedema of embryo c) Curled toe paralysis 62. Which condition is seen incase of
c) Perosis and haemorrhage d) Twisted legs Molybdenum deficiency syndromes
d) Development of faulty spine and limb 55. Which condition is seen incase of Biotin a) Chodrodystrophy
40. Trematode of eye of poultry 48. Which type of embryonic sign seen incase of deficiency b) Femoral head necrosis
a) Collyriclumfaba Manganese deficiency a) Black tongue c) Round heart disease
b) Echinostomarevolutum a) Skeletal deformities b) Star gazing d) Oil bird syndrome
c) Philopthalmusgalli b) Oedema of embryo c) Curled toe paralysis 63. Which condition is seen incase of Selenium
d) Billharziellapolonica c) Perosis and haemorrhage d) Fattyliver kidney syndrome deficiency syndromes
41. Trematodes of Respiratory System of d) Development of faulty spine and limb 56. Which condition is seen incase of Folic acid a) Chodrodystrophy
poultry: deficiency syndrome b) Femoral head necrosis
237 238
Dhruv N Desai Dhruv N Desai
c) Round heart disease d) Genital system d) All of above c) Viscerotropicvelogenic
d) Oil bird syndrome 72. Post mortem sign incase of peracute ILT d) All of above
64. Family of infectious laryngotrachitis a) Haemorragictracheitis 88. Highly virulent form of RD
a) Herpesviridae b) Blood stain mucous in trachea a) Doyle‘s
b) Poxviaridae c) Both a and b 80. Synonym of Ranikhet disease is b) Beach ‗s
c) Picornaviridae d) Caseousdiptheretic exudate a) New castle disease c) Hitchner‘s
d) Rota virus 73. Post mortem sign incase of acute ILT b) Doyle‘s disease d) Asymptomatic form
65. Outbreak of ILT mainly seen which age a) Haemorragictracheitis c) Both a and b 89. Which type of lesion seen incase of RD
group of birds b) Blood stain mucous in trachea d) None of above a) Haemorrage seen on tip of
a) 5 to 9 months c) Both a and b 81. Ranikhet disease belong which group of proventricular gland
b) 21 days d) Caseousdiptheretic exudates virus b) Haemorrage seen on caecal tonsil
c) 3 to 4 months 74. Synonym of Infectious avian a) Paramaxo virus c) Haemorrage on the tracheal mucosa
d) All age group encephalomyelitis b) Orthomaxovirus d) All of above
66. Predisposing factor which cause severe ILT a) Aftosa c) Pox virus
disease. b) Epidemic tremor d) Picorna virus
a) Defi. of Vit A c) Pink eye 82. Activity of longest projection
b) Excess ammonia in atmosphere d) Wattles disease a) Haemagglutination 90. Doyle‘s form mainly affect
c) Both a and b 75. Which system of our body primarily affected b) Nuraminidase a) Digestive tract
d) Defi of B complex in case of epidemic tremor c) Both a and b b) Respiratory tract
67. ILT virus mainly spread by which type a) Peripheral nervous system of young d) Ability of virus enveloped to fuse with c) Cardiovascular tract
a) Horizontal chick cell membrane. d) Genital tract
b) Vertical b) Central nervous system of young 83. Activity of smaller spikes 91. Beach‘s form of ranikhet disease mainly
c) Lateral chick a) Haemagglutination affect
d) All of above c) Musculoskeletal system of young b) Nuraminidase a) Respiratory system
68. Incubation period of ILT chick c) Both a and b b) Nervous system
a) 6 to 12d d) Respiratory system of young chick d) Ability of virus enveloped to fuse with c) Both of a and b
b) 3 to 4d 76. Signs seen in case of Epidemic tremor cell membrane. d) Digestive system
c) 20 to 24d a) Ataxia 84. How many form of RD is seen in poultry 92. Which form of RD is neurotropic velogenic
d) Not proper b) Paralysis flock a) Doyle‘s form
69. Characteristic feature of acute ILT c) Stunted growth a) 4 b) Beach form
a) Torticollis d) All of above b) 5 c) Hitchnner‘s
b) Dyspnoea 77. Causal organisum of epidemic tremor c) 3 d) Asymptomatic form
c) Convulsion a) Picorna virus d) 6 93. Which form of RD is pneumoencephalitic
d) Paralysis b) Pox virus 85. On base of virulence how many form of RD form
c) Adeno virus seen in poultry flock a) Doyle‘s form
d) Para maxovirus a) 4 b) Beach form
78. Which age group birds infected will show b) 5 c) Hitchnner‘s
70. Sign of ILT nervous sign c) 3 d) Asymptomatic form
a) Moist rales a) Upto 6 week d) 6 94. Which form of RD is a less virulent
b) Birds with wide open mouths and b) Upto 3 week 86. Which is a form of RD on base of virulence mesogenic form
gasping c) Upto 10 week a) Doyle‘s a) Doyle‘s form
c) Blood stain sputum d) All age group b) Lentogenic b) Beach form
d) All of above 79. Transmission of epidemic tremor occurs c) Beach‘s form c) Beaudett‘s
71. Incase of ILT lesions are mainly seen in though d) Asymptomatic d) Asymptomatic form
a) Upper respiratory tract a) Eggs 87. Other name of Doyle‘s form 95. Which form of RD is affect pigeon.
b) Lower respiratory tract b) Fomites a) Asiatic New Castle disease a) Doyle‘s form
c) Digestive system c) Mechanical carrier b) Typical Ranikhet disease b) Beach form
239 240
Dhruv N Desai Dhruv N Desai
c) Beaudett‘s d) IB c) Conjuctival b) Respiratory and urogenital
d) Asymptomatic form 104. Genus of the Gumboro disease virus d) Genital c) Respiratory and cardiovascular
96. Which stain of RD associated with a) Birnavirus 112. Vent pecking is commonly seen in- d) Urogenital and cardiovascular
Hitchner‘s form b) Paramaxovirus a) Pox
a) Lentogenic strain c) Orthomaxovirus b) IBD
b) Mesognic strain d) Orbivirus c) IB
c) Velognic strain 105. Which serotype of IBD affect chicken and d) RD 120. Which is most important natural host of
d) None of above cause pathogenicity 113. Post mortem lesion in Gumboro disease IB
97. Asymptomatic form is also known as a) Serotype 1 a) Haemorrage in thigh and leg a) Turkey
a) Enteric form b) Serotype 2 b) Haemorrage between proventiculus b) Chickens
b) Beach‘s form c) Serotype 3 and gizzard c) Pheasants
c) Doyle‘s form d) Serotype 4 c) Both a and b d) Quail
d) Hitchner‘s form 106. IBD exhibites in which form d) Haemorrage in proventriculus 121. Reproductive signs of IB in layer
98. Which type of sign seen in man in case of a) Acute 114. Which type of lesion seen on bursa in a) Shell less, misshaped eggs with watery
Ranikhet disease b) Chronic Gumboro disease contents
a) Orchitis c) Both a) Enlarged, inflamed b) Gasping
b) Intermittent fever d) None of above ,oedematous,creamcoloured c) Paralysis of wing and neck
c) Conjunctivitis 107. Which age group birds usually affected b) Atrophies ,3 to8 after symptoms d) Torticolis
d) Dermatitis with IBD started 122. 1st avian adenovirus isolated from:
99. Through which route virus enters the body a) More than 6 wk of age c) Both a and b a) Respiratory disease in quail ( quail
a) Respiratory b) 6 wk of age d) None of above bronchitis)
b) Intestinal c) Adults 115. Which type of necrosis seen in lymphoid b) Egg drop syndrome
c) Both route d) One day old bird follicle in case of IBD c) Turkey haemorragic enteritis
d) None of above 108. Which tissue is mainly affected with IBD a) Coagulative necrosis d) Haemorragic enteritis
virus b) Caseous necrosis 123. Inclusion Body Hepatitis usually which
a) Lymphoid tissues c) Fat necrosis group of birds
b) Nervous tissues d) Suppurative necrosis a) Broiler
100. Characteristic lesions in case of RD c) Cardiac tissues 116. Infectious bronchitis is belong to the b) Layer
a) Haemorrage in proventriculus d) All tissue family- c) Both
b) Haemorrage in caecal tonsils 109. Which organ is mainly affected with IBD a) Coronaviridae d) Dual purpose
c) Ulcerarion of the caecal tonsils virus b) Birnaviridae 124. Which condition is mainly associated with
d) All of the above a) Spleen c) Picornaviridae adenovirus in avian:
101. Nervous sign of Ranikhet disease b) Caecal tonsil d) Poxviridae a) IBH
a) Paralysis of wing and legs c) Bursa 117. Which organ is affected in case of b) EDS
b) Torticollis d) All of above infectious bronchitis c) Angara disease
c) Ataxia or circular movement a) Respiratory tract d) All of above
d) All of above b) Oviducts 125. Combine infection of which virus cause
102. Which factor able to aggravate post c) Kidney produce IBH infection
vaccinal reaction 110. Which cells is mainly affected in case of d) All of above a) IBD +IB
a) E.coli IBD infection 118. Main site of IB virus multiplication is b) IBD + RD
b) Mycoplasma gallispticum a) B-lymphocyte a) Digestive tract c) IBD + adenovirus
c) Low relative humidity b) T-lymphocyte b) Respiratory tract d) Adenovirus +RD +IB
d) All of above c) Both B and T lymphocytes c) Genital tract 126. Egg drop syndrome recognized in:
103. Synoname of infectious bursal disease d) Nerve cell d) Skin a) Netherland in 1976
a) Gumboro disease 111. Most important route of IBD infection 119. Which system of body mainly affected in b) Netherland in 1972
b) IBD a) Respiratory case of IB c) India 1976
c) Both a and b b) Oral a) Respiratory and Digestive d) India 1972
241 242
Dhruv N Desai Dhruv N Desai
127. EDSvirus agglutinates the RBCs of d) Decolouration of kidney a) Infectious stunting syndrome c) Swollen liver
a) Chicken 135. Which lesion is not a seen in CIA b) Inclusion body hepatitis d) Haemorragic enteritis
b) Duck a) Haemorrhage in skin, proventriculus, c) Marek‘s disease
c) Turkey on heart d) Avian influenza
d) All of above b) Atrophy of Thymus, Spleen, Bursa 143. Which is significant feature in stunted
128. EDS virus is naturally occuring in which c) Lymphoid depletion syndrome 150. Which serotype strain of MD is vary
birds d) Hydropericardium a) Enlarged liver markedly in pathogenicity
a) Ducks 136. Principle sites of CIA virus appears is: b) Atrophied bursa a) Sero type 1
b) Geese a) B cells c) Pale shanks b) Sero type 2
c) Both a and b b) Epithelial cells d) Swollen head c) Sero type 3
d) Fowl c) T cells 144. Osteodystropathies is common in broiler d) Sero type 4
129. 1st sign of manifestation of classsical EDS d) All of above chicken of more than 2 week of age. 151. Which age group chicks most commonly
is : 137. Which is not a immunosuppressive disease a) Infectious stunting syndrome affected with Marek‘s disease
a) Sudden fall in production that is occur a) CIA b) Inclusion body hepatitis a) 12 to 24 week
around peak egg production b) MD c) Marek‘s disease b) 2 week
b) Thin shell eggs c) IBD d) Avian influenza c) Upto 6 week
c) Misshaped eggs d) Avian pox disease 145. Characteristic feature of lymphoid d) One day old
d) All of above 138. Aplastic pale bonemarrow is seen in which leucosis: 152. MATSA is
disease of poultry. a) Enlargement of liver by diffuse or a) Marek‘s disease associated tumor
a) MD nodular infiltration of lymphoblasts. specific antigen
b) CIA b) Swollen liver b) Marek‘s disease associated tumor
130. Ascitic syndrome consist: c) Pox virus c) Swollen head specific antibody
a) Right ventricle failure d) RD d) Haemorragic enteritis c) Marek‘s disease associated thymus
b) Pulmonary hypertension 139. In which age group birds not seen any 146. Large number of immature red cells are specific antigen
c) Portal hypertension clinical sign in CIA presents in the blood d) Mucosal disease associated tumor
d) All of above a) Chick a) Lymphoid leucosis specific antigen
131. Some time outbreak of Ascities may b) Adults broiler b) Myeloblastosis 153. Bilateral paralysis of legs is character of
follow which disease c) Adults layer c) Myelocytomatosis which form:
a) IB d) None of above d) Erythroid leucosis a) Visceral form
b) IBD 147. In which condition seen Morocco leather b) Classical form
c) Respiratory Aspergillosis appearance in liver. c) Ocular form
d) RD a) Lymphoid leucosis d) Cutaneous form
132. Angara disease also called as 140. Other name of Infectious stunting b) Myeloblastosis 154. Cauliflower like ovary is character of
a) Hydropericardium syndrome syndrome c) Myelocytomatosis which form:
b) Hydroperitonitis syndrome a) Pale birds syndrome d) Erythroid leucosis a) Visceral form
c) Encephalitic disease b) Mal absorption syndrome 148. Lymphopoliferative disease of domestic b) Classical form
d) Muscular disease c) Infectious runting chicken c) Ocular form
133. Angora disease 1st reported in: d) All of above a) IBH d) Cutaneous form
a) India 1987 141. From these which is not a syndrome of b) New castale disease 155. Pearly eyes is a characteristic of which
b) Pakistan 1987 Infectious stunting syndrome c) MD form:
c) Sri Lanka 1987 a) Runting and stunting syndrome d) IB a) Visceral form
d) Bangladesh 1987 b) Helicopter feathering 149. Characteristic feature of ofmarek‘s b) Classical form
134. Which lesion is seen in Chicken c) Osteoporosis disease c) Ocular form
infectious anemia d) None of above a) Mononuclear cell infiltration of d) Cutaneous form
a) Anemic lesion 142. Broken and displaced primary feathers on peripheral nerves and also in various 156. Leukotic lesions in the skin are common.
b) Hypertrophy of kidney the head and neck (Helicopter feather) is viscera a) Visceral form
c) Mucous in trachea feature of : b) Helicopter feather b) Classical form
243 244
Dhruv N Desai Dhruv N Desai
c) Ocular form 26. b 73. d 120. b
d) Cutaneous form 27. a 74. b 121. a
157. Enlarged sciatic nerve is characteristic of 28. a 75. b 122. a
which disease 29. c 76. d 123. a
a) Mucosal disease 30. a 77. a 124. d
b) Encephalitis 31. c 78. a 125. c
c) Lymphoid leucosis 32. a 79. d 126. a
d) Marek‘s disease 33. b 80. c 127. d
**************************************** 34. d 81. a 128. c
**************************************** 35. a 82. c 129. a
***** 36. a 83. d 130. d
37. c 84. b 131. c
38. d 85. c 132. a
39. c 86. b 133. b
40. c 87. d 134. a
41. b 88. a 135. d
42. a 89. d 136. c
43. a 90. a 137. b
44. d 91. c 138. b
ANSWERS 45. c 92. b 139. b
46. a 93. b 140. d
47. b 94. c 141. d
1. a 48. c 95. c 142. a
2. b 49. d 96. a 143. c
3. c 50. d 97. a 144. a
4. c 51. a 98. c 145. a
5. a 52. b 99. c 146. d
6. a 53. c 100. d 147. b
7. a 54. d 101. d 148. c
8. b 55. d 102. d 149. a
9. a 56. a 103. c 150. a
10. b 57. b 104. a 151. a
11. a 58. c 105. b 152. a
12. a 59. d 106. a 153. b
13. c 60. d 107. b 154. a
14. a 61. a 108. a 155. c
15. d 62. b 109. d 156. d
16. b 63. c 110. a 157. d
17. a 64. a 111. b ****************************************
18. b 65. a 112. b *************************************
19. c 66. c 113. c
20. c 67. c 114. c
21. d 68. a 115. a
22. c 69. b 116. a
23. d 70. d 117. d
24. d 71. a 118. b SET-9
25. c 72. c 119. b
245 246
Dhruv N Desai Dhruv N Desai
1. Inflammation of seminal vesicles is called b) Coxitis d) Jaundice d) Folliculitis
as- c) Gonotis 18. Hassel‘s corpuscles are present in- 26. Abscess of hair follicle is called as-
a) Funiculitis d) Arthritis a) Thymus a) Acne
b) Orchitis 10. Inflammation of stifle joint is called as- b) Thyroid b) Boil
c) Seminal vesiculitis a) Om arthritis c) Parathyroid c) Carbuncle
d) Scirrhous cord b) Coxitis d) Pineal d) Folliculitis
2. Inflammation of glans penis is called as- c) Gonotis 19. Thickening of the epidermis due to 27. Cluster of boils situated close to each
a) Balanitis d) Arthritis hyperplasia of the cells of malpighian other, opening on to the skin through
b) Posthitis 11. Chronic arthritis may be seen in fowl in- layer is known as- several pores is known as-
c) Balanoposthitis a) Fowl typhoid a) Bulla a) Acne
d) Phimosis b) Fowl cholera b) Hyperkeratosis b) Boil
3. Twisting of neck with an unnatural c) Fowl pox c) Dyskeratosis c) Carbuncle
position of the head is called as- d) IBD d) Acanthosis d) Furuncle
a) Torticollis 12. Inflammation of bursa (joint) between 20. Abnormal thickening of stratum 28. Allergic condition which is characterized
b) Scoliosis ligamentumnuchae and atlas/axis is called granulosum layer is called as- by appearance of wheals on the skin is-
c) Kyphosis as- a) Bulla a) Acne
d) Lordosis a) Poll evil b) Hyperkeratosis b) Boil
4. Bow legs condition seen in- b) Fistulous withers c) Dyskeratosis c) Carbuncle
a) Rickets c) Navicular disease d) Acanthosis d) Urticaria
b) Osteoporosis d) Infectious synovitis 21. Thickening of skin in which all layers of 29. Which is also called as ‗wattle disease‘ in
c) Osteomalacia 13. Inflammation of bursa (joint) between skin is affected is called as- fowl-
d) None of the above ligamentumnuchae and thorasic spine is a) Pachyderma a) Fowl coryza
5. Test for detection of alkaline phosphatase called as- b) Acanthosis b) Fowl plague
in serum in osteoporosis is called as- a) Poll evil c) Lichenification c) Fowl cholera
a) Rothra‘s test b) Fistulous withers d) Erosion d) Fowl typhoid
b) Fouchet‘s test c) Navicular disease 22. Intracellular edema of epidermis is called 30. Which of the following statement is
c) Izuka‘s test d) Infectious synovitis as- correct-
d) Any of the above 14. Bursitis of carpal joint is called as- a) Scales a) Demoid cyst contain skin appendages
6. Inflammation of bone marrow is called as- a) Poll evil b) Papule b) Epidermoid cyst contain skin
a) Osteitis b) Bog spavin c) Wheal appendages
b) Periostitis c) Hygroma d) Spongiosis c) Equine sarcoid is caused by herpes
c) Spondylitis d) Synovitis 23. Pustular dermatitis caused by virus
d) Osteomyelitis 15. Bursitis of hock joint is called as- Staphylococcus sp. is known as- d) Ranikhet disease is caused by DNA
7. Which of the following disorder of a) Poll evil a) Seborrhea virus.
musculoskeletal system doesn‟t causes b) Bog spavin b) Impetigo 31. Constriction of pupil is known as-
lameness- c) Hygroma c) Urticaria a) Mydriasis
a) Ring bone d) Synovitis d) Eczema b) Myiosis
b) Spavin 16. Example of acute non-suppurative 24. Inflammation of sebaceous gland is called c) Choroid
c) Splint myositis is- as- d) None of the above
d) Laminitis a) Black quarter a) Acne 32. The condition in which lens of eye become
8. Inflammation of hip joint is called as- b) Strangles b) Boil opaque is called as-
a) Om arthritis c) Glanders c) Carbuncle a) Cataract
b) Coxitis d) All of the above d) Urticaria b) Coloboma
c) Gonotis 17. Death in myoglobinuria is due to- 25. Inflammation of hair follicle is called as- c) Entropion
d) Arthritis a) Asphyxia a) Acne d) Strabismus
9. Inflammation of shoulder joint is called as- b) Renal insufficieny leading to uremia b) Boil 33. Inflammation of eye lids is known as-
a) Om arthritis c) Toxemia c) Carbuncle a) Trichiasis
247 248
Dhruv N Desai Dhruv N Desai
b) Blepharitis b) Listeria sp. 49. Oily nasal discharge is feature of- 22. d
c) Keratitis c) Salmonella sp. a) Strangles 23. b
d) Hordeolum d) Bacillus sp. b) Glanders 24. a
34. Abscess formation in meibomian glands is 42. The condition in which there is increase in c) IBR virus 25. d
called as- intraocular pressure is known as- d) All of the above 26. b
a) Chalazion a) Cataract 50. Punched out ulcers in lungs are seen in- 27. c
b) Hordeolum b) Anterior synechia a) Strangles 28. d
c) Pannus c) Luxation b) Glanders 29. c
d) Stye d) Glaucoma c) Tuberculosis 30. a
35. Infectious keratoconjuctivitis in cattle is 43. Iridocyclitis in fowl is seen in- d) Pasteurellosis 31. b
caused by- a) ALC 32. a
a) Staphylococcus aureus b) IBD ****************************************
33. b
b) Pseudomonas aeruginosa c) MD **************************************
34. a
c) Morexellabovis d) RD ANSWERS 35. c
d) Pasteurellatularensis 44. Appearance of giant cell is pathognomonic 1. c 36. d
36. Inflammation of orbit is called as- of- 2. a 37. b
a) Cellulitis a) Anthrax 3. a 38. a
b) Orbitis b) Tuberculosis 4. a 39. b
c) Dacryoadenitis c) Listeriosis 5. c 40. c
d) Orbital cellulitis d) Pasteurellosis 6. d 41. a
37. Abscess formation of the follicles of an 45. Animal most susceptible for tuberculous 7. c 42. d
eyelid is known as- meningitis is- 8. b 43. c
a) Chalazion a) Swine 9. a 44. b
b) Hordeolum b) Cattle 10. c 45. b
c) Pannus c) Equine 11. b 46. c
d) Trichiasis d) Fowl 12. a 47. b
38. Infectious keratoconjuctivitis in sheep is 46. Animal most susceptible for tuberculous 13. b 48. a
caused by- osteomyelitis is- 14. c 49. b
a) Rickettsia conjuctiva a) Swine 15. b 50. b
b) Pseudomonas aeruginosa b) Cattle 16. a
c) Morexellabovis c) Equine ****************************************
17. b **************************************
d) Pasteurellatularensis d) Fowl 18. a
39. Infectious keratoconjuctivitis in fowl is 47. Lesions caused by avian strain of 19. d
caused by- tuberculosis in the intestines of horse 20. b SET-10
a) Rickettsia conjuctiva resembles- 21. a
b) Ricloasiaconjuctivae a) Lesions of intestines in hog cholera d) Vitamin K
c) Morexellabovis b) Lesions of intestines in Johne‘s disease 1. Fluke causing diseases of veins is- 3. Gout mostly occur in-
d) IBR virus c) Lesions of intestines in salmonellosis a) Schistosomaspindalis a) Cattle
40. Inflammation of iris and ciliary body d) Lesions of intestine in pullorum b) Fasciola hepatica b) Horses
called as- disease c) Paraamphistomumcervi c) Dogs
a) Anterior uveitis 48. Epitheloid cells may fuse to form syncytial d) Oesophagostomum spp. d) Fowl
b) Iridocyclitis mass termed as ‗symplasma stage‘ is seen 2. Which vitamin deficiency causes 4. Pale infract is not seen in-
c) Both of the above in- metaplasia of conjunctiva and cornea- a) Lung
d) None of the above a) Paratuberculosis a) Vitamin C b) Liver
41. Periodic ophthalmia or equine recurrent b) Tuberculosis b) Vitamin B c) Spleen
iridocyclitis is caused by- c) Actinomycosis c) Vitamin A d) Heart
a) Leptospira sp. d) Actinobacillosis
249 250
Dhruv N Desai Dhruv N Desai
5. Commonest site of metastasis is- 13. Chronic inflammation of Spermatic a) Haematoma b) By coitus
a) Lung cord is called as- b) Cavernous angioma or tumour of c) Both
b) Brain a) Scirrhous cord newly formed blood vessel d) Respiratory
c) Liver b) Funiculitis c) Mass of dilated previously existing 29. Which one of these findings would be of
d) Kidney c) Spermatitis blood vessels greatest assistance in establishing a
6. Epitheloid cell is modified- d) Corditis d) Cancer metastasis diagnosis
a) Lymphocyte 14. For histopathology of tissues they are 22. Proctitis is inflammation of- of enterotoxaemia in a sheep found dead?
b) Macrophage fixed in- a) Anus a) A fibrin clot in the pericardial sac and
c) Monocyte a) 10% formalin b) Prostate gland autolysed kidneys
d) Eosinophil b) 10% nitric acid c) Perineal gland b) Many large gram negative rods
7. Line of Zahn seen in- c) 5% formalin d) Caecum arranged singly in smears of the
a) Postmortem clot d) Distilled water 23. The accumulation of purulent exudates in mucosa of the small
b) Infarct 15. For histopathology of Bone or calcified the body cavity is known as- intestine
c) Embolus tissue they are decalcified in- a) Hydrothorax c) Severs acute pulmonary oedema
d) Thrombus a) 5% sodium nitrate b) Pneumothorax d) Cl. perfringens type D toxin in the
8. Anthrax in camel characteristically b) 5% nitric acid c) Chylithorax small intestine as determined by
shows- c) 10% formalin d) Empyema ELISA tests
a) Hemorrhagic enteritis d) 5% sulphuric acid 24. The primary pathological lesion produced 30.Which one of the following organisms is
b) Extremely enlarged liver 16. Tubercular nodules doesn‟t calcified by Brucellaovis infection in rams is: frequently isolated from lesions
c) Lymphadenitis in- a) Seminal vesiculitis resembling
d) S/c edema along with ventral part a) Pig b) Epididymitis tuberculosis in the submaxilliary lymph
of body b) Cattle c) Orchitis nodes of pigs?
9. Rinderpest produces following type of c) Dog d) Balanoposthitis a) Streptococci Group E
enteritis- d) Fowl b) Staphylococcus aureus
a) Fibrinous 17. Tumor of enamel tissue of tooth is 25. In the central nervous system, c) Pasteurellamultocida
b) Hemorrhagic called as- oligodendroglia are primarily d) Rhodococcus (Corynebacterium) equi
c) Serous a) Admentinoma concerned with: 31. Sleepy foal disease is an acute highly
d) Suppurative b) Odontoma a) Initiation of nervous impulses fatal septicaemia of new born foals
10. Kidney worm found in cysts in c) Teratoma b) Regulation of fluid and electrolyte characterize
perirenal tissue- d) None of the above. balance by kidney micro abscesses. The causative
a) Single worm 18. Which of the following is not a tumor- c) Formation and maintenance of myelin organism is:
b) Always in pair a) Teratoma d) Phagocytic activity a) Actinobacillusequuli
c) Always in two pair b) Mastocytoma 26. The characteristic muscle lesion of b) Rhodococcus (Corynebacterium) equi
d) Not present in peri-renal tissue, c) Granuloma blackleg (Clostridium chauvoei) is: c) Streptococcus equi
present in kidney d) Lipoma a) Necrotizing myositis d) Salmonella typhimurium
11. Which of the following is 19. Lesions seen in Liver in Fowl cholera b) Degenerative myopathy
incorrectlymatched- are- c) Muscular hypertrophy 32. Infectious avian encephalomyelitis virus
a) Glycogen- Best Carmine a) Severe ecchymotic hemorrhage d) Intestinal oedema with no muscle causes disease with nervous signs in
b) Amyloid- Best Carmine b) Multiple pin point necrotic foci lesion domestic
c) Fat- Sudan IV c) Severe large infracts on liver 27. Fatty change mainly affects the: fowl:
d) Amyloid- Congo Red d) None of the above. a) Nucleus a) 1-4 weeks of age
12. Mycoplasma in poultry causes- 20. Typical Lesion of Black Quarter- b) Cytoplasm b) 12-18 weeks of age
a) Fowl typhoid a) Crepitating sound of thigh muscle c) Nucleolus c) older than 25 weeks
b) Fowl cholera b) Wooden tongue d) Mitochondria d) of any age provided that they are not
c) Fowl plague c) Cysts in muscles 28. Themostimportant method of spread of immune
d) CRD d) Muscular dystrophy Brucellaabortus among cattle is:
21. Telangiectasis is a a) Ingestion
251 252
Dhruv N Desai Dhruv N Desai
33. In dogs with hepatocellular necrosis, c) Proliferative enteritis d) Hydropercardium syndrome. 15. b
which is most likely to be increased d) Proliferative abomasitis 48. Turkish Towel appearance of crop with 16. d
in serum? 41. In dogs with chronic kidney disease and ulcers are seen in- 17. a
a) Albumin renal secondary a) Candidiasis 18. c
b) Bilirubin hyperparathyroidism, all are b) Aspergillosis 19. b
c) Alkaline phosphatase (ALP) increased EXCEPT: c) Histoplasmosis 20. a
d) Alanine aminotransferase (ALT) a) Serum calcitriol d) Mycoplasmosis. 21. c
34. Which causes secondary absolute b) Serum phosphorus 49. White mouldy growth with accumulations 22. a
appropriate erythrocytosis? c) Urine protein:creatinine of cheesy material in air sac are 23. d
a) Dehydration d) Serum parathyroid hormone levels characteristic lesion of- 24. b
b) Polycythemia vera 42. In BAL fluid from sheep infected with a) Candidiasis 25. c
c) Splenic contraction Maedi-Visna virus, which increases b) Aspergillosis 26. a
d) Right-to-left cardiac shunt with disease c) Histoplasmosis 27. a
35. A morphologic feature of autophagy is: severity? d) Mycoplasmosis. 28. c
a) Organelle swelling a) Percentage of eosinophils 50. Microabscess in brain seen in- 29. d
b) Nuclear fragmentation b) Percentage of neutrophils a) Salmonellosis 30. a
c) Pericellular neutrophils c) Percentage of lymphocytes b) Fowl cholera 31. a
d) Cytoplasmic intravacuolar whorls d) Percentage of macrophages c) Rabies 32. a
36. In a 12-week-old rat with multifocal 43. Disease of poultry which is not caused by d) Listeriosis. 33. d
lymphohistocytic interstitial virus is- ************************************ 34. d
pneumonia and a) Chronic respiratory Disease ***************************** 35. d
perivascular lymphoid cuffing, the most b) Infectious Bronchitis ******* 36. c
appropriate diagnosis is: c) Fowl Pox 37. c
a) Sendai viral infection d) Ranikhet Disease. ANSWERS 38. c
b) Rat coronaviral infection 44. Canine have more _______________ 39. d
c) Rat respiratory viral infection than lymphocytes- 1. a 40. d
d) Mycoplasma pulmonisinfection a) Monocytes 2. c 41. a
37. In lambs, which one is most affected in b) Eosinophils 3. d 42. c
Type O Foot and Mouth Disease? c) Neutrophils 4. a 43. a
a) Feet d) Basophils 5. a 44. c
b) Teats 45. In rotavirus infection sample collected in 6. b 45. a
c) Heart sterile vial for confirmation is- 7. d 46. b
d) Mouth a) Fecal sample 8. d 47. a
38. An effector caspase is: b) Serum 9. a 48. a
a) Caspase 1 c) Lung 10. b 49. b
b) Caspase 2 d) None. 11. b 50. d
c) Caspase 3 46. Hydropic degeneration leads to 12. d
d) Caspase 4 ____________ formation. 13. a ****************************************
39. Chlamydophilacaviaein guinea pigs a) Papule 14. a *************************************
causes: b) Vesicle
a) Fibrinous polyarthritis c) Pustule
b) Necrotizing placentitis d) Scab.
c) Granulomatous enteritis 47. Watery yolk with caseous material in
1. Punched ulcers in abomasums of cattle are c. Haemonchus contortus
d) Seropurulent conjunctivitis bronchi is seen in- d. Ostertagia ostertagi
characteristic of :
40. In cattle, Cryptosporidiaandersonicauses: a) IB 2. Typical sign of ripened abscess is
a) Ulcerative colitis b) ILT a. Theilaria annulata
b) Necrotizing enteritis c) EDS-76 b. Babesia bigemina a. Swelling
253 254
Dhruv N Desai Dhruv N Desai
b. Pain a. Rumen 15. Skim milk powder is a byproduct of c) Decrease appreciably

c. Fluctuation b. Reticulum d) remain with-in normal range


a) Industrial
d. Pointing c. Omasum 21. Which of the following is a circulating
b) Grain blood cell that is capable of differentiating
3. Operation flood I was launched during d. Abomasum into a plasma cell?
c) Cereal
(a) 1960 9. Blood is added in blood medium agar at a. Neutrophil
following concentration: d) Roughage b. Basophil
(b) 1970
c. B lymphocyte
a) 5-10 % d. T lymphocyte
(c) 1980 b) 15 % 22. After ovulation, which of the following is
(b) 1982. c) 20 % most important for the rapid movement of
d) 2 % the oocyte to the ampular-isthmic junction
4. When world environmental day 10. The 5 carbon sugar compound present in of the oviduct?
celebrated….. DNA molecule is: 16. East coast fever in cattle is caused by :
a) 15th June a. Erythrose a. Fluid production by the isthmus
b) 5th July b. Deoxyribose (a) Theileria b. Cillilary beating in the ampulla
c) 15th July c. Ribose (b) Trypanosoma c. Muscle contractions in the ampulla
d) 5th June d. Ribulose (c) Babesia d. The presence of cumulus cells
5. Right side displacement of abomasums is 11. Study of birds which are not classed as (d) None surrounding the oocyte at
usually caused after 17. Wound does not heal is known as ovulation.
poultry is known as
23. Which is the hardest substance in the
a. Poultry Science a. Maggot wound
a) Immediately postpartum b. Ornithology animal body?
c. Bird Science b. Ulcer
b) 2-4 weeks postpartum a. Bone
d. Poultry Production
c) During gestation 12. Inhibition of aggregation of platelets c. Infected wound
b. Cartilage
d) 9-12 months postpartum a) Aspirine d. Contaminated wound
6. The term that refers to the percentage of c. Enamel
packed erythrocytes per unit volume of b) Urokinase 18. 73rd amendment act 1992 was introduced
d. Dentine
blood is the: to strengthen
c) Thromboxane A2
a. Differential Count a Dairy development
b. Hemoglobin d) Streptokinase
c. Hematocrit b Community development
d. Hemopoiesis 13. Gajrai grass is the fodder grass belong to 24. Nucleic acid present in virus
7. Which of the following enzymes are present group c Panchayat development
in the acrosome of the bovine sperm? a. Seasonal a) DNA
b. Annual d Co-operative development
a. Trypsin
b. Adenylate cyclase c. Perennial b) RNA
19. CFC‘s are responsible for depletion of…..
c. Phospholipase C (PLC) d. None
14. Which of the following will be the number a) Ozone c) Either DNA or RNA
d. Acrosin
b) Oxygen
of chromosomes in the sperm of a
c) Carbon d) Both DNA and RNA
Sahiwal bull? d) Nitrogen
20. Serum calcium and phosphorus 25. The synthesis of new DNA strand on
a) 30
concentration in nutritional dystrophies may template strand takes place in the direction
b) 60 of:
a) Increase above normal level a. 3‘-5‘
c) 50 b. 5‘-3‘
8. Which of the following is the smallest b) decrease above normal level c. In both the directions
compartment of the ruminant stomach? d) 25
d. In 3‘-5‘ on leading strand and in 5‘-3‘ a. Cervix d. Straw making
direction on lagging strand b. Uterine body 44. The strength of selection is expressed as
26. Study of birds which are classed as poultry c. Uterine horn, at the greater curvature
is known as (half way up the uterine horn) (a) Coefficient of selection
32. The main cause of death in case of burn d. At the distal tip of the uterine horn (b) Response to selection
a. Poultry Science
b. Ornithology during latter stage is 38. Which is the largest foramen in the skull? (c) Selection differential
c. Bird Science (d) None
a. Hypovolemia a. Foramen magnum 45. Inflammation of lymph node is called as-
d. Poultry Production
27. Verapamil blocks transport of b. Blood loss b. Supraorbital foramen (A) Lymphangitis
a) Sodium ion c. Asphyxia
c. Infra orbital foramen (B) Lymphadenitis
b) Chloride ion d. Secondary infection
d. Mantel foramen (C) Typhilitis
c) Calcium ion 33. Ranching is common practice in
39. Short hair like projections for attachment (D) Both (A) and (B).
d) Potassium ion (A) India and genetic transfer in bacteria are:
46. Example of NPN is
28. Subabul is the grass originated in the (B) Australia a) Flagella
country
a. Maxico (C) Japan b) Fimbria a) Albumin
b. U. S. A.
(D) USA c) Plasmid b) Amino acid
c. Brazil
d. India c) Prolamine
29. Genotype of purebred Rose comb hen 34. Humidity is measured by….. d) All
should be a) Luxmeter d) Lignin
40. The autonomously replicating extra-
b) Dry bulb thermometer
c) Wet bulb thermometer chromosomal double stranded DNA molecule
a) RrPp 47. Example of anaerobic protozoa is :
d) Chlorinometer present in bacteria is called:
b) RRPp 35. Increase survival rate of S. pullorum a. Plasmid (a) Leishmania
infected chick embryo is achieved by b. Episome (b) Trichomonas
c) RRpp drug c. Phage (c) Trypanosoma
d. Transposon (d) None of the above
d) rrpp a) Chloramphenicol 41. Modern chicken are descendents of the
which following wild species
30. Example of protein is b) Furazolidon a. Gallus soneratti
b. Gallus lafeyetti
c) Oxytetracycline c. Gallus varius 48. Last stage of wound healing is
a) Peptide
d. Gallus gallus
d) Colistin a. Wound contraction
b) Amine 42. The most appropriate anticoagulant used
36. The component of plasma responsible for for collection of blood for blood glucose b. Epithelization
c) Amino acid maintaining the osmotic pressure of blood estimation
c. Fibroplasia
d) Glutamine is:
a) Sodium EDTA
d. Vasodilatation
31. Caecal coccidiosis in fowl is due to which a. Plasmin
b) Sodium fluoride
b. Albumin
of the following : 49. The approach in which people have their
c. Fibrinogen c) Heparin
d. Gamma globulin say in programmed planning is
(a) Eimeria precox
(b) Eimeria mutis 37. Following artificial insemination in the d) Sodium oxalate (A) Democratic
(c) Eimeria tenella cow, increased retrograde flow will occur 43. Swath curing is the method apply to
(d) Eimeria brunetti if semen is placed in which of the a. Cutting the crop (B) Authoritative
following locations? b. Hay making
c. Silage making (C) Directive
(D) Laissez-faire d) 1% a) Ringers lactate

50. The metal should not be used for storage 56. The scientists associated with discovery of 62. Disease of poultry which is not caused by b) Normal saline
of rain water is….. ―restriction endonucleases‖: virus is-
a) Iron c) Dextrose saline
a. Lederberg (A) Chronic respiratory Disease
b) Galvanized iron b. Kelly & Smith d) Dextrose saline 20%
c) Lead c. Dulbecco
d) Copper (B) Infectious Bronchitis
d. Korenberg 68. Excessive destruction of erythrocytes is
51. Virion of avian infectious bronchitis has a 57. National Institutes of Nutrition (NIN) to (C) Fowl Pox characteristic of:
shape of Government of India has recommended
eggs and poultry meat consumption per (D) Ranikhet Disease. a. Thalassemia
a) Globules with cilia b. Aplastic anemia
capita per annum are 63. Wild game is reservoir host for which c. Pernicious anemia
b) crown like projection a. 18 eggs and 9 kg of poultry meat parasite : d. Hemolytic anemia
b. 180 eggs and 0.90 kg of poultry meat 69. Which of the following are examples of
c) Oval shaped body c. 180 eggs and 9.0 kg of poultry meat (a) Trypanosoma evansi the maturation function in the epididymis?
d. 180 eggs and 11.0 kg of poultry meat (b) Taenia a. Removal of protoplasmic droplets
d) elliptical round body 58. Receptors of CTZ stimulated by centrally (c) Entamoeba histolytica b. Concentration of sperm
52. Plasma is: acting emetics are (d) Coccidia c. Secretion of glycoproteins by principle
cells
a. Blood that has no red blood cells a) α1 d. Contractions of smooth muscle
b. The liquid portion of blood including 70. Which of the following is tendineous
b) α2
the clotting factors linear sheet present at the abdominal floor?
c. The liquid portion of blood minus the 64. The best treatment of fistula is
c) 1
clotting factors a. Linea alba
a. Antibiotics
d. The proteins of blood d) Dopamine
53. The concentration of sperm in the b. Rectus abdominis
b. Antibiotics and corticosteroids
ejaculate would be highest among which 59. For ensiling the fodder crop should contain
of the following farm animal species? c. Fascia
moisture percent c. Surgical removal
a. Bull
a. 40 to 50 d. Tendoachillis
b. Ram d. Counter irritants
b. 50 to 60
c. Boar
c. 65 to 70 71. Agar is a source of carbohydrate to:
d Stallion 65. In IRDP, SC/ST and physical handicapped
d. 70 to 75
54. Which of the bone is the hardest bone in beneficiaries are provided subsidy @ a) Most of the bacteria
60. What will be the phenotypic ratio in
the animal body?
progenies from a cross between creeper Percent b) Only a few bacteria
a. Femur and normal fowl
(A) 10 c) None of the bacteria
b. Tibia a) 1:1
d) All the bacteria
(B) 25
c. Petrous temporal b) 3:1
(C) 33
d. Humerus c) 9:7 72. The acquisition of DNA molecule by
(D) 50 bacterial cells from environment is called:
55. The term antibiotic was first used by: d) 2:1
66. Among following is the safest source of a. Transformation
61. DCP content of Guinea grass b. Transduction
a) Domagk drinking water…..
c. Conjugation
a) Shallow well d. None
b) Fleming b) Surface water
a) 50%
c) River
c) Waksman
d) Deep well 73. First poultry show was held at Boston in
b) 5%
d) Robert Koch 67. If vomition due to metabolic acidosis the year
c) 15% suggested fluid therapy is a. 1838
b. 1870
c. 1861 b. Umbilical ligament 93. Fragmentation of nucleus in a cell is
d. 1849 termed as-
74. Agar acts as a 80. Typical sign of moist gangrene of tail is c. Supraspinatous ligament
(A) Pyknosis
a) Cathartics a. Erected hairs d. Caudate ligament
(B) Karyorrhexis
b) Emollient purgative b. Immobility of tail 87. Bacteria that grow at 50-550 C are known
as: (C) Karyolysis
c) Bulk purgative c. Swelling
a) Psychrophiles (D) Chromatolysis
d) Osmotic purgative d. Cold to touch
b) Mesophiles 94. Concentrate is a feed containing more than
75. Mango seed kernel is the byproduct of 81. In First five year plan ------------- c) Thermophiles % TDN
industry programmed was started for development d) Halophiles
a. Canning industry of animal Husbandry common alone a) 20
b. Oil industry
c. Gluten industry (A) Operation flood I b) 10
d. None
(B) Key village scheme 88. The ability of the cell to acquire DNA c) 60
76. Proportion of Roan polled progenies from
from environment is called:
a cross between Red polled (Pp) and white (C) ICDP d) 30
a. Competence
polled (Pp) cow will be
b. Compatibility
(D) Gosadan c. Interference 95. Triangular and pyriform apparatus like
a) 0.25 cooked rice grain present in egg of :
d. None of the above
82. Causative agent of undulant fever is…..
b) 0.50 89. Per capita availability of poultry eggs and
a) Br. abortus (a) Moniezia sp.
meat are respectively
b) Br. melitensis (b) Hymenolepsis nana
c) 0.75 a. 44 nos. and 17.6 kg.
c) Br. suis (c) Dipylidium sp
d) Br. Equi b. 176 nos. and 44 kg. (d) Taenia sp.
d) 0.60 c. 44 nos. and 1.76 kg.
83. Coughing up of blood is termed as
d. 176 nos. and 4.4 kg
77. Bronze discolouration of Liver in poultry 90. Acid rebound effect is observed with
a) Haematemesis
is caused by-
b) Epistaxsis a) Sodium bicarbonate
(A) Pasteurellosis
c) Metrorrhagia b) Sodium citrate
(B) IBD Virus
d) Haemoptysis c) Sodium chloride 96. Common site of occurrence of haematoma
(C) Leptospirosis
in dog is
84. A hematocrit of 80 would be considered: d) Potassium iodide
(D) Salmonellosis
a. Ear
a. Polycythemia 91. Rotational stocking is the method of
78. Rich source of carbohydrates a. Storage of feed
b. Anemia b. Eyelid
c. Thrombocytopenia b. Feeding of animal
d. Leukemia c. Grazing of animal c. Tail tip
a) Barley 85. The ATP produced by sperm goes towards d. Management of pasture
b) Bone meal which of the following? 92. The cross over percentage ranges between d. Digit
c) Cotton seed a. Motility
d) Dub grass b. Maturation of sperm a) 80 -100 %
79. Tape worm increases in which of the c. Lysis of corona radiate
d. Transcription and translation b) 50 -100 %
following :
86. Which of the following is the longest
c) 0- 50 % 97. Prime goal of state department of animal
(a) Liver ligament in the animal body?
husbandry and dairying is to provide -------
(b) Caecum d) 50 – 80 % - To the farmers
(c) Small intestine a. Broad ligament
(d) None
(A) Subsidy d) 15 to 20% 116. All of the following conditions impair
104. First Genome sequenced was of coagulation except:
(B) Input a. Bacteriophage λ 111. All domestic animal is definitive host for
b. Bacteriophage φ X 174 which of the following : a. Vascular spasm
(C) Service b. Vitamin K deficiency
c. Haemophilus influenza
d. Homo sapiens (a) Moniezia sp. . c. Severe hypocalcaemia
(D) Semen (b) Hymenolepsis nana d. Liver disease
105. Total contribution of poultry production to
(c) Dipylidium sp 117. Spherical bodies, weighing 0.45 – 0.9 kg,
98. The causative agent of hydatidious is….. the National GDP of India is nearly
(d) Taenia sp. attached to the placenta of a normal calf
a) Echinococcus granulosa a. 1.0 % comprising of an outer skin enclosing a
b) Diphylobothrium spp. b. 0.1 % mass of adipose connective tissue is
c) Taenia solium c. 10 % known as
d) Taenia saginata d. 8 % a) Perosomus elumbis
99. Ptyakusn is 106. Which one of the following is a rate b) Amorphus globosus
limiting step in adrenaline synthesis? c) Otter calf
a) Excessive secretion from brunners gland
d) Schistosomus reflexus
a) Tyrosine to DOPA 118. Which one of the following is the longest
b) Excessive salivation 112. The best suture material for peritoneum in
b) DOPA to Dopamine dog is nerve in the animal body?
c) Decrease secretion from salivary gland
c) Dopamine to Nor-adrenaline a. Catgut # 1 a. Vagus
d) Decrease secretion of succus entericus
d) None of the above b. Catgut # 1/0 b. Sciatic
100. During hemoglobin recycling in the
spleen, heme is initially converted into: 107. The synonym of Anjan grass is c. Catgut # 2 c. Femoral
a. Dhaman d. Median
a. Bilirubin b. Shevari d. Catgut # 3
b. Stercobilin c. Ghamar
113. Programmed is the statement of situation - 119. Disposable articles are best sterilized by:
c. Urobilin d. Jinjavo
d. Urobilinogen 108. Slow and fast feathering trait in poultry is ---- problem and solution a) Hot air oven
101. Failure of the blood testis barrier would
directly prevent a) Sex linked (A) Physical resources b) Autoclave
a. Stem cell renewal. b) sex limited trait
b. Spermatocytogenesis. (B) Objectives c) Gamma radiation
c) sex influenced
c. Meiosis.
d) autosomal (C) Planning
d. Spermiogenesis. d) Alcohol
102. Which of the following is unpaired 109. Infectious Necrotic Hepatitis in sheep is
caused by- (D) Goal
skeletal muscle present in the animal body?
(A) Leptospira sp. 114. Sellar‘s staining technique is used for
a. Diaphragm 120. First bacterial genome sequenced was of
diagnosis of…..
(B) Fasciola hepatica a) Rabies a. Salmonella typimurium
b. Bicepbrachii b. Bacillus anthracis
b) Brucellosis
(C) Clostridium sp. c) Listeriosis c. Pseudomonas aeruginosa
c. Popliteus
d) Tuberculosis e. Haemophilus influenza
d. Quadriceps femoris (D) Heterakis gallinarum 115. Tympany of diaphragmatic hernia is 121. Female Hen has a following set of
chromosomes
103. Nucleic acid not found in plasmids is: 110. Green fodder to be ensiled should have a) Recurrent a. Xx
DM between b. Xw
a) DNA b) Persistent c. Zz
a) 50 to 60% d. Zy
b) RNA c) Both 1and 2 122. Which of the following drugs is used in
b) 30 to 35%
c) either DNA or RNA d) None of the above treating digitalis arrhythmia?
c) 20 to 25%
d) Both DNA and RNA a) Lignocaine
b) Quinidine 128. Which of the following is unpaired b. fundic part (b) LD50
skeletal muscle present in the animal body?
c) Procainamide c. Gizzard (c) KD50
a. Diaphragm
d) None of the above d. pyloric part (d) Half life dose
123. Medicago sativa is the botanical name of b. Bicepbrachii
a. Alfalfa 133. In modern smoke house, we can control 138. Exchange of non homologous
b. Berseem c. Popliteus chromosome material is known as
c. Guar a) Temperature
d. Cowpea d. Quadriceps femoris a) Translocation
124. Stage of cell division in which chiasmata b) Moisture
b) Crossingover
formation takes place is
c) Flavour
c) Duplication
a) Metaphase I
d) None of the above d) Synapsis
b) Pachytene
134. Which of the following is a opening of 139. The terms gene and genotype were coined
c) Diakinesis abomasum into the duodenum? by
129. Which one of the following is the longest
d) Anaphase a. Pylorus
nerve in the animal body? (a) Wilson
125. The animal resistant to Atherosclerosis is- b. cardia
a. Vagus (b) Johannsen
(A) Cattle c. rima oris
b. Sciatic (c) Mendel
(B) Swine d. Isthmus faucium
c. Femoral (d) Weisman
(C) Rabbit
d. Median 135. Stage of cell division in which chiasmata 140. Spherical bodies, weighing 0.45 – 0.9 kg,
(D) Poultry. formation takes place is attached to the placenta of a normal calf
130. Which of the following is the largest nerve comprising of an outer skin enclosing a
126. Lucerne hay contains % TDN in the animal body? a) Metaphase I mass of adipose connective tissue is
known as
a. Radial b) Pachytene a) Perosomus elumbis
a) 10 b) Amorphus globosus
b. Sciatic c) Diakinesis c) Otter calf
b) 30 d) Schistosomus reflexus
c. Femoral d) Anaphase
141. Obturator paralysis is more common in
c) 20 136. The Frequency for any class that is a) Mare
d. Median
d) 50 obtained by dividing the frequency for b) Cow
131 Which one of the following nerve that class by the total
127. In which tape worm, uterus is long supplying to blood vessel? number of observations is known as c) Bitch
transverse and dumble shape :
a. Vasomotar (a) Class frequency d) ewe
(a) Stilesia hepatica
b. Sensory (b) Relative frequency 142. Early insemination during estrous leads to
(b) Thysanosoma actinoides
(c) Thysaneizia giardia fertilization failure due to
c. Mixed (c) Cumulative frequency
(d) Anaplocephala magna
a) Ageing of sperms
d. Motar (d) Grouped frequency
b) Ageing of zygote
132. Which of the following is a spindle 137. The dose of a drug that kills 50 % of the
shaped glandular stomach of the bird? population is known as c) Ageing of ovum
a. proventriculus (a) ED50 d) none of above
148. Operation Flood III Was Launched during a. Brown (a) Prosthogonimum sp.
b. Yellow (b) Paramphistomes sp.
143. The relations of the dorsum of the fetus to (A) 1986 c. White (c) Fasciola hepatica
the quandrants of the maternal pelvis is d. Black (d) Fasciola gigantica
(B) 1990 156. Medusa head colonies are characteristic 164. Ovary much lobulated in which of the
a) Presentation feature of….. following :
(C) 1992
a) Rickettsia spp.
b) Position (a) Prosthogonimus sp.
(D) 1998 b) Bacillus anthracis
c) Clostridium botulinm (b) Paramphistomes sp.
c) Posture (c) Fasciola hepatica
149. The synonym of Anjan grass is d) Listeria spp.
d) None of the above a. Dhaman 157. Ring worm infection is caused by….. (d) Fasciola gigantica
a) Nocardia spp. 165. Haemoglobinuria is seen in-
b. Shevari
c. Ghamar b) Trichophyton spp.
(A) Theileriosis
d. Jinjavo c) Candida spp.
144. Sterno abdominal presentation is a 150. Medicago sativa is the botanical name of d) Dermatophillus spp. (B) Leptospirosis
a. Alfalfa 158. Curdling without pronounced acid
a) Posterior longitudinal presentation
b. Berseem production associated with milk and milk (C) Salmonellosis
b) Transverse ventral presentation c. Guar products…..
d. Cowpea a) Sweet curdling (D) Pasturellosis.
c) Anterior longitudinal presentation b) Acid curdling
166. Who is the father of Cellular Pathology?
c) Curdling
d) Transverse dorsal presentation d) Alkaline curdling (A) John Hunter
151. The nitrogen content (%) of urea fertilizer 159. Type of microbial association in which in
145. Balwant Ray Mehta committee was which food chain i.e. the metabolic
is (B) Robert Koch
appointed by products of one are utilized by the
a. 38
b. 42 other….. (C) Rudolph Virchow
(A) Govt. of India
c. 46 a) Metabolism (D) K.Cohnhiem.
(B) planning commission d. 50 b) Symbiosis
c) Fermentation
(C) State govt. Which of the following breeds of class is
160. known for egg production
(D) NGO
152. Licking of wall is the vices observed in
146. Post is example of --------aid. animal due to a. American breed
a. Mineral deficiencies b. Asiatic breeds
(A) Visual b. Internal parasites c. English breeds
c. Depraved appetite d. Mediterranean breeds 167. Pseudo Rabies is caused by-
(B) Audio d. All of the above 161 Which of the following breeds of class has
153. Wind sucking is the vice observed in the feathered shank (A) Lyssa virus
(C) A.V. a. American breed
species
b. Asiatic breeds (B) Picorna virus
(D) Projected a. Pig
b. Cattle c. English breeds
d. Mediterranean breeds (C) Paramyxo virus
147. Who is the architect of Indian modern c. Horse
d. None of above 162. Which of the following is smallest tape
dairy industry? (D) Herpes virus.
Mediterranean chicken breeds egg shell worm of poultry :
(A) Dr. V.Kurian 154. colour is 168. Pulpy Kidney Disease is caused by-
(a) Ralleitina tetragona
(B) Dr. Amrita Patel a. Brown (b) Ralleitina echinobothridia (A) Clostridium perfringens
b. White (c) Davainea proglotina
(C) Shree Parthi Bhatol c. Brown and white (d) Hymenolepis nana (B) Clostridium septicum
d. All of above 163. Indian liver fluke is which of the following
(D) MansiBhai American chicken breeds skin colour is : (C) Clostridium novyi
155.
(D) Clostridium tetani a. Adipocyte b) Reo virus c. Carbol fuchsin
b. Fibroblast d. None of the above
169. Turkey Egg Kidney is seen in- c. Mast cell c) Rabies virus 189. The concentration of double stranded
d. Plasma cell DNA molecule which gives absorbance
(A) Swine Pox 175. Which of the following describes a d) Rota virus
value of 1.0 at 260 nm is:
secretory process in which no cell a. 33 µg/ ml
(B) Swine Influenza 182. Holoenzyme is a combination of:
membrane components or cytosolic b. 40 µg/ ml
(C) Swine Fever contents are lost? a) enzyme and substrate c. 50 µg/ ml
d. Cannot be determined on the basis of
(D) Swine Erysipelas. a. Merocrine b) Apoenzyme and substrate absorbance value
b. Apocrine 190. The concentration of single stranded DNA
170. Antiseptics used for cleaning of eye is c. Holocrine c) Apoenzyme and coenzyme molecule which gives absorbance value of
d. Endocrine 1.0 at 260 nm is:
a. Dettol 176. The matrix of connective tissue is d) None of the above
a. 33 µg/ ml
composed of: b. 40 µg/ ml
b. Boric acid
c. 50 µg/ ml
a.Cells, fibers, and ground substance
c. Tr. iodine d. Cannot be determined on the basis of
b.Cells and fibers
absorbance value
d. Zinc oxide c.Fibers and ground substance
191. Which of the following cells lines the
d.Cells and ground substance
177. Following is not an effect of the ventricle of the brain?
171. More than two fracture fragments with 183. Following statements are correct except.
interconnecting fracture line in complete Ganglionic blockade a. Ependymal cell
fracture is known as a. Viruses multiply only in living cells.
a)Dry mouth b. Simple squamous cell
a. Multiple fracture b) Anhydrosis b. Viral nucleic acid directs cell
c) Tachycardia metabolism to synthesize viral components
d) Vasoconstriction c.simple cuboidal cell
b. Comminuted fracture
178. Magnesium sulphate has following effects c. Viruses are not able to perform their
d. Simple columner cell
c. Depressed fracture except own metabolic activities.

d. Fissure fracture a) CNS depressant d. Viral genetic information resides only in


b) Purgative DNA not in RNA
172. Pathognomic signs of fracture is c) Muscle relaxant
d) Diuretic 184. First Genome sequenced was of 192. The bony demarcation between abdominal
a. Crepitation a. Bacteriophage λ and pelvic cavities is:
179. Which one of the following is an osmotic b. Bacteriophage φ X 174
b. Pain diuretic? c. Haemophilus influenza a. Pelvic outlet
c. Swelling d. Homo sapiens
a)
Magnesium sulphate 185. First bacterial genome sequenced was of b. Pelvic diaphragm
b)
Ethacrynic acid
d. Loss of function a. Salmonella typimurium
c)
Spironolactone c. Plevic brim
b. Bacillus anthracis
173. Which of the following is a unicellular d)
None of the above
c. Pseudomonas aeruginosa
180. What is the site of action of carbonic d. Pelvic symphysis
gland that is typically found in mucosal e. Haemophilus influenza
anhydrase inhibitors? 186. First human protein produced in micro-
epithelium? 193 In which part of the cloaca in birds, bursa
a) Throughout the length of the tubule organism was: of fabricus opens?
a. Neuroepithelial cell a. Somatostatin
b) Loop of Henle
b. Myoepithelial cell b. Insulin a. Urodeum
c) PCT
c. Goblet cell c. Protopin
d) DCT
d. Friar cell d. None of the above b. Proctodeum
181. Viruses having Reverse transcriptase
174. Which of the following cells is primarily 187. The intercalating dye used to visualize
enzyme: c. Coprodeum
responsible for the production of collagen double stranded DNA is
and the amorphous ground substance in a) Retro virus a. Ethidium bromide d. Vent
loose connective tissue? b. Methylene blue
194 Which is the space between right and left (a) Sex linked a) Amputation of fetal limbs
pleural sac in thoracic cavity?
(b) Sex influenced b) Bisection of pelvis 209. When arable farming is mixed with livestock
a. Omentum raising it is known as
(c) Sex limited c) Transverse division of fetal trunk
b. Mediastinum (A) Mixed farming
(d) None d) Amputation of head and neck
c. Serous sac (B) Sole farming
205. Generally stallion attains the puberty at the
d. pleural sac age of (C) Co-operative farming
200. The method of sex determination in birds
195 Which of the following is the largest vein is a) 4 to 7 months (D) Slate farming
in the animal body?
(a) XO b) 9 to 12 months 210. Villagesurpanch is elected through
a. Saphenous vein
(b) XY c) 12 to 24 months (A) Secret ballot paper
b. Posterior venacava
(c) ZW d) 36 to 42 months (B) Member
c. Anterior venacava
(d) XA 206. Sertoli cell tumor of the testes in dog (C) Co-operative
d. Mammary vein secretes
(D) Nominated
196 Which of the following is the longest bone a) Testosterone
in birds? 201. Linkage between either dominant or 211. AMUL system of milk marketing follows the
recessive alleles is called b) Estrogen principle of
a. Femur
(a) Coupling linkage c) Androgen (A) Co-operative
b. Humerus
(b) Repulsion linkage d) Prolactin (B) Mutual work
c. Tarsometatarsus
(c) Complete linkage 207. Dystrophia adiposogenitalis is observed in (C) Subsidy purpose
d. Tibiotarsus the
(d) Incomplete linkage (D) Service
197. Mendel‘s work was rediscovered in the a) Dog
year 202. The diploid chromosome number in 212. GCMMF has made turnover of Rs.
chicken is b) Stallion
(a) 1895 (A) 5500 crore
(a) 38 c) Boar
b) 1900 (B) 6700 crore
(b) 74 d) Ram
(c) 1905 (C) 5000 crore
(c) 78 208. Large number of primary sperm
(d) 1913 abnormalities are indicative of (D) 8000 crore
(d) 60
198. Holandric inheritance is characterized by a) Ectopic testes 213. Who is the chairman of GCMMF?
203. Wry neck is mostly seen in
(a) Color blindness b) Testicular degeneration (A) Amrita patel
a) Bovine
(b) Muscular atrophy c) Testicular neoplasm (B) Vipul chaudhary
b) Caprine
(c) Hair in ear pinna d) Testicular fibrosis (C) Parthi Bhatol
c) Equine
(d) Baldness (D) Dr. Kurian
d) Canine
199. Hemophilia is a condition which is 214. Rural dairy extension programmed was the
inherited as 204. Recommended fetotomy procedure in part and parcel of
perosomus elumbis is
(A) ICDP b. Banglore c) Chlorinated hydrocarbons 237. Out of which of the following are
c. Hyderabad d) Activated chlorinate unsegmented :
(B) DPAP d. Anand 230. The following acid producing bacteria
convert lactose into lactic acid in milk….. (a)Round worm
(C) IRDP 223. Central Avian Research Institute (CARI) is a) Streptococcus cremoris
located at b) Staphylococcus aureus (b) Trematode
(D) JRY
c) Bacillus cereus
a. Izzatnagar (c) Both of the above
215. Electronic identity is made in animal by d) Clostridium botulinm
b. Banglore
a. Electron microscope (d) None of the above
c. Hyderabad 231. The most common bacterium causing joint
b. Computer
d. Anand pain….. 238. 'Gape worm' of poultry is :
c. Radium number
224. Normally which ovary & oviduct is
d. Electronic chip a) Brucella
216.This measurement of body has close relation functional in chicken b) Shigella (a) Ascaridia galli
with body weight in animal c) Salmonella (b) Heterakis gallinarum
a. Right ovary and Right oviduct (c) Syngamus trachea
a. Body length d) Clostridia
b. Left ovary and Left oviduct (d) Subulura brumpti
b. Paunch girth c. Right ovary and Left oviduct 239. Poll evil in Horse is caused by-
c. Height at wither d. Left ovary and right oviduct 232. Galacto toxins in milk are produced by…..
d. Heart girth a) Streptococci spp. (A) Clostridium tetani
225. Complete Parts of Oviduct in chronological b) Contact of milk with steel vessels
c) Contact of milk with copper vessel (B) Actinomyces bovis
order are
217.Blanketing is the practice utilize for d) Serratia spp.
a. Brighter look to body coat a. Infundibulum-Isthumus-Magnum-Uterus- (C) Brucella abortus
b. To keep hide in good condition Vagina
(D) Both (B) and (C).
c. Refard hair growth b. Infundibulum-Magnum-Isthumus-Uterus-
d. All of above Vagina 233. . Praziquental and tartar emetic is drug of
218.Wedge shaped body denotes c. Infundibulum-Magnum-Isthumus-Uterus
d. Infundibulum-Magnum-Isthumus-Vagina- choice for :
a. Beef character
b. Draught character Uterus
(a) Schistosoma sp.
c. Dairy character 226. Fertilization is take place in which part of 240. Nutritional roup in Poultry is caused due to
(b) Eurytrema sp.
d. None of above reproductive tract (c) Notocotylus sp. deficiency of-
219.Colostrum feeding in the calf should be done (d) All of the above
at a. Ovary 234. In schistosoma which of the following (A) Vitamin B
b. Isthumus
a. Within two hours after calving statement is true :
c. Infundibulum (B) Vitamin C
b. Within three days of calving
d. Uterus
c. Within twelve hours after calving (a) Male is longer than female
227. . Chloramphenicol residues and milk (C) Vitamin E
d. None of above (b) Female is longer than male
220.Age at first kidding in mehsani goat breed is products causes in consumers….. (c) Female & Male are of same size (D) Vitamin A.
a. 18 to 24 months a) Arthritis (d) None of the above
b. 24 to 30 months b) Aplastic anemia 235. Flame cell is excretory system of which of 241. Epithelial Pearls are seen in-
c. 12 to 18 months c) Blindness the following :
d. 30 to 36 months d) Anorexia (A) Sebaceous cell Adenoma
221. Which state rank first in duck population 228. Well established protozoan disease (a) Trematodes
transmitted through milk….. (b) Cestodes (B) Squamous cell carcinoma
a. West Bangal a) Cysticercosis (c) Nematodes
b. Assam b) Toxoplasmosis (d) Acanthocephala (C) Melanoma
c. Orissa c) Giardiasis 236. Normally eggs are operculated in which of
d. Karala (D) Venereal granuloma.
d) Salmonellosis the following :
229. Insecticides of group that constitute the 242. Blue Tongue in sheep is caused by-
222. Project Directorate on Poultry (PDP) is principal health hazard to consumers of milk (a) Round worm
located at and milk products….. (b) Trematode (A) Herpes virus
(c) Both of the above
a) Organic sulphates
a. Izzatnagar (d) None of the above (B) Birna virus
b) Organic phosphates
(C) Picorna virus d.80-100 days 253. Alkaline mucous glands are found in the a) Rats
submucosa of the: b) Cattle
(D) Orbi virus. c) Horse
a. Ileum d) All of the above
243. Most common Serotype of FMD virus in 248. Typical radiographic sign of osteo-arthritis b. Jejunum 260. Which of the following drugs produces only
India is- is c. Duodenum laxative effect even with increase in dose?
d. Cardiac region of the stomach
(A) A a. Increased joint space 254. The gallbladder: a) Magnesium sulphate
b) Anthraquinone
(B) C b. Decreased joint space a. Produces bile c) Castor oil
b. Is attached to the pancreas d) Liquid paraffin
(C) Asia-1 c. Irregular joint space c. Stores and concentrates bile 261. One of the following is not a salt of
d. Produces cholecystokinin bunamidine used against cestodial infestation in
(D) O. d. Irregular joint space with new bony
255. Which of the following sphincters is under
growth
244. In which disease post mortem of carcass is voluntary control? animals.
prohibited?
a. Pyloric a) Bunamidine P-toluene sulphonate
(A) Haemorragic septicemia 249. The radiographic signs of non union is b. Hepatopancreatic
c. Internal anal b) Bunamidine hydrochloride
(B) Rinder pest a. Radiolucency between fragments d. External anal
256. At the junction between the esophagus and c) Bunamide sodium
(C) Anthrax the stomach, the epithelial lining changes abruptly
d) Bunamidine hydroxynaphthoate
b. Increased radio density of either from __________________ to
(D) Brucellosis. fragments _________________. 262. Source of bacitracin is
245. The water used for moistening of POP cast c. Placement of fragments side by side a. Nonkeratinized stratified squamous; a) Streptomycin rimosus
should range between simple columnar
b. Simple columnar; nonkeratinized stratified b) Bacillus subtilis
̊
a. 30-35 C d. Rounding of both fragments squamous
c. Nonkeratinized simple squamous; c) Streptomyces aureqfaciens
̊
b.10-20 C 250. The radiographic diagnosis of intestinal stratified columnar
d. Stratified columnar; nonkeratinized simple d) Bacillus polymyxa
̊
c. 20-25 C obstruction is done by using
squamous
263. The most effective chemical disinfectant to
̊
d. 40-45 C a. Barium sulphate
kill FMD virus is:
246. The best procedure for complete b. Conray -420
a. 2 % formalin
diaphyseal fracture of tibia in bullock is c. Urographin 257. Which one of the following has maximum
b. 70 % alcohol
a. POP natriuretic effect?
d. Ipamidol
c. 2 % Sodium hydroxide
b. Hanging pin cast a) Spironolactone
251. . Holocrine secretion: b) Frusemide
d. 0.5 % phenol
c. Walking cast c) Mannitol
a. Occurs in sebaceous glands
d) Acetazolamide 264. Hendra and Nipah viruses belong to the
b. Occurs in endocrine glands
d. Thomas splint 258. The following is not an indication of PGF2
c. Involves little or no loss of cytoplasm family:
d. All of the above alpha
247. If fracture is stabilized with rigid fixation
252. The site of production of cholecystokinin and a. Paramyxoviridae
the clinical union occur at a) Synchronization of oestrus
secretin is the: b) Cystic ovaries b. Orthromyxoviridae
a. 15-20 days c) Persistent corpus luteum
a. Stomach
d) Induction of abortion c. Picornaviridae
b.20-30 days b. Pancreas
259. In which of the following animals emetics
c. Small Intestine d. Parvoviridae
c.40-50 days d. Large Intestine are not used?
271. The 260/280 nm ratio of pure DNA sample a. Glenoid cavity d. blood
should be:
265. Clinical manifestation of canine a. Less than 1.8 b. Cotyloid cavity 283. The point of crossover is known as
parvovirus infection is/are: b. 1.8
c. Articular groove (a) Tetrad
c. More than 1.8
a. Mycocarditis in pups d. 3.0 d. Semilunar notch (b) Chiasma
272. The 260/280 nm ratio less than 1.8 for a
b. Haemorrhagic diarrhea
DNA sample reflects: 279. The cell membranes is mainly composed (c) Synapsis
c. Leukopenia
a. Protein contamination of
b. RNA contamination (d) Recombination
d. All of above
266. Bluetongue virus has: c. Both a. a single layer of protein molecules
d. None 284. The The maximum probability of making
a. 20 serotypes Type-I error is known as
b. 24 serotypes b. a protein bilayer
273. Oligo dT attached to resin is used for the
c. 7 serotypes (a) Confidence interval
column based isolation of
d. 9 serotypes c. a phospholipids bilayer
a. Prokaryotic DNA
267. Find the wrong match: b. Eukaryotic DNA (b) Test of significance
a. Borrel bodies- Fowl pox c. Prokaryotic m-RNA
b. Guarneri bodies- small pox d. a polysaccharide bilayer
d. Eukaryotic m-RNA (c) Level of significance
c. Negribodies- Rabies 274. The chemical that can be used for
d. All are correct precipitation of DNA is:
268. Bovine diarrhea virus belongs to the 280. Which organelles is the site for ATP (d) Rejection region
a. Ethyl alcohol
family b. Isoamyl alcohol production?
c. Phenol 285. Which of the following traits will show
a. Flaviviridae a. nucleoli higher rate of genetic improvement under
d. None
b. Reoviridae 275. The chemical method of DNA sequencing is: mass selection programme
a. Maxam Gilbert method b. mitochondria
c.Togaviridae b. Sanger method a) Service period in cattle
c. gogli complex b) Body weight at 8 week in
c. Both
d.Rhabdoviridae d. None poultry
d. ribosomes c) Litter size in pig
276. Which of the following is a oval articular
269. Viral triad include projection? d) Twinning rate in goat
286. Diallele crossing is usually practiced in
a. Rinder pest virus, Measles virus a. Condyle
and Canine distemper virus (a) Cattle
b. Trochlea 281. During which stage mitosis do the (b) Sheep
b. Rinder pest virus, Mumps virus, (c) Race horse
chromosomes line up in the middle of cell
Measles virus c. Head (d) Poultry
a. prophase 287. Marker assisted selection is more
c. Measles, mumps and Rubella d. Facet effective for traits of
viruses b. anaphase
277. Which of the following is a ventricle of (a) High heritability
d. Rinder pest virus, Reo virus and hind brain? c. metaphase (b) Low heritability
Rhabdo virus (c) Medium heritability
a. Third ventricle d. telophase (d) Both b) and c)
270. The concentration of single stranded RNA
b. Fourth ventricle 282. Which of the following is not a connective
molecule which gives absorbance value
tissue? 288 ‗Operation Flood‘ scheme was operated by
of 1.0 at 260 nm is: c. Lateral ventricle
a. 33 µg/ ml a. bone (a) NDDB, Anand
b. 40 µg/ ml d. None of above (b) NDRI, Karnal
c. 50 µg/ ml b. cartilage (c) IVRI, Izatnagar
d. Cannot be determined on the basis of 278. Which of the following is a spherical (d) NDRI, Bangalore
absorbance value shallow articular depression? c. muscle
294. Terminology used for the high per cent of (A) NGO d. Ewe with loss of teat
abnormal sperms
(B) Govt.
a) Teratozoospermia
(C) People
304.The average life span of horse is ( Years )
b) Aspermia
289. Inbreeding would more severely affect (D) Middlemen a. 2.5 to 18 Years
c) Asthenozoospermia b. 3 to 16 Years
(a) Pre weaning gain in sheep c. 3 to 15 Years
(b) Litter size in pig d) Necrozoospermia d. 3 to 12 Years
(c) Fat % in cow 299. NDRI is located at 305.Sunandini is the cross bred developed from
(d) Mature body weight in doe 295. Inability to withdraw the penis back in to the a. Local cattle Kerala
prepuce (A) Jaipur
b. Local cattle Karnataka
(B) Kernel c. Sahiwal
290. Smooth muscle around spermatic cord to a) Paraphimosis d. Tharparkar
help in thermoregulation of testes is 306.Simmental is the breed of cow belong to
b) Posthitis (C) Jabalpur
breed
a) Tunica dartos
c) Phimosis (D) Cochin a. Exotic dairy cow
b) Cremester muscle b. Exotic beef cow
d) Preputial prolapse 300. CSWRI is located at c. Indigenous dairy cow
c) Pampiniform plexus d. Indigenous beef cow
296. Effect of novelty of stimulus females reduces (A) Avikanagar
307.This breed not belong to mysore type cattle
d) Gubernaculum refractory period in males is known as group
(B) Izzatnagar
a. Burgur
291. Mitochondria concentrates close to the a) Balling up effect (C) Modinagar b. Alambadi
axoneme and forms the c. Khillari
b) Coolidge effect (D) Mathura d. Nimari
a) End piece of the tail 308.Buffalo in philipines is known as
c) Bruce effect 301. In mixed farming income from main
b) Neck of the sperm a. Kerban
d) Pederasty enterprise is b. Shin nive
c) Mid piece of the tail c. Arana
297. Sabarmati Ashram gosala founded in (A) 49% d. Carabao
d) Annulus 309. This buffalo breed is come under endangered
(A) 1815 (B) 50%
categories
292. Unnatural tactile stimulation and ejaculation a. Bhadawari
(B) 1925 (C) 60%
is known as b. Jafarabadi
(C) 1915 (D) 70% c. Banni
a) Onanism d. Jerangi
(D) 1945 302. NDDB was established in the year 310. The inner surface of which section of
b) Coolidge effect
oviduct is lined with goblet cell that secrete
(A) 1965
c) Pederasty albumen
297. Central council of Gosamverdhan was started (B) 1955
d) Balling up a. Isthumus
(C) 1975 b. Magnum
293. A complete series of cellular associations (A) 1952 c. Uterus
along a seminiferous tubule (D) 1985 d. Infundibulum
(B) 1955 311. Doubled yolked egg is more common in
a) Spermatogenic wave
(C) 1958 a. Older birds
b) Spermiostasis b. Pullet
(D) 1960 303. Crone is the synonym of the
c. Both a and b
c) Spermiogenesis a. Young sow d. None of above
298. State farming is managed by b. Broken mouth sow 312 Hens usually moult in the which following
d) Cycle of seminiferous epithelium c. Old aged ewe
order (B) Botriomycosis

a. Head-Neck-Body-Wing-Tail 319. Cold sterilization means….. (C) Haemorrhagic Septicemia


b. Head- Neck-Wing-Body-Tail
c. Tail-Wing-Neck-Head-Body a) Sterilization at low temperature (D) Actinobacillosis.
d. Tail-Wing-Body-Neck-Head b) Sterilization by radiations
c) Flash pasteurization 327. Alimentary canal present in which of the 333. Mode of transmission of IBR virus is-
The order of disappearance of pigment
313 d) None of the above following :
(bleaching) from body in 320. Anthrax is also called as….. (A) Venereal
a. Vent-Eye ring-Ear lobes-Beak- a) Splenic fever (a) Trematodes
b) Desert fever (b) Cestodes (B) Inhalation
Shanks
b. Vent- Beak- Eye ring-Ear lobes- c) Undulant fever (c) Nematodes
d) All of the above (d) Acanthocephala (C) Both
Shanks
c. Shanks-Beak-Ear lobes-Eye ring- 321. Trichinella cyst can be destroyed by…..
(D) None of the above.
Vent a) Salting
d. Vent-Shanks-Beak-Ear lobes-Eye b) Smoking 334. Maedi is primarily a disease of-
ring c) Both of the above
d) None of the above 328. Which of the following is the kidney worm
The pigment fist leaves those structures (A) Sheep affecting respiratory system
314 having 322. Scrapie is a….. of dog :
a) Progressive fatal disease of CNS (B) Cattle affecting reproductive system
(a) Trichuris sp.
a. Poor blood circulation b) Disease of young animals
(b) Capillaria sp. (C) Sheep affecting nervous system
b. Best blood circulation c) The causal agent is antigenic
(c) Seteria digitata
c. No correlation with blood d) All of the above
(d) Dioctophyma renale (D) Cattle affecting nervous system.
circulation 323. Knott‘s technique is a concentration
329. In which egg are lemon shaped and with
d. Both a and b method for detection of following parasite in
The urophygial gland is located on dorsal plug on both side of egg :
315 area of blood
(a) Trichuris sp.
(a) Trichomonas spp. (b) Capillaria sp.
a. Tail 335. Equine Plague is also called as-
(c) Seteria digitata
b. Back
(b) Trypanosoma evansi (d) Dioctophyma renale
c. Head (A) Equine viral arteritis
(c) Microfilariae 330. Mad itch is mostly a disease of-
d. Neck
Most chicken breeds have how many (d) All above (B) Glanders
(A) Caprine
316 number of toes on each foot 324. Which parasite cause destruction of host
tissue by breakage of lymph vessels : (C) Strangles
(B) Bovine
a. Four
b. Three (a) Chiggers (D) African Horse sickness.
(C) Swine
c. Five (b) Filarids
(c) Strongylus 336. Sore mouth in cattle is seen in-
d. Two (D) Ovine
317. Immediate test to judge the quality of (d) Ascarids
(A) Blue tongue
milk….. 325. Oxyuris equi in horse found in which of 331. Tigroid Heart is seen in cattle affected
a) Electro impedance method the following : with- (B) Bovine malignant catarrh
b) Alcohol test
c) Organoleptic tests (a) Small intestine (A) Bovine malignant catarrhal (C) Rinder pest
d) Sediment test (b) Caecum, colon
(c) Oesphagus (B) Botulism (D) Vesicular Stomatitis
318. Psychrophils of significance in milk
(d) Caecum, colon, rectum
hygiene is/are….. 326. Presence of eggs of the Oxyuris equi can (C) Bovine viral diarrhea 337. Protective wears for radiography are made
a) Pseudomonas
be diagnosed by which of the following : up of
b) Listeria monocytogenes (D) Foot and mouth disease.
c) Both a and b (a) Faeces a. Lead
d) None of the above 332. Wooden Tongue in cattle is seen in-
(b) Perianal swab
(c) Both of the above b. Iron
(A) Actinomycosis
(d) None of the above
c. Zinc
d. Tungste a. Secrete digestive enzymes 354. Kanamycin derivate is
b. Remove waste materials
338. The best treatment of long bone fracture is c. Regulate the release of bile a) Amikacin
d. Secrete water in order to regulate blood
a. POP bandages volume b) Spectinomycin
350. Which of the following is not produced by
b. Intramadulary pinning 343. General anesthesia of equine is performed c) Atramycin
an enteroendocrine cell?
now a days by using
c. Intramadulary nailing d) Spiramycin
a. Pepsin
a. Chloral hydrate b. Cholecystokinin
d. Bone plating 355. Polymixin E is also called as
c. Gastrin
b. Xylazine
339. Suturing of lacerated nostril is performed d. Secretin a) Novobiocin
under nerve block c. Xylazine + Ketamine
b) Bacitracin
a. Infra-orbital d. Chlormag
c) Colistin
b. Retro bulbar 344. Histologically, the stomach wall is unique
because it contains: d) Kanamycin.
c. Mandibular 351. An anti-estrogen used in advanced breast
a. No lamina propria cancer is 356. An agent used against anaerobic bacteria as
d. Supra-orbital b. 1 extra layer in its muscularis mucosae well as protozoa is
a) Tamoxifen
c. 1 extra layer in its muscularis externa
340. Extirpation of incisor tooth in cattle is d. An adventitia in addition to a double- a) Mebendazole
performed under nerve block b) Precarbazine
membraned serosa
b) Metronidazole
345. The gastric gland cell whose absence c) Mitotane
a. Infra-orbital
could lead to pernicious anemia is the: c) Methicillin
b. Mandibular d) Cisplatin
a. Chief cell d) Marbofloxacin
c. Retro bulbar b. Goblet cell
c. Mucous neck cell 357. Neomycin B is also called as
d. Cornual d. Parietal cell
346. The layer of the digestive tube wall which a)Kanamycin
341. The best anaesthetic technique for contains blood vessels, lymphatic nodules, and a
laparotomy in cattle is b) Amikacin
rich supply of elastic fibers is the: 352. A semi synthetic derivative of diterpene is
a. Local infiltration c) Tobramycin
a. Mucosa a) Clindamycin
b. Submucosa d) Gentamicin
b. Paravertebral c. Muscularis Externa b) Ticarbiciliin
d. Serosa 358. Flaviviridae is
c. Field block 347. Mechanical digestion occurs in the: c) Tiamulin
d. Anterior caudal epidural a. dsDNA
a. Stomach d) Lincomycin
b. Cecum b.ds RNA
342. Docking in adult dog is performed under c. Pharynx 353. Drug used as anthelmentic by producing
d. Esophagus its effect by GABA mediated hyper polarization c.ss DNA
a. Epidural
348. The entry of bile into the duodenum is is
b. Local infiltration controlled by the: d. ssRNA
a) Albendazole
c. Ring block a. Liver sinusoids 359. Following are the morbilli viruses except
b. Common pancreatic duct b) Mebendazole
d. General anaesthesia c. Pyloric sphincter a. Mumps virus
d. None of the above c) Fenbendazole
b. Measles virus
349. A major function of the large intestine is
d) Ivermectin
to: c.PPRV
d.Rinderpest virus c. Pulse Field electrophoresis a. sternum a. cranial nerve V
d. None
b. patella b. spinal nerves supplying the intercostal
muscles c.
360. Following is/are the character(s ) of c. os cardis hypothalamus
368. The blotting technique used for the
Streptococcus detection of DNA molecule is d. calcaneus
a. Gram positive cocci called: d. a neuromuscular junction
b. Catalase –Ve a. Southern blot
b. Northern blot 379. The pons, medulla and cerebellum
c. Arranged in chain
d. All of these c. Western blot 375. Which of the following joints is an together form the
361. Cold enrichment is required for the d. Eastern blot example of an amphiarthrosis?
369. The blotting technique used for the a. forebrain
isolation of
detection of RNA molecule is a. the temporomandibular joint
a. Listeria monocytogenes b. midbrain
called:
b. Erysipelothrix
a. Southern blot c. hindbrain
c. Staph.aureus
b. Northern blot b. Sutures of the skull
d. Clostridium tetani
c. Western blot d. cerebral hemispheres
362. Bacteria responsible for food
d. Eastern blot c. between the bodies of the vertebrae
poisoning
370. The blotting technique used for the 380. Testosterone is secreted by which cell?
a. Staph.aureus
detection of protein is called:
b. Clostridium botulinum
a. Southern blot d. ischiopubic symphysis a. islets cells
c. Bacillus cereus
b. Northern blot
d. All of these b. sertoli cells
c. Western blot
d. Eastern blot
363. Dysgonic species of 371. In southern blot, the labeled nucleic acid c. Brunner‘s glands
Mycobacterium is used to detect complementary sequence is
a. M.bovis called: d. cells of Leydig
b. M.avium a. Template
c. M.tuberculosis 376. What is the unit of contraction in a 381. In the foetal circulation, the shunt that
b. Primer
d. M.phlei c. Probe muscle? connects the pulmonary artery and aorta is
364. Satellite growth on blood agar d. None of the above called as
plate in presence of Staph. aureus a. motor unit
372. Which type of tissue covers the external and
is characteristic of a. ductus venosus
internal surfaces of the body? b. sarcomere
a. Pasteurella
b. Haemophillus b. foremen ovale
a. connective c. origin
c. Actinobacillus
d. Mycoplasma c. ductus arteriosus
b. skin d. insertion
365. The di-deoxy chain termination
method of DNA sequencing is: d. falciform ligament
c. areolar 377. Which of the following muscle is not a
a. Maxam gilbert method
component of the Achilles tendon? 382. Genetic drift in small population is
b. Sanger‘s method d. epithelial
an example of
c. Pyrosequencing method
a. semimembranosus (a) Systematic process
d. Nanopore sequencing method 373. What is found within the peritoneal
366. The pH of Tris saturated phenol cavity? b. biceps femoris (b) Dispersive process
used for the purpose of DNA
isolation should be: a. pleural fluid (c) Both
a. 5.0
b. 6.0 b. the pericardium (d) None
c. semiitendinosus
c. 7.0
d. 8.0 c. peritoneal fluid 383. The maximum frequency of recombinant
d. superficial digital flexor
367. DNA molecule as big as 10 MB progeny is
d. liquor pericardi
can be separated by: 378. Which of the following structures is not
a. Polyacrylamide gel electrophoresis 374. Which of the following is a splanchnic part of the peripheral nervous system? (a) 50 %
b. Agrose gel electrophoresis (b) 25 %
bone?
(c) 42 % (a) Intensity of selection 393. The penile protrusion is followed by a) Cows
(d) 63 %
(b) Standardized selection a) Erection b) Buffalo

(c) Selection differential b) Mounting c) Mare

384. The diploid number of chromosomes are (d) None of the above c) Intromission d) Ewes
equal in
(a) Sheep and goat 389. Scent glands, source of pheromones d) Pelvic thrust
(b) Cattle and goat located dorsally and medially at the horns
(c) Buffalo and sheep 394. The required number of progressive 399. The first veterinary college was started in
of
motile sperms at A.I. in cattle is the year 1886 at
a) Ram
(d) Man and rhesus monkey a) 50 millions (A) Kolkata
b) Buck
385. A negative correlation coefficient between b) 30 millions (B) Mumbai
the two variables X and Y indicates that c) Bull
c) 10 millions (C) Madras
(a) Large values of X are associated with
small values of Y d) Stallion
d) 3 millions (D) Ludhiyana
(b) Large values of X are associated with 390. Sertoli cell produces the protein hormone
395. Presence of musculo membranous
large values of Y which suppresses the production of FSH
attachment on ventral aspect of glans penis
a) Androgen binding protein to the preputial mucosa
(c) Small values of X are associated with
small values of Y b) Inhibin a) Balanoposthitis
(d) None of the above answers are correct b) corkscrew penis 400. The first dairy co-operative was started in
c) Luteinizing hormone
the year 1913 at
386. Which of the following chemical is used as a d) Estrogen c) Penile tumor
mutagen? (A) Anand
391. Spreading of adherent acrosomal granule d) Penile frenulum
(a) Mustard gas (B) Allahabad
over the surface of spermatid nucleus
396. Accentrically placed thickening of the
(b) Colchisine acrosome is known as (C) Jaipur
a) Golgi phase
(c) FCS a) Diadem defect (D) Pune
b) Cap phase
(d) All of these b) Acrosomal cap 401. Toxic jaundis is also known as
c) Acrosomal phase
387. Which of the following is sex linked trait in c) Sterilizing tail stump (A).Post haepatic jaundis
d) Maturation phase
poultry?
d) Knobbed spermatozoa (B).Haepatic jaundis
(a) Colour pattern of plumage
397. Flehman‘s reaction is not observed in (C).Pre haepatjc jaundis
(b) Brring pattern of plumage
a) Bull (D). Obstructive jaundis
392. Infertility in a bull due to Inability to
(c) Comb pattern fertilize
b) Boar 402. Siderosis means
(d) Abumin height a) Penile deviation
c) Stallion (A).Deposition of calcium in lung
388. The difference between mean phenotypic b) Penile franulum
d) Ram (B). Deposition of iron in lung
values of the progeny of
selected parents and whole of the c) Testicular Hypoplasia (C). Deposition of silicon in lung
398. Bicornual transverse presentation is more
population before selection is known as
d) Penile Neoplasm common in (D). Deposition of silver particle in lung
403. Van den Bergh test for obstructive jaundis (C). Right shift 414. Secretary of Panchayat samity (D) 95%

(A). Direct (D). Both (A) & (C) (A) DDO 419. Two forms of social stratification

(B).Indirect 409.Blood in vomitus (B) TDO (A) Cast class

(C).Biphasic (A). Haematamiasis (C) Collector (B) Color race

(D). Both (B) & (C) (B). Haemoptysis (D) Mamlatdar (C) Custom value

404. In abscess which type of necrosis is seen? (C).Epistaxis 415. National Institute of Rural Development is (D) Norm belief
located at
(A). Coagulative necrosis (D).Melena 420. Tegur in Karnataka is the breeding
(A) Hyderabad farm for
(B).Liquifective necrosis 410. Bleeding from the oviduct is designated as: a. Gir
(B) Chennai b. Tharparkar
(C).Caseative necrosis a) Epitaxis c. Khillar
(C) Bangalore d. None of these
(D).Fat necrosis b) Hemosalpinx 421. Malas are the professional breeder
(D) Jaipur for
405..First change after death is c) Hematocele
a. Sahiwal
(A). Alger mortis d) Hematemasis b. Red sindhi
c. Rathi
(B).Rigor mortis 411. Condition which is hereditary and sex linked d. Ongole
in which clotting is delayed: 416. Specialized farm income from main 422. This breed produce the ―Primium
(C).Formation of bloat market milk‖ due to yellow color
enterprise is
a) Apoplexy of milk
(D).Both (B) & (C) a. Jersey
(A) <50%
b) Hemophilia b. Brown swiss
406. Inflammation of crop (B) >50% c. Guernsey
c) Brown induration d. Ayreshire
(A). Blephritis (C) 50% 423. Ration calculation for individual
d) Epistaxis animal does not require this thing
(B).Ingluvitis (D) 60% a. Weight of animal
412. On the basis of rainfall, temperature, and b. Lactation yield
(C). Typhlitis soil the country can be divided into --- 417. The first Veterinary University was c. Pregnanacy
(D). Gonitis established in the year 1986 at d. Parity of animal
Animal husbandry region 424. Maize fodder (green) is comes
407.Cart wheel appearance of nuclease found in (A) Hyderabad under the grade in quality
(A) 5 a. Excellent
(A).Plasma cell (B) Trichur b. Good
(B) 6
c. Medium
(B). Basophils (C) Bangalore d. Poor
(C) 7
425. This practice is not comes under
(C). Eosinophils (D) Chennai milking method
(D) 8
a. Intermittent milking
(D). Monocyte 413. Small scale farming also known as b. Knuckling
418. Contribution of sight in learning is c. Full hand
(A) Faimly farming d. Stripping
408..Extreme elevation of leucocyte in peripheral (A) 80%
(B) Sole farming 426. This practice does not required for
blood is known as (B) 87% sexual stimulation in bull
(C) Individual farming a. False mount
(A).Shift to left (C) 90% b. Changing teaser
(D) Co-operative farming
(B). Leukamoid reaction c. Changing semen collector
d. Restraining mount a. Withdrawal of Feed and Water a) 100-110 mm of mercury 450. Spinose ear tick mainly pathogenic in
427. The score points for cow graded b. Withdrawal of Light b) 120-145 mm of mercury which of the following stage :
―very good‖ are c. Increase level of dietary Zinc c) 146-165 mm of mercury
a. 80 to 85 d. All of above d) 166-175 mm of mercury (a) Larval stage
b. 85 to 90 Cold room temperature for hatching egg 443. Unconventional meat….. (b) Nymphal stage
c. 90 to 95 435 storage is a) Beef (c) Adult stage
d. 70 to 80 b) Mutton (d) All of the above
428. The key village scheme was a. 48-52 ºF c) Crocodile meat
launched in India during the year b. 58-62 ºF 444. Booling test on meat detect…..
a. First five year plan c. 68-72 ºF a) PH
b. Second five year plan d. 78-82 ºF b) Colour
c. Fifth five year plan 451. The chief cause of 'strike in the sheep' is
436 Setter temperature for hatching egg is c) Odour
d. Sixth five year plan 445. Electrical stunning is widely used in….. which of the following :
429. Operation flood project received a. 92.5-93 ºF
a) Cattle and poultry (a) Larvae of Musca sp.
donation of butter oil and scheme b. 99.5-100 ºF
b) Pigs and poultry (b) Larvae of Sarcopaga sp.
milk from c. 102.5-103 ºF
c) Buffaloes and poultry (c) Larvae of Lucilia sp.
a. European union d. 97.5-98 ºF
446. Study of organism in relation to their (d) All the above
b. U. S. A. 437 Hatcher temperature for hatching egg is environment is Known as…
c. European dairies 452. Second intermediate host of oviduct fluke
d. European economic community a. 94-95 ºF are :
b. 99-100 ºF a)Ecology
c. 98-99 ºF (a) Dragon flies
430 In poultry true stomach is d. 96-97 ºF b)Ecosystem
(b) Water snail
a. Gizzard Physiological zero (0) temperature bellow (c) Grasshopper
c)Epornitic
b. Proventriculus 438 which embryo growth is arrested in eggs (d) None
c. Crop d)Oncology 453. Insects which possess sponging mouth
d. Abomasums a. 75 ºF parts and do not bite are :
b. 65 ºF 447. Which disease is Ectoparasitic zoonosis
The end product of protein metabolism in c. 85 ºF (a) Fleas
431 poultry is mainly d. 95 ºF a) Scabies (b) House flies
439 Position of hatching egg in setter is (c) Mosquitoes
b) Taeniasis (d) Deer flies
a. Urea
b. Uric acid
a. Narrow end up 454. Scientific name of horse fly is ……
b. Broad end up c) Both a) & b)
c. Ammonia
c. Horizontal (a) Tabanus equinum
d. Urates d) None of above
d. Vertical (b) Oestrus ovis
(c) Simulium sp
Which type of egg producer molt late in the 448. How to control reservoir of infection (d) None of the above
432 season and rapidly
440. Ematiation in meat is caused by….. a) Treatment
455. Which of the following is a non granular
a) Inadequate intake of nutrients WBC?
a. Good egg producer
b. Poor egg producer b) Pathological conditions b) Health education
c) Advanced age of animal a. Lymphocytes
c. Average egg producer
d. None of above d) Over eating c) Both a) & b)
b. Neutrophil
441. Clostridium botulinum secretes toxin
under….. d) None of above
Which type of egg producer molt early in the c. Basophil
433 season and slowly a) Aseptic condition 449. Rabies also knokwn as….
b) Aerobic condition d. Eosinophil
a. Good egg producer c) Anaerobic condition a)Lyssa
b. Poor egg producer d) Aerobic as well as anaerobic condition 456. In Classification of helminthes the higher
c. Average egg producer b)Rage taxa platyhelminthes containing
d. None of above
c)Tallwut helminthes of veterinary importance are :
434 Forced molting is done by 442. In sheep the normal arterial blood pressure
is….. d)All of above (a) Flat worm
(b) Round worm (A) Botulism 468. Substance responsible for increase
(c) Thornyheaded worms penetration of Lyssa virus-
(d) None (B) Tetanus
457. In Filarial nematode, the life cycle is…… (A) Hyaluronidase
(C) Both 473. To capture wild elephant the anesthetic
(a) Direct (B) Erythriol used is
(b) Direct and Indirect (D) None of the above
(c) Indirect (C) Protagen a. Xylazine
(d) None 464. Diamond skin disease is primarily a
458. Morocco leather condition is caused by : disease of- (D) Amylase b. Etorphine

(a) Hemonchus contortus (A) Horse 469. Crop mycosis in poultry is caused by- c. Medazolam
(b) Ostertagia ostertagia
(c) Cooperia curtesi (B) Lion (A) Bacteria d. Morphin
(d) None
(C) Sow (B) Mycoplasma 474. The sedation in camel is performed by
using xylazine intravenously
(D) Turkey (C) Fungi
a. 2-3 ml
(D) Virus
459. Strongylus vulgaris is responsible for :
b. 8-10 ml
470. Brooder‘s pneumonia in poultry is caused
(a) Colic in ruminants
(b) Paralysis in horse by- c. 25 -30 ml.
465. In Johne‘s disease, corrugation is not the
(c) Colic in horse
feature in- (A) Candida albicans d. 30-40 ml
(d) None
460 Bunostomum trigonocephalum is hook (A) Cattle (B) Aspergillus fumigatus
worms of :
(B) Sheep (C) Haemophilus paragallinarum
(a) Sheep and Goat
(b) Cattle (C) Horse (D) Pasturella multocida
(c) Dog
(d) None (D) Both (B) and (C) 471. The best general anesthetic for canine is 475. The best inhalation anesthetic for closed
461. Disease caused by Clostridium septicum circuit is
is- 466. Most susceptible species for Haemorrhagic a. Xylazine
septicemia- a. Cyclopropane
(A) Black Quarter b. Thiopental
(A) Sheep b. Ether
(B) Enterotoxaemia c. Acepromazine
(B) Buffalo
(C) Braxy d. Ketamine c. Nitrous oxide
(C) Cattle
(D) Tetanus 472. To deliver live pups, cesarean section is d. Chloroform
(D) Pig performed under
462. Sulphur granules in yellowish pus is seen 476. Deep chested dog is prone to following
in- 467. Erythritol sugar plays important role in a. Thiopentone surgical condition
pathogenesis of-
(A) Glanders b. Pentobarbitone a. Intestinal obstruction
(A) Clostridium spp.
(B) Strangles c. Acepromazine b. Diaphragmatic hernia
(B) Brucella spp.
(C) Staphylococcosis d. Diazepam + Local anesthesia c. Gastric dilatation and torsion
(C) Bacillus spp.
(D) Actinomycosis d. Gastric ulcer
(D) Corynebacterium spp.
463. Toxins of organism causes peripheral 477. Pathognomic sign of sharp molar is
nerve paralysis in cattle-
a. Quidding a. Right atrium c) Promazine
b. Left atrium
b. Halitosis c. Interventricular septum d) Reserpine
d. Interatrial septum
c. Salivation 483. What hypophyseal structure receives 496. Xylazine does not have the following effect
signals from the hypothalamus via the
d. All of above hypophyseal portal system? a) Analgesic

478. The development of pouch in lower neck a. Follicular medulla 491. In body netobimin is b) Muscle relaxant
just after feeding in case of buffalo is b. Adenohypophysis converted into
noticed in c) Sedative
c. Neurohypophysis
d. Pars intermedia a) Albendazole
a. Pyloric stenosis d) Antipyretic
484. Low blood glucose typically results in the
secretion of all of the following EXCEPT: b) Mebendazole
b. Oesophageal ulcer
a. Glucagon c) Fenbendazole
c. Choke
b. Thyroxine
d) Lobendazole
d. Oesophageal diverticulum c. hGH 497. Which one of the following has high water to lipid
d. PTH 492. Which one of the followings not an partition coefficient?
479. Continuous lacrimation in canine is 485. What hormone increases intestinal calcium
anaesthetic?
characteristic sign of absorption? a) Chlorpromazine
a) Phencyclidine b) Ether
a. Obstruction of lacrimal duct a. Calcitriol c) Halothane
b. Calcitonin b) Xylazine d) None of the above
b. Entropion c. Parathormone
498. With which of the following
d. Pancreatic polypeptide c ) Ketamine
c. Conjunctivitis 486. The ______ cells of the pancreas secrete anaesthetic is eructation reflex not affected.
insulin. d) Cyclopropane
d. Ectropion a) Ketamine
a. Chief 493. Diazepam does not possess the following
480. Myringotomy in canine is performed for b) Xylazine
b. Principal action
the correction of c. Alpha c) Phenobarbitone
d. Beta a) Sedative
a. Otitis interna 487. Oxytocin is secreted by the: d) Equithesin
b) Anticonvulsant
b. Otitis media a. Adenohypophysis 499. Which of the following statements is false?
b. Neurohypophysis c) Analgesic
c. Otitis externa c. Zona glomerulosa a) Thiopentone is ultra short acting
d. Pars intermedia d) Anxiolytic
d. Othaematoma barbiturate
488. Hyposecretion of cortisol can cause: b) Thiopentone is administered by
494. The full life of drug is usually approximately
481. Each of the following is an amino acid intramuscular route
a. Cretinism
derivative EXCEPT: a) Twice its half-life c) Thiopentone administered typically
b. Diabetes mellitus
c. Diabetes insipidus b) Ten times its half- life shows barbiturate apnoea.
a. Epinephrine
d. Addison's disease d) Thiopentone is yellowish powder used as
b. Melatonin
489. The space in the middle of the thoracic c) Five times its half- life a sodium salt.
c. Thyroxine
d. TSH cavity where the heart resides is the: 500. Which one of the following is an example
482. Which of the following hormones does not d) None of the above of physical antagonism?
a. Pericardial cavity
act via a second messenger system? b. Pericardium 495. The following is not a phenothiazine a) Administration of activated charcoal in
c. Pleural Cavity derivative poisoning
a. Glucagon
d. Mediastinum
b. Epinephrine b) Relief of acidity
490. The foramen ovale in the fetal heart is a) Triflupromazine
c. GH using antacids
d. Testosterone located in the:
b) Chlorpromazine
c) Administration of 509. Spiti is the horse breed found d. All of above 525. The source of ―Reverse transcriptase‖
Atropine in organophosphate at enzyme used for c-DNA synthesis is:
poisoning a. J&K 517. Calf hood vaccination is advisable for a. Moloney murine leukemia virus
b. Himachal Pradesh a. Brucellosis (MuLV)
d) None of the above c. Hariyana b. Salmonellosis b. Avian myeloblastosis virus (AMV)
d. Punjab c. Pasteurellosis c. Both
501. The body region of the sheep in which best 510. The disease found in sheep d. Neonatal calf diarrhoea d. None
quality wool is found during monsoon 518. The germ tube production is characteristic 526. Quantitative studies using PCR technique
a. Breech a. Foot rot of can be done with:
b. Shoulder b. F. M. D. a. Candida albicans a. RT-PCR
c. Head c. Enterotoxaemia b. Corynebacterium pyogenes b. Real Time PCR
d. Tail d. Rinderpest c. Cryptococcus neoformans c. Micro array
502. A growing female sheep between sixth 511. Bacteria which require living medium for d. None of the above
their growth is d. Pseudomonas aeruginosa 527. Most type II restriction endonucleases
month to maturity.
a. Mycoplasma and Leptospira 519. Experimentally Mycobacterium leprae recognize and cleave DNA within particular
a. Doeling sequence of 4-8 nucleotides which have two
can be cultivated on
b. Wedder fold rotational symmetry. Such sequences are
b. Chlamydia and Rickettsia a. Bacterial media with mycobactin.
c. Hogget called:
b. Cell culture system.
d. Lamb a. Palindromes
c. Rickettsia and Mycoplasma c. Nine banded armadillo
503. Native place of sannem is b. Short tandem repeats
d. None of the above
a. England d. None of these c. Both
520. Viruses that exist in cells and cause
b. Switzerland d. None
recurrent disease are considered
c. U. S. A. 512. Pallisade arrangement is characteristic of 528. For restriction analysis of DNA molecule,
a. Oncogenic
d. China a. Corynebacterium the type of ―restriction endonucleases‖ used
b. Cytopathic
504. Major breeding season of Indian sheep is are:
c. Latent
a. June to august b. E.coli a. Type I RE
d. Resistant
b. December to January c. Campylobacter b. Type II RE
521. For transformation reactions competent E.
c. March to april c. Type III RE
coli cells can be prepared by treating log phase
d. Year round d. Listeria d. None
E. coli cells with:
505. The term used for meat of rabbit is 513. Mycoplasma organisms are pleomorphic 529. c-DNA can be synthesized on a RNA
a. Calcium Chloride
a. Mutton in nature due to template using following primers:
b. Magnesium Chloride
b. Pork a. Absence of cell wall a. Sequence specific primers
c. EDTA
c. Chevon b. Oligo dT
d. None of the above
d. Venison b. Absence of rigid cell wall c. Random hexamers
522. A DNA molecule from external source can
c. Small in size be inserted into the host cells by following d. All of the above
d. Species specific in nature methods: 530. Isoschizomers are Restriction enzymes
514. Periodic opthalmia in horses is a sequel of a. Heat shock treatment which:
506. Location of rabbit breeding farm in a. Glanders b. Electroporation a. Recognize and cut the same sequence
Gujarat b. Mycoplasmosis c. Lipofection b. Recognize the same sequence but cut
a. Bhuj c. Equine leptospirosis d. All of the above site vary
b. Ankleshwar d. Babesiosis 523. In a PCR reaction two short c. Both
c. Morbid oligonucleotide used which flank the DNA d. None
d. Mandvi sequence to be amplified is called: 531. What is the name of the main lymphatic
507. India has total number of sheep breed a. Primer duct that arises in the abdomen?
a. 40 515. Chlamydia can be stained with following b. Probe
b. 15 stains except: c. Template a. tracheal duct
c. 20 a. Gram‘s stain d. None of the above
d. 28 524. The source of Taq polymerase used in the b. cisterna chyli
b. Macchiavello stain
c. Gimenez stain PCR reaction is:
508. Gurej sheep breed found in d. Castaneda stain a. E. coli c. right lymphatic duct
a. J&K 516. Tuberculin test is based on b. Thermus aquaticus
b. Himachal Pradesh a. Delayed hypersensitivity c. Both
c. Hariyana b. Arthus reaction d. None d. cisterna magna
d. Punjab c. Anaphylactic reaction
532. Which part of the respiratory system is 537. The space between the incisors and the d. Epitendineum
also responsible for the production of cheek teeth of rabbits and rodents is
sound? known as the 542. Which one of the following is a 547. The response to selection increases when
exoskeleton?
a. hyoid apparatus a. diastema (a) Proportion of individuals selected
a. Hoof decreases
b. eustachian tube b. philtrum
b. Metacarpus (b) Proportion of individuals selected
c. pharynx c. dewlap increases
c. Sternum
d. larynx d. acromion (c) Heritability of the trait is low
d. Ulna
533. Which muscle is responsible for increasing 538. Visceral skeleton present in camel (d) None of the above
the volume of the thoracic cavity during 543. Which one of the gland is a apocrine
a. Os penis gland? 548. The heritability of a character in narrow
inspiration?
sense will be more when it is
a. diaphragm b. Os phrenic a. Mammary gland controlled by
(a) Genes with non-additive effects
b. hypxial c. Os cardis b. Testes

c. external oblique d. Os rostrum c. Parotid salivary gland (b) Genes with additive effects

d. epaxial d. Pancreas (c) Both (a) and (c)

534. The basin shaped structure in the center of 539. Which is the type of placenta present in 544. Part of the peritoneum that covers the
the kidney is called bitch? fallopian tube with abdominal cavity: (d) None of the above

a. cortex a. Zonary a. Mesometrium

b. hilus b. Diffuse b. Mesosalphinx

c. pelvis c. Discoidal c. Mesoovarium 549. The genotypic value of quantitative traits


d. Cotyledonary d. Broad ligament is
d. medulla
540. Total number of incissor teeth in ox 545. Which of the following plays a part in (a) Sum total of effects
535. In which layer of the skin are the sensory
nerve endings found? thermoregulation?
a. Six
a. Hair (b) sum total of gene effects
a. hypodermis b. Four
b. Claws (c) Sum total of additive effects
b. epidermis c. Eight
c. dermis c. Sebaceous gland
d. Ten (d) None of the above
d. subcutis d. Meibomian gland
550. The specific combining ability of a line is
536. Which is the most developed special sense 546. Which of the following is used for due to gene effects
in the birds? predicting future performance of individuals
(a) Epitasis
541. Outer covering of the nerve is (a) Heritability
a. sight
a. Endoneurium
b. touch (b) Repeatability (b) Dominance
b. Perineurim (c) Genetic correlation
c. smell (d) None of the above (c) Over-dominance
c. Epineurium
d. taste
(d) All of above (b) Family selection 560. Diploid number of chromosomes in sheep
(c) Pedigree selection is
551. The dry period in buffaloes can be reduced
by (a) 60
(d) Any of these 565. Androgen Binding protein is secreted by
(a) Individual selection (b) 54
556. The regression of offspring on mid parent a) Ley dig cells
provides the estimate of (c) 38
(b) Better management b) Sertoli cells
(a) ½ h 2 (d) 64
(c) Family selection c) Spermatogonia
(b) ¼ h2 561. Both hind limbs retained in the uterus
beneath the body of the posteriorly d) Efferent ducts
(c) h2
(d) Progeny testing presented fetus 566. Eversion of galeacapitis and crater shaped
(d)  h depressions in the nucleus
a) Dog sitting posture
552. The change in the fat % when selection is a) Dag defect
made for higher milk yield b) Breech presentation
is due to c) Poll presentation b) Diadem defect
557. The accuracy of estimating breeding value c) Corck screw defect
(a) Natural selection d) Wry neck
of a sire increased by
(b) (b) Direct selection 562. Acute angulation of the vertebral column d) Abaxial defect
(a) Decreasing the number of
(c) Indirect selection of the fetus causing dorsal approximation
sires under test 567. Effecting a change in the presentation is
(d) None of these (b) Decreasing the number of of its head and tail is a known as
progeny of sire
553. The estimate of heritability of egg yield (c) Increasing the number of a) Perosomus elumbis a) Rotation
cannot be obtained by progeny of sire
(d) None of the above b) Amorphus globosus b) Version
(a) Paternal half-sib correlation 558. The Osborne index for cockerel selection
c) Otter calf c) Extension
is based on information from
d) Schistosomus reflexus d) Flexion
(b) Regression of daughter on dam (a) Individual performance and
dam and sire families. 563. The release of which hormone is 568. Non dilation of cervix causing dystocia in
(c) Regression of daughter on sire associated with the Fergusion‘s reflex .
(b) Dam and sire families ewes is known as
a) Estroegn a) Vertex
(d) Maternal half-sib correlation
(c) Progeny and pedigree b) Oxytocin b) Foot nape
554. The precision of heritability estimate is
known by its (d) Individual performance and c) Progesterone c) Ring womb
(a) Standard error pedigree
d) Relaxin d) wry neck
(b) Magnitude 559. The generation interval can be reduced by
using 564. Penile Transmissible Neoplasia is 569. Pattern assumed by the semen upon drying
(c) Method of estimation observed in the on a glass slide
(a) Genetic markers
(d) Deviation from a) Ram a) Fern pattern
555. For selection of individuals for traits b) Stallion
(b) Progeny information b) Crenellation pattern
measured after life, we will prefer
(c) Life time information c) Dog c) Cork screw pattern
(a) Indirect selection
(d) Multi-trait selection d) Bull d) Zig Zag pattern
570. Test used to know the functional integrity 576. Intense educational activity for motivation 586. AMUL was established in
of sperm membrane and mobilization of community to action
Is known as 581. Chepest, Oldest & most effective aids of (A) 1846
a) Sperm mucus penetration test teaching is
(A) Awareness (B) 1848
b) Hypo osmotic swelling test (A) Black Board
(B) Campaign (C) 1850
c) Hamster egg penetration test (B) White board
d) High temperature viability test (C) Demonstration (D) 1950
(C) Display board
(D) G.D.
571. The end of the diestrus period is due to (D) Interactive board
a. Recruitment of the ovulatory 577. Diagram presentation of facts or ideas is
follicular wave. known as 582. A Statement of assumption validity of
which is yet to be tested is known as 587. Air and water are nature of goods
b. A decrease in estradiol negative
feedback. (A) Chart (A) Material
c. Regression of the corpus luteum. (A) Hypothesis
d. An increase in estradiol levels (B) Poster (B) Nominated
(B) Theory
coming from the preovulatory follicle.
572. In the mare, transition from the non- (C) Diagram (C) Free
(C) Principles
breeding to the breeding season is caused by
(D) Book (D) Consumable
a. A decrease in melatonin. (D) Philosophy
b. A decrease in the day length. 578. A statement of policy to guide decision
c. A switch from positive to negative 583. A miniature replica of an object is called 588. The demand is salt is
feedback by estradiol. and action in a consistent manner is
d. The presence of progesterone (A) Specimen (A) Inelastic
(A) Principle
coming from the corpus luteum.
(B) Model (B) Elastic
573. Granulosa cells are similar to Sertoli Cells (B) Law
in that both (C) Prefect elastic
a. Have FSH receptors. (C) Philosophy (C) Sample
b. Have LH receptors. (D) Non elastic
c. Produce testosterone. (D) Hypothesis (D) Picture
d. Have very low levels of cAMP.
584. A System of social relationship in and
574. The blood testis barrier is due to
a. The inhibition to growth of blood through which we live is known as
vessels in the seminiferous tubules. 579. Call mode by farmers at the veterinary
hospital for getting information is called (A) Society
b. The presence of gap junctions
between Sertoli cells. (B) Communication
c. The presence of tight junctions (A) Office call 589. AGMARK indicates
between Sertoli cells. (C) Cast
d. The basement membrane between (B) Personal call (A) Quality & Purity
the Sertoli cells and the interstitial (D) Creed
(C) Community call (B) Quality
space where the Leydig cells are
located. 585. Combination of seeing, hearing, and (C) Quantity
(D) Tall call
575. It has been demonstrated that dogs can be doing may help to retain----------in
trained to detect estrus in cattle. Which of the 580. A visual display which is never used teaching Learning (D) Purity
following senses is being used by the dog to
alone for teaching programmed is
accomplish this task? (A) 50% 590. Top level function is known as
a. Smell (A) Chart
b. Sight (B) 60% (A) Management
c. Touch (B) Flipchart
d. Hearing (C) 70% (B) Administration
(C) Poster
(D) 80% (C) Bureaucracy
(D) Banner
(D) Line of control b. 3 – 6 kg body cavity. Such hen are known as a. Cage house
c. 1 – 2 kg b. Deep litter house
591. Herd registration scheme in Gujarat is d. 0.5 – 1 kg a. Poor layer c. Both A & B
running for the breed 600. Stag is the term utilize for b. External layer d. None of the above
a. Mehasana buffalo a. Castrated goat c. Internal layer
b. Gir cattle b. Castrated horse d. Faulty layer Pullets grown during the period when most
c. Dangi cattle c. Castrated sheep Pigments responsible for egg shell colour is
d. Banni buffalo d. Castrated pig 609 produced by 616 of the days have decreasing light are known
592. The number of calving pen required on as
601. Tongue rolling is the type of behaviour in
dairy farm depends on cow a. Magnum a. In season flocks
a. Breedable females a. Detrimental b. Vagina b. Out season flocks
b. Fertility rate b. Stereotyped c. Uterus c. Mid season flocks
c. Calving interval c. Retired d. Isthmus d. Odd season flocks
d. All of above d. Apathetic The brown colour of eggshell is due to
593. Dehorning in calves can be done at 602. Pig farming is the most developed in 610 which pigments
a. 1 to 5 days Pullets grown during the period when most
a. Himachal Pradesh
b. 7 to 0 days b. West Bengal a. Carotenoids 617 of the days have increasing light are known
c. 11 to 20 days c. Bihar b. Porphyrin as
d. More than 30 days d. Madhya Pradesh c. Xanthophylls
594. The pulsation ratio in milking machine 603. Certification agency in organic farming is d. None of above a. In season flocks
should be kept within the limit of b. Out season flocks
a. IARI 611 Preheating or pre-warming of egg is done c. Mid season flocks
a. 1 : 1 b. NDRI
b. 2.5 : 1 c. APEDA a. Before candling d. Odd season flocks
c. Both a & b d. NPDP b. Before Cold storage
d. None 604. Hissardale crossbred sheep breed c. Before Setting Birds were placed in the laying house just
d. Before Hatching
developed from 618 before the onset of egg production is known
595. The major input for maximum contribution a. Australian merino Eggs of average size and quality incubated as
to cost of milk production b. Bikaneri in air with 50 to 60 % relative humidity,
a. Labour c. Both 612 they will lose approximately _____ % of a. Housing
b. Electricity d. None of above their initial weight in 19 days of incubation b. Shifting
c. Feeding c. Laying
d. Breeding a. 6 % d. None of above
596. The buffalo population (Millions) in 605. Fat tailed sheep breed found in India is b. 12 % Replacing old males in a flock with a set of
Gujarat state as per livestock census 2007 a. Macheri c. 18 % new and younger male after about two-
a. 8.77 b. Chokla d. 24 % 619 thirds of the egg production period is known
b. 7.97 c. Bhakarwal Best time for artificial insemination to get as
c. 2.01 d. Marwari 613 maximum fertility in poultry is
d. 4.64 A narrow band surrounding the yolk that is a. Replacement of flock
606 almost void of blood vessels is known as a. Early morning b. Spiking the flock
b. Before noon c. Placement of flock
a. Stream c. After Noon d. All of above
b. Stigma d. Late evening Distance between the bulbs in poultry house
c. Strake
597. Nilgai belong to the animal in category 620 should be _____times the distance from the
a. Wild ruminant d. Strike The ammonia concentration in the poultry bulb to the bird level.
b. Wild non-ruminant Chickens lay eggs on successive days is 614 house should not be more than
c. Domestic ruminant 607 known as a. 0.5 times
d. Domestic non-ruminant a. 2.5 ppm b. 5.0 times
598. The gestation period (days) of leopard is a. Pause b. 25 ppm c. 15 times
a. 62 – 65 b. Clutches c. 50 ppm d. 1.5 times
b. 92 – 95 c. Persistency d. 75 ppm 621. Herpes simplex seen in….
c. 112 – 120 d. Both b and c
Sometime the infundibulum loses power to a) Cattle
d. 150 -180 608 615 Nitrogen fixation is poor in
599. The birth weight of piglet in kg is pick up a yolks, and yolks accumulate in
a. 2 – 4 kg b) Dog
c) Elephant 633. Ozone layer found at which height (b) Caecal worm
(c) Stomach worm
d) None of above a)10-20 km (d) None
638. The condition ‗Sweating blood‘ in horse is
622. Volumetric method for determination of 628 Example of cyclozoonosis… b)70-80km caused by :
fat% of milk.
a) Toxoplasmosis c)40-50km (a) Parafilaria equi
a. Waste fall balance (b) Dirofilaria immitis
b) Taeniasis d)90-100km
(c) Parafilaria multipapillosa
b. Soxhlet method (d) None
c) Echinococcosis
639. Seteria labiato-papillosa of cattle causing :
c. Richmond sliding method
d) Both b) & c)
(a) Lumber paralysis in horse
d. Garber method.
629. Example of algal zoonosis 634. Optimum temperature range for adult pig (b) Lumber paralysis in sheep and goat
623. Which of the disinfectant is effective is between (c) Lumber paralysis in cattle
a) Scrub typhus (d) None
against wide range of micro organism in
a) 0-3 0c 640. The largest nematode of domestic animal
floor,wall and equipments b) Actinomycosis is :
a)quick time b) 40-500c
c) Fascioliasis (a) Macracanthorhynchus hirudinaceus
b)cresol 2-3% in hot water c) 4-300c (b) Dioctophyma renale
c)costic soda 5% d) Protothecosis (c) Ascaris sum
d)Nil d)30-350c (d) None
630. In anthrax by which reaction Bacillus 641. Acute fasciolosis is common in :
624. Coagulation of water by chemical is
organism detected in blood smear 635. Which is Iron bacterium
essential in
(a) Cattle
a) Mac Fadyean‘s reaction a)Crenothrix (b) Sheep
a)slow sand filter
b)rapid sand filter (c) Buffalo
b) Zeil Neelsen staining b)Gellionella (d) Horse
c)boiling 642. Triclabendazole is a drug of choice for :
d)None c) Ascoli‘s precipitation test c)Klebsiella
625. The desirable limit for fluoride should (a) Liver fluke
d) None of above d)Both a) & b)
be(mg/litre) (b) Haemonchus nematode
631. Medium which is used in isolation of (c) Lung worm
a) 0.2 (d) None
T.B. organism
b) 1.5 643. The blood fluke are usually present in :
c) 2.5 A )XLD agar
(a) Hepatic vein
d) 0.6-1.8 (b) Portal vein
626. In strong sewage B.O.D. is (mg/litre) b) LJ medium
(c) Mesenteric vein
c) EMJH medium (d) None
a) 50 636. Incineration of carcass done by heating at
b) 100 d) blood agar which temperature
c) 300
d) None 632. Greenish colors of water is developed a)6000c
627. Who was first chief of VPH in WHO due to…
b)8000c
a) Iron 644. Taenia multiceps is tape worm of dog and
a) Dr.Guerin
c)13000c it‘s larval stage present in sheep/cattle
b) Dr.Daniel E.Salmon b) Organic matter
d)10000c is known as :
c) Dr.Karl F.Meyer c) Algae flora
637. Oxyuris equi are commonly known as : (a) Coenurus serialis
d) Dr.Martin Kaplan d) None of above (b) Hydatid
(a) Pin worm (c) Coenurus cerebralis
(d) None 652. Circling disease in cattle is caused by- (C) Mode of transmission (D) Mucosal disease
645. Sputum is a diagnostic material in two
infections of dog (A) Listeria monocytogenes (D) Pathogenesis 663. Black head disease is predominately a
disease of-
(a) Ancylostoma caninum and Dictophyma (B) Erysipelothrix rhusiopathiae
renale (A) Cattle caused by Parasite
(b) Filaroides and Angiostrongylus (C) Streptococcus equi 658. Which bacterium is predisposed by
(c) Schistosoma nasale and Stephanurus Fasiola hepatica infestation- (B) Poultry caused by Parasite
(D) Chlamydia psittacii
(d) None
646. Smallest and most pathogenic tapeworm of (A) Bacillus spp. (C) Horse caused by Virus
653. Intranuclear inclusion bodies are seen in-
poultry is : (B) Clostridium spp. (D) Pig caused by Virus
(A) Pox diseases
(a) Amoebotaenia (C) Leptospira spp. 664. Which of the following is correctly
(b) Davainea proglotina (B) Herpes virus infection matched-
(c) Raillietina echinobothrida (D) Pasturella spp.
(d) None (C) Adeno virus infection (A) Tubercular lesions are calcified-
647. A common term for myiasis caused by 659. Which is the most potent aflatoxin-
(D) Lyssa virus infection Buffalo
members of the calliphoridae is :
(A) M1 (B) Johne‘s disease-Foul smelling diarrhea
(a) Blow-fly Strike 654. Negri bodies are seen in Rabies which are-
(b) Pediculosis (B) M2 (C) Avian spp.- Dry pus
(A) Intranuclear
(c) Mange
(d) None (C) B1 (D) Lamb dysentery- Clostridium
(B) Intracytoplasmic
648. The larval of Oestrus ovis are commonly perfringens type D
(D) B2
referred to as : (C) Both
660. Curled toe paralysis in chicken is due to 665. Post mortem of chick shows foul smelling
(a) Gnats (D) May be intranuclear or yellow-brown watery Yolk, fibrinous
(b) Warble deficiency of-
intracytoplasmic perihepatitis
(c) Bots
(A) Vitamin B12
(d) None 655. Enlargement of Bursa of fabricius in and pericarditis, suspect the disease
649. The condition ‗false gid‘ in sheep is poultry is seen in- (B) Vitamin B1
caused by : (A)Infectious Coryza
(A) CRD (C) Niacin
(a) Multiceps multiceps (B) Fowl Typhoid
(b) Oestrus ovis larva (B) IB (D) Vitamin B2
(c) Setaria digitata (C) Coli Bacillosis
(d) None (C) RD 661. Mn deficiency is chicken will lead to-
650. Ornithodorous moubata is : (D) Infectious Bronchitis
(D) IBD (A) Pica
(a) Hard tick
(b) Soft tick 656. Zebra marking is predominant feature of- (B) Star grazing condition
(c) Mites
(d) Flea (A) Johne‘s disease (C) Crazy chick disease
666. Haemorrhages at the tip of the
651. Animal Protozoa are unicellular and they
(B) Tuberculosis (D) Slipped Tendon proventricular gland is the pathognomic lesion s
are :
seen in-
(C) Rinder pest 662. Phosphorous deficiency in soil will
(a) Eukaryotic
predispose the cattle to- (A)Ranikhet disease
(b) Prokaryotic (D) Both (A) and (C)
(c) Monera (A) Haemorrhagic septicemia (B) Chronic respiratory disease
(d) None 657. CBPP differs from CCPP in-
(B) Botulism (C) Infectious bursal disease
(A) Both occur in same species
(C) Anthrax (D) Avian influenza
(B) Sequestra formation
667. The non-invasive confirmative diagnosis 672. Common site for obstructive uroliths in d. Autoclave
of diaphragmatic hernia in buffalo is bullock is
677. Sweeny is characterized by atrophy of
a. Auscultation a. Prescrotal
a. All muscles
b. Contrast radiography b. Neck of bladder
b. Gastrocnemious muscle 682. Which blood vessel does NOT bring
c. Rumenotomy c. Pelvic urethra deoxygenated blood directly to the heart?

d. Plain radiography d. Post scrotal c. Scapular muscle a. Pulmonary vein


b. Coronary Sinus
673. The surgical correction of teat pea in cattle d. None of above c. Inferior Vena cava
is performed by d. Superior Vena Cava
668. Liptek test is performed for the diagnosis 678. Laminitis means that 683. If there is a blockage between the AV
of a. Teat slitter node and the AV bundle, how will this
a. Animal walks lame
affect the appearance of the EKG?
a. LDA b. Hudson teat spider
b. Inflammation of joints
a. PR interval would be smaller
b. Vagal indigestion c. Teat tumour extractor
c. Inflammation of laminae b. QRS interval would be shorter
c. Ruminal acidosis d. Teat dilator c. There would be more P waves than
d. None of above QRS complexes
d. Ruminal tympany d. There would be more QRS complexes
679. Treatment of upward fixation of patella than P waves
669. Metallic foreign bodies are mostly 684. A valve damaged by rheumatic fever fails
recovered from a, Planter tenotomy to open completely. This is known as:
674. Umbilical hernia is most commonly seen
a. Esophagus in b. Medial patellar desmotomy a. Stenosis
b. Heart Block
b. Reticulum a. Cattle c. Middle patellar desmotomy
c. Ischemia
d. MI
c. Rumen b. Dog 685. Accodrding to the Frank-Starling Law of
d. Cunion tenotomy
the heart, CO is directly related to:
d. Abomasum c. Pig
680. Inflammation of stifle joint is known as
a. Ventricular muscle mass
670. The basic cause of vagal indigestion in d. Horse b. Heart Rate
cattle is a. Laminitis
c. Amount of blood returning to the heart
675. During surgery arterial bleeding is
b. Omarthritis d. ESV
a. Trauma controlled by 686. The T Wave on an EKG represents:
b. Inflammatory conditions a. Haemostat c. Gonitis
a. Ventricular Depolarization
d. Joint mice b. Ventricular Repolarization
c. Ruminal impaction b. Thumb forceps
c. Atrial Depolarization
d. Liver dysfunction c. Allies tissue forcep 681. Oozing of inflammatory exudates at d. Atrial Repolarization
coronary band in horse hoof is known as e. Ventricular Systole
671. Distal intestinal obstruction in bullock is d. Rat tooth forceps 687. Cardiac output is equal to:
diagnosed by a. Arthritis
676. The best technique for making instruments a. HR x SV
a. Clinical signs germ free is b. Side bones b. HR/SV
c. EDV - ESV
b. Hematology a. Direct flame c. Quittor d. (EDV-SV) x HR
688. Which of the following represents the
c. Per rectal examination b. Boiling water d. Curb
outermost layer of the heart?

d. History c. Chemical a. Epicardium


b. Parietal pericardium
c. Myocardium b. Brain a) Dopamine
d. Subendocardium c. Spinal cord
689. Which of the following cell types is d. Nuclei b) Neostigmine 706. Furosamide causes all except one
responsible for skeletal muscle 696. A neuron with many short dendrites and a
single long axon is a: c) Atropine a) Metabolic alkalosis
regeneration?
a. Multipolar neuron d) Benzodiazepam
a. Myoepithelial cell
b. Myofibril b. Bipolar neuron b) Hypocalcaemia
701. Which of following is used for relief of heaves in
c. Satellite cell c. Unipolar neuron
d. None of the above horse? c) Hypokalemia
d. Myofibroblast
697. Graded Dose- Response curve has
a) Oxytocin
a) Shape of hyperbola on simple graph d) Oligouria
b) Atropine
paper and 'S' shape on semi- log paper
690. How many T-tubules lie within a single 707) Generally, which of the following is in the
c) Methanol correct order as dosage is increased?
skeletal muscle sarcomere? b) Shape of 'S' on both simple and semi-log
a) ED50 < LD50 < TD50
graph paper d) Frusemide
a. 1 b) ED50 < TD50 < LD50
b. 2 c) Shape of 'S' on simple graph paper and 702. Which of following drug increases blood c) LD50 < TD50 < ED50
c. 3 shape of hyperbola on semi log paper. pressure, heart rate and force of contractions. d) LD50 < ED50 < TD50
d. 4 e) TD50 < LD50 < ED50
691. The connective tissue layer that bundles d) Shape of hyperbola on both simple and a) Epinephrine 708) Which of the following is considered the
skeletal muscle fibers into fascicles is the: semi log graph paper. therapeutic index (or ratio)?
b) Atropine a) T.I. = TD50 / ED50
a. Perichondrium
b. Perineurium c) Laetolol b) T.I. = LD50 / ED50
c. Perimysium
d. Epimysium d) Pindalol c) T.I. = ED50 / TD50
692. An overlap of actin and myosin filaments
703. Which of followings is a not a saline diuretics. d) T.I. = ED50 / LD50
occurs in the:
698. Which of the following inhibits uptake of
acetycholine in to vesicles a)Magnesium sulphate e) A & B
a. A Band b)Mannitol
b. I Band c)Sorbitol
c. Z Line a) Vesamicol 709) Which of the following is considered the
d)Acetazolemide brand name?
d. H Band 704. What is the site of action of carbonic
b) Cobra toxin a) Paracetmol
693. Which of the following does not describe
anhydrase inhibitors? b) Crocin
skeletal muscle fibers? c) Bungaro toxin
c) ß-blocker
a) Throughout the length of the tubule
a. Striated b) Loop of Henle c) ―off label‖ use
b. Typically voluntary d) Botulinum toxin c) PCT d) Antipyretics
c. Multinucleate d) DCT 710) Which of the following is NOT a protein
d. Branched 699 Which of following is G-protein coupled 705. In which of the following animals emetics target for drug binding?
694. Which of the following cells is reponsible receptors? a) Side of action (transport)
are not used?
for myelin formation in the peripheral b) Enzymes
nervous system? a) Muscurinic a) Rats c) Carrier molecules
b) Cattle d) Ion channels
a. Astrocyte b) Nicotinic c) Horse
b. Oligodendrocyte 711. The general steps in the viral
d) All of the above
c. Schwann cell c) Alpha adrenergic receptors multiplication cycle are:
d. Microglial cell a. Adsorption, penetration, replication,
695. The peripheral nervous system includes d) a and b maturation and release.
the: b. Endocytosis, penetration, replication,
700. Which of following is used in the treatment of assembly and lysis.
a. Somatic nervous system myasthenia gravis
c. Adsorption, uncoating, replication, c. Adeno virus 729. Which of the following has the highest d. None of the above
assembly and budding. d. Parvo virus density: 737. At 600 nm, one unit optical density (OD)
d. Endocytosis, penetration, replication, 721. Swine Influenza virus associated with a. Relaxed genomic DNA of E. coli culture corresponds roughly to:
maturation and exocytosis. current pandemic in humans is b. Supercoiled DNA a. 1 X 106 cells/ml
712. In dog canine adenovirus infection a. H1N1 c. Plasmid b. 1 X 107 cells/ml
produces b. H2N2 d. RNA c. 1 X 108 cells/ml
a. Pink eye c. H3N2 d. 1 X 109 cells/ml
b. Blue eye d. H5N1 738. EDTA present in lysis solution has
c. Pearly eye following functions:
d. None a. Chelates Mg++ ions and thus inhibits
730. Homo polymer tail can be added by using
713. Choose the laboratory animal most the activity of enzyme DNase
enzyme:
suitable for typing FMD virus b. Removes Mg++ ions that are essential
a. Ligase
a. Guinea pig for preserving the overall structure of
722. Chief source of leptospira is b. Phosphate kinase
b. Hamster cell envelope
a. Blood c. Terminal deoxytransferase
c. Weaned mice c. Both of the above
b. Urine d. None
d. Rat d. None of the above
c. Milk 731. The plasmid which is maintained in the
739. Which of the following match is incorrect:
d. Faeces host cell in multiple copies are called:
714. Biological vector of Bluetongue virus is a. SDS: Cell Lysis
723. Growth of brucella organisms is favored a. Relaxed
a. Aedes b. EDTA: Chelating Mg++ ions
due to b. Stringent
b. Culicoides c. Proteinase K: Degradation of Protein
a. Erythritol c. Conjugative
c. Anopheles d. Isoamyl alcohol: precipitation of DNA
b. Sorbitol d. None
d. House fly 740. CTAB used is isolation of DNA forms
c. Glucose 732. The plasmid which is maintained in the
715. Double stranded segmented RNA with 10- complexes with:
d. Protein host cell in limited number of copies are
12 segments is the features of family a. DNA
724. E.coli called:
a. Reoviridae b. Proteins
a. Grows at 15-400 C. a. Relaxed
c. Carbohydrates
b. Retroviridae b. Stringent
b. Lactose fermenter d. None of The above
c. Orthomyxoviridae c. Conjugative
c. Motile 741. Guanidinium thiocynate is useful in DNA
d. Paramyxoviridae d. None
d. All of these isolation because:
716. First anti rabies vaccine was developed by 733. RNAs that catalyze biological reactions,
725. Spore forming bacteria a. It forms complexes with DNA
a. Edward Jenner such as self-splicing introns, are known as:
a. Bacillus molecule
b. Louis Pasteur a. Enzyme
b. Clostridium b. It denatures and dissolves all
c. Robert Koch b. Ribozyme
c. Both a and b biochemical substances other than
d. Robert Hook c. Sliceosome
d. None nucleic acid
717. Separation of RBCs from virus is called d. None
726. Isoschizomers are Restriction enzymes c. In its presence DNA binds tightly to
a. Elution 734. The 2 µm plasmid is found in:
which: silica particles
b. HA a. Escherichia coli
a. Recognize and cut the same sequence d. Both b & c
c. HI b. Pneumococcus
b. Recognize the same sequence but cut 742. Which one of the following type of
d. Eclipse c. Bacillus anthracis
site vary
718. The viral agent produces diphagic fever, d. Sacchromyces cerevisiae placenta is found in goat?
c. Both
respiratory distress, nervous symptoms and 735. Which of the following statement is true
d. None a. Zonary
hard pad disease in dog is for life cycle of Lysogenic phages:
727. Neoschizomers are Restriction enzymes
a. Canine distemper virus. a. They immediately induce lysis of host
which: b. Cotyledonary
b. ICH virus cells for release of new virions
a. Recognize and cut the same sequence
c. Rabies Virus b. Phage DNA is integrated with the host
b. Recognize the same sequence but cut c. Diffuse
d. IBH virus DNA and retained for several
site vary
719. Virus having RNA dependent DNA generations
c. Both d. Discoidal
polymerase c. Both the above depending upon the
d. None
a. Retrovirus environmental condition 743. The lymphatis are absent in
728. The enzymes used for joining two DNA
b. Reovirus d. None of the above
molecule is:
c. Rabies virus 736. Genes cloned with M 13 based vector can a. Intestine
a. DNA gyrase
d. Rubella virus be obtained in the form of:
b. DNA ligase
720. Chicken pox in man is caused by a. Single stranded DNA b Uterus
c. Topoisomerase
a. Pox virus b. Double stranded DNA
d. Helicase
b. Herpes virus c. Single stranded RNA c. Brain
d. Udder c. Zonary b. Horse a. Subarachoid space

744 The pancreatic acinus is characterized by d. Discoidal c. Dog b. Subdural space


presence of c. Epidural space
749 Which of the following nerve is the largest d. Pig d. Dural space
a. Hassal‘s corpusle cranial nerve? 759 Which one of the following vein is called
as milk vein in cow?
b. Centro-acinar cells a. Trigeminal
a. External jugular vein
c. central artery b. Vagus
754 Animal which walks on the hoof is called b. Subcutaneous abdominal vein
d. central vein c. Sciatic as?

745 The accentric placed central artery is d. None of the above a. Plantigrade c. Umbilical vein
found in
750 Which one of the following contains b. Unguligrade d. None of above
a. Hepatic lobule cerebrospinal fluid?
760 All the spinal nerves are:
b. Osteon a. Epidural space c. Digitigrade
a. Mixed type
c. Spinal cord b. Subdural space d. None of above
b. Sensory type
d. Spleen c. Subarachnoid space 755 Which type of placenta present in cow?
c. Motor type
d. None of above a. Syndesmochorial
d. None of the above
751 Which one of the following ligament is b. Epi theleochorial
great importance for treatment of upward 761 Which one of the following foetal
fixation of patella in the bullock? c. Endotheliochorial membrane encloses embryo in fluid sac?
746 The purkinjee cells are found in a. Medial patellar ligament d. Haemochorial a. Chorion
a. Heart b. Middle patellar ligament 756 Which of the following animal presents b. Amnion
ovulation fossa on its ovary?
b. Cerebrum c. Yolk sac
c. Lateral patellar ligament a. Cow
c. Cerebellum d. Allantois
d. Cruciate ligament b. Mare
d. Spinal cord
752 Which one of the following species c. Bitch
747 The tendon cell is
presents single occipital condyle in skull? d. Doe
a. Fibroblast
a. cattle 757 Which of the following structure produces
b. Myoblast voice in the fowl?
b. Horse
c. Osteoblast a. Larynx 762 Down syndrome is an example of
c. Dog
d. Chondroblast b. Pharynx (a) Monosomy
d. Poultry
748 Which one of the following placenta found c. Syrinx (b) Trisomy
in mare? 753 Which one fo the following animal vomits
only through nostril because of its very d. None of the above (c) Triploidy
a. Cotyledonary long and well developed soft palate?
758 Which is the space present between (d) Polyploidy
b. Diffuse a. Ox vertebral canal and spinal duramater?
763 Outward expression of a trait is called
(a) Genotype (d) None of these (a) Nomal male (a) Fecundity

(b) Phenotype 769 The criss cross pattern of inheritance is (b) Wool production
seen in (b) Nomal female
(c) Karyotype (c) Body weight
(a) Sex limited trait (c) Super male
(d) all of these (d) Milk production.
(b) Sex linked trait
764. Colour blindness in human is (d) Super female
(c) Sex influence trait
(a) Sex limited trait 774 Genotype of purebred Pea combed birds is
(d) None of above
(b) Sex linked trait (a) RrPp 779 The choice of exotic breed for improving
milk production in cattle in
(c) Sex influenced trait (b) RRpp hilly areas is
(d) None of above (c) rrPP (a) HF
770 The sudden heritable changes in genetic
765 What should be the performance of material is called as (d) rrpp (b) Jersey
progeny above the herd average in order to
declare the bull as a proven bull? (a) Duplication 775 The phenotypic ratio of recessive epistasis (c) both
is
(a) 20 % (b) Mutation (d) none
(b) 5 % (a) 9:3:4
(c) 10 % (c) Deletion 780 Red Kandhari cattle belongs to
(d) 15 % (b) 9:7
766 The nullisomy is an example of (d) None of these (a) U.P.
(a) Anuploidy (c) 12:3:1
(b) M.P.
(b) Euploidy (d) 15:1
(c) A.P.
(c) Both 776 The medium sized milch breed of buffalo
771 Mutation resulting from replacement of which requires less maintenance is (d) Maharastra
(d) None base pair of purine with purine or
pyrimidine with pyrimidine is called as (a) Murrah 781 The exotic cattle breed of choice for
767 The structural change in a chromosome in improving milk production is
which a segment is oriented in (a) Transition (b) Jafarabadi (a) HF
a reverse order is called as
(b)Tansversion (c) Surti (b) Jersey
(a) Tranlocation
(c) Translocation (d) None (c) Brown Swiss
(b) Duplication
(d) None of these 777 Central Institute for research on goat is (d) Guernsey
(c) Deletion located at
772 The Coiling Pattern of Shell in Snail is an 782 Vibriosis in a herd can best be diagnosed
(d) Inversion example of (a) Karnal by

768 The regression of offspring on mid-parent (a) Extra nuclear inheritance (b) Izzatnagar a) Serum agglutination test
value estimates
(b) Maternal inheritance (c) Makhdoom b) Vaginal mucus agglutination test
(a) ½ h2
(c) Cytoplasmic inheritance (d) Hissar c) Milk ring test
(b) ¼ h2
(d) all of above d) Rose Bengal plate test
(c) h2
773 The sex index (X/A) value of 0.5 in 778 Booroola gene in Garole sheep refers to
Drosophila indicates
783 Induction of estrus by premature 788 The true bacterial venereal disease is 793 Defective fetal deglutition is thought to be a) 30-40 days
regression of the corpus luteum can be one of the causes of
done by using a.) Brucellosis b) 40-60 days
a.) Hydroallantois
a) PRID b.) Campylobacteriosis c) 150-170 days
b.) Hydroamnios
b) GnRH c.) Genital tuberculosis d) 90-120 days
c.) Hydrocephalus
c) PGF2 α d.) Leptospirosis 799 Causative organism for contagious
d.) Fetal ascitis equine metritis
d) HCG 789 Endometrial cups secrete PMSG hormone
during 794 ―Buller cow‖ is representing the a) Campylobacter fetus
784 Under which of the following conditions
animal is acyclic a.) 20-60 days a.) Follicular cyst b) Trypenosoma equiperdum
b.) 100-180 days
a) Delayed ovulation b.) Luteal cyst c) Herpes virus
c.) 40-120 days
b) Silent heat c.) Cystic corpora lutea d) Teylorella equigenitalis
d.) 180-250 days
c) Anovulation d.) Par ovarian cyst 800 Abortion storm is a characteristic sign of
790 In bitch the abdominal palpation can be
d) None of the above best performed for early pregnancy 795 Yellowish or whitish, thick, opaque uterine a) Vibriosis
diagnosis during secretion nourishing the ovum and embryo
785 Time of ovulation in a cow is is called b) Trychomoniasis
a.) 15-20 days c) Brucellosis
a) 12-24 hrs before the end of estrum a.) Lochia
b.) 28-32 days d) Leptospirosis
b) 30-40 hrs after the end of estrum b.) Yolk sac
c.) 45-50 days 801 The location of seminal vesicles in the
c) About the last day of estrum c.) Uterine milk
d.) 55-60 days bovine is
d) 10-15 hrs after the end of estrum d.) Oviductal fluid
791 Which of the following vulval suture a) On the floor of pelvis
786 Flehman‘s reaction is not observed in technique has the least disadvantages for 796 Length of diestrus period of estrus cycle in
bovine is b) Caudal to the bladder, around the
a) Bull the retention of prepartum cervicovaginal neck of bladder
prolapse? a) 10 days
b) Boar c) On the either side of the pelvis
a.) Caslicks operation b) 13 days urethra near ischiatic arch
c) Stallion
b.) Matress suture c) 18 days d) None of the above
d) Ram
c.) Quill suture d) 15 days
d.) Buhner`s suture 797 Period of embryo in cattle is
787 The condition in which fetal bones
crepitate within the uterus on rectal 792 Cuboni test is done to detect the presence a) 13- 45 days
examination of
b) 18-50 days 802 Head office of Directorate of Marketing
a) Mummification a.) Progesterone and inspection is located at
c) 10-12 days
b) Hydroallantois b.) PMSG (A) Delhi
d) 45-280 days
c) Maceration c.) Estrogen (B) Ahmedabad
798 Fremitus can be best felt earliest during
d) None of the above d.) None of the above pregnancy in cattle (C) Faridabad
(D) Jaipur b. Second 820. It is not a sexual behavior of bull. a. White rock
c. Fifth a. chin resting b. Cornish
803 CACP headquarter is located at d. First b. libido c. New Hampshire
811. Gujarat has total breeds of buffaloes. c. courtship d. Australorp
(A) Jaipur a. Two d. reactivity 829White leghorn are white because
b. Four 821. Banas dairy is located at.
(B) Pune c. Three a. Palanpur a. No colour gene
d. Six b. Deesa b. A dominant gene which inhibits color
(C) Kolkata c. Recessive white gene
812. First ring on a horn at the age in cattle. c. Bharuch
a. 2 years d. Mehsanas d. They have silver gene
(D) Delhi
b. 4 years 822. Khillar is the breed found at. 830The wild ............is the ancestor of all
804. It is the sheep breed with high fecundity c. 3years a. Gujarat domestic duck breeds
trait d. 5years b. Andhrapradesh
a. Nellore 813. Principle object of running dairy farm. c. Maharashtra a. Mallard
b. Garole a. Meat production d. Karnataka b. Campbell
c. Black Bengal b. Milk production 823. Calf starter is a feed c. Red fowl
d. Nail c. Sale of heifers a. liquid feed for calf d. Both a & b
805. Ringing is the practice followed by sheep d. Fodder b. liquid feed for heifer 831The black and white barring in barred
owner as 814. Sahiwal cow is breed pertaining to the c. solid feed for calf Plymouth rock is due to ............barring gene
a. Tied bell on neck group. d. solid feed for heifer
b. Pass through ring of grasses a. Milch a. Sex linked gene
c. Clipping of hair around neck b. Dual b. Sex limited gene
d. Clipping of hair around penis c. Draught 824) Calcium requirement in layer poultry per day c. Sex influence gene
806. Milling is the process of wool related to d. None d. Dominant gene
is about
a. Removal of grease and dirt 815. Calf starter is a feed for. 832The wild ............is the ancestor of all
b. Raises individual fibres on surface a. Calf a. 0.5 % domestic duck breeds
c. Marking at brisket with colour b. Heifer b. 1.5 %
d. Process of removal of c. Cow c. 3.5 % e. Mallard
objectionable part d. Buffalo d. 10 % f. Campbell
807. Low burr content (LB) is graded in wool at 816. Nose ring should be applied in bull at the g. Red fowl
burr content level age. h. Both a & b
825Incubation period if chicken egg
a. Below 2 % a. 1.5 years 833In a sex-linked cross involving barring, the
b. Below 3 % b. 2 years a. 19 days female parent is a ...............
c. 3 to 6 % c. 2.5 years b. 23 days
d. More than 6 % d. 3 years c. 21 days a. Barred Plymouth Rock
808. Vautha fair is famous for trading of 817. Hormone responsible for letdown of milk. d. 18 days b. Non Barred Plymouth Rock
a. Horse a. Oxytocin c. Both
b. Cow b. Prolactine d. None of above
826Nutritional roup in poultry is due to
c. Camel c. Lactate
d. Hyluronidase deficiency of
d. Donkey
809. It is the location were disowned and a. Vit A 834
In a sex-linked cross involving silver and
orphan animal are kept b. Vit B6 gold, the silver gene carrying ...........parent is
a. Animal pound c. Vit K used
b. Gaushala d. Vit E
c. Panjarapole 818. Dehorning is done in the calf at the age. a. Male
d. None a. 4-10 days 827which of the fowl has a single medium wattle b. Female
b. 10-20 days c. Both
c. 3 months a. red jungle fowl d. None of above
d. 6 months b. ceylon jungle fowl 835
In a sex-linked cross involving silver and
819. Common chemical used for dehorning. c. grey jungle fowl gold, the gold gene carrying ........parent is
810. India ranks total milk production in the a. Sodium bromide d. javan jungle fowl used
world. b. Potassium iodide 828The best breed for using as male line in
a. Third c. Caustic potash broiler production is a. Male
d. HCL b. Female
c. Both antibacterial activity a) Troposphere d)vehicle.
d. None of above b) stratosphere
836
In a sex-linked cross involving feathering a. Lysozyme c) mesosphere 855 Test use for diagnosis of brucellosis in
gene, a late feathering .......parent is used b. Avidine cattle.
c. Transferine d) thermosphere
a. Male d. All of above a) MRT
b. Female 844 Host which provides a medium for larval
c. Both or asexual phase of life cycle of an b) coagulation test
e. None of above infectious agent. 850 Method of carcass disposal are
837
Which one is sex linked c)hensa test
a) Intermediate host a) cremation
a. dwarfism d) gmelin test.
b)Final host
b. nakedness b) burial
c)Obligatory host 856 Yellow fever cause by which virus.
c. Albinism
d. rapid feathering d)None of the above
c) flamegium a)flavi virus
838
Egg shell treatment is done to reduce the rate 845 Zoonotic disease are perpetuated in
of ............loss nature by a single vertebrate species. d) incineration b)lyssa virus

a. Oxygen loss a) Cyclozoonosis 851 The term refers to smoke mixed with c)hendra virus
b. Carbon dioxide loss dust
c. Both a and b b) Direct zoonosis d)picorna virus.
d. Energy loss a)smog
c) metazoonosis 857 Constant present of a disease or organism
839
Shank length and width is a good indicator of
b)smust in a community-
........ d) Amphizoonosis
c)soot a)Epidemic
a. Egg production 846 These are the bacterial zoonosis.
b. Meat production d)mist
c. Skeleton size a)brucellosis b)Sporadic
d. Health of birds 852 Rotten egg odour in water is due to
840
Shank length and width is a good indicator of b)leptospirosis c)Endemic
........ a) Hyrogen sulfide d)Panzootic
c)listeriosis
a. Egg production b) Algae
b. Meat production d)all of the above
c. Skeleton size c) Cyanide
d. Health of birds 847 Yello fever is transmitted by.
841
Poultry need one more essential aminoacid d) Ammonia
a)Ades aegypti
........than cattle
853. Standard of ventilation
b)Argus
a. Lysine
a) cubic space 858 Diseases transmitted from man to lower
b. Metheonin c)Ioxides
c. Glysine b)air space vertebrate animals called as
d. Cystene d)Simules
c)floor space a) Zooanthropozoonosis
842
Hens egg contains about.....grams of protein 848 Epedemic in bird population.
d)all of the above b)Anthropozoonosis
a)Epizootic
854 Type of transmission caused by physical c) Amphixenosis
a. 6-7 b)Epidemic
contact. d)Sporozoonosis
b. 12-13
c. 2-3 c)Epornitics
a) direct 859 Unit of radiation
d. 21-22
d)all of the above
843
A component of egg white having b)indirect a) Meds
849 Zone comprises the ozone layer.
c)contect
b) Reds a) Musca spp. 872 The budy of an arachnid is divided into a) Infestation
b) Chrysomia spp. two parts the anterior gnathosoma and b) Bites
c) calori c) Calliphora spp. posterior c) Anaphylactic shock
d) Sarcophaga spp. d) All of the above
d)Joule a) Podosoma 880 The term ‗entomophobia‘ means
b) Idiosoma
860 Who is the first chief of VPH in WHO? a) A fear of insects
c) Opisthosoma
d) Prosoma b) The science of insect classification
a) Dr martin Kalpan c) The study of insects
873 A key role in the transmission of artropod
d) The study of insect behaviour
b)Dr Smith borne diseases is played by the
881 Which of the insects listed below could
cause myiasis?
c)Dr nayadu 866 ‗Flea collars‘ of dogs and cats are usually a) Circulatory system
b) Excretory system
impregnted with a) Dragon fly
d)Dr B B Jack c) Respiratory system
d) Digestive system b) Flea
a) BHC c) Screwworn fly
861 Test for residual chlorine
b) DDT d) Mite
a)chlorine c) Malathion
d) Dichlorovas
b)florine 867 The condition ‗butchers jelly‘ is caused by
the larvae of
c)organoleptic 874 Hexagonal discal cell is present in the
a) Oestrus ovis
wings of
d)orthitolidine b) Gastrophillus intestinalis
c) Hypoderma lineatum a) Tabanus spp.
862 The term reffered to mixture of smoke d) Callitroga hominivorax 882 In insects, the body part which acts as a
b) Stomoxys spp.
and fog 868 The two pairs of antennae are present in c) Phlebotomus spp. protective structure and which provides for
artropodas belonging to the class d) All the above the attachment of muscles is known as the
a) smog
875 wool-staining in sheep is caused by
a) Arachnida a) Endoskeleton
b)must b) Crustaecia b) Sclerites
a) Oestrus ovis
c) Insecta b) Damalinia ovis c) Abdomon
c)sute d) Myriapoda d) Exoskeleton
c) Melophagas ovinus
869 Formation of hairballs in the stomach of d) None of the above
d)mist
calves may occur due to 876 Brown-dog tick is
863 ozogase present in ozonlayer of
a) Fly infestation a) Rhipicephalus appendiculatus
stratosphere
b) Lice infestation b) Rhipicephalus sanguineus
a)0.5 to 1 mg/l c) By both of the above c) Haemaphysalis leachi
d) None of the above d) None of the above 883 Insects which posses sponging mouth parts
b)0.2 to 0.3 mg/l 870 Which of the following is not a larviparous 877 ‗Gavac‘ vaccine is used against and do not bite are
fly
a) Hyalomma anatolicum a) House flies
c)4 to 5 mg/l
a) Oestrus ovis b) Dermacentor andersoni b) Fleas
d)2 to 3 mg/l b) Sarcophaga dux c) Haemaphysalis leachi c) Mosquitoes
c) Pseudolynchia canariensis d) Boophilus microplus d) Deer flies
864 Tick paralysis is mainly caused by ticks d) Gastrophillus intestinalis 878 ‗Sweet itch‘ in horses is caused by 884 Which of the following is not correctly
belonging to 871 The following is known as wing louse of matched-?
a) Simulium indicum
poultry
a) Hyalomma b) Phlebotomus sergenti (A) Zn deficiency-Pig
b) Ixodes spp. a) Lipeurus caponis c) Culicoides robertsi
c) Boophilus spp. d) Climex lecturalius (B) Epidemic tremor-Virus
b) Goniodes gigas
d) Amblyomma spp. c) Menopon gallinae 879 How are hosts affected directly by
(C) Siderosis-silica dust
865 Accidental myiasis causing fly d) Menacanthus stramineus arthropods?
(D) Alkali disease-Se (A) Calcification absent (B) Infectious Bronchitis (B) CRD

885 Pachymeningitis is inflammation of- (B) Liver and bones are most commonly (C) Infectious Lanyngiotracheitis (C) Fowl cholera
affected
(A) Piamater (D) Avain Encephalomyelitis (D) Fowl typhoid
(C) Lungs are most commonly affected
(B) Brain 895 Dohle‘s bodies are toxic granules of- 900 In Angara disease, the pathological finding
(D) Intradermal test is performed on is-
(C) Duramater wattle. (A) Macrophages
(A) Haemopericardium
(D) Spinal cord (B) Eosinophils
(B) Hydropericardium
886 Liquifactive necrosis is most commonly 891 Post mortem of cattle reveals too much (C) Neutrophils
seen in- emaciated carcass,mucosa of intestine (C) Myocarditis
(D) Lymphocytes
(A) Kidney thrown into corrugated folds, most (D) Pneumopericardium
probable cause will be- 896 East coast fever is caused by-
(B) Liver 901 In Left side heart failure, the heart failure
(A) Rinder pest (A) Theleria parva cells are seen in-
(C) Heart
(B) Johne‘s disease (B) Theleria annulata (A) Lungs
(D) Brain
(C) Tuberculosis (C) Babesia bovis (B) Heart
887 Which of the following is correctly
matched-? (D) Pasterellosis. (D) Anaplasma centrale (C) Kidney

(A) Picorna virus-Ranikhet disease 892 Spondylitis is inflammation of- 897 Edema consisting of gelatinous material in (D) Spleen
neck and brisket region seen in cattle in-
(B) Lumpy skin disease- Pox virus (A) Prepuce 902 Sway back condition is seen due to
(A) Black Quarter deficiency of-
(C) Diamond skin disease-Herpes virus (B) Vertebrae
(B) Deganala disease (A) Cu
(D) Paramyxo virus-FMD (C) Bone
(C) Botulism (B) Co
888 Enlargement of Sciatic nerve is seen in- (D) Spermatic cord
(D) Haemorrhagic Septicemia (C) Mn
(A) Ranikhet disease 893 Which of the following is correct
regarding poultry diseases-? (D) Se
(B) Marek‘s disease
(A) In pullorum disease, green constant 898 Which is the main chemical mediator of 903 Most pathogenic species/disease affecting
(C) Chronic respiratory disease diarrhea is seen inflammation-? Snakes-
(D) Infectious Coryza (B) Face is swollen and edematous in (A) Serotonin (A) Pasteurellosis
Haemophilus infection
889 Apennosis is- (B) Bradykinin (B) Histomoniasis
(C) Bloody mucous expelled from trachea
(A) Intracellular edema of epidermis (C) Histamin (C) Salmonellosis
in Infectious Bronchitis
(B) Congenital lack of feathers in fowl (D) Interleukin-1 (D) Listeriosis.
(D) In pullorum disease, nervous signs are
(C) Absence of pineal gland seen along with diarrhoea
904 Abnormal large amount of granulation
(D) Lack of cell differentiation during 894 In which outbreak at poultry farm tissue is known as
embryogenesis maximum mortality of birds will be expected?
899 Big liver disease is also known as- a. Proud flesh
890 Which is incorrect about avain (A) Ranikhet disease
(A) IBD b. Callus
tuberculosis-?
c. Adenoma c. Vesicocele a. Dermoid cyst d. Maximum

d. Wound d. Hysterocele b. Hypopia 920 Reflecting layer is present in

905 Who introduced the basic principles of 910 Radical surgery is done to c. Exophthalmos a. X-ray film
surgery
a. Conserve damaged tissue d. Conrneal ulcer b. X-ray tube
a. W. S. Halsted
b. Remove damaged tissue 915 Exposure factor for taking radiograph is c. Intensifying screen
b. Joseph Lister
c. Eliminate root cause a. mA d. All the above
c. W. T. G. Morten
d. Correct malformations b. kVp 921 Frequency mainly used for the purpose of
d. Michal Harward diagnostic ultrasound
911 Preanesthetic medication is employed to c. Second
make a. 1 to 10 MHz
d. All of above
a. Induction of anaesthesia smooth b. More than 50 MHz
916 Amputation of horn can be done under
906 Gangrene occurs most commonly in b. Recovery complication minimum c. Less than 20000Hz
buffaloes at a. Cornual nerve block
c. Animal more controllable d. None of the above
a. Udder b. Caudal epidural
d. All of above
b. Ear pinna c. Anterior epidural
912 Reaction of living tissue to injury is 922 Barium sulphate is exclusively used for
c. Tail known as d. Retrobulbar nerve block
a. Outlining alimentary tract
d. All of above a. Infection 917 Adhesion of iris to cornea is known as
b. Outlining urinary tract
907 Perineal hernia is common in b. Inflammation a. Anterior synechia
c. Outlining spinal canal
a. Mares c. Infestation b. Posterior synechia
d. Outlining abdominal cavity
b. Castrated dogs d. Sarcoma c. Epiphora
923 Reducing agent used in x-ray developer
c. Spayed bitches 913 Irreducibility of hernia is a due to d. Chemosis
a. Metol
d. Adult uncastrarted dogs a. Adhesion 918 Surgical operation for providing drainage
from middle ear is known as b. Sodium carbonate
908 Peterson's block is practiced to b. Strangulation
anaesthetize a. Zepps' operation c. Potassium bromide
c. Incarceration
a. Teeth b. Bulla osteotomy d. Sodium sulphite
d. All of above
b. Eye c. Hyovertebrotomy 924 Cells found in the choroid plexus that
secrete cerebrospinal fluid are:
c. Flank d. Ventriculectomy
a. Astrocytes
d. Limb 919 FFD for taking diagnostic radiography b. Microglia
should be c. Ependymal cells
909 If the content of hernia is urinary bladder, d. Oligodendrocytes
then it is termed as a. Minimum 925 Action potentials are conducted more
rapidly in:
a. Enterocele 914 One of the following is the congenital b. 90 cm
defect of eye a. Small diameter axons than large
b. Epiplocele c. 36 inch
diameter axons
b. Large diameter axons than small d. Visceral layer of the Bowman's b. Anaphylactic shock c) Movement of drug from the gut into
diameter axons Capsule general circulation
c. Unmyelinated axons than myelinated 932 Glucose is: c. Hypovolemic shock d) The drug causes dilation of coronary
axons vessels
d. Axons that lack a wrapping of a. Filtered, reabsorbed, and secreted d. All of the above
Schwann cells b. Filtered, and reabsorbed, but not side effects?
926 Neurotransmitters are stored in vesicles secreted 939 A patient with a hypothalamic tumor has a) Paracetamol
that are located primarily in specialized c. Filtered, and secreted, but not hypersecretion of ADH. Which of the b) Aspirin
portions of the: reabsorbed following BP readings would be most c) Meloxicam
d. Filtered, and neither secreted nor likely for this patient? d) None of above
a. Soma reabsorbed 945) Most drugs are either ____ acids or ____
b. Axon 933 While the kidneys process about 180L of a. 95/65 bases.
c. Dendrite blood-derived fluids per day, the amount a) Strong; Strong
d. Perikaryon that actually leaves the body is: b. 115/80
927 Which of the following organelles is b) Strong; Weak
a. 50%, or 90L c. 120/60
responsible for the appearance of Nissl c) Weak; Weak
bodies in the cell bodies of motor neurons? b. 100%, or 180L
d. 165/100
c. 10%, or 18L
d. 1%, or 1.8L d) All of above
a. Smooth endoplasmic reticulum 940 The space in the middle of the thoracic 946) Which of the following enteral
b. Rough endoplasmic reticulum 934 The fluid in the capsular space is similar cavity where the heart resides is the: administration routes has the largest first-pass
c. Golgi apparatus to plasma except that it does not contain a a. pericardial cavity effect?
d. Mitochondria significant amount of: b. pericardium a) SL (sublingual)
928 Which of the following structures is a c. pleural cavity
component of a reflex arc? a. Glucose d) mediastinum b) Buccal
b. Sodium 941 Blood returning from the lungs enters the
a. Afferent neuron c. H+ heart through the: c) Rectal
b. Efferent neuron d. Albumin a. pulmonary semilunar valve
c. Effector organ 935 Aldosterone causes: b. mitral valve d) Oral
d. All of the above c. right ventricle
929 The testosterone-producing cells of the a. Decreased K+ in the urine d. left atrium 948) Which of the following would receive
b. Increased Na+ in the urine 942 The precursor of ketone body is drug slowly?
testes are called:
c. Increased urine output a. Acetyl CoA a) Liver
a. Sertoli cells d. Decreased urine output
b. Granulosa cells 936 Which of the following is not one of the 3 b.Acetoacetic acid b) Brain
c. Spermatogonia external coverings of the kidney?
d. Leydig cells c. Betahydroxybutyric acid c) Fat
a. Renal capsule
b. Adipose capsule d. Cholesterol d) Muscle
c. Renal fascia
d. Renal adventitia 943) For intravenous (IV) dosages, what is the 949) Pharmacokinetics is the effect of the ____
930 An oocyte surrounded by one layer of 937 The addition of a strong acid to the bioavailability assumed to be? and Pharmacodynamics is the effect of the
a) 0% ____.
squamous follicle-like cells is most likely extracellular fluid would result in the
a) Drug on a drug; Body on the drug
a: increased formation of: b) 25% b) Body on the drug; Drug on a drug
- c) Drug on the body; Body on the drug
a. Primordial follicle a. NaHCO3 c) 75% d) Body on the drug; Drug on the body
b. Primary follicle b. H2CO3
950) Which of the following is NOT an action
c. Secondary follicle c. OH- d) 100% of the body on a drug?
d. Graafian follicle d. All of the above
a) Absorption
931 Podocytes make up the: 944) Which of the following is NOT a b) Distribution
pharmacokinetic process? c) Metabolism
a. Visceral layer of the nephron 938 Severe vomiting can result in: a) Alteration of the drug by liver enzymes
b. Visceral layer of the glomerulus d) Side effects
b) Drug metabolites are removed in the 951) Which of the following is the amount of a
c. Visceral layer of the renal capsule a. Septic shock urine drug absorbed per amount administered?
a) Bioavailability
b) Bioequivalence b) Heart c. 4 -5 % d. Acid fast stain: Staphylococci
c) Kidney 10 %
c) Drug absorption d) Liver
964. Nagler‘s reaction with Cl.perfringens on 971. Strauss reaction is positive for
d) All of above 958) movement of a drug between egg yolk agar is due to a. Brucella abortus
body compartments? a. Haemolysin b. Pseudomonas mallei
a) Partition constant b. Hyaluronidase c. Actinobacillus ligneresii
952) h b) Degree of ionization
c. Lecithinase d. All of above
e c) Size
d. Leucocidin 972. The Weil-Felix test is an agglutination
blood-brain barrier (BBB)? d) All of the above 965. Cytochemically Rickettssia are reaction between.
a) Large and lipid-soluble 959 Following is responsible for agglutination a. Weak gram positive a. Antibody against Rickettsia and
of RBC in animal body. b. weak gram negative antigen from Pseudomonas
b) Large and lipid-insoluble
c. Acid fast b. Antibody against Rickettsia and
a) Abrin d. None antigen from Staphylococci.
c) Small and lipid-soluble
966. Corynebacteria can be characterized as c. Antibody against Rickettsia and
b) Ricin a. Non motile, non sporing, aerobic, gram antigen from Brucella.
d) Small and lipid-insoluble
positive bacilli d. Antibody against Rickettsia and
c) Renin b. Motile, non sporing, aerobic, Gram antigen from Proteus.
953)
phase II substrate? positive bacilli 973. IMViC pattern of Salmonella is
d) Thiamine
a) Glucuronic acid c. Motile, sporing, aerobic, Gram positive a. + + - -
960 Bovine Bunker is synonym to bacilli b. + - + -
b) Sulfuric acid d. Non motile, sporing, anaerobic, Gram c. - - + +
a) Urea toxicity positive bacilli d. - + - +
c) Acetic acid 967. The cells parasitized in host by Rickettsia 974. Virus having criss cross pattern on its
b) Salt toxicity are surface is
d) Amino acids a. Erythrocytes a. Contagious pustular dermatitis virus
c) Oxalates toxicity b. Vascular endothelial cells b. Fowl pox virus
c. Neutrophills c. Adeno virus
d) Selenium toxicity d. Parvo virus
d. Monocytes 975. Virus having cyclic DNA is
961 Severe and acute pneumonia (Drowning a. Papova virus
954) i syndrome) indicates probability of 968. Dimorphic fungi produces b. Parvovirus
on is phase II and NOT phase I? a.Mold type growth at 370 C
poisoning due to c. Pestivirus
a) Oxidations b. Yeast type growth at 220 C d. Poxvirus
b) Reductions a) ANTU c. Mold type growth at 370 C and Yeast 976. Diploid genome is the characteristic of
c) Conjugations virus family
type growth at 220 C.
d) Deaminations b) OCTP a. Retroviridae
d. Yeast type growth at 370 C.and mold
b. Rhabdoviridae
Cytochrome-P450 system? c) MB type growth at 220 C
c. Reoviridae
a) Metabolism of substances 969. The organism present in high d. None of these
b) Detoxification of substances d) OP concentration in pigeon droppings is 977. Bovine viral diarrhea virus belongs to
c) Decreasing pH of compartments a . Candida albicans family
containing substances 962 All of following except one is not used for b. Cryptococcus neoformans a. Togaviridae
d) A & B doping in animal c.Rhinosporidium seeberi b. Reoviridae
955) Weak acids are excreted faster in ____ d. Aspergillus flavus c. Herpes viridae
urine and weak bases are excreted faster a) Furasemide
d. Flaviviridae
in____ urine.
a) Acidic; Alkaline b) Phenylbutazone
b) Alkaline; Acidic
c) Steroids 978. Which animal is not susceptible to
c) Acidic; Neutral
d) Alkaline; Neutral vesicular exanthema virus.
d) Adrenaline 970. Which of the following is incorrect match.
a. Pig
956) Which organ is responsible for metabolism
963. The dry matter requirement of goat is a. Diene‘s stain: Mycoplasma
in the ―first pass effect‖? b. Horse
b. Fontana stain: Spirochaetes
a) Brain a. 2 – 3 % c. Cow
c. Machiavello stain: Chlamydia
b. 6 – 8 % d. All of above
979. Samples suspected for Pox virus are b. Break internal phospho-diester bonds c. Codon choice b. Okazaki fragment
inoculated in embryonated eggs by the route within DNA molecule d. None of the above c. Klenow fragment
of c. Removes nucleotides only in 3‘-5‘ 994. A technique that can be used to construct a d. None of the above
a. Allantoic cavity direction clone contig by identifying overlapping 1000. Which of the following is not a stop
b. Amniotic cavity d. Removes nucleotides only in 5‘-3‘ fragments of cloned DNA: codon:
c. CAM direction a. Gene walking a. AUG
d. Yolk sac b. Chromosome walking b. UAA
980. Bovine ephemeral fever virus belongs to 986. Endonucleases: c. Clone fingerprinting c. UGA
the family a. Removes nt one at a time from the end d. None of the above d. UAG
a. Herpes viridae of DNA molecule 995. The ability of two different types of 1001. Which of the following is a start codon:
b. Rhabdoviridae b. Break internal phospho-diester bonds plasmid to co-exist in same cell is called: a. AUG
c. Reoviridae within DNA molecule a. Competence b. GUG
d. Togaviridae c. Removes nts only in 3‘-5‘ direction b. Compatibility c. Both
981. EDS-76 is d. Removes nts only in 5‘-3‘ direction c. Interference d. None
a. Herpes virus 987. The enzyme used to remove phosphate d. None of the above
b. Corona virus group at the 5‘ end of DNA molecule is: 996. The non coding sequence within genes that 1002. Which of the following is incorrect about
c. Adenovirus a. Alkaline phosphatase do not translate into protein is: genetic code:
d. Poxvirus b. Polynucleotide kinase a. Intron a. It is non overlapping
982. Paired serum samples means c. Terminal deoxytransferase b. Exon b. It is redundant
a. Serum sample collected at two d. Topoisomerase c. Transposons c. It is ambiguous
different stages of a disease from an 988. The enzyme used to add phosphate group d. None of the above d. None of the above
animal at the free 5‘ end of DNA molecule is: 997. The coding sequence within genes that
b. From two different animals. a. Alkaline phosphatase translate into protein is:
c. Samples divided in two aliquots. b. Polynucleotide kinase a. Intron
d. Samples collected from two parents. c. Terminal deoxytransferase b. Exon
f. Newcastle disease virus infection in d. Topoisomerase c. Transposons
human can cause 989. The enzyme that changes the conformation d. None of the above
a. Generalized infection of covalently closed circular DNA by 998. The process of modification of RNA after
b. Contagious respiratory infection. introducing of removing supercoils is: transcription in which introns are removed
c. Contgious conjunctivitis and mild a. Alkaline phosphatase and exons are joined is called:
influenza like symptoms b. Polynucleotide kinase a. Splicing
d. Encephalitis c. Terminal deoxytransferase b. Recombination
983. Which of the following showing buoyant d. Topoisomerase c. Translocation
density of DNA, RNA and Protein is 990. Eco RI produces: d. Assortment
correct: a. Blunt end 999. A relatively short fragments of DNA
a. RNA>DNA>Protein b. Sticky end synthesized on the lagging strand during
b. Protein>RNA>DNA c. Both DNA replication is called:
c. DNA> RNA>Protein d. None a. Primer
d. Protein>DNA>RNA 991. The specific position on a DNA molecule
984. Intercalation of ethidium bromide (etBr) in ANSWER KEY
where DNA replication begins is called:
DNA molecule will: a. Replication fork Sr. Answe Sr. Answe Sr. Answer Sr. No Answer Sr. No Answer
a. Increase the buoyant density of DNA b. Origin of replication
molecule No r No r No
c. Start Point of replication
b. Decreases the buoyant density of DNA d. None of the above 1. a 46 b 91 c 136 b 181 a
molecule 992. An algorithm used to search homology is:
c. Does not affect the buoyant density of a. BLAST 2. d 47 b 92 c 137 b 182 c
DNA molecule b. CLAST
d. None of the above c. PUBSCAN 3. b 48 a 93 b 138 a 183 d
d. HOMOSCAN
993. The fact that not all codons are used equal 4. d 49 a 94 c 139 b 184 b
985. Exonucleases: frequency in the genes of a particular
a. Removes nucleotides one at a time organism is called: 5. B 50 c 95 a 140 b 185 d
from the end of DNA molecule a. Codon preference
b. Codon bias
6. C 51 b 96 a 141 b 186 a 35. d 80 d 125 a 170 b 215 D

7. d 52 b 97 c 142 a 187 c 36. b 81 b 126 d 171 b 216 D

8. b 53 b 98 a 143 b 188 a 37. a 82 a 127 a 172 a 217 D

9. a 54 c 99 a 144 b 189 c 38. a 83 d 128 a 173 c 218 C

10. b 55 c 100 a 145 b 190 a 39. b 84 a 129 a 174 b 219 A

11. b 56 b 101 c 146 a 191 a 40. a 85 a 130 b 175 a 220 A

12. a 57 c 102 a 147 a 192 c 41. d 86 c 131 a 176 c 221 A

13. c 58 d 103 b 148 a 193 b 42. b 87 c 132 a 177 b 222 C

14. a 59 c 104 b 149 a 194 b 43. c 88 a 133 a 178 d 223 A

15. a 60 a 105 a 150 a 195 b 44. a 89 c 134 a 179 c 224 B

16. a 61 d 106 b 151 b 196 d 45. b 90 a 135 b 180 c 225 B

17. b 62 a 107 a 152 d 197 b Sr. Answe Sr. Answe Sr. Answer Sr. No Answer Sr. No Answer
No r No r No
18. c 63 a 108 a 153 c 198 c
226 c 271 b 316 a 361 a 406 B
19. a 64 c 109 c 154 b 199 A
227 c 272 a 317 c 362 d 407 A
20. d 65 d 110 b 155 b 200 C
228 b 273 d 318 c 363 a 408 B
21. c 66 d 111 d 156 b 201 A
229 b 274 a 319 c 364 b 409 A
22. B 67 a 112 b 157 b 202 C
230 a 275 a 320 a 365 b 410 b
23. c 68 d 113 b 158 a 203 C
231 a 276 a 321 a 366 d 411 b
24. c 69 a 114 a 159 c 204 C
232 c 277 b 322 a 367 c 412 a
25. b 70 a 115 a 160 d 205 C
233 a 278 a 323 c 368 a 413 a
26. a 71 c 116 a 161 b 206 B
234 b 279 c 324 b 369 b 414 b
27. c 72 a 117 b 162 c 207 A
235 a 280 b 325 d 370 c 415 a
28. a 73 d 118 a 163 d 208 B
236 b 281 c 326 b 371 c 416 b
29. c 74 c 119 c 164 a 209 A
237 c 282 c 327 c 372 d 417 d
30. d 75 a 120 d 165 b 210 A
238 c 283 b 328 d 373 c 418 b
31. c 76 c 121 b 166 c 211 A
239 d 284 c 329 a 374 c 419 a
32. d 77 d 122 b 167 d 212 B
240 d 285 b 330 c 375 d 420 d
33. b 78 a 123 a 168 a 213 C
241 b 286 d 331 d 376 b 421 d
34. c 79 c 124 b 169 c 214 A
242 d 287 b 332 d 377 a 422 c

243 d 288 a 333 c 378 c 423 d

244 c 289 b 334 a 379 C 424 b Sr. Answe Sr. Answe Sr. Answer Sr. No Answer Sr. No Answer
No r No r No
245 a 290 b 335 d 380 D 425 b
451 c 496 d 541 c 586 a 631 b
246 d 291 c 336 d 381 C 426 d
452 a 497 b 542 a 587 c 632 c
247 c 292 a 337 a 382 B 427 b
453 b 498 a 543 a 588 a 633 c
248 d 293 a 338 d 383 a 428 a
454 a 499 b 544 b 589 a 634 c
249 d 294 a 339 a 384 b 429 d
455 a 500 a 545 a 590 b 635 d
250 a 295 a 340 b 385 A 430 b
456 a 501 B 546 b 591 b 636 c
251 a 296 c 341 b 386 A 431 b
457 c 502 C 547 b 592 d 637 a
252 c 297 b 342 d 387 B 432 a
458 b 503 B 548 b 593 b 638 c
253 c 298 a 343 c 388 C 433 b
459 c 504 A 549 b 594 c 639 b
254 c 299 b 344 c 389 B 434 d
460 a 505 D 550 d 595 c 640 b
255 d 300 a 345 d 390 B 435 b
461 c 506 D 551 b 596 a 641 b
256 a 301 a 346 b 391 B 436 b
462 d 507 A 552 c 597 a 642 a
257 b 302 a 347 a 392 C 437 c
463 a 508 A 553 a 598 b 643 c
258 b 303 c 348 d 393 B 438 a
464 c 509 B 554 a 599 c 644 c
259 b 304 c 349 b 394 C 439 b
465 d 510 A 555 d 600 d 645 b
260 d 305 a 350 a 395 D 440 b
466 b 511 b 556 c 601 b 646 b
261 b 306 b 351 a 396 D 441 c
467 b 512 a 557 c 602 b 647 a
262 b 307 d 352 c 397 B 442 b
468 a 513 b 558 b 603 c 648 c
263 c 308 d 353 d 398 C 443 a
469 c 514 c 559 a 604 c 649 b
264 a 309 a 354 a 399 B 444 c
470 b 515 a 560 b 605 c 650 b
265 d 310 b 355 a 400 B 445 b
471 b 516 a 561 b 606 b 651 a
266 b 311 b 356 b 401 B 446 a
472 d 517 a 562 d 607 b 652 a
267 d 312 a 357 c 402 B 447 a
473 b 518 a 563 b 608 c 653 c
268 a 313 a 358 d 403 A 448 a
474 b 519 c 564 c 609 c 654 b
269 a 314 b 359 a 404 B 449 d
475 a 520 c 565 b 610 b 655 d
270 b 315 a 360 d 405 A 450 D
476 c 521 a 566 b 611 c 656 c 681 c 726 a 771 a 816 b 861 D

477 d 522 d 567 b 612 b 657 b 682 a 727 b 772 d 817 a 862 A

478 d 523 a 568 c 613 d 658 b 683 c 728 b 773 a 818 a 863 D

479 a 524 b 569 b 614 b 659 c 684 a 729 d 774 c 819 d 864 B

480 a 525 c 570 c 615 a 660 d 685 c 730 c 775 a 820 d 865 A

481 d 526 b 571 c 616 a 661 d 686 b 731 a 776 c 821 a 866 D

482 d 527 a 572 a 617 b 662 b 687 a 732 b 777 c 822 d 867 C

483 b 528 b 573 a 618 a 663 b 688 a 733 b 778 a 823 a 868 B

484 d 529 d 574 c 619 b 664 c 689 c 734 d 779 b 824 c 869 B

485 a 530 a 575 a 620 d 665 c 690 b 735 b 780 d 825 c 870 D

486 d 531 b 576 b 621 d 666 a 691 c 736 a 781 a 826 a 871 A

487 b 532 d 577 b 622 d 667 b 692 a 737 d 782 b 827 d 872 B

488 d 533 a 578 c 623 c 668 a 693 d 738 c 783 c 828 b 873 D

489 d 534 c 579 a 624 b 669 b 694 c 739 d 784 d 829 b 974 A

490 d 535 c 580 c 625 a 670 b 695 a 740 a 785 d 830 a 875 C

491 a 536 a 581 a 626 c 671 c 696 a 741 d 786 b 831 a 876 B

492 b 537 a 582 a 627 d 672 d 697 a 742 b 787 c 832 a 877 D

493 c 538 b 583 b 628 d 673 c 698 a 743 c 788 b 833 a 878 C

494 a 539 a 584 b 629 d 674 a 699 d 744 b 789 c 834 b 879 D

495 d 540 c 585 c 630 a 675 A 700 b 745 d 790 b 835 a 880 A

701 b 746 c 791 d 836 b 881 C

702 a 747 a 792 c 837 d 882 D

Sr. Answe Sr. Answe Sr. Answer Sr. No Answer Sr. No Answer 703 a 748 b 793 b 838 b 883 A
No r No r No
704 c 749 a 794 a 839 c 884 C
676 d 721 a 766 a 811 b 856 a
705 b 750 c 795 c 840 c 885 C
677 c 722 b 767 d 812 c 857 c
706 d 751 a 796 b 841 c 886 D
678 c 723 a 768 c 813 b 858 a
707 b 752 d 797 a 842 a 887 B
679 b 724 d 769 b 814 a 859 B
708 b 753 b 798 d 843 a 888 B
680 c 725 c 770 b 815 a 860 A
709 b 754 b 799 d 844 a 889 B
710 d 755 a 800 c 845 b 890 C 916 A 961 A

711 a 756 b 801 b 846 d 891 B 917 A 962 D

712 a 757 c 802 c 847 a 892 B 918 B 963 B

713 c 758 c 803 d 848 c 893 B 919 B 964 C

714 b 759 b 804 b 849 b 894 C 920 C 965 B

715 a 760 a 805 d 850 b 895 C 921 C 966 A

716 b 761 b 806 b 851 b 896 A 922 A 967 B

717 a 762 b 807 b 852 a 897 D 923 A 968 D

718 a 763 b 808 d 853 d 898 C 924 C 969 B

719 a 764 b 809 c 854 c 899 A 925 B 970 D

720 b 765 a 810 d 855 a 900 B 926 B 971 D

927 B 972 D

Sr. Answe Sr. Answe Sr. Answer Sr. No Answer Sr. No Answer 928 d 973 D
No r No r No
929 D 974 A
901 A 946 C 991 b
930 A 975 A
902 A 947 D 992 a
931 d 976 A
903 C 948 C 993 b
932 b 977 D
904 A 949 D 994 b
933 D 978 B
905 A 950 D 995 b
934 D 979 C
906 C 951 A 996 a
935 D 980 B
907 D 952 C 997 b
936 d 981 C
908 B 953 D 998 a
937 B 982 A
909 C 954 C 999 b
938 C 983 a
910 C 955 D 1000 a
939 D 984 b
911 D 956 B 1001 c
940 D 985 a
912 B 957 D 1002 c
941 D 986 b
913 D 958 D
942 D 987 a
914 A 959 B
943 D 988 b
915 D 960 a
944 D 989 d
945 d 990 b
………………………….....
22. Post parturient haemoglobinurea is also known as ………………………..and…………
…………………………………….
23. Sodium and potassium rich diet …………………………………. The chance of development of
milk fever.
24. …………………………………….posture occurs in milk fever.
25. Myoglobin in urine can be differentiated from hemoglobin by ………………………….
26. Hypomagnesaemic tetany is also known as …………………....,…………………………,
………………………
27. ………………………………………………syndrome occurs primarily in light horses.
28. Young green grass are ………………………………. In magnesium than mature grass.
29. A decrease in the ratio of magnesium : calcium will stimulate the secretion of …………
………………………………………. Which is responsible for titanic sign.
30. Drug of choice for malignant hyperthermia is ………………………………….
31. Porcine stress syndrome is also known as ……………………………………..
32. Baby pig cannot utilize …………………………….. and ……………………………….
Excepting………………………..
33. Pregnancy toxaemia is highly fatal metabolic disease of ……………………………….. and
…………………….
Occur in late pregnancy.
34. Hypoglycaemia Is also known as……………………………or………………………
1. The incidence of ketosis is higher in……………………………lactation. 35. Porcine stress syndrome is due to ……………………………………
2. Primary ketosis usually develops during ………………………………..period. 36. In preganacy toxaemia the level of cortisone is……………………………………….
3. Alimentary form of ketosis is also known as …………...or………………ketosis. 37. Pregnancy toxaemia is also known as ……………………………………
4. Nervous form of ketosis is mainly due to……………………………………. Or…………………………………………or……………………………………..
5. Milk fever type of ketosis is also known as ………………………………………… 38. Absence of corneal reflex and star gazing posture may be noted in ……………………….
6. Milk fever type ketosis is a condition of ………………………….. associated with 39. A decrease in the ratio of magnesium : calcium will stimulate secretion of ………………
………………………………… which is responsible for tetanic sign.
7. Ketosis is basically a problem related to………………………………………….balance. 40. Normal level of Mg in the blood is……………………………………….
8. Clinical ketosis usually occurs when plasma glucose level goes below ……………..and NEFA 41. ………………….....specific gravity of urine is found in
concentration goes greater than …………………………. ………………………………………………myoglobinurea.
9. Glucorticoids reduced ketone body formation by utilization of ………………………… 42. Paralytic myoglobinurea is popularly known as ………………………………………
10. Vetalog (triameinolene) is very much active against ……………………………………… 43. Eclampsia in bitches is generally restricted to ………………………………………… breed of dog
11. Anabolic steroid never increase blood …………………………….level but increases (Spitz) and generally not susceptible in ……………………………...breed of dog (Alsatian).
blood……………………. Level and help in the elimination of preformed …………….. 44. Biot‘s spot is associated with deficiency of …………………………………….
12. Parturient paresis is a metabolic disease occurring most commonly within ………………………..of 45. Enzyme ……………… Causes conversion of carotene to vitamin A in …………………
parturition. 46. In kerato malacia cornea become soft due to……………………………… necrosis.
13. Parturient paresis also known as ………………………….or…………………………… 47. Day blindness is also known as…………………………………………………….
14. Parturient paresis is frequently found in high yielding………………………………and rare in sow. 48. Night blindness is also known as…………………………………………………..
15. When milk fever is associated with hypocalcaemia, hypophosphataemia and hypomagnesemia it is 49. Configuration of epithelial tissue is maintained by……………………………………..
also known as ……………………….or……………………… 50. ……………………………….hormone helps in the conversion of carotene to vitamin A.
16. The normal ratio of Ca to Mg in blood is …………………………………. 51. During heat stress additional ………………………… Should be supplemented to the feed.
17. Detection of Ca in the urine of animal is done by…………………………………….test. 52. Mature pasture is deficient in……………………….. content.
18. A decreased ionized ……………………………………. Lead to milk fever. 53. Low …………………………….levels retards the conversion of carotene to vitamin A.
19. Chances of development of milk fever is more in …………………………………due to 54. …………………………………. And ………………………………………fertilizers interfere with
……………………..binding. the conversion of carotene too vitamin A.
20. Carbol (C.B.G.) contains ………………….parts Ca-gluconate and …………………parts boric acid. 55. …………………………….. is the store house of vitamin A.
21. Eclampsis in bitches is also known as …………………….,………………,or……………
56. Animal deficient in …………………………………..Absorbs vitamin A insufficiently. 92. Mineral essential for metabolism of propionate is…………………………………………
57. Karatomalacia is a ………………………………… Ocular sign in vitamin A deficiency. 93. Copper deficiency produces nervous manifestations known as …………………………
58. Corneal scar is a …………………………………… Ocular sign in vitamin A deficiency. and............................................
59. Conjunctival xerosis is usually confined to ………………………… part of conjunctiva. 94. Copper deficiency causes depigmentation of hairs and wool known as……………………
60. …………………………… associated with Vitamin A deficiency appears as a small silver grey 95. Copper deficiency causes myocardial degeneration and this condition is known
raised plaque on the conjunctiva. as……………………..
61. In corneal xerosis cornea turns hazy due to…………………………………….. and 96. Copper deficiency causes …………………………. Oestrus in cattle.
assumes……………………. 97. In dairy cattle excess thickening of skin known as ……………………………………Is due to
62. In corneal xerosis anterior chamber of the eye may be filled with cellular exudates leading to a deficiency of Zn.
condition known as……………………………………….. 98. Deficiency of Zn causes imperfect keratinization of the epithelial cells of the skin in pigs known
63. Cells of eye responsible for vision in day light are…………………………………. as……………………………………..
64. Cells of eye responsible for vision in night are ……………………………….. 99. ……………………………………… is known as anti infection vitamin.
65. Corneal opacity is a condition frequently observed in …………………………and 100. Chondromalacia is due to…………………………………
………………………… 101. Deficiency of vitamin A in fowl causes…………………………….
66. Corneal scar may be taken up as a sequelae to…………………………………………….. 102. Vitamin A is required for maintenance of specialized ………………………of the body.
67. In hypo vitaminosis A there is decrease in …………………………………. Activity of bones. 103. Hyper-vitaminosis causes development of ………………………………(intense osteoclastic
68. Hyperkeratinized condition of skin is known as ………………………….. activity).
69. ……………………………….. and ……………………………………. Diet accentuates the 104. Crazy chick disease is due to deficiency of …………………………
formation of calcium phosphate caliculi in the bladder and kidney. 105. Vitamin C is water……………………….. and heat……………………………
70. Deficiency of ……………………………….. leads to loss of cell mediated immunity. 106. Battery sickness among fowl is due to deficiency of ………………………………
71. The varieties of vitamin D having physiological function are………………………… 107. Beri-beri is due to deficiency of…………………………………………
and……………… 108. …………………………………………. Paralysis is due to deficiency of riboflavin.
72. Abnormal dryness of the conjunctiva occurs in xeropthalmia due to non 109. In dog deficiency of niacin causes……………………or………………………………
functioning…………………………. 110. Canine pellagra is formerly called as……………………………………………….
73. Vitamin d helps in the absorption of ………………………….. from gut. 111. Pellagra which means rough skin is manifested by………………………………………
74. Vitamin D2 and D3 are………………………….. effective in most of the animals but in case of 112. Hyperplasia of bone marrow is due to deficiency of………………………………………
chicken …………………………… is more effective. 113. Biotin has an important role in …………………………… fixation.
75. In older animals vitamin D deficiency causes ………………………………….. 114. Fatty liver condition is due to deficiency of ……………………………
76. Deficiency of …………………………………….. causes pilioencephalomalacia. 115. In fowl perosis is due to deficiency of …………………………and……………………
77. …………………………….., ……………………………….., ………………………Is due to 116. Vitamin B12 is also known as ………………………………………………. factor.
deficiency of nicotinic acid. 117. ……………………………………………… Is due to acute poisioning of selenium.
78. A disease called ………………………… is seen in sheep in Britain where there is loss of wool and 118. Alkali disease is due to chronic poisioning of…………………………………….
debility. 119. Deficiency of ………………………………….. causes fatty liver and heavy deposition of
79. Pine is a disease of sheep which is due to deficiency of ………………………………. hemosiderin in the spleen are noticed at postmortem. This condition is enzootic in New Zealand and
80. Vitamin E functions in animal body mainly as …………………………………………. is known as……………………………………..
81. ………………………………………… disease of pigs is a deficiency sign of vitamin E. 120. Zinc is an important constituent of enzymes…………………and………………………
82. Vitamin E is also known as ……………………………………………. Factor. 121. In sheep a condition known as …………………………………………. Is due to deficiency of Cu
83. White muscle disease is due to deficiency of …………………………………….. in which wool loses its crimps.
84. ……………………………………. syndrome in horses is due to deficiency of vitamin E. 122. Deficiency of copper may cause fatal syncope in cattle known as………………………
85. Vitamin K1 is also known as……………………….. 123. ……………………………………….. is necessary for the synthesis of cartilage matrix.
86. Vitamin K2 is also known as………………………………. 124. The epiphyseal cartilage fails to ossify due to deficiency of……………………………
87. Vitamin K 3 is also known as……………………………….. 125. Slipped tendon is due to deficiency of ………………………………………..
88. Vitamin K is ……………………… soluble and heat………………………… 126. Pica or ………………………………Is due to deficiency of ……………………………
89. Vitamin K takes part in normal ……………………………………… process of blood. 127. In horses a condition developed called ……………………………………………. Also called big
90. Sweet clover disease is due to dicoumarol poisioning due to deficiency head,………….. disease or barn disease is due to excess intake of phosphorus.
of…………………………….. 128. The normal inorganic phosphorus content of blood is …………………………………
91. About 90% of copper remains in plasma as …………………………………… 129. Calcium and phosphorus must be fed in a definite ratio of……………………………
130. Causative agent of kennel cough is…………………………………….. 169. Sulkowitch test is based on detection of ……………………… in……………………..
131. Major infectious diseases of lambs which causes mortality 170. Carbol conatins ……………………..parts Ca-gluconate and ………………… parts boric acid.
are…………………..…and…………………… 171. During toxicity of carbol the antidote ……………………… may be given.
132. Viral infection causing cerebellum hypoplasia in calves is………………………………. 172. The ratio of Ca : P should be ……………….. in cattle during last month of pregnancy to prevent
133. Enzootic ataxia in lamb is caused by deficiency of……………………………… milk fever.
134. Lamb dysentery is caused by……………………………. 173. Eclampsia in bitches can be differentially diagnosed from………………………………. ,
135. Inflammation of external aspect of umbilicus is known as…………………………… ………………………. , …………………………… and………………………………..
136. Feeding of ……………………………….. To neonates provides passive immunity. 174. Lahu-mutna is other name for…………………………………………………….
137. Clostral immunoglobulin present in intestine protects new born against………………… 175. Red water is caused due to deficiency of ………………………….. in diet.
138. Colisepticemia is a common cause of ………………………………. Mortality. 176. ………………………………… are formed due to oxidative changes in haemoglobin of RBC in
139. Tracheobronchitis is a highly contagious canine illness characterized by inflammation of the post parturient hemoglobinurea.
upper………………………… 177. Normal phosphorus amount in blood of cattle is………………..
140. Kennel cough can be …………………………… And …………………………… 178. Monday morning sickness is caused due to excessive accumulation of ………………… in muscles.
141. Parvo virus infection produces antibodies are categorized as………………. And later 179. Normal magnesium level of cow‘s blood is………………………………
as………………… 180. ………………………………………….is a disease of civilization.
142. The parvo virus has got affinity towards ………………………. And the ……………… system. 181. The most susceptible breed of cow to downer cow syndrome is…………………………
143. In porcine strees syndrome the death occurs due to excess ………… level causing cardiac arrest. 182. If the mastitis and metritis are the cause of downer cow syndrome then the prognosis
144. The inherited defect of HAL gene causes……………………………. In porcine. is…………………………………………
145. Other name of milk fever type of ketosis is………………………………. 183. If in case of downer cow syndrome both hind legs are spread laterally then the prognosis
146. The enzyme which is required for conversion of fatty acid to glucose is………………………. is……..……………………………..
147. NEFA stands for…………………………………………… 184. Pregnancy toxaemia is caused due to deficiency of………………………………..
148. Blood NEFA on oxidation produces……………………….. 185. ………………………………. Is the choice of drug in porcine stress syndrome.
149. Blood NEFA on esterification produces……………………………. 186. The dose of dantrolene is…………………………………
150. The amount of glucose required for synthesis of lactose in high yielding dairy cattle 187. …………………………………………… is also known as pro-viatmin D3.
is……………… 188. The vitamin produced in skin on exposure to sunlight is………………………………….
151. In a cow, yielding 20-40 litres of milk daily will drain out about …………… fat and 189. The vitamin also known as antisterlity factor is………………………………
about……………… protein per day. 190. Mineral present in enzyme glutathionate peroxidase is………………………………….
152. Antiketogenic volatile fatty acid is………………………………. 191. Vitamin which cannot be obtained from plant sources is…………………………………
153. ……………………….. Hormone stimulates formation of glucose from fat and protein. 192. ……………………………… acts as coenzymes for carboxylase.
154. Precursor of Acetyl CoA is…………………………………… 193. Polyneuritis may be caused due to deficiency of………………………………..
155. Woody appearance of cow is due to……………………… 194. Maturation of RBC delays due to deficiency of…………………………………
156. In ketosis, the DLC picture shows……………………………….. 195. ……………………………….. is also known as anti-pellagra factor.
157. In clinical ketosis, usually plasma glucose level is below……………………. And NEFA greater 196. ………………………………..is also known as antidermatitis vitamin.
than…......................... 197. Other name for vitamin B6 is………………………………
158. …………………………… can be done for diagnosis of ketone bodies in urine. 198. Other name for Niacin is…………………………………….
159. In rothera test, if urine is not available then………………………………….. can also be used for 199. Anti-anaemic vitamin is…………………………
diagnosis. 200. Cynacoblamin is obtained from fungus………………………………………………
160. Nervous form of ketosis can be differentiated from nitrate poisioning by the absence of 201. …………………… acts as coenzyme in oxidation of tyrosine and phenylalanine.
…………………... In ketosis. 202. The vitamin that is destroyed in rumen is………………………………………
161. Parturient paresis occur most commonly within ………………….. days of parturition. 203. Water soluble vitamins are…………………………..
162. There is deficiency of…………………………… mineral in parturient paresis. 204. Fat soluble vitamins are…………………………………..
163. Hypocalcemia occurs in most cows with their ………………………….. calving. 205. Iron deficiency anaemia caused due to parasitic burden is characterized
164. Milk fever occurs commonly in………………………….. and………………….. season. by…………………………Condition.
165. The normal ratio of calcium and phosphorus in blood of cow is…………………… 206. Haemitinic mixture contains ferritic ammonium citrate…………………..gm, copper
166. The amount of calcium in colostrum is about …………………. Times more than normal milk. sulfate…………… mg, and cobalt sulfate……………………mg.
167. ………...... shape posture appears in cattle in milk fever. 207. About ……………………. Of copper remains in plasma as……………………………
168. Non parturient milk fever occurs in about………………………….. percent cases. 208. Runting is seen in……………………………….
209. Starvation and crushing are major cause of pre-weaning mortality in………………………… 246. Animals which are unable to rise after 24 hours and also after two treatments of calcium can be
210. Piglets is subjected to cold stress when the temperature goes below………………………. classified as…………………………………..
211. Colisepticaemia is common cause of …………………………… mortality. 247. Therapeutic dose of vitamin A in a calf suffering from hypovitaminosis A is……………
212. New born should be fed colostrums with in …………………….. of birth. ……………...I.U./Kg. B.wt.
213. Young one of porcine are more susceptible to……………………………. Deficiency. 248. Excessive Ca without complementary P & vitamin D may lead to……………………….
214. Secondary deficiency of copper may occur due to excess of …………………………… 249. Treatment of acute hypothiaminosis in goat is……………………………….
215. High level of copper causes ……………………………… of liver. 250. Normal range of blood calcium in cow is…………………..
216. The requirement of copper is about……………………………… in an adult animal.
217. Metallic component of enzyme carbonic anhydrase is………………………………
218. Metallic component of enzyme alkaline phosphatase is………………………………
219. Canine distemper is also known as…………………………………. Disease. ANSWER – 24. S-shaped 47. Amblyopia
220. In cattle deficiency of zinc causes excess thickening of skin known as………………… 25. Spectroscopic 48. Nyctopia
221. Due to deficiency of zinc in pigs, there is imperfect keratinization of epithelial cells of skin known 1. Third examination 49. Vitamin A
as……………………………….. 2. Post partum 26. Lactation tetany, 50. Thyroxine
222. …………………….. responsive dermatitis often respond to zinc therapy. 3. Digestive from, wheat pasture 51. vitamin A
223. Normal level of zinc in cattle is………………………… ketogenic poisoning, grass 52. Carotene
224. ………………….. is a component of cyanocoblamin. 4. Hypoglycemia stagger 53. Phosphorus
225. Dietary ……………..increases while…………………decreases the chances of production of milk 5. Complicated ketosis 27. Tying up syndrome 54. Nitrate and nitrite
fever. 6. Hypoglycemia, 28. Poor 55. Liver
226. Selenium is absorbed from…………………………….. but not from…………………… hypocalcaemia 29. Acetylcholine 56. Vitamin E
and………………….. 7. Negative energy esterase 57. Primary
227. Non inflammatory degenerative necrotic changes in cardiac and skeletal muscles due to deficiency balance 30. Dantrolene 58. Secondary
of selenium is known as…………………………….. 8. 35 mg/dl, 100µeq/l 31. Autosomal recessive 59. Bulbar
228. In acute enzootic muscular dystrophy……………………….. muscles are affected. 9. Acetyl CoA gene 60. Biot‘s spot
229. In sub-acute enzootic muscular dystrophy……………………….. muscles are affected. 10. Ketosis 32. Sucrose, starch, 61. Cellular infiltration,
230. In white muscle disease white areas of muscles shows …………………………….. degeneration. 11. Glucose, nitrogen, lactose bluish milky
231. Hepatosis dietecia is seen in pigs due to deficiency of…………….. and………………… ketone bodies 33. Sheep, goat appearance
232. There is reduction of enzyme…………………………………………….. level in blood and tissues 12. 72 hours 34. Baby pig disease, 3 62. Hypopyon
in white muscle disease. 13. Calving paralysis, days pig disease 63. Cone
233. Cobalt acts as a source of amino acid…………………………….. milk fever 35. Malignant 64. Rods
234. Cobalt is conserved in liver as…………………… 14. Jersey cow hypothermia 65. Calf, dog
235. Cobalt bound to plasma protein is known as………………………. 15. Milk fever 36. Increased 66. Xeropthalmia
236. Cobalt is released into cell cytosol as ……………………………………… syndrome, milk fever 37. Kidding sickness, 67. Osteoblastic
237. Liver and muscle specific enzymes……………………… ,……………………………. complex sleepy sick, twin 68. Xerodermia
And………………………. Increases in cobalt deficiency. 16. 6:1 lamb disease 69. Low vitamin A,
238. Iodine deficiency can be caused in animals when there is excessive feeding of ……………….. 17. Sulkowitch 38. Pregnancy toxaemia calcium
mineral. 18. Calcium 39. Acetylcholine 70. Vitamin A
239. An example of goiterogenic substance is………………………… 19. Metabolic alkalosis, esterase 71. D2 and D3
240. …………………………….. is caused due to deficiency of iodine. albumin 40. 2.3 mg/dl 72. Goblet cells
241. Naval ill is called so because the infection starts at………………………….. and may reach upto 20. 83, 17 41. High 73. Ca
joints. 21. Lactation tetany, 42. Monday morning 74. Equally, D3
242. The inflammation of umbilicus is called…………………………….. puerperal tetany, post sickness 75. Osteomalacia
243. Cabbage conatins…………………………………… which acts as goiterogen. parturient tetany 43. Small breed of dog, 76. Thiamine
244. In Monday morning sickness, mostly………………………….. limbs are affected. 22. Hypophosphatemia, Large (alsation) 77. Diarrohea, dysentery,
245. In milk fever there are three stages………………………………….. , nutritional 44. Vitamin A dementia
…………………………………. And…………………………………. hemoglobinurea 45. Carotene 78. Pine
23. Increases 46. Coagluative 79. Vit. B12
80. Antioxidant 122. Falling disease 159. Milk 199. Cyanocoblamin 219. Hard pad/carre‘s 236. Hydroxocobalam
81. Mulberry heart 123. Manganese 160. Diarrhea 200. Streptomyces disease ine
disease 124. Manganese 161. 3 grisens 220. Hyperkeratosis 237. CPK, SGOT,
82. Antisterlity 125. Manganese 162. Calcium 201. Ascorbic acid 221. Parakaratosis SGPT
83. Vitamin E 126. Allotriopathy, 163. 3-7th 202. Ascorbic acid 222. Zinc 238. Calcium
84. Tying up phosphorus 164. Later winter, 203. Vitamin B, 223. 20-120mg/dl 239. Thiouracil
85. Phytonadione 127. Osteitis fibrosa, spring vitamin C 224. Cobalt (cabbage)
86. Farnoquinone miller‘s 165. 2.3:1 or 2:1 204. Vitamin – 225. Cation, Anion 240. Goiter
87. Menadione 128. 4-6 mg/100 ml 166. 12 A,D,E,K 226. Duodenum, 241. Umbilical cord
88. Fat, stable 129. 2:1 167. S 205. Bottle jaw abomasum, rumen 242. Oomphelitis
89. Coagulation 130. Bordetella 168. 5-7 % 206. 1.2, 60, 15 227. White muscle 243. Thiouracil
90. Vitamin K bronchiseptica 169. Ca, urine 207. 90 %, disease/ stiff lamb 244. Hind
91. Ceruloplasmin 131. Struck, lamb 170. 83, 17 ceruplasmin disease 245. Very fleeting
92. Cobalt dysentery 171. MgSO4 208. Pig 228. Cardiac stage, stage of sitting
93. Neonatal ataxia, 132. BVD-MD 172. 1:3.3 209. Porcine 229. Skeletal on sternum, stage of
sway back 133. Copper 173. Tetanus, 210. 93° F 230. Zenker‘s lateral placement of
94. Achromotrichia 134. Clostridium strychnine poisoning, 211. Calf 231. Selenium, body
95. Falling disease perfringens type-B rabies, epilepsy 212. 3 hours Vitamin E 246. Downer cow
96. Delayed 135. Naval ill 174. Post parturient 213. Iron 232. Glutathione syndrome
97. Hyperkeratosis 136. Colostrum hemoglobinurea 214. Molybdenum peroxidase 247. 10-15 lakh
98. Parakaratosis 137. Diarrhea 175. Phosphorus 215. Cirrhosis 233. Methionine 248. Osteomalacia
99. Vitamin A 138. Calf 176. Heinz bodies 216. 5 PPM 234. Methylcobalamin 249. Yeast
100. Hypervitaminosis 139. Respiratory tract 177. 4-7 mg/dl 217. Zinc e 250. 8-10.5 mg/dl
101. Nutritional roup 140. Viral, bacterial 178. Glycogen 218. Zinc 235. Transcobalamine
102. Epithelial surface 141. Ig M, Ig G 179. 2.3 mg/dl
103. Osteoporosis 142. Lymphocytes, 180. Hypomagnesemi
104. Vitamin E lymphatic system c tetany
105. Soluble, labile 143. Potassium 181. Holstein breed
106. Battery sickness 144. Porcine stress 182. Hopeful
107. Vitamin B1 syndrome 183. Hopeless
108. Curled toe 145. Complicated 184. Blood glucose
109. Black tongue, ketosis 185. Dentrolene
canine distemper 146. CoA 186. 4-5 mg/kg, I/V
110. Stuttgart disease 147. Non Esterified 187. 7-
111. Dermatitis Fatty Acid dehydrocholesterol
112. Pyridoxine 148. Ketone bodies 188. Vitamin D3
113. CO2 149. Triglycerides 189. Vitamin E
114. Choline 150. 1 kg 190. Selenium
1. To improve the negative energy balance in d. The ―input‖ into the animal
115. Choline and Mg 151. 0.4 kg 191. Vitamin B12
a dairy animal which of the following must be less than ―output‖.
116. Anti pernicious 152. Propionate 192. Thiamine statement stands correct 2. Compton metabolic test for prediction of
anaemia 153. Thyroxine, 193. Thiamin/Aneurin a. The ―input‖ into the animal health status of dairy animals based on
117. Blind stagger cortisone 194. Cyanocoblamin must be equal or less than blood biochemical profiles includes the
118. Selenium 154. Fatty acid 195. Nicotinic ―output‖. following tests except one;
119. Cobalt 155. Hypoglycemia acid(niacin) b. The ―input‖ into the animal a. Blood Manganese, Iodine
120. Carboxyl 156. Lymphocytosis 196. Pyridine(Vitamin must be equal to ―output‖. b. Blood Sodium, nitrogen
c. The ―input‖ into the animal c. Serum Copper, Iron, Potassium
anhydrase, carboxy 157. 35 mg/dl, B6 )
must be equal or more than d. All the above
peptidase 1000µeq/l 197. Pyridoxine ―output‖. 3. Ketosis in dairy cattle can be caused by;
121. Steely wool 158. Rothera 198. Nicotinic acid
a. Glucose availability, negative 10. Wasting type of bovine ketosis can be longer periods to horse can cause which of c. Nitrate poisioning
energy balance, imperfect confused by: the following: d. All the above
NEFA utilization. a. Indigestion a. Phosphorus deficiency 27. Biot‘s spot is associated with:
b. Glucose availability, negative b. Abomasal displacement b. Milk fever a. Vitamin B deficiency
energy balance, perfect NEFA c. Traumatic reticulitis c. Osteodystropic fibrosa b. Vitamin C deficiency
utilization. d. All the above d. Rickets c. Vitamin A deficiency
c. Glucose non-availability, 11. Milk fever can be prevented by feeding the 19. Azoturia is seen in horses : d. Vitamin E deficiency
negative energy balance, animal pre-partuma: a. During exercise after resting on 28. Which of the following vitamin acts an
imperfect NEFA utilization. a. High calcium diet full ration antioxidant:
d. Glucose non- availability, b. High protein diet b. During rest on full ration a. Vitamin A
negative energy balance, c. High phosphorus and low c. During exercise on poor ration b. Vitamin E
perfect NEFA utilization. calcium diet d. All the above c. Vitamin C
4. Muscular dystrophy can be treated by d. None of the above 20. Pica is caused by: d. Both b and c
supplementing : 12. Polioencephalomalacia in animals occurs a. Dietary deficiency of bulk 29. Menadione is also known as
a. Vitamin E due to deficiency of b. Dietary deficiency of nutrients a. Vitamin K1
b. Phosphorus a. Thiamin c. Boredom b. Vitamin K2
c. Calcium b. Vitamin D d. All the above c. Vitamin K3
d. Protein c. Vitamin E 21. Hypomagnesemic tatany may occur after: d. Vitamin K5
5. Feeding of cereals containing high d. All the above a. Long stressful transport 30. farnoquinone is also known as
contents of potassium for longer period 13. Symptoms of hypomagnessemic tetany in b. Grazing on grass dominant a. Vitamin K1
may cause: animals occur when the: pasture b. Vitamin K2
a. Lactation tetany a. Hypomagnesemia alone c. Grazing on pastures heavily top c. Vitamin K3
b. Iron deficiency anaemia b. Hypocalcaemia alone dressed with N and K fertilizers d. Vitamin K5
c. Ketosis c. Hypocalcaemia and d. All the above 31. Approximately how much percent of body
d. Milk fever hypomagnesemia both 22. In the healthy animals, intake of iron from iron remains functionally as haemoglobin,
6. Xeropthalmia occurs due to deficiency of: d. Hypophosphatemia alone gastro intestinal tract contents into myoglobin, etc.:
a. Riboflavin 14. The production disease of farm animals circulation is: a. 10%
b. Ascorbic acid include: a. 10% b. 35%
c. Vitamin A a. Ruminal load b. 20% c. 50%
d. Vitamin D b. Selenium vitamin-E in- c. 15% d. 70%
adequency d. Less than 1% 32. Iron deficiency may be as a result of:
c. Rumen acidosis/alkalosis 23. If iron is taken for long time, it may cause: a. Magnesium deficiency
d. Nine of the above a. Constipation b. Cobalt deficiency
7. Majority of clinical cases of parturient b. Diarrohea c. Copper deficiency
paresis occurs: 15. In the Compton metabolic profile test the c. Phosphorus deficiency d. Manganese deficiency
a. During advance stage of blood Is analysed for d. All the above 33. Secondary deficiency of copper may be
pregnancy a. Calcium 24. Oral dose of iron preparation in anaemia in due to:
b. During first 48 hours of b. Inorganic phosphorus cattle is: a. Low level of molybdenum
parturition c. Blood urea nitrogen a. 20 – 25 gms b. High level of molybdenum
c. Between 10-25 days after d. Total serum proteins b. 25 – 30 gms c. Low level of iron
parturition 16. Peat scour is due to which of the c. 5 – 10 gms d. High level of iron
d. One month after parturition following: d. 50 – 60 gms 34. Neonatal ataxia can be caused due to
8. Enzootic ataxia in lambs is caused by a. Deficiency of cobalt 25. In iron deficiency there is: deficiency of
deficiency of which of the following: b. Deficiency of copper a. Microcytic hypochromic a. Zinc
a. Copper c. Deficiency of molybednum anaemia b. Copper
b. Selenium d. Deficiency of iron b. Macrocytic hypochromic c. Iron
c. Manganese 17. Pregnancy toxaemia in ewes occurs anaemia d. Molybdenum
d. Iron during: c. Microcytic normochromic 35. Depigmentation of hairs and wool known
9. Molybdenum poisioning can cause a. Early pregnancy anaemia as achromotrichia is caused due to
deficiency of : b. Mid-pregnancy d. Microcytic normochromic a. Hyper manganesemia
a. Copper c. Parturition anaemia b. Hypo manganesemia
b. Iron d. Last month of pregnancy 26. Nervous form of ketosis can be confused c. Iron deficiency
c. Zinc 18. Feeding of wheat bran containing high with: d. Copper deficiency
d. Selenium contents of phosphorus, in excess for a. Rabies 36. Falling disease is caused due to
b. Lead poisoning a. Hyper manganesemia
Dhruv N Desai Dhruv N Desai
b. Hypo manganesemia c. Vitamin C c. Absolute ketosis 59. Night blindness due to deficiency of
c. Iron deficiency d. All the above d. Nervous ketosis vitamin A is:
d. Copper deficiency 44. In Nutritional myopathy there is a non 52. Does a normal cow produce ketone a. Primary ocular sign
37. Deficiency of zinc causes imperfect inflammatory degenerative or necrotic bodies? b. Secondary ocular sign
keratinization of the epithelial cells of the changes in cardiac and skeletal muscles a. Yes & may cause ketosis c. Biot‘s spot
skin which is also known as caused due to deficiency of: b. No d. Due to corneal xerosis
―parakaratosis‖. This disease is caused in a. Copper c. Yes & they are used in her 60. In keratomalacia there is softening of
a. Cattle b. Molybdenum body cornea due to:
b. Pig c. Cobalt d. Both a and c are correct a. Liquefactive necrosis
c. Poultry d. Selenium b. Colliquative necrosis
d. Goats 45. Mulberry heart disease and yellow fat c. Caeseous necrosis
38. Deficiency of zinc causes imperfect disease in pigs is caused due to: 53. Hematological change in ketosis is : d. Fat necrosis
keratinization of the epithelial cells of the a. Deficiency of selenium and a. Eosinophilia 61. Consider the following statements;
skin which is also known as ―hyper- Vitamin C b. Neutrophilia I. Due to deficiency of vitamin A
karatosis‖. This disease is caused in b. Deficiency of selenium and c. Lymphocytosis there may be increase in the
a. Cattle Vitamin E d. Monocytosis osteoblastic activity during growth
b. Pig c. Deficiency of copper and 54. Normal calcium : phosphorus in blood is stage.
c. Poultry Vitamin C about: II. There is decrease in osteoclastic
d. Goats d. Deficiency of copper and a. 2:1 activity due to hypovitaminosis A.
39. The blood sample of an animal seems to vitamin E b. 4:1
be anaemic reveals the presence of ―Heinz 46. Which of the following animal disease is c. 1:6 Which of the above statement/s is/are
bodies‖. The animal is probably suffering more commonly found to occur in sub- d. 6:1 correct?
from Himalayan region: 55. Milk fever of bitch is known as
a. I
a. Zinc deficiency a. Anaemia a. Downer‘s syndrome
b. II
b. Iron deficiency b. Goiter b. Parturient paresis
c. Both I and II are true
c. Copper deficiency c. Rickets c. Eclampsis
d. Both I and II are false
d. Magnesium deficiency d. Pica d. None of them
62. Consider the following statements;
40. Consider the following statement: 47. Photophobia in animals can be caused due 56. An animal was brought to the hospital
I. Vitamin D2 is also known as
I. Cobalt acts as growth inducer to deficiency of: suffering from hyperkeratosis of skin. The
cholecalciferol.
II. Cobalt acts as a source of amino a. Thiamine animal may be suffering from deficiency
II. Vitamin D3 is also known as
acid – cysteine b. Pyridoxine of :
ergosterol.
III. Cobalt helps in erythropoiesis c. Riboflavin a. Vitamin A
process d. Biotin b. Vitamin C Which of the above statement/s is/are
Which of the above statement/s 48. Of the three varieties of tocopherols which c. Vitamin D correct?
is/are correct? is most active d. Vitamin E
a. 1 only a. α - tocopherol 57. Which hormone helps in the conversion of a. I
b. 2 and 3 only b. β - tocopherol carotene to vitamin A: b. II
c. 1 and 3 only c. ϒ- tocopherol a. FSH c. Both I and II are true
d. 1,2,3 d. Both a and c b. Thyroxin d. Neither I and II is true
41. Deficiency of cobalt in diet causes: 49. Which of the following cow seldom c. Calcitonin 63. Cortinase enzyme is very much required
a. Microcytic anaemia suffers from ketosis: d. Adrenaline for conversion of carotene to vitamin A in
b. Macrocytic anaemia a. Low producing 58. Consider the following statements: a. Liver
c. Hypochromic anaemia b. Plain diet fed cow I. Nitrate and nitrite fertilizers b. Gut
d. Megaloblastic anaemia c. Both a and b interfere with the conversion of c. Intestinal mucosa
42. Cobalt is essential for the production of d. None of the above carotene to Vitamin A. d. Muscles
vitamin B12. The production of vitamin 50. Ketonurea with hypoglycemia occur in II. Cathartic drugs increase the 64. There may be vitamin A absorption
B12 occurs in which disease: availability of vitamin A from gut. insufficiency due to deficiency of:
a. Liver a. Ketosis a. Vitamin C
b. Muscles b. Acidosis Which of the above statements are true? b. Vitamin H
c. Rumen c. Diabetic ketoacidosis c. Vitamin B complex
a. I only
d. Abomasum d. None of them d. Vitamin E
b. Both I and II
43. Which of the following acts as 51. Ketosis occur during adequate dietary 65. Day blindness is also known as:
c. II only
antioxidants: supply is known as: a. Nyctalopia
d. Both are false
a. Selenium a. Starvation ketosis b. Amblyopia
b. Methionine b. Secondary ketosis c. Myopia
Dhruv N Desai Dhruv N Desai
d. Hypermetropia a. Only I c. Ascorbic acid 90. In man 3D (dementia, diarrhea, and
b. Both II and II d. Pyridoxine dermatitis) is associated with the
66. Nyctalopia is due to defective formation c. Only II 81. A disease called pine is seen in sheep in deficiency of which vitamin?
of: d. Both are true Britain due to deficiency of: a. Riboflavin
a. Visual red a. Biotin b. Nicotinic acid
b. Visual blue b. Cyanocobalamin c. Pantothenic acid
c. Visual purple 73. The amount fo 1,2,5-dihydroxy c. Ascorbic acid d. Choline
d. Visual green cholecalciferol produced by the kidney is d. Biotin 91. Photophobia in animals is caused due to
67. The cells of eye which helps in the day controlled by: 82. Acrodyna is caused due to deficiency of : deficiency of:
vision are: a. Thyroid hormone a. Pyridoxine a. Folic acid
a. Rod cells b. Parathyroid hormone b. Cyanocobalamin b. Nicotinic acid
b. Cone cells c. Calcitonin c. Biotin c. Riboflavin
c. Corneal cells d. Thyrotropin hormone d. Pantothenic acid d. Ascorbic acid
d. Retinal cells 74. Vitamin E functions as an antioxidant in 83. Streptomyces grisens is a rich source of: 92. Which of the following vitamin is not
68. There may be formation of renal stones association with the selenium containing a. Choline required by dog to be added in the feed:
due to: enzyme: b. Ascorbic acid a. Choline
a. High vitamin A and low calcium in a. Hydroperoxidase c. Cyanocobalamin b. Ascorbic acid
diet b. Glutathione peroxidase d. Pyridoxine c. Cyanocobalamin
b. High vitamin A and High calcium c. Alkaline peroxidase 84. Antidote of arsenic poisoning is d. Pyridoxine
in diet d. Guanine peroxidase a. Folic acid 93. Bottle jaw condition is caused due to
c. Low vitamin A and low calcium in 75. ‗Tying up‘ syndrome in horse is due to b. Nicotinic acid deficiency of:
diet deficiency of: c. Pantothenic acid a. Copper
d. Low vitamin A and high calcium in a. Vitamin E d. Ascorbic acid b. Zinc
diet b. Vitamin A 85. Which vitamin is required for DNA c. Iron
69. The precursors of vitamin D2 and vitamin c. Vitamin D synthesis: d. Calcium
D3 are: d. Vitamin B complex a. Folic acid 94. Which of the following pair of metals is
a. 7-dehydrocholesterol and 76. ‗Mulberry heart disease‘ can be caused in b. Nicotinic acid required for the synthesis of hemoglobin:
ergosterol respectively pigs due to deficiency of: c. Pantothenic acid a. Copper and magnesium
b. 7-hydroxycholesterol and a. Vitamin E d. Ascorbic acid b. Copper and iron
ergosterol respectively b. Vitamin A 86. Which vitamin takes part in methylation c. Copper and selenium
c. Ergosterol and 7- c. Vitamin D and thus helps in phospholipid d. Iron and magnesium
dehydrocholesterol respectively d. Vitamin B complex metabolism: 95. Normal copper range in animals is:
d. Ergosterol and 7- 77. ‗White muscle disease‘ of young calf is a. Folic acid a. 0.5-1.5 µg/ml
hydroxychloesterol respectively due to deficiency of: b. Choline b. 0.5-1.5 mg/ml
70. Which of the following vitamin is a. Vitamin E c. Pantothenic acid c. 0.5-1.5 gm/ml
produced in the skin on exposure to b. Vitamin A d. Biotin d. 0.5-1.5 gm/dl
sunlight: c. Vitamin D 87. Which vitamin is destroyed in rumen: 96. Consider the following statements;
a. Vitamin D2 d. Vitamin B complex a. Folic acid I. Copper absorption and retention is
b. Vitamin D3 78. Which of the following vitamin is also b. Nicotinic acid not affected by Ca and Zn.
c. Vitamin D12 known as antisterlity hormone: c. Pantothenic acid II. Copper deficiency can be caused
d. Vitamin D5 a. Vitamin E d. Ascorbic acid by high level of molybdenum.
71. Rickets due to deficiency of vitamin D is b. Vitamin A 88. Which vitamin is required for fast healing
caused in c. Vitamin D of wounds: Which of the above statement/s is/are true?
a. Neonates d. Vitamin B complex a. Choline
a. I only
b. Young animals 79. Which of the following vitamin cannot be b. Ascorbic acid
b. II only
c. Adult animals obtained from plant sources: c. Cyanocobalamin
c. Both I and II
d. Old animals a. Pantothenic acid d. Pyridoxine
d. None
72. Consider the following statements; b. Cyanocobalamin 89. Aneurin is the other name for:
97. Consider the following statements;
I. Osteomalacia is a disease of older c. Nicotinic acid a. Pyridoxine
I. Copper absorption and retention is
animals due to deficiency of d. Biotin b. Thiamine
affected by Ca and Zn.
vitamin D. 80. Which of the following vitamin acts as co- c. Biotin
II. Copper deficiency causes ‗sway
II. It is a common condition found in enzyme in the oxidation of tyrosine and d. Choline
back‘ and ‗neonatal ataxia‘.
farm animals. phenylalanine:
a. Choline Which of the above statement/s is/are true?
Which of the above statement/s is/are true? b. Biotin
Dhruv N Desai Dhruv N Desai
a. I only 102. Match list I with list II and select the 106. Consider the following statements; d. Biotin
b. II only correct answer using the code given below 1. Vitamin A deficiency mostly occurs 112. Ketosis / acetonemia is defined as:
c. Both I and II the list during the latter half of gestation and a. Relative lack of CHO in the body
d. None List I is characterized by absorption or by b. Absolute lack of CHO in the body
98. Consider the following diseases in List II birth of a dead calf. c. Both a and b
animals; (Type of ketosis) 2. Vitamin A requirements are higher d. None of them
i. Sway back disease (Characteristic) 1 for female animals. 113. Time of occurrence of ketosis in animal
ii. Achromotrichia A. Primary ketosis Which of the statement/s given above is/are is which of the following:
iii. Falling disease B. Secondary ketosis correct? a. One month before calving
iv. Spectacle disease C. Starvation ketosis a. 1 only b. In new born calf
D. Alimentary ketosis 2 b. 2 only c. In calf at one year age
Which of the above is are caused due to c. Both 1 and 2 d. One month after calving
deficiency of copper? 3 d. Neither 1 and 2 114. Starvation ketosis result in symptoms
107. Consider the following conditions: related to which form of ketosis:
a. i and iv
4 1. Hypocalcaemia a. Wasting form
b. i, ii and iii
2. Hypophosphatemia b. Nervous form
c. Ii and iii Code 3. Neurological muscle paralysis c. Both a and b
d. All of the above
4. Ischemic muscle necrosis d. None
99. Consider the following statements; A B C D
115. Ketogenic amino acid are which of the
1. If vitamin A is available over a. 2 4 3 1 Etiology of downer cow syndrome includes following:
and above body requirements, it b. 2 3 4 1 which of the above? a. Acetate and propionate
is stored in the animal body in c. 3 4 1 2
b. Propionate and butyrate
substantial amounts. d. 3 1 4 2 a. 1 and 2 only
c. Acetate and butyrate
2.Vitamin E is not stored in the 103. A mare with a history of recent foaling b. 1 and 4 only
d. All of the above
animal body in large amounts was presented in a veterinary clinic with c. 1,2 and 4
116. High protein in diet lead to ketosis due to
for any length of time and hence symptoms of sweating, stiffness in gait and d. 2,3 and 4
excess production of :
a regular dietary supply is anurea, the intravenous administration of 108. The brain parts of chick affected by
a. Propionate
required. calcium provided immediate symptomatic encephalomalacia caused by vitamin E
b. Butyrate
relief. What would be the most likely deficiency are:
Which of the above statement/s is/are true? c. Acetate
disease? a. Cerebellum
d. None
a. Azoturia b. Strail hemisphere
a. 1 only 117. Border line ketosis/ spontaneous ketosis/
b. Eclampsia c. Medulla oblongata
b. 2 only feeding ketosis occurs in:
c. Strangles d. Mesencephelon
c. Both 1 and 2 a. High yielding cattle
d. Tetanus 109. Star gazing attitude in poultry is
d. None of the above b. Low yielding cattle
104. Lactation tetany in high yielding dairy observed in the deficiency of:
100. In cattle, symptoms like abnormal c. In heifer
cow is a well recognized production a. Riboflavin
appetite and chewing of wood, bones, rags d. In 7th lactation of cow
disease. Which of the following factors b. Thiamine
and other foreign materials indicate the 118. Milk fever type ketosis have which of
serve as a predisposing cause for the c. Niacin
deficiency of the following:
occurrence of same? d. Pantothenic acid
a. Co a. Hypoglycemia
a. High ammonia content of rumen 110. Consider the following:
b. NaCl b. Hypoglycemia and
b. Increased Mn and decreased Co 1. Vitamin B complex
c. P hypomagnesaemia
concentration in diet 2. Vitamin C
d. All the above c. Hypoglycemia and hypocalcaemia
c. Feeding high energy diet which 3. Calcium
101. Ketosis can be prevented by which of the d. Hypocalcaemia and
interferes with Mn absorption 4. Iron
combination of feed additives? hypomagnesaemia
d. None of the above is justified Meat is the rich source of which of the
a. Dicalcium phosphate and calcium 119. End result of stress in high yielding cow
predisposing cause of the above?
propionate is:
syndrome a. 1 and 2
b. Sodium propionate and propylene a. Acidosis
105. Which one of the following trace mineral b. 1 and 3
glycol b. TRP
is required for growth chicks and c. 2 and 3
c. Sodium bicarbonate and propylene c. Pneumonia
maintenance of xanthine oxidase content d. 1 and 4
glycol d. Ketosis
of tissues? 111. Cerebrocortical necrosis is a condition
d. Calcium phosphate and niacin 120. Which of the following cow seldom
a. Selenium associated with
suffers from ketosis:
b. Zinc a. Riboflavin
a. Low producing
c. Molybdenum b. Pantothenic acid
b. Plain diet fed
d. Copper c. Thiamin
c. Both a and b
Dhruv N Desai Dhruv N Desai
d. None of them 130. Ketolactia is a term for which of the 139. Ketone level in milk and urine in ketosis d. 3:1
121. Which vitamin deficiency may cause following? is: 148. Normal ratio of calcium to magnesium in
ketosis? a. Ketone bodies in urine a. 40 mg/100 ml in milk & 500-1000 blood is
a. Vitamin B1 b. Ketone bodies in milk mg/100 ml in urine a. 2:1
b. Vitamin B12 c. Ketone bodies in blood b. 500-1000 mg/100 ml in milk & 40 b. 4:1
c. Vitamin B5 d. Ketone bodies in milk and food mg/100 ml in urine c. 6:1
d. Vitamin C 131. Ketonaemia is a term for which of the c. 250 mg/100 ml in milk & 80 d. 8:1
122. Level of acetoacetic acid, β-hydroxy following? mg/100 ml in urine 149. Which reflex is lost in IInd stage of milk
butyrate and free fatty acids in the blood in a. Ketone bodies in urine d. 80 mg/100 ml in milk & 250 fever:
ketosis is: b. Ketone bodies in milk mg/100 ml in urine a. Pupillary reflex
a. 0.1 mg/100 ml, 8%, 9% c. Ketone bodies in blood 140. Hematological change in ketosis is b. Rectal reflex
b. 0.1 mg/100 ml, 30%, 9% d. Ketone bodies in milk and food which of the following: c. Both a and b
c. 7%, 15%, 28% 132. Ketonurea is a term for which of the a. Eosinophilia d. None
d. 7%, 30%, 28% following? b. Neutropenia 150. Test for milk fever is:
123. Test of urine for ketosis is which of the a. Ketone bodies in urine c. Lymphocytosis a. Sulphudryl test
following: b. Ketone bodies in milk d. All the above b. Sulkowitch test
a. Rothera test c. Ketone bodies in blood 141. Ketone body test in milk is called c. Ascoli test
b. Ross test d. Ketone bodies in milk and food a. Rothera test d. Rors test
c. California test 133. Ketosis occur during adequate dietary b. Ross test 151. Milk fever like stances with normal
d. Benzedene test supply is known as: c. California mastitis test voiding of feces and urine and normal
124. Which of the following is Rothera a. Starvation ketosis d. Turbidity test appetite indicates:
reagent? b. Primary ketosis a. Downer cow syndrome
a. Ammonium sulfate c. Relative ketosis 142. Level of dextrose for treatment of b. Parturient paresis
b. Sodium nitrate d. Secondary ketosis ketosis: c. Eclampsis
c. Sodium nitro prusside a. 5% dextrose 500 ml I/V d. None
d. All of the above 134. Ketosis occur when dietary supply is less b. 10% dextrose 500 ml I/V or S/C 152. Milk fever of bitch is known as:
125. Is ketosis a self limiting disease? than requirement is known as: c. 50% dextrose 500 ml S/C a. Downer cow syndrome
a. Yes a. Starvation ketosis d. 50% dextrose 500 ml I/V b. Parturient paresis
b. No b. Diabetic ketosis 143. Triamacilone corticoid used for Rx of c. Eclampsis
c. In some extent c. Relative ketosis which disease: d. None
d. None d. Absolute ketosis a. Milk fever 153. Composition of CBG is which of the
126. Pregnancy toxaemia is which of the 135. Ketosis represents the use of which of b. Ketosis following:
following? the following: c. Acidosis a. Mg gluconate 80%, Boric acid
a. Caprine ketosis a. CHO d. Alkalosis 20%
b. Ovine ketosis b. Fat 144. In milk fever the temperature is b. Ca gluconate 83%, Mg gluconate
c. Cow ketosis c. Protein generally which of the following: 17%
d. Both a and b d. Vitamins a. Subnormal c. Ca gluconate 83%, boric acid 17%
127. Ketonurea with hyperglycemia occurs in 136. Cause of ovine pregnancy toxemia is b. Normal d. Calcium gluconate 83%, DW 17%
which disease: which of the following: c. Increase 154. Prerequisite for CBG administration is
a. Ketosis a. Increase of adrenal cortical level d. None which of the following:
b. Acidosis b. Stress of late pregnancy 145. Level of calcium in normal blood is: a. Warm up to body temperature
c. Diabetic ketoacidosis c. Increase demands of nutrition by a. 1-2 mg % b. Slow I/V infusion
d. None twins b. 2-6 mg % c. Both a and b
128. Ketonurea with hypoglycemia occurs in d. All the above c. 6-8 mg % d. None
which disease: 137. Iron deficiency anaemia is a common d. 9-10 mg % 155. Rapid CBG leads to:
a. Ketosis anemias in: 146. Level of calcium in case of milk fever is: a. Rapid recovery
b. Acidosis a. Cattle and horses a. 1-2 mg % b. Ventricular fibrillation
c. Diabetic ketoacidosis b. Adult pigs b. 2-6 mg % c. Toxemia
d. None c. Piglets and dogs c. 6-8 mg % d. None of the above
129. Deficiency of insulin causes which of the d. Piglets only d. 9-10 mg % 156. Ventricular fibrillation by rapid CBG
following? 138. Which of the following occurs in ketosis: 147. Normal ratio of calcium to phosphorus in administration is cured by:
a. Ketosis a. Vinegar smell in urine blood is a. CuSO4
b. Acidosis b. Vinegar smell in milk a. 2:1 b. K2SO4
c. Diabetic ketoacidosis c. Both of above b. 4:1 c. CaSO4
d. None d. None c. 6:1 d. Mg
Dhruv N Desai Dhruv N Desai
157. Temperature in Eclampsia is which of d. Methyl red 176. Salt used to differentiate myoglobin and a. Downer cow syndrome
the following: 167. Monday morning sickness is also known hemoglobin: b. Fatty cow syndrome
a. Subnormal as: a. NaCl c. Grass staggers
b. Normal a. Enzootic hemoglobinurea b. MgSO4 d. Milk fever
c. Increase b. Paralytic myoglobinurea c. NH4SO4 185. Ophisthotonus posture occurs in which
d. None c. Chronic Cu poisoning d. NH4OH disease:
158. Hypophosphatemia is also known as: d. All the above 177. Serum creatinine phosphokinase enzyme a. Tetanum and grass tetany
a. Post parturient hemoglobinurea 168. Bracken fern poisoning is which of the level increased in which of the following b. Milk fever and botulism
b. Lahu mutana following: disease: c. Downer cow syndrome
c. Red water a. Enzootic hemoglobinurea a. Hypomagnesaemia/ grass tetany d. Crush syndrome
d. All of the above b. Paralytic myoglobinurea b. Azoturia 186. In creeper cow disease which condition
159. Striking consequence of the c. Chronic Cu poisoning c. Chronic Cu poisoning occurs:
Hypophosphatemia is: d. All the above d. Excess of vitamin A a. Hypocalcaemia
a. Leucosis 169. In which of the following the color of the 178. High green grass feeding leads to which b. Hypoglycemia
b. Hemolysis urine is red: disease: c. Hypokalemia
c. Polycythemia a. Enzootic hemoglobinurea a. Hypomagnesaemia/ grass tetany d. Hypophosphatemia
d. Polyurea b. Paralytic myoglobinurea b. Azoturia 187. 3 day pig or baby pig disease represents
160. Body formed in RBC in lahumutana is: c. Chronic Cu poisoning c. Chronic Cu poisoning which of the following condition:
a. Heinz body d. All the above d. All the above a. Hypocalcaemia
b. Howell jolly body 170. Black water or tying up occur due to 179. Consider the following statement: b. Hypoglycemia
c. Both a and b secretion of which substance in urine: I. Tissue culture vaccines are used against c. Hypokalemia
d. None a. Hemoglobin canine distemper. d. Hypophosphatemia
161. Excess of which mineral leads to red b. Ketone bodies II. Measles virus vaccine is used against 188. Only source of CHO for piglet is which
water: c. Myoglobin canine distemper. of the following:
a. Mg d. Sugar Which of the above is true? a. Milk
b. P 171. Accumulation of which substance in a. I b. Grain
c. Ca muscle leads to Azoturia: b. II c. Both a and b
d. S a. Myoglobin c. Both I & II d. None
162. Death in red water is due to which of the b. Hemoglobin d. Both are not used 189. Course of downer cow syndrome is:
following: c. Sarcolactate 180. Wheat pasture poisoning is due to: a. 1-2 weeks
a. Anemia d. Ketone body a. Mg b. 2-4 weeks
b. Septicemia 172. Color of urine in black water becomes: b. Mn c. 4-6 weeks
c. Toxemia a. Pink c. Cu d. 6-8 weeks
d. All the above b. Coffee d. Fe 190. Well fed highly obesed cow during latter
163. Red water disease can be due to c. Black 181. Calves exclusively fed on milk lead to part of pregnancy show:
deficiency of which of the following: d. Red brown which of the following: a. Crush syndrome
a. P 173. A horse trying to lift hind quarter a. Whole milk tetany b. Downer cow syndrome
b. Cu indicates which disease: b. Grass tetany c. Fatty cow syndrome
c. Mg a. Azoturia c. Fatty cow syndrome d. All the above
d. Both a and b b. Hypomagnesaemia d. Downer cow syndrome 191. In which disease the Ca, P, Mg and
164. Normal level of P in blood is: c. Hemoglobinurea 182. Pregnancy toxaemia of cow is known as: Glucose blood level remains normal:
a. 0.5-3 mg/100 ml d. None of them a. Whole milk tetany a. Ketosis
b. 3-5 mg/100 ml 174. Most common muscle affected in b. Grass tetany b. Milk fever
c. 4-7 mg/100 ml myopathy of Azoturia: c. Fatty cow syndrome c. Downer cow syndrome
d. 5-9 mg/100 ml a. Deltoid d. Downer cow syndrome d. None
165. Level of phosphorus in red water is: b. Femoral 183. Post parturient recumbency is a 192. A metabolic disease of unknown
a. 0.5-3 mg/100 ml c. Facial symptom of which of the following etiology and is characterized by paresis is
b. 3-5 mg/100 ml d. Gluteal disease: known as:
c. 4-7 mg/100 ml 175. If red urine on centrifugation have a. Whole milk tetany a. Ketosis
d. 5-9 mg/100 ml supernatant of red color it indicate: b. Grass tetany b. Milk fever
166. After administration of which medicine a. Presence of myoglobin c. Fatty cow syndrome c. Downer cow syndrome
the color of urine becomes red: b. Presence of hemoglobin d. Downer cow syndrome d. Fatty cow syndrome
a. Phenothiazine c. Presence of Phenothiazine 184. Septic mastitis, hepatosis, myocardosis, 193. If animal quickly responds to calcium
b. Pyridium d. Presence of parasite nerve and muscle injuries and low level of therapy then we predict which of the
c. Methylene blue phosphorus, Mg, K, Ca are the etiology of: following disease:
Dhruv N Desai Dhruv N Desai
a. Ketosis c. Ketosis a. Ca, Vitamin D a. P
b. Milk fever d. None b. Ca, P, vitamin D b. Cu
c. Downer cow syndrome 201. Puerperal tetany is which of the c. P, vitamin D c. Fe
d. None following: d. Vitamin D d. Ca
194. If in biochemical test (benzedine test) on a. Eclampsia 219. Enterotoxaemia produced by E. coli are
microscopic examination, there is 8 b. Milk fever 210. Rickets is uncommon in which of the of two types viz. Heat stable and Heat
RBC/high field or more than animal is c. Ketosis following: labile. Which is most common?
suffering from which disease: d. None a. Pup a. Heat stable
a. Hematurea 202. Which of the following treatment is b. Foal b. Heat labile
b. Hemoglobinurea given in Eclampsia: c. Lamb c. Both are common
c. Myoglobinurea a. 10 % calcium gluconate d. Piglet d. Both are less common
d. All the above b. Phenobarbital Na 211. Less susceptibility of rickets is in which 220. Dog sitting posture of horse followed by
195. If there is less than 8 or no RBC / high c. Cortisone of the following: lateral recumbency is which of the
field the animal is suffering from which of d. All the above a. Pup following:
the following disease: 203. Which of the following is incorrect b. Foal a. PEM
a. Hematurea statement in relation to porcine stress c. Lamb b. Azoturia
b. Hemoglobinurea syndrome: d. Piglet c. Eclampsia
c. Myoglobinurea a. Death occurs due to excess K+ 212. Joint commonly involved in rickets is: d. All the above
d. All the above level causing cardiac arrest. a. Knee joint 221. Cause of death in Azoturia is which of
196. When blood may be voided in the form b. Death occurs due to low K+ level b. Shoulder joint the following:
of clot and causes deep red to brown causing respiratory failure. c. Pastern joint a. Uremia & nephrosis
coloration of urine then it is: c. Death occurs due to excess Mg2+ d. Intervertebral joint b. Decubital septicemia
a. Hematurea level causing cardiac arrest. 213. Excess of parathyroid hormone leads to c. Myoglobinurea
b. Hemoglobinurea d. Death occurs due to low Mg2+ level excessive removal of calcium from bone d. All the above
c. Myoglobinurea causing cardiac arrest. known as:
d. All the above 204. Which type of anemia occurs in sweet a. Osteodystrophy fibrosa 222. Primary lesion in Azoturia is which of
197. Which of the following occurs in chronic clover poisoning: b. Osteoporosis the following:
copper poisoning and in blood transfusion a. Microcytic Hypochromic c. Osteomalacia a. Myopathic
reactions: b. Macrocytic Hypochromic d. Osteoid leukemia b. Neuropathic
a. Hematurea c. Macrocytic normochromic 214. Depraved appetite/ pica is due to c. Both a and b
b. Hemoglobinurea d. Microcytic normochromic deficiency of which of the following: d. None
c. Myoglobinurea 205. Which is diagnosed by spectroscope: a. P 223. Secondary lesion of Azoturia is:
d. All the above a. Hematurea b. Ca a. Myopathic
198. Which of the following statement is b. Hemoglobinurea c. Mn b. Neuropathic
incorrect? c. Myoglobinurea d. Mg c. Both a and b
a. The order of ketogenic substance is d. None of the above 215. Molecular destruction of cement and d. None
concentrate > ensilage > hay 206. If tetany and hyperesthesia do not dentine due to Ca, P deficiency which 224. Muscles involved in Azoturia is which of
b. In ketosis animal refuse to eat grain disappear in 2nd stage of milk fever small cavity formation on table known as: the following:
and ensilage but continue to eat indicate: a. Scurvy a. Gluteal
hay a. Ca deficiency b. Dental caries b. Quadriceps
c. In ketosis the temperature, pulse b. Ca and P deficiency c. Fluorosis c. Rectus femoris
and respiration is normal c. Ca and Mg deficiency d. All the above d. All the above
d. Milk yield is fully regain after d. K deficiency 216. Indirect Ca deficiency due to heavy 225. Enzootic goiter commonly occurs in:
recovery from ketosis 207. Rickets in pup is deficiency of which of feeding of phosphorus known as: a. Hilly area
199. Which of the following is not the form of the following: a. Pica b. Plains
Hypomagnesemic tetany: a. Ca, Vitamin D b. Osteomalacia c. Costal area
a. Lactation tetany b. Ca, P, vitamin D c. Osteodystrophy fibrosa d. Both a and c
b. Summer tetany c. P, vitamin D d. Polioencephalomalacia 226. Demyelination and cerebral ataxia is due
c. Transit tetany d. Vitamin D 217. HypovitaminosisC in dog is known as: to deficiency of which of the following:
d. Milk tetany 208. Rickets in piglet is due to deficiency of: a. Scurvy a. Cu
200. In which of the following disease, there a. Ca, Vitamin D b. Dental caries b. Ca
is Ophisthotonus posture, rise of b. Ca, P, vitamin D c. Fluorosis c. K
temperature: c. P, vitamin D d. All the above d. Na
a. Eclampsia d. Vitamin D 218. Bottle jaw occurs due to deficiency of 227. Polioencephalomalacia is due to
b. Milk fever 209. Rickets in calf is due to deficiency of: which of the following: deficiency of:
Dhruv N Desai Dhruv N Desai
a. Vitamin A 235. In parvo virus infection there is an initial 243. In rodenticide poisoning, which of the d. Vitamin D
b. Vitamin C surge of: following vitamin injection is rhe drug of 244. The common cause of calf mortality is:
c. Vitamin B1 a. Ig G choice: a. Colisepticaemia
d. Vitamin D b. Ig D a. Vitamin A b. Hypoglycemia
228. Post hemorrhagic anemia is always: c. Ig A b. Vitamin C c. Pneumonia
a. Sickle cell anaemia d. Ig M c. Vitamin K
b. Parasitic anaemia 236. Lamb dysentery prophylaxis can be d. Vitamin A deficiency d. Only III
c. Both a and b given to lambs: 245. The most specific drug for the treatment of 248. Which of the following is not a form of
d. None a. In the month of winter Azoturia is: canine distemper:
229. Deficiency anaemia is due to deficiency b. In the month of autumn a. Corticosteroids a. Pulmonary
of: c. In lambing season b. Thiamine hydrochloride b. Digestive
a. Co d. In any month of year c. Antihistaminic c. Ocular
b. I 237. Canine distemper affect mostly pups of d. Antibiotics d. Cardiac
c. Fe age: 246. Which of the cell is most affected in canine 249. Certain plants are responsible for causing
d. Mg a. Above 12 months distemper: PPH because of:
230. Folic acid deficiency can cause which b. Within 1 week after birth a. Respiratory epithelial cell a. Toxic substance
type of anaemia: c. Upto 3-6 months of age b. Neuron b. High saponin content
a. Hypochromic d. Over 9 months of age c. Intestinal villi c. High fiber content
b. Hyperchromic 238. Porcine stress syndrome is a: d. RBC d. High potassium content
c. Microcytic a. Calcium deficiency disorder 247. Consider the following statements: 250. Hard pad disease or canine distemper is
d. Both a and c b. Vitamin deficiency disorder I. High glucose level retards fat mobilization. synonym for:
231. Mannose binding test is performed in c. Mineral deficiency disorder II. Insulin retards fat mobilization. a. Carres
case of: d. Genetic disorder III. Nor-epinephrine retards fat mobilization. b. Canine parvo virus
a. Calf scour 239. Malignant hyperthermia of porcine is Which of the above is correct? c. Canine influenza
b. White diarrhea caused due to defective gene which is: a. I & II d. Both a and c
c. Only a a. BAL gene b. I, II & III
d. Both a and b b. HAL gene c. II & III
232. Hemolytic anaemia in the post parturient c. GAL gene 22. b
animals is due to: d. FAL gene 23. b
a. Phosphate deficiency 240. Possible cause of downer cow syndrome ANSWER:- 24. d
b. Copper deficiency is/are:
25. a
c. Molybdenosis a. Persistent hypocalcaemia 1. C
d. All the above b. Hypokalemia 26. d
2. A
c. Hypomagnesaemia 27. c
3. c
d. All the above 28. b
4. a
233. Consider the following statements: 241. Consider the following statements; 29. c
I. Anticanine distemper serum may be tried I. There is congenital goiter in all 5. d
30. b
to safe guard the life of the patient. domesticated animals. 6. c
31. d
II. Antiviral canine distemper vaccine is II. Impairment of sexual urge in 7. b
32. c
also available for CD. breeding bull 8. a
Which of the above is correct? 33. a
9. a
a. I Which of the above are the clinical signs 34. b
of Iodine deficiency? 10. d
b. II 35. d
11. c
c. Both I & II 36. d
a. I 12. a
d. None 37. b
b. II 13. c
234. A young pup was brought to the clinics 38. a
c. Both I and II
with signs of inappetance, refusal of food, 14. d
d. None 39. c
polydipsia, frothy yellow colored vomitus, 15. c
242. Increase in CSF pressure is observed in: 40. c
retching and restlessness. The probable 16. c
a. Hypocalcaemia 41. d
diagnosis that you will make is that pup is 17. d
b. Hypovitaminosis A
suffering from: 42. c
c. Hypomagnesaemia 18. c
a. Parvo virus enteritis 43. a
d. Hypovitaminosis E 19. a
b. Parvo virus enteritis 44. d
c. Parvo virus lymphangitis 20. d
45. b
d. Canine distemper 21. d
Dhruv N Desai Dhruv N Desai
46. b 93. c 140. d 187. b
47. b 94. b 141. a 188. a
48. a 95. a 142. d 189. a
49. d 96. b 143. b 190. c
50. c 97. c 144. a 191. d
51. b 98. d 145. d 192. c
52. c 99. c 146. b 193. b
53. a 100. d 147. a 194. a
54. a 101. b 148. c 195. b
55. c 102. b 149. a 196. a
56. a 103. a 150. b 197. b
57. b 104. a 151. a 198. d
58. a 105. c 152. c 199. b
59. b 106. c 153. c 200. a
60. b 107. c 154. c 201. a
61. d 108. a 155. b 202. d
62. d 109. b 156. d 203. a
63. c 110. d 157. c 204. a
64. d 111. c 158. d 205. c
65. b 112. c 159. b 206. b
66. c 113. d 160. a 207. a
67. b 114. a 161. d 208. a
68. d 115. c 162. a 209. b
69. c 116. b 163. d 210. b
70. b 117. a 164. c 211. b
71. b 118. c 165. a 212. a
72. a 119. d 166. a 213. a
73. b 120. c 167. b 214. a
74. b 121. b 168. a 215. b
75. a 122. a 169. a 216. c
76. a 123. a 170. c 217. a
77. a 124. d 171. a 218. c
78. a 125. a 172. b 219. b
79. b 126. d 173. a 220. b
80. c 127. c 174. d 221. d
81. b 128. a 175. a 222. a
82. a 129. c 176. c 223. b
83. c 130. b 177. a 224. d
84. a 131. c 178. a 225. a
85. a 132. a 179. b 226. a
86. b 133. d 180. a 227. c
87. d 134. a 181. a 228. b
88. b 135. b 182. c 229. c
89. b 136. c 183. d 230. d
90. b 137. d 184. a 231. d
91. c 138. c 185. c 232. a
92. b 139. a 186. a 233. a

Dhruv N Desai Dhruv N Desai


234. a 243. c
235. d 244. a
236. c 245. b
237. c 246. a
238. d 247. a
239. b 248. d
240. d 249. b
241. c 250. d
242. a

Dhruv N Desai

You might also like